Microeconomics Study Guide

1085

Click here to load reader

description

This is a compilation of study guides and self quizzes from Microeconomics

Transcript of Microeconomics Study Guide

Page 1: Microeconomics Study Guide

Oligopoly firms may produce either standardized or differentiated products.TrueFalse

True

Which of the following is correct as it relates to cost curves?

Marginal cost intersects average total cost at the latter's minimum point.

The law of diminishing marginal utility states that

beyond some point additional units of a product will yield less and less extra satisfaction to a consumer.

Answer the next question(s) on the basis of the following cost data:

Refer to the above data. The marginal cost of producing the sixth unit of output is:

$8

The long run is characterized by

he ability of the firm to change its plant size

Suppose that, when producing 10 units of output, a firm's AVC is $22, its AFC is $5, and its MC is $30. This

firm's total cost is $270

The law of diminishing marginal utility explains why

demand curves slope downward

The theory of consumer behavior assumes that:

consumers behave rationally, attempting to maximize their satisfaction

An industry comprised of four firms, each with about 25 percent of the total market for a product is an example of

oligopoly.

The total output of a firm will be at a maximum where:

MP is zero.

Page 2: Microeconomics Study Guide

The basic characteristic of the short run is that

the firm does not have sufficient time to change the size of its plant.

Firms seek to maximize:

total profit.

Which of the following is not a characteristic of pure competition?

price strategies by firms

Refer to the above diagram. The vertical distance between ATC and AVC reflects:

the average fixed cost at each level of output.

Refer to the above data. In the long run the firm should use plant size "A" for:

10 to 30 units of output.

Refer to the above information. The marginal cost of the third unit of output is

$15.

Refer to the above data. The value for X is

15

Refer to the above short-run production and cost data. In Figure B curve (3) is

MC and curve (4) is AVC.

Accounting profits equal total revenue minus

total explicit costs.

Refer to the above information. The average total cost of 3 units of output is:

$35.

When diseconomies of scale occur:

the long-run average total cost curve rises.

Refer to the above short-run production and cost data. The curves of Figures A and B suggest that:

marginal cost reaches a minimum where marginal product is at its maximum.

Page 3: Microeconomics Study Guide

Refer to the above data. The value for W is:

20.

The marginal utility of the last unit of apples consumed is 12 and the marginal utility of the last unit of bananas consumed is 8. What set of prices for apples and bananas, respectively, would be consistent with consumer equilibrium?

$6 and $4

A consumer who has a limited budget will maximize utility or satisfaction when the:

ratios of the marginal utility of each product purchased divided by its price are equal.

If a firm increases all of its inputs by 10 percent and its output increases by 10 percent, then:

it is encountering constant returns to scale.

The Sunshine Corporation finds that its costs are $40 when it produces no output. Its total variable costs (TVC) change with output as shown in the accompanying table. Use this information to answer the following question(s).

Refer to the above information. The average fixed cost of 3 units of output is:

$13.33.

An industry comprised of a very large number of sellers producing a standardized product is known as:

pure competition.

Answer the next question(s) on the basis of the following information:

Refer to the above information. Average total cost is:

TFC+TVC/Q

Refer to the above data. The value for Y is:

45.

The short run is characterized by:

fixed plant capacity.

Page 4: Microeconomics Study Guide

The first Pepsi yields Craig 18 units of utility and the second yields him an additional 12 units of utility. His total utility from three Pepsis is 38 units of utility. The marginal utility of the third Pepsi is:

8 units of utility.

If total utility is increasing, marginal utility:

is positive, but may be either increasing or decreasing.

In the above diagram curves 1, 2, and 3 represent the:

marginal, average, and total product curves respectively.

As the firm in the above diagram expands from plant size #3 to plant size #5, it experiences:

economies of scale.

Use the following data to answer the next question(s). The letters A, B, and C designate three successively larger plant sizes.

Refer to the above data. In the long run the firm should use plant size "C" for:

all units of output greater than 80.

Economies of scale are indicated by:

the declining segment of the long-run average total cost curve.

To maximize utility a consumer should allocate money income so that the:

marginal utility obtained from the last dollar spent on each product is the same.

The theory of consumer behavior assumes that consumers attempt to maximize:

total utility.

A purely competitive seller is:

a "price taker."

When a consumer shifts purchases from product X to product Y the marginal utility of:

X rises and the marginal utility of Y falls.

Answer the next question(s) on the basis of the following cost data:

Page 5: Microeconomics Study Guide

Refer to the above data. The average total cost of producing 3 units of output is:

$16.

Answer the next question(s) on the basis of the following cost data:

Refer to the above data. The average fixed cost of producing 3 units of output is:

$8.

The Sunshine Corporation finds that its costs are $40 when it produces no output. Its total variable costs (TVC) change with output as shown in the accompanying table. Use this information to answer the following question(s).

Refer to the above information. The total cost of producing 3 units of output is:

$105.

Ben is exhausting his money income consuming products A and B in such quantities that MUa/Pa= 5 and MUb/Pb= 8. Ben should purchase:

more of B and less of A.

Average fixed cost:

declines continually as output increases.

An industry comprised of 40 firms, none of which has more than 3 percent of the total market for a differentiated product is an example of:

monopolistic competition.

Refer to the above diagram. At output level Q total variable cost is:

0BEQ.

Where total utility is at a maximum, marginal utility is:

zero.

Refer to the above diagram. At output level Q total cost is:

0BEQ plus BCDE.

Page 6: Microeconomics Study Guide

Economies and diseconomies of scale explain:

why the firm's long-run average total cost curve is U-shaped.

In the above figure, curves 1, 2, 3, and 4 represent the:

MC, ATC, AVC, and AFC curves respectively.

In which of the following industry structures is the entry of new firms the most difficult?

pure monopoly

Marginal cost is the:

change in total cost that results from producing one more unit of output.

If MUa/Pa= 100/$35 = MUb/Pb= 300/? = MUc/Pc= 400/?, the prices of products b and c in consumer equilibrium:

are $105 and $140 respectively.

Marginal product is:

the increase in total output attributable to the employment of one more worker.

If a firm decides to produce no output in the short run, its costs will be:

its fixed costs.

Refer to the above diagram. At output level Q total fixed cost is:

BCDE.

Suppose that MUx/Pxexceeds MUy/Py. To maximize utility the consumer who is spending all her money income should buy:

more of X and less of Y.

Suppose you have a limited money income and you are purchasing products A and B whose prices happen to be the same. To maximize your utility you should purchase A and B in such amounts that:

their marginal utilities are the same.

The basic difference between the short run and the long run is that:

at least one resource is fixed in the short run, while all resources are variable in the long run.

Fixed cost is:

Page 7: Microeconomics Study Guide

any cost which does not change when the firm changes its output.

Refer to the above data. The value for Z is:

-5

In the long run:

all costs are variable costs.

To economists, the main difference between the short run and the long run is that:

in the long run all resources are variable, while in the short run at least one resource is fixed.

Answer the next question(s) on the basis of the following cost data:

Refer to the above data. The total variable cost of producing 5 units is:

$37

Economic profits are calculated by subtracting:

explicit and implicit costs from total revenue.

Refer to the above short-run production and cost data. In Figure A curve (1) is:

average product and curve (2) is marginal product.

If a firm increases all of its inputs by 10 percent and its output increases by 15 percent, then:

it is encountering economies of scale.

If a firm doubles its output in the long run and its unit costs of production decline, we can conclude that:

economies of scale are being realized.

Marginal product is:

the increase in total output attributable to the employment of one more worker.

The theory of consumer behavior assumes that:

consumers behave rationally, attempting to maximize their satisfaction.

Answer the next question(s) on the basis of the following cost data:

Page 8: Microeconomics Study Guide

Refer to the above data. The marginal cost of producing the sixth unit of output is:

$8.

Refer to the above diagram. At output level Q total variable cost is:

0BEQ.

In the long run:

all costs are variable costs.

1. 'Economies of scale' refers to:A. the notion that small firms are less bureaucratic and, therefore, more efficient than corporations.B. public investments in highways, schools, utilities etc.C. the fact that large producers may be able to use more efficient technologies.D. the reallocation of labour from less productive to more productive uses.

Page 9: Microeconomics Study Guide

C

2. Economic cost can best be defined as:A. any contractual obligation which results in a flow of money expenditures from an enterprise to resource suppliers.B. any contractual obligation to labour, or material suppliers.C. compensations which must be received by resource owners to ensure their continued supply.D. all costs, exclusive of payments to fixed factors of production.

C

3. Which of the following constitutes an implicit cost to the Jackson Manufacturing Company?A. Payments of wages to its office workers.B. Property taxes.C. Rent paid for the use of equipment owned by the Schultz Machinery Company.D. Depreciation charges on company-owned equipment.

D

4. Suppose that a business incurred implicit costs of $200 000 and explicit costs of $1 million in a specificyear. If the firm sold 4000 units of its output at $300 per unit, its accounting profits were:A. $100 000, and its economic profits were zero.B. $200 000, and its economic profits were zero.C. $100 000, and its economic profits were $100 000.D. zero, and its economic loss was $200 000.

B

5. Economic profits are calculated by subtracting:A. explicit costs from total revenue.B. implicit costs from total revenue.C. implicit costs from normal profits.D. explicit and implicit costs from total revenue.

D

6. Normal profit is:A. determined by subtracting implicit costs from total revenue.B. determined by subtracting explicit costs from total revenue.C. the return to the entrepreneur when economic profits are zero.D. the average profitability of an industry over the preceding 10 years.

C

Page 10: Microeconomics Study Guide

7. Which of the following definitions is correct?A. Accounting profit + economic profit = normal profit.B. Economic profit - accounting profit = explicit costs.C. Economic profit = accounting profit - implicit costs.D. Economic profit - implicit costs = accounting profits.

C

8. The basic characteristic of the short run is that:A. 'barriers to entry' prevent new firms from entering the industry.B. the firm does not have sufficient time to change the size of its plant.C. the firm does not have sufficient time to cut its rate of output to zero.D. the firm does not have sufficient time to change the amounts of any of the resources it employs.

B

9. To economists, the main difference between 'the short run' and 'the long run' is that:A. the law of diminishing returns applies in the long run, but not in the short run.B. in the long run, all resources are variable while, in the short run, at least one resource is fixed.C. fixed costs are more important to decision making in the long run than they are in the short run.D. in the short run all resources are fixed, while in the long run, all resources are variable.

B

10. The law of diminishing returns indicates that:A.as extra units of a variable resource are added to a fixed resource, the extra or marginal product willdecline beyond some point.B. because of economies and diseconomies of scale, a competitive firm's long-run average cost curve will be U-shaped.C. the demand for goods produced by purely competitive industries is down sloping.D. beyond some point, the extra utility derived from additional units of a product will yield for the consumer smaller and smaller extra amounts of satisfaction.

A

11. Which of the following statements concerning the relationships between total product (TP), averageproduct (AP) and marginal product (MP) is not correct?A. AP continues to rise so long as TP is rising.B. AP reaches a maximum before TP reaches a maximum.C. TP reaches a maximum when the MP of the variable input becomes zero.D. MP cuts AP at the maximum AP.

Page 11: Microeconomics Study Guide

A

12. The following is output data for a firm. Assume that the amounts of all non-labour resources are fixed.Number of workers Units of output0 01 402 903 1264 1505 1656 180Refer to the above information. Diminishing returns become evident with the addition of:A. the fourth worker.B. the third worker.C. the second worker.D. the first worker.

B

13. The following is the total and marginal product for a firm.Number of workers Total product Marginal product0 0 —1 8 82 103 254 305 36 34Refer to the above information. When two workers are employed:A. total product is 20.B. total product is 18.C. average product is 10.D. total product cannot be determined from the information given.

B

14. Which of the following is most likely to be a fixed cost?A. Shipping charges.B. Property insurance premiums.C. Wages for unskilled labour.D. Expenditures for raw materials.

B

15. The cost of any output is minimised when:A. the marginal product per dollar's worth of each resource used is the same.

Page 12: Microeconomics Study Guide

B. the marginal product of each resource used is the same.C. the price of each resource used is the same.D. none of the above.

A

16. Which of the following is most likely to be a variable cost?A. Fuel and power payments.B. Interest on bonded indebtedness.C. Rental payments on IBM equipment.D. Real estate taxes.

A

17. Marginal cost may be defined as the:A. rate of change in total fixed cost which results from producing one more unit of output.B. change in total cost which results from producing one more unit of output.C. change in average variable cost which results from producing one more unit of output.D. change in average total cost which results from producing one more unit of output.

B

18. Average fixed cost:A. is intersected by marginal cost at its minimum point.B. may be found for any output by adding average variable cost and average total cost.C. graphs as a U-shaped curve.D. declines so long as output increases.

D

19. The vertical distance between ATC and AVC reflects:A. the average fixed cost at each level of output.B. marginal cost at each level of output.C. the presence of economies of scale.D. implicit costs.

A

20. Assume that in the short run, a firm which is producing 100 units of output has average total costs of $200and average variable costs of $150. The firm's total fixed costs are:A. $5000.B. $500.C. $0.50.D. $50.

A

Page 13: Microeconomics Study Guide

21. The following is the total output and cost data for a firm.Total Output Total Cost($)0 241 333 413 484 545 616 69Refer to the above cost data. The average total cost of producing 3 units of output:A. is $14.00.B. is $16.00.C. is $13.50.D. cannot be determined from the information given.

B

22. The following is the total output and cost data for a firm.Total Output Total Cost($)0 241 333 413 484 545 616 69Refer to the above cost data. The marginal cost of producing the sixth unit of output:A. is $24.B. is $16.C. is $8.D. cannot be determined from the information given.

C

23. If a technological advance reduces the amount of variable resources needed to produce any given level ofoutput, this will cause:A. the AVC curve to shift downward.B. the MC curve to shift downward.C. the ATC curve to shift downward.D. all of the above.

D

24. The Sunshine Corporation finds that its costs are $40 when it produces no output. Its total variable costs(TVC) change with output as shown in the table below.

Page 14: Microeconomics Study Guide

Output TVC($)1 302 503 654 855 110Refer to the above information. This firm:A. is making an economic profit of $260.B. is realising a loss of $125.C. may be either realising a profit or a loss.D. is selling its output in a competitive market.

C

25. Economies and diseconomies of scale explain:A. the profit-maximising level of production.B. why the firm's long-run average cost curve is U-shaped.C.why the firm's short-run marginal cost curve cuts the short-run average variable cost curve at itsminimum point.D. the distinction between fixed and variable costs.

B

26. In the long run:A. all costs are variable costs.B. all costs are fixed costs.C. variable costs equal fixed costs.D. fixed costs are greater than variable costs.

A

27. The long-run average cost curve:A. will rise if diminishing returns are encountered.B. will fall if diminishing returns are encountered.C. will rise if economies of scale are incurred.D. is based on the assumption that all resources are variable.

D

28. If a firm doubles its output in the long run and its unit costs of production decline, we can conclude that:A. technological progress has occurred.B. economies of scale are being realised.C. the firm is encountering diminishing returns.D. diseconomies of scale are being encountered.

Page 15: Microeconomics Study Guide

B

29. The minimum efficient scale of a firm:A. is realised somewhere in the range of diseconomies of scale.B. occurs where marginal product becomes zero.C. is in the middle of the range of constant returns to scale.D. is the smallest level of output at which long-run average cost is minimised.

D

30. If an industry's long-run average cost curve has an extended range of constant returns to scale, this impliesthat:A. technology precludes both economies and diseconomies of scale.B. the industry will be a natural monopoly.C. both relatively small and relatively large firms can be viable in the industry.D. the industry will be comprised of a very large number of small firms.

C

31. If a firm increases all of its inputs by 10% and its output increases by 15%, we can say that:A. it is encountering diseconomies of scale.B. it is encountering economies of scale.C. the law of diminishing returns is taking hold.D. the firm's long-run ATC curve will be rising.

B

32. As successive units of labour are applied to a set of fixed resources, the marginal product of labour:A. diminishes at all levels of production.B. may initially increase, then diminish, but will never become negative.C. may initially increase, then diminish, and ultimately become negative.D. is always less than average product.

C

33. The short run is a period of time during which all costs are fixed costs.A. TrueB. False

...

34. The law of diminishing returns accounts for the fact that the long-run average cost curve is U-shaped.A. TrueB. False

Page 16: Microeconomics Study Guide

...

35. Diseconomies of scale stem, primarily, from the difficulties in managing and coordinating a large-scalebusiness enterprise.A. TrueB. False

...

36. At zero units of output, a firm's variable costs are zero.A. TrueB. False

...

37. Average fixed costs diminish so long as output increases.A. TrueB. False

...

38. If marginal cost lies below average variable cost, average variable cost must be falling.A. TrueB. False

...

39. When economic profits equal zero, normal profits are negative or equal zero.A. TrueB. False

...

40. In the long run, all resource inputs are fixed instead of variable.A. TrueB. False

...

41. The law of diminishing marginal utility explains why short-run production costs increase directly with afirm's level of output.A. TrueB. False

...

Page 17: Microeconomics Study Guide

42. The short-run marginal-cost curve is upward sloping because of the law of diminishing marginal returns.A. TrueB. False

...

43. Economies of scale are generally in evidence over an entire range of output.A. TrueB. False

...

44. Economies of scale are generally in evidence over low to medium levels of output.A. TrueB. False

...

45. If economies of scale are quickly exhausted, and diseconomies of scale set in at a low level of outputrelative to the market, the industry will have a small number of large firms.A. TrueB. False

...

46. Explicit costs are resources for which the firm does not pay, but which have an opportunity cost, whileimplicit costs are resource for which direct monetary payment is made.A. TrueB. False

...

47. An example of an explicit cost of production would be:A. the cost of forgone labour earnings for an entrepreneur.B. the lost opportunity to invest in other capital markets when the money is invested in one's business.C. the cost of flour for a baker.D. None of the above is correct.

...

48. An example of an implicit cost of production would be:A.the income an entrepreneur could have earned working for someone else.B. the cost of raw materials for producing bread in a bakery.

Page 18: Microeconomics Study Guide

C. the cost of a delivery truck in a business that rarely makes deliveries.D. All of the above are correct.

...

49. Which of these assumptions is often realistic for a firm in the short run?A. The firm can vary both the size of its factory and the number of workers it employs.B. The firm can vary the size of its factory, but not the number of workers it employs.C.The firm can vary the number of workers it employs, but not the size of its factory.D. The firm can vary neither the size of its factory nor the number of workers it employs.

...

50. The marginal product of an input in the production process is the increase in:A. total revenue obtained from an additional unit of that input.B. profit obtained from an additional unit of that input.C. total revenue obtained from an additional unit of that input.D. quantity of output obtained from an additional unit of that input.

...

51. Fixed costs can be defined as costs that:A. vary inversely with production.B. vary in proportion with production.C. are incurred only when production is large enough.D. are incurred even if nothing is produced.

...

52. When marginal cost is less than average total cost:A. marginal cost must be falling.B. average variable cost must be falling.C. average total cost is falling.D. average total cost is rising.

...

53. Marginal cost is equal to average total cost when:A. average variable cost is falling.B. average fixed cost is rising.C. marginal cost is at its minimum.D. average total cost is at its minimum.

...

54. If marginal product is rising:A. marginal cost will be falling.B. marginal cost will be increasing.

Page 19: Microeconomics Study Guide

C. marginal cost will be constant.D. There is no relationship between marginal product and marginal cost.

...

Oligopoly firms may produce either standardized or differentiated products.TrueFalse

True

Which of the following is correct as it relates to cost curves?

Marginal cost intersects average total cost at the latter's minimum point.

The law of diminishing marginal utility states that

beyond some point additional units of a product will yield less and less extra satisfaction to a consumer.

Answer the next question(s) on the basis of the following cost data:

Refer to the above data. The marginal cost of producing the sixth unit of output is:

$8

The long run is characterized by

he ability of the firm to change its plant size

Suppose that, when producing 10 units of output, a firm's AVC is $22, its AFC is $5, and its MC is $30. This

firm's total cost is $270

The law of diminishing marginal utility explains why

demand curves slope downward

The theory of consumer behavior assumes that:

consumers behave rationally, attempting to maximize their satisfaction

Page 20: Microeconomics Study Guide

An industry comprised of four firms, each with about 25 percent of the total market for a product is an example of

oligopoly.

The total output of a firm will be at a maximum where:

MP is zero.

The basic characteristic of the short run is that

the firm does not have sufficient time to change the size of its plant.

Firms seek to maximize:

total profit.

Which of the following is not a characteristic of pure competition?

price strategies by firms

Refer to the above diagram. The vertical distance between ATC and AVC reflects:

the average fixed cost at each level of output.

Refer to the above data. In the long run the firm should use plant size "A" for:

10 to 30 units of output.

Refer to the above information. The marginal cost of the third unit of output is

$15.

Refer to the above data. The value for X is

15

Refer to the above short-run production and cost data. In Figure B curve (3) is

MC and curve (4) is AVC.

Accounting profits equal total revenue minus

total explicit costs.

Refer to the above information. The average total cost of 3 units of output is:

$35.

When diseconomies of scale occur:

Page 21: Microeconomics Study Guide

the long-run average total cost curve rises.

Refer to the above short-run production and cost data. The curves of Figures A and B suggest that:

marginal cost reaches a minimum where marginal product is at its maximum.

Refer to the above data. The value for W is:

20.

The marginal utility of the last unit of apples consumed is 12 and the marginal utility of the last unit of bananas consumed is 8. What set of prices for apples and bananas, respectively, would be consistent with consumer equilibrium?

$6 and $4

A consumer who has a limited budget will maximize utility or satisfaction when the:

ratios of the marginal utility of each product purchased divided by its price are equal.

If a firm increases all of its inputs by 10 percent and its output increases by 10 percent, then:

it is encountering constant returns to scale.

The Sunshine Corporation finds that its costs are $40 when it produces no output. Its total variable costs (TVC) change with output as shown in the accompanying table. Use this information to answer the following question(s).

Refer to the above information. The average fixed cost of 3 units of output is:

$13.33.

An industry comprised of a very large number of sellers producing a standardized product is known as:

pure competition.

Answer the next question(s) on the basis of the following information:

Refer to the above information. Average total cost is:

TFC+TVC/Q

Refer to the above data. The value for Y is:

Page 22: Microeconomics Study Guide

45.

The short run is characterized by:

fixed plant capacity.

The first Pepsi yields Craig 18 units of utility and the second yields him an additional 12 units of utility. His total utility from three Pepsis is 38 units of utility. The marginal utility of the third Pepsi is:

8 units of utility.

If total utility is increasing, marginal utility:

is positive, but may be either increasing or decreasing.

In the above diagram curves 1, 2, and 3 represent the:

marginal, average, and total product curves respectively.

As the firm in the above diagram expands from plant size #3 to plant size #5, it experiences:

economies of scale.

Use the following data to answer the next question(s). The letters A, B, and C designate three successively larger plant sizes.

Refer to the above data. In the long run the firm should use plant size "C" for:

all units of output greater than 80.

Economies of scale are indicated by:

the declining segment of the long-run average total cost curve.

To maximize utility a consumer should allocate money income so that the:

marginal utility obtained from the last dollar spent on each product is the same.

The theory of consumer behavior assumes that consumers attempt to maximize:

total utility.

A purely competitive seller is:

a "price taker."

Page 23: Microeconomics Study Guide

When a consumer shifts purchases from product X to product Y the marginal utility of:

X rises and the marginal utility of Y falls.

Answer the next question(s) on the basis of the following cost data:

Refer to the above data. The average total cost of producing 3 units of output is:

$16.

Answer the next question(s) on the basis of the following cost data:

Refer to the above data. The average fixed cost of producing 3 units of output is:

$8.

The Sunshine Corporation finds that its costs are $40 when it produces no output. Its total variable costs (TVC) change with output as shown in the accompanying table. Use this information to answer the following question(s).

Refer to the above information. The total cost of producing 3 units of output is:

$105.

Ben is exhausting his money income consuming products A and B in such quantities that MUa/Pa= 5 and MUb/Pb= 8. Ben should purchase:

more of B and less of A.

Average fixed cost:

declines continually as output increases.

An industry comprised of 40 firms, none of which has more than 3 percent of the total market for a differentiated product is an example of:

monopolistic competition.

Refer to the above diagram. At output level Q total variable cost is:

0BEQ.

Page 24: Microeconomics Study Guide

Where total utility is at a maximum, marginal utility is:

zero.

Refer to the above diagram. At output level Q total cost is:

0BEQ plus BCDE.

Economies and diseconomies of scale explain:

why the firm's long-run average total cost curve is U-shaped.

In the above figure, curves 1, 2, 3, and 4 represent the:

MC, ATC, AVC, and AFC curves respectively.

In which of the following industry structures is the entry of new firms the most difficult?

pure monopoly

Marginal cost is the:

change in total cost that results from producing one more unit of output.

If MUa/Pa= 100/$35 = MUb/Pb= 300/? = MUc/Pc= 400/?, the prices of products b and c in consumer equilibrium:

are $105 and $140 respectively.

Marginal product is:

the increase in total output attributable to the employment of one more worker.

If a firm decides to produce no output in the short run, its costs will be:

its fixed costs.

Refer to the above diagram. At output level Q total fixed cost is:

BCDE.

Suppose that MUx/Pxexceeds MUy/Py. To maximize utility the consumer who is spending all her money income should buy:

more of X and less of Y.

Suppose you have a limited money income and you are purchasing products A and B whose prices happen to be the same. To maximize your utility you should purchase A and B in such amounts that:

Page 25: Microeconomics Study Guide

their marginal utilities are the same.

The basic difference between the short run and the long run is that:

at least one resource is fixed in the short run, while all resources are variable in the long run.

Fixed cost is:

any cost which does not change when the firm changes its output.

Refer to the above data. The value for Z is:

-5

In the long run:

all costs are variable costs.

To economists, the main difference between the short run and the long run is that:

in the long run all resources are variable, while in the short run at least one resource is fixed.

Answer the next question(s) on the basis of the following cost data:

Refer to the above data. The total variable cost of producing 5 units is:

$37

Economic profits are calculated by subtracting:

explicit and implicit costs from total revenue.

Refer to the above short-run production and cost data. In Figure A curve (1) is:

average product and curve (2) is marginal product.

If a firm increases all of its inputs by 10 percent and its output increases by 15 percent, then:

it is encountering economies of scale.

If a firm doubles its output in the long run and its unit costs of production decline, we can conclude that:

economies of scale are being realized.

Marginal product is:

Page 26: Microeconomics Study Guide

the increase in total output attributable to the employment of one more worker.

The theory of consumer behavior assumes that:

consumers behave rationally, attempting to maximize their satisfaction.

Answer the next question(s) on the basis of the following cost data:

Refer to the above data. The marginal cost of producing the sixth unit of output is:

$8.

Refer to the above diagram. At output level Q total variable cost is:

0BEQ.

In the long run:

all costs are variable costs.

Page 27: Microeconomics Study Guide

 1. The basic formula for the price elasticity of demand coefficient is: A. absolute decline in quantity demanded/absolute increase in price.B. percentage change in quantity demanded/percentage change in price.C. absolute decline in price/absolute increase in quantity demanded.D. percentage change in price/percentage change in quantity demanded.

 

AACSB: AnalyticBlooms: Level 1 RememberDifficulty: 1 EasyLearning Objective: 04-01 Discuss price elasticity of demand and how it can be applied.McConnell - Chapter 04 #2Topic: Price elasticity of demand 

2. In which price range of the accompanying demand schedule is demand elastic?

    A. $4-$3B. $3-$2C. $2-$1D. below $1

 

AACSB: AnalyticBlooms: Level 4 AnalyzeDifficulty: 3 HardLearning Objective: 04-01 Discuss price elasticity of demand and how it can be applied.McConnell - Chapter 04 #39Topic: Price elasticity of demandType: Table 

3. Suppose the price elasticity of demand for bread is 0.20. If the price of bread falls by 10 percent, the quantity demanded will increase by: A. 2 percent and total expenditures on bread will rise.B. 2 percent and total expenditures on bread will fall.C. 20 percent and total expenditures on bread will fall.D. 20 percent and total expenditures on bread will rise.

 

AACSB: AnalyticBlooms: Level 2 UnderstandDifficulty: 2 MediumLearning Objective: 04-02 Explain the usefulness of the total revenue test for price elasticity of demand.McConnell - Chapter 04 #50Topic: Total revenue test 

Page 28: Microeconomics Study Guide

4. Suppose that the price of peanuts falls from $3 to $2 per bushel and that, as a result, the total revenue received by peanut farmers changes from $16 to $14 billion. Thus: A. the demand for peanuts is elastic.B. the demand for peanuts is inelastic.C. the demand curve for peanuts has shifted to the right.D. no inference can be made as to the elasticity of demand for peanuts.

 

AACSB: Reflective ThinkingBlooms: Level 3 ApplyDifficulty: 2 MediumLearning Objective: 04-02 Explain the usefulness of the total revenue test for price elasticity of demand.McConnell - Chapter 04 #65Topic: Total revenue test 

5. The elasticity of demand for a product is likely to be greater: A. if the product is a necessity, rather than a luxury good.B. the greater the amount of time over which buyers adjust to a price change.C. the smaller the proportion of one's income spent on the product.D. the smaller the number of substitute products available.

 

AACSB: AnalyticBlooms: Level 1 RememberDifficulty: 1 EasyLearning Objective: 04-01 Discuss price elasticity of demand and how it can be applied.McConnell - Chapter 04 #74Topic: Determinants of price elasticity 

6. An antidrug policy which reduces the supply of heroin might: A. increase street crime because the addict's demand for heroin is highly inelastic.B. reduce street crime because the addict's demand for heroin is highly elastic.C. reduce street crime because the addict's demand for heroin is highly inelastic.D. increase street crime because the addict's demand for heroin is highly elastic.

 

AACSB: Reflective ThinkingBlooms: Level 4 AnalyzeDifficulty: 3 HardLearning Objective: 04-01 Discuss price elasticity of demand and how it can be applied.McConnell - Chapter 04 #118Topic: Applications of price elasticity 

Page 29: Microeconomics Study Guide

7. The main determinant of elasticity of supply is the: A. number of close substitutes for the product available to consumers.B. amount of time the producer has to adjust inputs in response to a price change.C. urgency of consumer wants for the product.D. number of uses for the product.

 

AACSB: AnalyticBlooms: Level 1 RememberDifficulty: 1 EasyLearning Objective: 04-03 Describe price elasticity of supply and how it can be applied.McConnell - Chapter 04 #85Topic: Elasticity of supply 

    

 

McConnell - Chapter 04 

8. Refer to the above diagram and assume that price decreases from $10 to $2. The coefficient of the price elasticity of supply (midpoint formula) relating to this price change is about: A. 4 and supply is elastic.B. 1 and supply is unit elastic.C. .5 and supply is inelastic.D. .25 and supply is inelastic.

 

AACSB: AnalyticBlooms: Level 3 ApplyDifficulty: 2 MediumLearning Objective: 04-03 Describe price elasticity of supply and how it can be applied.McConnell - Chapter 04 #98Topic: Elasticity of supplyType: Graph 

Page 30: Microeconomics Study Guide

9. If the supply of product X is perfectly elastic, an increase in the demand for it will increase: A. equilibrium quantity but reduce equilibrium price.B. equilibrium quantity but equilibrium price will be unchanged.C. equilibrium price but reduce equilibrium quantity.D. equilibrium price but equilibrium quantity will be unchanged.

 

AACSB: Reflective ThinkingBlooms: Level 4 AnalyzeDifficulty: 3 HardLearning Objective: 04-03 Describe price elasticity of supply and how it can be applied.McConnell - Chapter 04 #103Topic: Elasticity of supply 

10. Supply curves tend to be: A. perfectly elastic in the long run because consumer demand will have sufficient time to adjust fully to changes in supply.B. more elastic in the long run because there is time for firms to enter or leave the industry.C. perfectly inelastic in the long run because the law of scarcity imposes absolute limits on production.D. less elastic in the long run because there is time for firms to enter or leave an industry.

 

AACSB: Reflective ThinkingBlooms: Level 2 UnderstandDifficulty: 2 MediumLearning Objective: 04-03 Describe price elasticity of supply and how it can be applied.McConnell - Chapter 04 #105Topic: Elasticity of supply 

11. Suppose the income elasticity of demand for toys is +2.00. This means that: A. a 10 percent increase in income will increase the purchase of toys by 20 percent.B. a 10 percent increase in income will increase the purchase of toys by 2 percent.C. a 10 percent increase in income will decrease the purchase of toys by 2 percent.D. toys are an inferior good.

 

AACSB: AnalyticBlooms: Level 3 ApplyDifficulty: 2 MediumLearning Objective: 04-04 Apply cross elasticity of demand and income elasticity of demand.McConnell - Chapter 04 #127Topic: Cross and income elasticity 

Page 31: Microeconomics Study Guide

12. We would expect the cross elasticity of demand between Pepsi and Coke to be: A. positive, indicating normal goods.B. positive, indicating inferior goods.C. positive, indicating substitute goods.D. negative, indicating substitute goods.

 

AACSB: Reflective ThinkingBlooms: Level 3 ApplyDifficulty: 2 MediumLearning Objective: 04-04 Apply cross elasticity of demand and income elasticity of demand.McConnell - Chapter 04 #132Topic: Cross and income elasticity 

13. The first Pepsi yields Craig 18 units of utility and the second yields him an additional 12 units of utility. His total utility from three Pepsis is 38 units of utility. The marginal utility of the third Pepsi is: A. 26 units of utility.B. 6 units of utility.C. 8 units of utility.D. 38 units of utility.

 

AACSB: AnalyticBlooms: Level 3 ApplyDifficulty: 2 MediumLearning Objective: 06-01 Define and explain the relationship between total utility; marginal utility; and the law of diminishing marginal utility.McConnell - Chapter 06 #13Topic: Law of diminishing marginal utility 

14. If total utility is increasing, marginal utility: A. is positive, but may be either increasing or decreasing.B. must also be increasing.C. may be either positive or negative.D. will be increasing at an increasing rate.

 

AACSB: Reflective ThinkingBlooms: Level 2 UnderstandDifficulty: 2 MediumLearning Objective: 06-01 Define and explain the relationship between total utility; marginal utility; and the law of diminishing marginal utility.McConnell - Chapter 06 #23Topic: Law of diminishing marginal utility 

Page 32: Microeconomics Study Guide

15. Where total utility is at a maximum, marginal utility is: A. negative.B. positive and increasing.C. zero.D. positive but decreasing.

 

AACSB: AnalyticBlooms: Level 2 UnderstandDifficulty: 2 MediumLearning Objective: 06-01 Define and explain the relationship between total utility; marginal utility; and the law of diminishing marginal utility.McConnell - Chapter 06 #24Topic: Law of diminishing marginal utility 

16. Suppose you have a limited money income and you are purchasing products A and B whose prices happen to be the same. To maximize your utility you should purchase A and B in such amounts that: A. their marginal utilities are the same.B. their total utilities are the same.C. their marginal and total utilities are proportionate.D. the income and substitution effects associated with each are equal.

 

AACSB: Reflective ThinkingBlooms: Level 2 UnderstandDifficulty: 2 MediumLearning Objective: 06-02 Describe how rational consumers maximize utility by comparing the marginal utility-to-price ratios of all the products they could possibly purchase.McConnell - Chapter 06 #37Topic: Theory of consumer behavior 

17. Suppose that Ms. Thomson is currently exhausting her money income by purchasing 10 units of A and 8 units of B at prices of $2 and $4 respectively. The marginal utility of the last units of A and B are 16 and 24 respectively. These data suggest that Ms. Thomson: A. has preferences that are at odds with the principle of diminishing marginal utility.B. considers A and B to be complementary goods.C. should buy less A and more B.D. should buy less B and more A.

 

AACSB: AnalyticBlooms: Level 4 AnalyzeDifficulty: 3 HardLearning Objective: 06-02 Describe how rational consumers maximize utility by comparing the marginal utility-to-price ratios of all the products they could possibly purchase.McConnell - Chapter 06 #39Topic: Theory of consumer behavior 

Page 33: Microeconomics Study Guide

 Answer the question on the basis of the following marginal utility data for products X and Y. Assume that the prices of X and Y are $4 and $2 respectively and that the consumer's income is $18.

   

 

McConnell - Chapter 06 

18. Refer to the above data. What quantities of X and Y should be purchased to maximize utility? A. 2 of X and 1 of YB. 4 of X and 5 of YC. 2 of X and 5 of YD. 2 of X and 6 of Y

 

AACSB: AnalyticBlooms: Level 5 EvaluateDifficulty: 3 HardLearning Objective: 06-02 Describe how rational consumers maximize utility by comparing the marginal utility-to-price ratios of all the products they could possibly purchase.McConnell - Chapter 06 #54Topic: Utility maximization and the demand curveType: Table 

19. Refer to the above data. What level of total utility will the utility-maximizing consumer realize? A. 96 utilsB. 108 utilsC. 72 utilsD. 142 utils

 

AACSB: AnalyticBlooms: Level 5 EvaluateDifficulty: 3 HardLearning Objective: 06-02 Describe how rational consumers maximize utility by comparing the marginal utility-to-price ratios of all the products they could possibly purchase.McConnell - Chapter 06 #55Topic: Utility maximization and the demand curveType: Table 

Page 34: Microeconomics Study Guide

20. Refer to the above data. If the price of X decreases to $2, then the utility-maximizing combination of the two products is: A. 2 of X and 5 of Y.B. 4 of X and 6 of Y.C. 6 of X and 3 of Y.D. 4 of X and 5 of Y.

 

AACSB: AnalyticBlooms: Level 5 EvaluateDifficulty: 3 HardLearning Objective: 06-03 Explain how a demand curve can be derived by observing the outcomes of price changes in the utility-maximization model.McConnell - Chapter 06 #56Topic: Utility maximization and the demand curveType: Table 

21. To the economist, total cost includes: A. explicit and implicit costs, including a normal profit.B. neither implicit nor explicit costs.C. implicit, but not explicit, costs.D. explicit, but not implicit, costs.

 

AACSB: AnalyticBlooms: Level 1 RememberDifficulty: 1 EasyLearning Objective: 07-01 Explain why economic costs include both explicit (revealed and expressed) costs and implicit (present but not obvious) costs.McConnell - Chapter 07 #6Topic: Economic costs 

22. If you owned a small farm, which of the following would most likely be a fixed cost? A. harvest laborB. hail insuranceC. fertilizerD. seed

 

AACSB: Reflective ThinkingBlooms: Level 3 ApplyDifficulty: 2 MediumLearning Objective: 07-03 Describe the distinctions between fixed and variable costs and among total; average; and marginal costs.McConnell - Chapter 07 #61Topic: Short-run production costs 

Page 35: Microeconomics Study Guide

23. Marginal cost is the: A. rate of change in total fixed cost that results from producing one more unit of output.B. change in total cost that results from producing one more unit of output.C. change in average variable cost that results from producing one more unit of output.D. change in average total cost that results from producing one more unit of output.

 

AACSB: AnalyticBlooms: Level 1 RememberDifficulty: 1 EasyLearning Objective: 07-03 Describe the distinctions between fixed and variable costs and among total; average; and marginal costs.McConnell - Chapter 07 #64Topic: Short-run production costs 

24. Other things equal, if the fixed costs of a firm were to increase by $100,000 per year, which of the following would happen? A. Marginal costs and average variable costs would both rise.B. Average fixed costs and average variable costs would rise.C. Average fixed costs and average total costs would rise.D. Average fixed costs would rise, but marginal costs would fall.

 

AACSB: Reflective ThinkingBlooms: Level 3 ApplyDifficulty: 2 MediumLearning Objective: 07-03 Describe the distinctions between fixed and variable costs and among total; average; and marginal costs.McConnell - Chapter 07 #78Topic: Short-run production costs 

25. As output increases, total variable cost: A. increases more rapidly than does total cost.B. increases continuously at a decreasing rate.C. increases at a decreasing rate and then at an increasing rate.D. increases at a constant rate.

 

AACSB: Reflective ThinkingBlooms: Level 2 UnderstandDifficulty: 2 MediumLearning Objective: 07-03 Describe the distinctions between fixed and variable costs and among total; average; and marginal costs.McConnell - Chapter 07 #109Topic: Short-run production costs 

Page 36: Microeconomics Study Guide

 Answer the question on the basis of the accompanying table that shows average total costs (ATC) for a manufacturing firm whose total fixed costs are $10:

   

 

McConnell - Chapter 07 

26. Refer to the above data. The total cost of producing 4 units of output is: A. $31.B. $87.C. $127.D. $134.

 

AACSB: AnalyticBlooms: Level 3 ApplyDifficulty: 2 MediumLearning Objective: 07-03 Describe the distinctions between fixed and variable costs and among total; average; and marginal costs.McConnell - Chapter 07 #134Topic: Short-run production costsType: Table 

27. Refer to the above data. The average variable cost of 4 units of output is: A. $33.50.B. $28.50.C. $19.00.D. $21.00.

 

AACSB: AnalyticBlooms: Level 3 ApplyDifficulty: 2 MediumLearning Objective: 07-03 Describe the distinctions between fixed and variable costs and among total; average; and marginal costs.McConnell - Chapter 07 #135Topic: Short-run production costsType: Table 

Page 37: Microeconomics Study Guide

28. Refer to the above data. The marginal cost of the fourth unit of output is: A. $2.B. $12.C. $37.D. $16.

 

AACSB: AnalyticBlooms: Level 3 ApplyDifficulty: 2 MediumLearning Objective: 07-03 Describe the distinctions between fixed and variable costs and among total; average; and marginal costs.McConnell - Chapter 07 #136Topic: Short-run production costsType: Table 

29. In the long run: A. all costs are variable costs.B. all costs are fixed costs.C. variable costs equal fixed costs.D. fixed costs are greater than variable costs.

 

AACSB: AnalyticBlooms: Level 1 RememberDifficulty: 1 EasyLearning Objective: 07-04 Use economies of scale to link a firms size and its average costs in the long run.McConnell - Chapter 07 #144Topic: Long-run production costs 

    

 

McConnell - Chapter 07 

Page 38: Microeconomics Study Guide

30. The above diagram shows the short-run average total cost curves for five different plant sizes of a firm. If in the long run the firm should produce output 0x, it should do it with a plant of size: A. #4.B. #3.C. #2.D. #1.

 

AACSB: Reflective ThinkingBlooms: Level 4 AnalyzeDifficulty: 3 HardLearning Objective: 07-04 Use economies of scale to link a firms size and its average costs in the long run.McConnell - Chapter 07 #149Topic: Long-run production costsType: Graph 

31. If an industry's long-run average total cost curve has an extended range of constant returns to scale, this implies that: A. technology precludes both economies and diseconomies of scale.B. the industry will be a natural monopoly.C. both relatively small and relatively large firms can be viable in the industry.D. the industry will be comprised of a very large number of small firms.

 

AACSB: Reflective ThinkingBlooms: Level 2 UnderstandDifficulty: 2 MediumLearning Objective: 07-04 Use economies of scale to link a firms size and its average costs in the long run.McConnell - Chapter 07 #163Topic: Long-run production costs 

32. Diseconomies of scale arise primarily because: A. the short-run average total cost curve rises when marginal product is increasing.B. of the difficulties involved in managing and coordinating a large business enterprise.C. firms must be large both absolutely and relative to the market to employ the most efficient productive techniques available.D. beyond some point marginal product declines as additional units of a variable resource (labor) are added to a fixed resource (capital).

 

AACSB: AnalyticBlooms: Level 2 UnderstandDifficulty: 2 MediumLearning Objective: 07-04 Use economies of scale to link a firms size and its average costs in the long run.McConnell - Chapter 07 #165Topic: Long-run production costs 

Page 39: Microeconomics Study Guide

    

 

McConnell - Chapter 07 

33. Refer to the above diagram. Economies of scale: A. are evident over the entire range of output.B. occur over the 0Q1 range of output.C. begin at output Q3.D. occur only over the Q1Q3 range of output.

 

AACSB: Reflective ThinkingBlooms: Level 3 ApplyDifficulty: 2 MediumLearning Objective: 07-04 Use economies of scale to link a firms size and its average costs in the long run.McConnell - Chapter 07 #167Topic: Long-run production costsType: Graph 

34. Refer to the above diagram. Minimum efficient scale: A. occurs at some output greater than Q3.B. is achieved at Q1.C. is achieved at Q3.D. cannot be identified in this diagram.

 

AACSB: Reflective ThinkingBlooms: Level 3 ApplyDifficulty: 2 MediumLearning Objective: 07-04 Use economies of scale to link a firms size and its average costs in the long run.McConnell - Chapter 07 #169Topic: Long-run production costsType: Graph 

Page 40: Microeconomics Study Guide

35. The long-run average total cost curve: A. displays declining unit costs so long as output is increasing.B. indicates the lowest unit costs achievable when a firm has had sufficient time to alter plant size.C. has a shape which is the inverse of the law of diminishing returns.D. can be derived by summing horizontally the average total cost curves of all firms in an industry.

 

AACSB: Reflective ThinkingBlooms: Level 2 UnderstandDifficulty: 2 MediumLearning Objective: 07-04 Use economies of scale to link a firms size and its average costs in the long run.McConnell - Chapter 07 #171Topic: Long-run production costs 

    

 

McConnell - Chapter 07 

 Answer the question on the basis of the following cost data:

   

 

McConnell - Chapter 07 

36. Refer to the above data. The total variable cost of producing 5 units is: A. $61.B. $48.C. $37.D. $24.

 

AACSB: AnalyticBlooms: Level 3 ApplyDifficulty: 2 MediumLearning Objective: 07-03 Describe the distinctions between fixed and variable costs and among total; average; and marginal costs.McConnell - Chapter 07 #80Topic: Short-run production costsType: Table 

Page 41: Microeconomics Study Guide

37. Refer to the above data. The marginal cost of producing the sixth unit of output is: A. $24.B. $12.C. $16.D. $8.

 

AACSB: AnalyticBlooms: Level 3 ApplyDifficulty: 2 MediumLearning Objective: 07-03 Describe the distinctions between fixed and variable costs and among total; average; and marginal costs.McConnell - Chapter 07 #83Topic: Short-run production costsType: Table 

38. The total output of a firm will be at a maximum where: A. MP is at a maximum.B. AP is at a minimum.C. MP is zero.D. AP is at a maximum.

 

AACSB: Reflective ThinkingBlooms: Level 2 UnderstandDifficulty: 2 MediumLearning Objective: 07-02 Relate the law of diminishing returns to a firms short-run production costs.McConnell - Chapter 07 #49Topic: Short-run production relationships 

 Answer the question on the basis of the following output data for a firm. Assume that the amounts of all non-labor resources are fixed.

   

 

McConnell - Chapter 07 

Page 42: Microeconomics Study Guide

39. Refer to the above data. Diminishing marginal returns become evident with the addition of the: A. sixth worker.B. fourth worker.C. third worker.D. second worker.

 

AACSB: AnalyticBlooms: Level 3 ApplyDifficulty: 2 MediumLearning Objective: 07-02 Relate the law of diminishing returns to a firms short-run production costs.McConnell - Chapter 07 #36Topic: Short-run production relationshipsType: Table 

40. Suppose that a business incurred implicit costs of $200,000 and explicit costs of $1 million in a specific year. If the firm sold 4,000 units of its output at $300 per unit, its accounting profits were: A. $100,000 and its economic profits were zero.B. $200,000 and its economic profits were zero.C. $100,000 and its economic profits were $100,000.D. zero and its economic loss was $200,000.

 

AACSB: AnalyticBlooms: Level 3 ApplyDifficulty: 2 MediumLearning Objective: 07-01 Explain why economic costs include both explicit (revealed and expressed) costs and implicit (present but not obvious) costs.McConnell - Chapter 07 #14Topic: Profits

Page 43: Microeconomics Study Guide

1. The demand for a product is inelastic with respect to price if:

A. consumers are largely unresponsive to a per unit price change.B. the elasticity coefficient is greater than 1.C. a drop in price is accompanied by a decrease in the quantity demanded.D. a drop in price is accompanied by an increase in the quantity demanded.

2. If a firm can sell 3,000 units of product A at $10 per unit and 5,000 at $8, then:

A. the price elasticity of demand is 0.44.B. A is a complementary good.C. the price elasticity of demand is 2.25.D. A is an inferior good.

3. In which of the following instances will total revenue decline?

A. price rises and supply is elasticB. price falls and demand is elasticC. price rises and demand is inelasticD. price rises and demand is elastic

4. Gigantic State University raises tuition for the purpose of increasing its revenue so that more faculty can be hired. GSU is assuming that the demand for education at GSU is:

A. decreasing.B. relatively elastic.C. perfectly elastic.D. relatively inelastic.

5. For a linear demand curve:

A. elasticity is constant along the curve.B. elasticity is unity at every point on the curve.C. demand is elastic at low prices.D. demand is elastic at high prices.

6. If a demand for a product is elastic, the value of the price elasticity coefficient is:

A. zero.B. greater than one.C. equal to one.D. less than one.

7. Price elasticity of demand is generally:

A. greater in the long run than in the short run.B. greater in the short run than in the long run.C. the same in both the short run and the long run.D. greater for "necessities" than it is for "luxuries."

Page 44: Microeconomics Study Guide

8. Suppose that a 20 percent increase in the price of normal good Y causes a 10 percent decline in the quantity demanded of normal good X. The coefficient of cross elasticity of demand is:

A. negative and therefore these goods are substitutes.B. negative and therefore these goods are complements.C. positive and therefore these goods are substitutes.D. positive and therefore these goods are complements.

9. Consumer surplus:

A. is the difference between the maximum prices consumers are willing to pay for a product and the lower equilibrium price.B. the difference between the maximum prices consumers are willing to pay for a product and the minimum prices producers are willing to accept.C. the difference between the minimum prices producers are willing to accept for a product and the higher equilibrium price.D. rises as equilibrium price rises.

10. We would expect the cross elasticity of demand between dress shirts and ties to be:

A. positive, indicating normal goods.B. positive, indicating inferior goods.C. negative, indicating substitute goods.D. negative, indicating complementary goods.

11. We would expect the cross elasticity of demand between Pepsi and Coke to be:

A. positive, indicating normal goods.B. positive, indicating inferior goods.C. positive, indicating substitute goods.D. negative, indicating substitute goods.

12. 45. If a 1% increase in fees for legal services leads to a 5% increase in total revenue for lawyers, the price elasticity of demand for legal services must be: 

A. elastic. B. of unitary elasticity.C. inelastic.D. perfectly inelastic.

13. The “incidence of a tax” is the term used to indicate:

A. Responsibility for collecting the tax.B. Who actually bears the economic tax burden.C. Who the tax is initially levied on.D. The regressive rate structure of the tax.

Page 45: Microeconomics Study Guide

14. Approximately 40 luxury boats (price $100,000 or more) are produced each year.  How much revenue could the government expect to raise from a $1,000 excise tax on luxury boats?

A. Exactly equaled to $40,000.B. Less than $40,000.C. More than $40,000.D. Approximately $400,000.

15. Sally recently got a raise from $500 per week to $550 per week.  As a result, she now purchases six steaks per week rather than five.  This indicates that:

A. Steak is an inferior good for Sally.B. Steak is a normal good for Sally.C. Sally has an inelastic demand for steak.D. Sally has an elastic demand for steak.

Lesson 7 - The Theory of Consumer Behavior 1. Utility:

A. is synonymous with usefulness.B. is want-satisfying power.C. is easy to quantify.D. rarely varies from person to person.

2. The ability of a good or service to satisfy wants is called:

A. utility maximization.B. opportunity cost.C. revenue potential.D. utility.

3. The first Pepsi yields Craig 18 units of utility and the second yields him an additional 12 units of utility. His total utility from three Pepsis is 38 units of utility. The marginal utility of the third Pepsi is:

A. 26 units of utility.B. 6 units of utility.C. 8 units of utility.D. 38 units of utility.

4. Total utility may be determined by:

A. multiplying the marginal utility of the last unit consumed by the number of units consumed.B. summing the marginal utilities of each unit consumed.C. multiplying the marginal utility of the last unit consumed by product price.D. multiplying the marginal utility of the first unit consumed by the number of units consumed.

Page 46: Microeconomics Study Guide

5. To maximize utility a consumer should allocate money income so that the:

A. elasticity of demand on all products purchased is the same.B. marginal utility divided by its market price is the same for all products.C. total utility derived from each product consumed is the same.D. marginal utility of the last unit of each product consumed is the same.

6. Suppose that MUx/Px exceeds MUy/Py. To maximize utility the consumer who is spending all her money income should buy:

A. less of X only if its price rises.B. more of Y only if its price rises.C. more of Y and less of X.D. more of X and less of Y.

7. Suppose you have a limited money income and you are purchasing products A and B whose prices happen to be the same. To maximize your utility you should purchase A and B in such amounts that:

A. their marginal utilities divided by its market price are the same.B. their total utilities are the same.C. their marginal and total utilities are proportionate.D. the income and substitution effects associated with each are equal.

8. A consumer is maximizing her utility with a particular money income when: A. the total utility derived from each product consumed is the same.B. MUa/Pa = MUb/Pb = MUc/Pc = ... = MUn/Pn.C. MUa = MUb = MUc = ... = MUn.D. Pa = Pb = Pc = ... = Pn.

9. The theory of consumer behavior assumes that consumers attempt to maximize:

A. the difference between total and marginal utility.B. total utility.C. average utility.D. marginal utility.

10. When a consumer shifts purchases from product X to product Y the marginal utility of: A. X falls and the marginal utility of Y rises.B. X rises and the marginal utility of Y falls.C. both X and Y rises.D. both X and Y falls.

11. Which of the following has been a significant factor in DVDs replacing video cassettes (VCs) in the retail home video market?

A. DVDs are now less than one-half the price of VCs.B. A scarcity of production capacity has curtailed the manufacture of VCs.C. Most consumers perceive DVD sound and video reproduction to be of higher quality.D. The price of DVD players has increased dramatically.

Page 47: Microeconomics Study Guide

12. The diamond-water paradox arises because: A. essential goods may be cheap while nonessential goods may be expensive.B. the marginal utility of certain products increases, rather than diminishes.C. essential goods are always higher priced than nonessential goods.D. we sometimes fail to use money as a standard of value.

13. All of the following would reduce property crime by increasing its "price," except:

A. imposing greater penalties for those who are caught and convicted.B. using more sophisticated security systems.C. enhancing the legitimate earnings of potential criminals.D. cutting out the middlemen ("fences") by selling stolen goods via Internet auction sites.

14. High-wage consumers are over-represented among airline and taxi passengers because:A. They pay lower money prices for these services than do low-wage consumers.B. These commodities have income elasticities less than zero.C. The opportunity cost of spending more time on time-consuming transportation is higher for high-wage consumers.D. The price elasticity of demand for airline and taxi services is quite high.

15. If you receive a gift whose market price is $20, but you consider it to be worth only $10, then:

A. there is a $10 or 50 percent value gain.B. there may or may not be a value loss.C. there is a $10 or 50 percent value loss.D. you can be relatively certain the giver was a sibling or other close relative.

Lesson 8 - Costs and Supply of Goods1. Which of the following constitutes an implicit cost to the Johnston Manufacturing Company?

A. payments of wages to its office workersB. rent paid for the use of equipment owned by the Schultz Machinery CompanyC. depreciation charges on company-owned equipmentD. economic profits resulting from current production

2. Implicit costs are:

A. regarded as costs by accountants but not by economists.B. payments that a firm makes to other firms or individuals who supply resources to it.C. nonexpenditure costs.D. costs that vary proportionately with output.

3. Accounting profits are typically:

A. greater than economic profits because the former do not take explicit costs into account.B. equal to economic profits because accounting costs include all opportunity costs.C. smaller than economic profits because the former do not take implicit costs into account.D. greater than economic profits because the former do not take implicit costs into account.

Page 48: Microeconomics Study Guide

4. Normal profit is:

A. determined by subtracting implicit costs from total revenue.B. determined by subtracting explicit costs from total revenue.C. the return to the entrepreneur when economic profits are zero.D. the average profitability of an industry over the preceding 10 years.

5. Which of the following is a short-run adjustment?

A. A local bakery hires two additional bakers.B. Six new firms enter the plastics industry.C. The number of farms in the United States declines by 5 percent.D. BMW constructs a new assembly plant in South Carolina.

6. Average fixed cost:

A. equals marginal cost when average total cost is at its minimum.B. may be found for any output by adding average variable cost and average total cost.C. graphs as a U-shaped curve.D. declines continually as output increases.

7. Marginal cost:

A. equals both average variable cost and average total cost at their respective minimums.B. is the difference between total cost and total variable cost.C. rises for a time, but then begins to decline when diminishing returns set in.D. declines continuously as output increases.

8. If a technological advance increases a firm's labor productivity, we would expect its:

A. average total cost curve to rise.B. average total cost curve to fall.C. total cost curve to rise.D. average total cost curve to be unaffected.

9. Economies and diseconomies of scale explain:

A. the profit-maximizing level of production.B. why the firm's long-run average total cost curve is U-shaped.C. why the firm's short-run marginal cost curve cuts the short-run average variable cost curve at its minimum point.D. the distinction between fixed and variable costs.

10. In the long run:

A. all costs are variable costs.B. all costs are fixed costs.C. variable costs equal fixed costs.D. fixed costs are greater than variable costs.

Page 49: Microeconomics Study Guide

11. (Last Word) A cost that cannot be partly or fully recovered through any subsequent action is known as a:

A. variable cost.B. fixed cost.C. marginal cost.D. sunk cost.

12. Other things equal, if the prices of a firm's variable inputs were to fall:

A. one could not predict how unit costs of production would be affected.B. marginal cost, average variable cost, and average fixed cost would all fall.C. marginal cost, average variable cost, and average total cost would all fall.D. average variable cost would fall, but marginal cost would be unchanged.

13.  In the diagram below, curves 1, 2, and 3 represent:

A. average variable cost, marginal cost, and average fixed cost respectively.B. total variable cost, total fixed cost, and total cost respectively.C. total fixed cost, total variable cost, and total cost respectively.D. marginal product, average variable cost, and average total cost respectively.

Page 50: Microeconomics Study Guide

14. In the figure below, curves 1, 2, 3, and 4 represent the:

A. ATC, MC, AFC, and AVC curves respectively.B. AFC, MC, AVC, and ATC curves respectively.C. MC, ATC, AVC, and AFC curves respectively.D. ATC, AVC, AFC, and MC curves respectively.

Page 51: Microeconomics Study Guide

15. Refer to the diagram below. The vertical distance between ATC and AVC reflects:

A. the law of diminishing returns.B. the average fixed cost at each level of output.C. marginal cost at each level of output.D. the presence of economies of scale.

 

 

Lesson 9 - Competitive Markets1. Which of the following industries most closely approximates pure competition?

A. agricultureB. farm implementsC. clothingD. steel

2. Which of the following is not a basic characteristic of pure competition?

A. considerable nonprice competitionB. no barriers to the entry or exodus of firmsC. a standardized or homogeneous productD. a large number of buyers and sellers

3. If a firm in a purely competitive industry is confronted with an equilibrium price of $5, its

Page 52: Microeconomics Study Guide

marginal revenue:

A. may be either greater or less than $5.B. will also be $5.C. will be less than $5.D. will be greater than $5.

4. Firms seek to maximize:

A. per unit profit.B. total revenue.C. total profit.D. market share.

5. A competitive firm will maximize profits at that output at which:

A. total revenue exceeds total cost by the greatest amount.B. total revenue and total cost are equal.C. price exceeds average total cost by the largest amount.D. the difference between marginal revenue and price is at a maximum.

6. A purely competitive firm's short-run supply curve is:

A. perfectly elastic at the minimum average total cost.B. upsloping and equal to the portion of the marginal cost curve that lies above the average variable cost curve.C. upsloping and equal to the portion of the marginal cost curve that lies above the average total cost curve.D. upsloping only when the industry has constant costs.

7. Suppose you find that the price of your product is less than minimum AVC. You should:

A. minimize your losses by producing where P = MC.B. maximize your profits by producing where P = MC.C. close down because, by producing, your losses will exceed your total fixed costs.D. close down because total revenue exceeds total variable cost.

8. If a purely competitive firm shuts down in the short run:

A. its loss will be zero.B. it will realize a loss equal to its total variable costs.C. it will realize a loss equal to its total fixed costs.D. it will realize a loss equal to its total costs.

9. In the short run a purely competitive firm will always make an economic profit if:

A. P = ATC.B. P > AVC.C. P = MC.D. P > ATC.

Page 53: Microeconomics Study Guide

10. In a purely competitive industry:

A. there will be no economic profits in either the short run or the long run.B. economic profits may persist in the long run if consumer demand is strong and stable.C. there may be economic profits in the short run, but not in the long run.D. there may be economic profits in the long run, but not in the short run.

11. Long-run competitive equilibrium:

A. is realized only in constant-cost industries.B. will never change once it is realized.C. is not economically efficient.D. results in zero economic profits.

12. A constant-cost industry is one in which:

A. resource prices fall as output is increased.B. resource prices rise as output is increased.C. resource prices remain unchanged as output is increased.D. small and large levels of output entail the same total costs.

13. A purely competitive firm is precluded from making economic profit in the long run because:

A. it is a "price taker."B. its demand curve is perfectly elastic.C. of unimpeded entry to the industry.D. it produces a differentiated product.

14. A decreasing-cost industry is one in which:

A. contraction of the industry will decrease unit costs.B. input prices fall or technology improves as the industry expands.C. the long-run supply curve is perfectly elastic.D. the long-run supply curve is upsloping.

15. Allocative efficiency is achieved when the production of a good occurs where:

A. P = minimum ATC.B. P = MC.C. P = minimum AVC.D. total revenue is equal to TFC.

Page 54: Microeconomics Study Guide

Lesson 10 - Monopolies1. Which of the following is correct?

A. Both purely competitive and monopolistic firms are "price takers."B. Both purely competitive and monopolistic firms are "price makers."C. A purely competitive firm is a "price taker," while a monopolist is a "price maker."D. A purely competitive firm is a "price maker," while a monopolist is a "price taker."

2. Pure monopolists may obtain economic profits in the long run because:

A. of advertising.B. marginal revenue is constant as sales increase.C. of barriers to entry.D. of rising average fixed costs.

3. Which of the following is not a barrier to entry?

A. patentsB. X-inefficiencyC. economies of scaleD. ownership of essential resources

4. A natural monopoly occurs when:

A. long-run average costs decline continuously through the range of demand.B. a firm owns or controls some resource essential to production.C. long-run average costs rise continuously as output is increased.D. economies of scale are obtained at relatively low levels of output.

5. The pure monopolist's demand curve is:

A. identical with the industry demand curve.B. of unit elasticity throughout.C. perfectly inelastic.D. perfectly elastic.

6. Price exceeds marginal revenue for the pure monopolist because the:

A. law of diminishing returns is inapplicable.B. demand curve is downsloping.C. monopolist produces a smaller output than would a purely competitive firm.D. demand curve lies below the marginal revenue curve.

Page 55: Microeconomics Study Guide

7. An unregulated pure monopolist will maximize profits by producing that output at which:

A. P = MC.B. P = ATC.C. MR = MC.D. MC = AC.

8. The supply curve for a monopolist is:

A. perfectly elastic.B. upsloping.C. that portion of the marginal cost curve lying above minimum average variable cost.D. nonexistent.

9. Economic profit in the long run is:

A. possible for both a pure monopoly and a pure competitor.B. possible for a pure monopoly, but not for a pure competitor.C. impossible for both a pure monopolist and a pure competitor.D. only possible when barriers to entry are nonexistent.

10. X-inefficiency refers to a situation in which a firm:

A. is not as technologically progressive as it might be.B. encounters diseconomies of scale.C. fails to realize all existing economies of scale.D. fails to achieve the minimum average total costs attainable at each level of output.

11. Price discrimination refers to:

A. selling a given product for different prices at two different points in time.B. any price above that which is equal to a minimum average total cost.C. the selling of a given product at different prices that do not reflect cost differences.D. the difference between the prices a purely competitive seller and a purely monopolistic seller would charge.

12. Which of the following conditions is not required for price discrimination?

A. Buyer with different elasticities must be physically separate from each other.B. The good or service cannot be resold by original buyers.C. The seller must be able to segment the market, that is, to distinguish buyers with different elasticities of demand.D. The seller must possess some degree of monopoly power.

13. Which government price control lets a regulated monopoly set two prices:  A fixed charge and a rate charge?

A. Ramsey PricingB. Marginal Cost PricingC. Average Cost Pricing

Page 56: Microeconomics Study Guide

D. X-inefficiency

14. What is it called when a monopoly uses its wealth and power to influence the government in its favor?

A. Ramsey PricingB. Rent seeking behaviorC. Average Cost PricingD. X-inefficiency

15. If a regulatory commission wants to provide a natural monopoly with a fair return, it should establish a price that is equal to:

A. minimum average fixed cost.B. average total cost.C. marginal cost.D. marginal revenue.

Lesson 11 - Oligopolies and Monopolistic Competition1. Monopolistic competition means:

A. a market situation where competition is based entirely on product differentiation and advertising.B. a large number of firms producing a standardized or homogeneous product.C. many firms producing differentiated products.D. a few firms producing a standardized or homogeneous product.

2. Nonprice competition refers to:

A. competition between products of different industries, for example, competition between aluminum and steel in the manufacture of automobile parts.B. price increases by a firm that are ignored by its rivals.C. advertising, product promotion, and changes in the real or perceived characteristics of a product.D. reductions in production costs that are not reflected in price reductions.

3. If the number of firms in a monopolistically competitive industry increases and the degree of product differentiation diminishes:

A. the likelihood of realizing economic profits in the long run would be enhanced.B. individual firms would now be operating at outputs where their average total costs would be higher.C. the industry would more closely approximate pure competition.D. the likelihood of collusive pricing would increase.

4. Nonprice competition refers to:

A. low barriers to entry.B. product development, advertising, and product packaging.C. the differences in information which consumers have regarding various products.D. an industry or firm in long-run equilibrium.

Page 57: Microeconomics Study Guide

5. When a monopolistically competitive firm is in long-run equilibrium:

A. P = MC = ATC.B. MR = MC and minimum ATC > P.C. MR > MC and P = minimum ATC.D. MR = MC and P > minimum ATC.

6. Product variety is likely to be greater in:

A. monopolistic competition than in pure competition.B. pure competition than in monopolistic competition.C. homogenous oligopoly than in monopolistic competition.D. homogenous oligopoly than in differentiated oligopoly.

7. The term oligopoly indicates:

A. a one-firm industry.B. many producers of a differentiated product.C. a few firms producing either a differentiated or a homogeneous product.D. an industry whose four-firm concentration ratio is low.

8. The mutual interdependence that characterizes oligopoly arises because:

A. the products of various firms are homogeneous.B. the products of various firms are differentiated.C. a small number of firms produce a large proportion of industry output.D. the demand curves of firms are kinked at the prevailing price.

9. The copper, aluminum, cement, and industrial alcohol industries are examples of:

A. interproduct competition.B. homogeneous oligopoly.C. monopolistic competition.D. differentiated oligopoly.

10. Mutual interdependence means that each oligopolistic firm:

A. faces a perfectly elastic demand for its product.B. must consider the reactions of its rivals when it determines its price policy.C. produces a product identical to those of its rivals.D. produces a product similar but not identical to the products of its rivals.

11. Concentration ratios measure the:

A. geographic location of the largest corporations in each industry.B. degree to which product price exceeds marginal cost in various industries.C. percentage of total sales accounted for by the four largest firms in the industry.D. number of firms in an industry.

Page 58: Microeconomics Study Guide

12. The Herfindahl index for a pure monopolist is:

A. 100.B. 10,000.C. 100,000.D. 10.

13. Interindustry competition means that:

A. in oligopolistic industries a few large firms compete with one another in bidding down product price.B. in some markets the producers of a particular product might face competition from products produced by other industries.C. firms that sell a product at one stage of production are faced with firms that buy the product at the next stage of production.D. in most industries there are usually a number of firms producing identical products.

14. OPEC provides an example of:

A. a tacit understanding.B. noncollusive oligopoly.C. an international cartel.D. a monopolistically competitive industry.

15. Suppose the Herfindahl Indexes for industries A, B, and C are 1,200, 5,000, and 7,500 respectively. These data imply that:

A. market power is greatest in industry A.B. market power is greatest in industry B.C. market power is greatest in industry C.D. industry A is more monopolistic than industry C.

Page 59: Microeconomics Study Guide

1. The demand for airline pilots results from the demand for air travel. This fact is an example of:

A. resource substitutability. B. rising marginal resource cost. C. elasticity of resource demand. D. the derived demand for labor.

2. The demand for a resource depends primarily on:

A. the supply of that resource. B. the demand for the product or service that it helps produce. C. the price of that input. D. the elasticity of supply of substitute inputs.

3. The MRP curve for labor:

A. intersects the firm's labor demand curve from above. B. is the firm's labor demand curve. C. lies below the firm's labor demand curve. D. lies above the firm's labor demand curve.

4. Marginal product is:

A. the output of the least skilled worker. B. a worker's output multiplied by the price at which each unit can be sold. C. the amount an additional worker adds to the firm's total output. D. the amount any given worker contributes to the firm's total revenue.

5. Assume that a restaurant is hiring labor in an amount such that the MRC of the last worker is $16 and her MRP is $12. On the basis of this information we can say that:

A. profits will be increased by hiring additional workers. B. profits will be increased by hiring fewer workers. C. marginal revenue product must exceed average revenue product. D. the restaurant is maximizing profits.

6. The MRP curve is the resource demand curve for:

A. neither the purely competitive nor the imperfectly competitive seller. B. the imperfectly competitive seller, but not the purely competitive seller. C. the purely competitive seller, but not the imperfectly competitive seller. D. both the purely competitive and imperfectly competitive seller.

7. If two resources are highly substitutable for one another:

A. a decrease in the price of one will increase unit costs of production. B. an increase in the price of one will increase the demand for the other. C. an increase in the price of one will reduce the demand for the other. D. a decline in the price of one will increase the demand for the other.

8. Elasticity of resource demand is measured by the:

A. absolute change in resource quantity demanded divided by the absolute change in resource price. B. percentage change in resource quantity demanded divided by the percentage change in resource price. C. absolute change in resource price divided by the absolute change in resource quantity demanded. D. percentage change in resource price divided by the percentage change in resource quantity demanded.

9. The labor demand curve of a purely competitive seller:

A. slopes downward because the elasticity of demand is always less than unity. B. slopes downward because of diminishing marginal productivity. C. is perfectly elastic at the going wage rate. D. slopes downward because of diminishing marginal utility.

Page 60: Microeconomics Study Guide

10. Marginal resource cost is:

A. the increase in total resource cost associated with the production of one more unit of output. B. the increase in total resource cost associated with the hire of one more unit of the resource. C. total resource cost divided by the number of inputs employed. D. the change in total revenue associated with the employment of one more unit of the resource.

11. Economists refer to expenditures on training, education, and skill development designed to increase the productivity of an individual as:

A. Overhead expenditures. B. Investments in human capital. C. Non-exhaustive expenditures. D. Social capital.

12. An unexpected decrease in the demand for accountants will lead to a(n):

A. Increase in the earnings of accountants. B. Increase in the incentive of students to prepare for a career in accounting. C. Reduction in the future supply of accountants. D. Increase in the employment opportunities of accountants.

13. If businesses found that changing economic conditions made it attractive for them to hire a larger number of economics majors, we would expect:

A. Economics majors to receive a greater return on their human capital investment. B. An increase in the employment opportunities for economics majors. C. Higher wages for economics majors. D. All of the above.

14. If a construction boom leads to an increase in the price of lumber, how will the higher prices influence the wood furniture market?

A. The demand for wood furniture will decline and furniture prices will fall. B. There will be a shortage of wood furniture. C. There will be a surplus of wood furniture. D. The supply of wood furniture will decline and furniture prices will increase.

15. If the firm has a MP = 5 and a MR = $5, how much revenue will one more unit of labor contribute if this firm hires another employee?

A. The MRP = $1. B. The MRP = $5. C. The MRP = $25. D. Not enough information is given.

Answers:

1. D 2. B 3. B 4. C 5. B 6. D7. B 8. B 9. B 10. B 11. B 12. ;C13. D 14. D 15. C

Lesson 13 - The Labor Market

1. Increases in the productivity of labor result partly from:

A. the law of diminishing returns. B. improvements in technology. C. reductions in wage rates. D. increases in the quantity of labor.

2. The real wage will rise if the nominal wage:

Page 61: Microeconomics Study Guide

A. falls more rapidly than the general price level. B. increases at the same rate as labor productivity. C. increases more rapidly than the general price level. D. falls more rapidly than the general price level.

3. The economic term for a sole employer in a nonunion community is:

A. monopsonist. B. monopolist. C. bilateral competitor. D. bilateral monopolist.

4. Which of the following is most likely to be an example of monopsony?

A. the market for fast-food workers in a large summer resort town. B. the market for card dealers in Las Vegas. C. the market for major league baseball umpires. D. the market for retail sales clerks in a major city.

5. Other things equal, the monopsonistic employer will pay a:

A. lower wage rate and hire fewer workers than will a purely competitive employer. B. higher wage rate but hire fewer workers than will a purely competitive employer. C. lower wage rate but hire a larger number of workers than will a purely competitive employer. D. higher wage rate and hire a larger number of workers than will a purely competitive employer.

6. A monopsonist pays a wage rate that is:

A. less than the MRP of labor. B. equal to the firm's marginal resource (labor) cost. C. equal to the MRP of labor. D. greater than the MRP of labor.

7. In a labor market characterized by bilateral monopoly the wage rate will:

A. be logically indeterminate. B. be established at the level desired by the union. C. be established at the level desired by the employer. D. always be established at the competitive level.

8. Wage differentials can arise from:

A. both the demand-side and supply-side of labor markets. B. the demand-side of labor markets only. C. the supply-side of labor markets only. D. neither the demand-side or supply-side of labor markets.

9. Compensating differences in wages:

A. compensate workers for differences in their human capital. B. are wage differences that compensate for differences in the desirability of jobs. C. describe the tendency for the wages of all occupations to adjust to the median level. D. do not exist if jobs have different nonmonetary characteristics.

10. Data on education and earnings reveal:

A. negative age-earnings profiles for male workers. B. no relationship between the two. C. a positive relationship between the two. D. a negative relationship between the two.

11. The principal-agent problem arises in labor markets because:

Page 62: Microeconomics Study Guide

A. a firm may realize excessively large profits. B. workers may provide less-than-expected work effort. C. compensating wage differences do not pay for differences in the nonmonetary aspects of jobs. D. human capital investments vary among workers.

12. Which one of the following best exemplifies the principal-agent problem in the employer-employee relationship?

A. A worker takes 20 minute coffee-breaks although the employer allots only 15 minutes for this purpose. B. A worker is on the job 50 hours per week although only 40 hours are required for promotion. C. A worker opts for early retirement in response to the firm's incentive plan. D. A worker's productivity is independent of the wage paid.

13. Author Joe Writer receives $2 per book sold. This payment is a(n):

A. piece rate. B. royalty. C. bonus. D. example of profit-sharing.

14. A firm might choose to pay its employees a wage higher than that which would clear the market because:

A. the higher wage raises the opportunity cost of shirking. B. the higher wage may shift the labor demand curve to the left. C. the firm will have higher turnover, allowing "new blood" to invigorate older workers who have a greater tendency to shirk. D. this policy reduces the proportion of experienced to inexperienced workers, resulting in a lower overall wage bill.

15. Benefits mandated by the government are most likely paid for by:

A. The government through direct subsidies. B. The employer in the form of lower profits. C. The worker through reductions in other components of the wage package. D. Material suppliers in the forms of lower resource prices.

Answers:

1. B 2. C 3. A 4. C 5. A 6. A7. A 8. A 9. B 10. C 11. B 12. A13. B 14. A 15. C

Lesson 14 - Market Failure and Government Intervention

1. Unlike a private good, a public good:

A. has no opportunity costs. B. has benefits available to all, including nonpayers. C. produces no positive or negative externalities. D. is characterized by rivalry and excludability.

2. Which of the following is an example of a public good?

A. a weather warning system B. a television set C. a sofa D. a bottle of soda

3. The market system does not produce public goods because:

A. there is no need or demand for such goods. B. private firms cannot stop consumers who are unwilling to pay for such goods from benefiting from them. C. public enterprises can produce such goods at lower cost than can private enterprises. D. their production seriously distorts the distribution of income.

4. Because of the free-rider problem:

A. the market demand for a public good is overstated. B. the market demand for a public good is nonexistent or understated.

Page 63: Microeconomics Study Guide

C. government has increasingly yielded to the private sector in producing public goods. D. public goods often create moral hazard and adverse selection problems.

5. A demand curve for a public good is determined by:

A. summing vertically the individual demand curves for the public good. B. summing horizontally the individual demand curves for the public good. C. combining the amounts of the public good that the individual members of society demand at each price. D. multiplying the per-unit cost of the public good by the quantity made available.

6. A positive externality or spillover benefit occurs when:

A. product differentiation increases the variety of products available to consumers. B. the benefits associated with a product exceed those accruing to people who consume it. C. a firm produces at the P = MC output. D. economic profits are zero in the long run.

7. The Coase theorem:

A. applies only to circumstances in which externalities are extensive and bargaining costs are high. B. holds that the median voter will decide the outcome of elections. C. states that in some circumstances majority voting can yield inconsistent results. D. suggests that in some circumstances government intervention is not needed to resolve externality problems.

8. The tendency for society to overuse and therefore abuse common resources is called the:

A. law of conservation of matter and energy. B. law of diminishing returns. C. tragedy of the commons. D. the Coase theorem.

9. The Clean Air Act of 1990:

A. places taxes on utilities emitting sulfur dioxide, the major cause of acid rain. B. sets aside billions of dollars to promote recycling. C. allows utilities to buy and sell sulfur dioxide emission credits provided by government. D. places taxes on producers who emit toxic chemicals.

10. As it applies to insurance, the adverse selection problem is the tendency for:

A. those most likely to collect on insurance to buy it. B. those who buy insurance to take less precaution in avoiding the insured risk. C. sellers to price discriminate. D. sellers to restrict output and charge high prices.

11. As it applies to insurance, the moral hazard problem is the tendency for:

A. those most likely to collect on insurance to buy it. B. those who buy insurance to take less precaution in avoiding the insured risk. C. sellers to price discriminate. D. sellers to restrict output and charge high prices.

12. Professional buyers of antiques often have more information about the value of antique objects than do the sellers. This illustrates:

A. the Coase theorem. B. the moral hazard problem. C. the free-rider problem. D. asymmetric information.

13. Alex, Kara, and Lucian are the only three people in a community and Alex is willing to pay $20 for the 5th unit of a public good; Kara, $15, and Lucian, $25. Government should produce the 5th unit of the public good if the marginal cost is less than:

A. $25.

Page 64: Microeconomics Study Guide

B. $15. C. $60. D. $300.

14. If the marginal benefit for government to build a new highway is $5 million, but the new highway would cost $10 million, what should government do?

A. government should build the highway. B. government should not build the new highway. C. government should build the highway and raise taxes on people who will use the highway. D. not enough information is given.

15. What is global warming?

A. The earth is becoming warmer, because of the buildup of greenhouse gases in the atmosphere. B. The earth is becoming cooler, because greenhouse gases are accumulating in the atmosphere. C. The earth is becoming warmer, because the amount of greenhouse gases in the atmosphere is decreasing. D. The belief that the earth will enter another ice age.

Answers:

1. B 2. A 3. B 4. B 5. A 6. B7. D 8. C 9. C 10. A 11. B 12. D13. C 14. B 15. A

Lesson 15 - Gross Domestic Product (GDP)

1. National income accountants can avoid multiple counting by:

A. including transfer payments in their calculations. B. counting both intermediate and final goods. C. only counting final goods. D. only counting intermediate goods.

2. Gross domestic product (GDP) measures and reports output:

A. as an index number. B. in percentage terms. C. in dollar amounts. D. in quantities of physical units (for example, pounds, gallons, and bushels).

3. Tom Atoe grows tomatoes for home consumption. This activity is:

A. excluded from GDP in order to avoid double counting. B. excluded from GDP because an intermediate good is involved. C. productive but is excluded from GDP because no market transaction occurs. D. included in GDP because it reflects production.

4. Net exports are:

A. that portion of consumption and investment goods sent to other countries. B. exports plus imports. C. exports less imports. D. imports less exports.

5. GDP differs from NDP in that:

A. GDP is based on gross exports, while NDP is based on net exports. B. GDP includes, but NDP excludes, taxes on production and imports. C. net investment is used in calculating GDP and gross investment is used in calculating NDP. D. gross investment is used in calculating GDP and net investment is used in calculating NDP.

6. If depreciation exceeds gross investment:

Page 65: Microeconomics Study Guide

A. the economy's stock of capital may be either growing or shrinking. B. the economy's stock of capital is shrinking. C. the economy's stock of capital is growing. D. net investment is zero.

7. Real GDP measures:

A. current output at current prices. B. current output at base year prices. C. base year output at current prices. D. base year output at current exchange rates.

8. If nominal GDP rises:

A. real GDP may either rise or fall. B. we can be certain that the price level has risen. C. real GDP must fall. D. real GDP must also rise.

9. A large underground economy results in an:

A. understated GDP. B. overstated GDP. C. understated GDP price index. D. overstated GDP price index.

10. The growth of GDP may understate changes in the economy's economic well-being over time if the:

A. distribution of income becomes increasingly unequal. B. quality of products and services improves. C. environment deteriorates because of pollution. D. amount of leisure decreases.

11. As defined in national income accounting, investment includes:

A. business expenditures on machinery and equipment. B. all consumption. C. imports, but not exports. D. all nonfood items.

12. The concept of net domestic investment refers to:

A. the amount of machinery and equipment used up in producing the GDP in a specific year. B. the difference between the market value and book value of outstanding capital stock. C. gross domestic investment less net exports. D. total investment less the amount of investment goods used up in producing the year's output.

13. The amount of after-tax income received by households is measured by:

A. discretionary income. B. national income. C. disposable income. D. personal income.

14. What is a problem with the GDP measure?

A. GDP accounts for environmental pollution but the estimate is not accurate B. GDP assumes GDP is shared equally in the economy C. GDP includes quality improvements, but it is not accurate D. GDP does not include the hidden economy

15. Approximately how large (by economists' best guess) is the hidden economy in the United States?

Page 66: Microeconomics Study Guide

A. 10% B. 25% C. 50% D. 100%

Answers:1. C 2. C 3. C 4. C 5. D 6. B7. B 8. A 9. A 10. B 11. A 12. D13. C 14. D 15. A

Lesson 16 - Economic Growth

1. Growth is advantageous to a nation because it:

A. promotes faster population growth. B. lessens the burden of scarcity. C. eliminates the economizing problem. D. slows the growth of wants.

2. Given the annual rate of economic growth, the "rule of 70" allows one to:

A. determine the accompanying rate of inflation. B. calculate the size of the GDP gap. C. calculate the number of years required for real GDP to double. D. determine the growth rate of per capita GDP.

3. If the economy's real GDP doubles in 18 years, we can:

A. not say anything about the average annual rate of growth. B. conclude that its average annual rate of growth is about 5.5 percent. C. conclude that its average annual rate of growth is about 2 percent. D. conclude that its average annual rate of growth is about 4 percent.

4. Recurring upswings and downswings in an economy's real GDP over time are called:

A. recessions. B. business cycles. C. output yo-yos. D. total product oscillations.

5. During a severe recession, we would expect output to fall the most in:

A. the health-care industry. B. the clothing industry. C. agriculture. D. the construction industry.

6. The phase of the business cycle in which real GDP is at a minimum is called:

A. the peak. B. a recession. C. the trough. D. the pits.

7. Market economies have been characterized by:

A. occasional instability of employment and price levels. B. uninterrupted economic growth. C. persistent full employment. D. declining populations.

Page 67: Microeconomics Study Guide

8. Kara voluntarily quit her job as an insurance agent to return to school full-time to earn an MBA degree. With degree in hand she is now searching for a position in management. Kara presently is:

A. cyclically unemployed. B. structurally unemployed. C. frictionally unemployed. D. not a member of the labor force.

9. The unemployment rate of:

A. women greatly exceeds that of men. B. whites is roughly equal to that of African-Americans. C. white-collar workers exceeds that of blue-collar workers. D. teenagers is much higher than that of adults.

10. The natural rate of unemployment:

A. is fixed over time. B. is found by adding the cyclical and structural unemployment rates. C. may change from one decade to another. D. cannot be influenced by public policy.

11. Assuming the total population is 100 million, the civilian labor force is 50 million, and 47 million workers are employed, the unemployment rate is:

A. 3 percent. B. 6 percent. C. 7 percent. D. 53 percent.

12. Assume the natural rate of unemployment in the U.S. economy is 5 percent and the actual rate of unemployment is 9 percent. According to Okun's law, the negative GDP gap as a percent of potential GDP is:

A. 4 percent. B. 8 percent. C. 10 percent. D. 2 percent.

13. Cost-push inflation:

A. is caused by excessive total spending. B. shifts the nation's production possibilities curve leftward. C. moves the economy inward from its production possibilities curve. D. is a mixed blessing because it has positive effects on real output and employment.

14. Who is least likely to be hurt by unanticipated inflation?

A. a disabled laborer who is living off accumulated savings B. an owner of a small business C. a secretary D. a pensioned steelworker

15. A burst stock market bubble might adversely affect the economy by:

A. causing rapid inflation. B. greatly reducing net exports. C. causing a severe negative wealth effect and engendering pessimism about the economy's future. D. raising interest rates.

Answers:1. B 2. C 3. D 4. B 5. D 6. C7. A 8. C 9. D 10. C 11. B 12. B13. C 14. B 15. C

Page 68: Microeconomics Study Guide

Lesson 15 - Macroeconomic Relationships

1. With an MPS of .4, the MPC will be:

A. 1.0 minus .4. B. .4 minus 1.0. C. the reciprocal of the MPS. D. .4.

2. The consumption schedule shows:

A. that the MPC increases in proportion to GDP. B. that households consume more when interest rates are low. C. that consumption depends primarily on the level of business investment. D. the amounts households intend to consume at various possible levels of aggregate income.

3. A decline in disposable income:

A. increases consumption by moving upward along a specific consumption schedule. B. decreases consumption because it shifts the consumption schedule downward. C. decreases consumption by moving downward along a specific consumption schedule. D. increases consumption because it shifts the consumption schedule upward.

4. Which of the following is correct?

A. APC + APS = 1. B. APC + MPS = 1. C. APS + MPC = 1. D. APS + MPS = 1.

5. The investment demand curve portrays an inverse (negative) relationship between:

A. investment and real GDP. B. the real interest rate and investment. C. the nominal interest rate and investment. D. the price level and investment.

6. A high rate of inflation is likely to cause a:

A. high nominal interest rate. B. low nominal interest rate. C. low rate of growth of nominal GDP. D. decrease in nominal wages.

7. Investment spending in the United States tends to be unstable because:

A. expected profits are highly variable. B. capital goods are durable. C. innovation occurs at an irregular pace. D. all of these contribute to the instability.

8. The multiplier is:

A. 1/MPC. B. 1/(1 + MPC). C. 1/MPS. D. 1/(1 - MPS).

9. If the MPC is .70 and gross investment increases by $3 billion, the equilibrium GDP will:

A. increase by $10 billion. B. increase by $2.10 billion. C. decrease by $4.29 billion. D. increase by $4.29 billion.

Page 69: Microeconomics Study Guide

10. The practical significance of the multiplier is that it:

A. equates the real interest rate and the expected rate of return on investment. B. magnifies initial changes in spending into larger changes in GDP. C. keeps inflation within tolerable limits. D. helps to stabilize the economy.

11. The actual multiplier effect in the U.S. economy is less than the multiplier effect in the text examples because:

A. the real-world MPS is larger than the MPS in the examples. B. in addition to saving, households use some of any increase in income to buy imported goods and to pay additional taxes. C. the gap between the nominal interest rate and the real interest rate widens as the economy expands or contracts. D. the MPC in the United States is greater than 1.

12. The multiplier is useful in determining the:

A. full-employment unemployment rate. B. level of business inventories. C. rate of inflation. D. change in GDP resulting from a change in spending.

13. In annual percentage terms, investment spending in the United States is:

A. less variable than real GDP. B. less variable than consumption spending. C. less variable than the price level. D. more variable than real GDP.

14. When consumption and saving are graphed relative to real GDP, an increase in personal taxes will shift:

A. both the consumption and saving schedules downward. B. both the consumption and saving schedules upward. C. the consumption schedule upward and the saving schedule downward. D. the consumption schedule downward and the saving schedule upward.

15. The greater is the marginal propensity to consume, the:

A. smaller is the marginal propensity to save. B. higher is the interest rate. C. lower is the average propensity to consume. D. lower is the price level.

Answers:1. A 2. D 3. C 4. A 5. B 6.A7. D 8. C 9. A 10. B 11. B 12. D13. D 14. A 15. A

A leftward shift in the supply curve of product X will increase equilibrium price to a greater extent the:

A. more inelastic the demand for the product.

B. more elastic the supply curve.

C. larger the elasticity of demand coefficient.

D. more elastic the demand for the product.

Page 70: Microeconomics Study Guide

 

2) In a competitive market economy firms will select the least-cost production technique because:

A. "dollar voting" by consumers mandates such a choice.

B. such choices will result in the full employment of available resources.

C. to do so will maximize the firms' profits.

D. this will prevent new firms from entering the industry.

 

3) If the demand for farm products is price inelastic, a good harvest will cause farm revenues to:

A. decrease

B. increase

C. either increase or decrease, depending on what happens to supply.

D. be unchanged.

 

4) Camille's Creations and Julia's Jewels both sell beads in a competitive market. If at the market price of $5, both are running out of beads to sell (they can't keep up with the quantity demanded at that price), then we would expect both Camille's and Julia's to:

A. raise their price and increase their quantity supplied.

B. raise their price and reduce their quantity supplied.

C. lower their price and increase their quantity supplied.

D. lower their price and reduce their quantity supplied.

 

5) Since their introduction, prices of DVD players have fallen and the quantity purchased has increased. This statement:

A. suggests that the demand for DVD players has increased.

B. suggests that the supply of DVD players has increased.

C. constitutes an exception to the law of supply in that they suggest a downward sloping supply curve.

D. constitutes an exception to the law of demand in that they suggest an upward sloping demand curve.

Page 71: Microeconomics Study Guide

 

6) If price is above the equilibrium level, competition among sellers to reduce the resulting:

A. shortage will decrease quantity demanded and increase quantity supplied.

B. surplus will increase quantity demanded and decrease quantity supplied.

C. shortage will increase quantity demanded and decrease quantity supplied.

D. surplus will decrease quantity demanded and increase quantity supplied.

 

7) A firm that is motivated by self interest should:

A. hire each input so the productivity of each is equal at the margin.

B. employ the combination of resources that will produce the profit-maximizing output at the minimum cost.

C. always use large amounts of cheap inputs and small amounts of expensive inputs in producing its output.

D. always use large amounts of the most productive inputs and small amounts of the least productive inputs in producing its output.

 

8) If technology dictates that labor and capital must be used in fixed proportions, an increase in the price of capital will cause a firm to use:

A. more labor as a consequence of the output effect.

B. more labor as a consequence of the substitution effect.

C. less labor as a consequence of the output effect.

D. less labor as a consequence of the substitution effect.

 

9) If a firm is selling in an imperfectly competitive product market, then:

A. the marginal products of successive workers can be sold at a constant price.

B. A. average product will be less than marginal product for any number of workers hired.

C. the marginal products of successive workers must be sold at lower prices.

D. the marginal products of successive workers can be sold at higher prices.

Page 72: Microeconomics Study Guide

 

10) Which of the following represents a long-run adjustment?

A. a supermarket hires four additional clerks

B. a farmer uses an extra dose of fertilizer on his corn crop

C. unable to meet foreign competition, a U.S. watch manufacturer sells one of its branch plants

D. a steel manufacturer cuts back on its purchases of coke and iron ore

 

11) In the short run the Sure-Screen T-Shirt Company is producing 500 units of output. Its average variable costs are $2.00 and its average fixed costs are $.50. The firm's total costs:

A. are $1,100.

B. are $2.50.

C. are $1,250.

D. are $750.

 

12) In which of the following industries are economies of scale exhausted at relatively low levels of output?

A. newspaper printing

B. aircraft production

C. automobile manufacturing

D. concrete mixing

 

13) A firm can hire six workers at a wage rate of $8 per hour but must pay $9 per hour to all of its employees to attract a seventh worker. The marginal wage cost of the seventh worker is:

A. $21.

B. $9.

C. $10.

D. $15.

 

Page 73: Microeconomics Study Guide

14) Construction workers frequently sponsor political lobbying in support of greater public spending on highways and public buildings. One reason they do this is to:

A. increase the price of substitute inputs.

B. restrict the supply of construction workers.

C. increase the elasticity of demand for construction worke

Page 74: Microeconomics Study Guide

1. Marginal utility can be: A. positive, but not negative.B. positive or negative, but not zero.C. positive, negative, or zero.D. decreasing, but not negative. 

     2. Refer to the above data. The value for X is: A. 15.B. 5.C. 55.D. 10. 

3. Marginal utility is the: A. sensitivity of consumer purchases of a good to changes in the price of that good.B. change in total utility obtained by consuming one more unit of a good.C. change in total utility obtained by consuming another unit of a good divided by the change in the price of that good.D. total utility associated with the consumption of a certain number of units of a good divided by the number of units consumed. 

     

4. Refer to the above diagram. The marginal utility of the third unit of X is: A. 5.B. 4.C. 2.D. 15. 

5. Where total utility is at a maximum, marginal utility is: A. negative.B. positive and increasing.C. zero.D. positive but decreasing. 

Page 75: Microeconomics Study Guide

6. To maximize utility a consumer should allocate money income so that the: A. elasticity of demand on all products purchased is the same.B. marginal utility obtained from the last dollar spent on each product is the same.C. total utility derived from each product consumed is the same.D. marginal utility of the last unit of each product consumed is the same. 

7. The marginal utility of the last unit of apples consumed is 12 and the marginal utility of the last unit of bananas consumed is 8. What set of prices for apples and bananas, respectively, would be consistent with consumer equilibrium? A. $4 and $6B. $6 and $4C. $8 and $12D. $16 and $9 

8. If a rational consumer is in equilibrium, which of the following conditions will hold true? A. MUa = MUb = MUc = ... = MUn.B. The marginal utility of each good purchased will be zero.C. The marginal utility of the last dollar spent on each good purchased will be the same.D. The total utility obtained from each good purchased will be the same. 

9. A consumer's demand curve for a product is downsloping because: A. total utility falls below marginal utility as more of a product is consumed.B. marginal utility diminishes as more of a product is consumed.C. time becomes less valuable as more of a product is consumed.D. the income and substitution effects precisely offset each other. 

 Answer the question on the basis of the following marginal utility data for products X and Y. Assume that the prices of X and Y are $4 and $2 respectively and that the consumer's income is $18.

    

10. Refer to the above data. What quantities of X and Y should be purchased to maximize utility? A. 2 of X and 1 of YB. 4 of X and 5 of YC. 2 of X and 5 of YD. 2 of X and 6 of Y 

11. An explicit cost is: A. omitted when accounting profits are calculated.B. a money payment made for resources not owned by the firm itself.C. an implicit cost to the resource owner who receives that payment.D. always in excess of a resource's opportunity cost. 

Page 76: Microeconomics Study Guide

 The following is cost information for the Creamy Crisp Donut Company:Entrepreneur's potential earnings as a salaried worker = $50,000Annual lease on building = $22,000Annual revenue from operations = $380,000Payments to workers = $120,000Utilities (electricity, water, disposal) costs = $8,000Value of entrepreneur's talent in the next best entrepreneurial activity = $80,000Entrepreneur's forgone interest on personal funds used to finance the business = $6,000 

12. Refer to the above data. Creamy Crisp's accounting profit is: A. $150,000.B. $380,000.C. $230,000.D. $294,000. 

13. Refer to the above data. Creamy Crisp's economic profit is: A. $150,000.B. $80,000.C. $230,000.D. $94,000. 

14. Refer to the above data. If, other things equal, Creamy Crisp's revenue fell to $286,000: A. its implicit costs, including a normal profit, would exceed its explicit costs.B. it would earn a normal profit but not an economic profit.C. it would suffer an economic loss.D. its accounting profit would fall to zero. 

 Answer the question on the basis of the following output data for a firm. Assume that the amounts of all non-labor resources are fixed.

   

15. Refer to the above data. The marginal product of the sixth worker is: A. 180 units of output.B. 30 units of output.C. 15 units of output.D. negative. 

16. Refer to the above data. Average product is at a maximum when: A. five workers are hired.B. four workers are hired.C. three workers are hired.D. two workers are hired.  

Page 77: Microeconomics Study Guide

17. To economists, the main difference between the short run and the long run is that: A. the law of diminishing returns applies in the long run, but not in the short run.B. in the long run all resources are variable, while in the short run at least one resource is fixed.C. fixed costs are more important to decision making in the long run than they are in the short run.D. in the short run all resources are fixed, while in the long run all resources are variable. 

     

18. In the above diagram the range of diminishing marginal returns is: A. 0Q3.B. 0Q2.C. Q1Q2.D. Q1Q3. 

 Use the following data to answer the question:

   19. Refer to the above data. The average product (AP) when two units of labor are hired is: A. 8.B. 9.C. 10.D. 18. 

20. Refer to the above data. Diminishing returns begin to occur with the hiring of the _________ unit of labor. A. firstB. secondC. thirdD. seventh 

21. Refer to the above data. Marginal product becomes negative with the hiring of the __________ unit of labor. A. thirdB. fourthC. sixthD. seventh 

Page 78: Microeconomics Study Guide

22. Fixed cost is: A. the cost of producing one more unit of capital, for example, machinery.B. any cost which does not change when the firm changes its output.C. average cost multiplied by the firm's output.D. usually zero in the short run. 

23. If a firm decides to produce no output in the short run, its costs will be: A. its marginal costs.B. its variable costs.C. its fixed costs.D. zero. 

24. A purely competitive seller is: A. both a "price maker" and a "price taker."B. neither a "price maker" nor a "price taker."C. a "price taker."D. a "price maker." 

25. The demand curve in a purely competitive industry is _____, while the demand curve to a single firm in that industry is _____. A. perfectly inelastic, perfectly elasticB. downsloping, perfectly elasticC. downsloping, perfectly inelasticD. perfectly elastic, downsloping 

26. A competitive firm in the short run can determine the profit-maximizing (or loss-minimizing) output by equating: A. price and average total cost.B. price and average fixed cost.C. marginal revenue and marginal cost.D. price and marginal revenue. 

27. When a firm is maximizing profit it will necessarily be: A. maximizing profit per unit of output.B. maximizing the difference between total revenue and total cost.C. minimizing total cost.D. maximizing total revenue. 

28. Suppose you find that the price of your product is less than minimum AVC. You should: A. minimize your losses by producing where P = MC.B. maximize your profits by producing where P = MC.C. close down because, by producing, your losses will exceed your total fixed costs.D. close down because total revenue exceeds total variable cost. 

Page 79: Microeconomics Study Guide

 Answer the question on the basis of the following data confronting a firm:

    29. Refer to the above data. If the firm's minimum average variable cost is $10, the firm's profit-maximizing level of output would be: A. 2.B. 3.C. 4.D. 5. 

30. If a firm is confronted with economic losses in the short run, it will decide whether or not to produce by comparing: A. marginal revenue and marginal cost.B. price and minimum average variable cost.C. total revenue and total cost.D. total revenue and total fixed cost. 

31. A firm finds that at its MR = MC output, its TC = $1,000, TVC = $800, TFC = $200, and total revenue is $900. This firm should: A. shut down in the short run.B. produce because the resulting loss is less than its TFC.C. produce because it will realize an economic profit.D. liquidate its assets and go out of business. 

     

 32. Refer to the above diagram for a purely competitive producer. The firm will produce at a loss at all prices: A. above P1.B. above P3.C. above P4.D. between P2 and P3. 

33. The primary force encouraging the entry of new firms into a purely competitive industry is: A. normal profits earned by firms already in the industry.B. economic profits earned by firms already in the industry.C. government subsidies for start-up firms.D. a desire to provide goods for the betterment of society.  

Page 80: Microeconomics Study Guide

34. If a purely competitive firm is producing at the MR = MC output level and earning an economic profit, then: A. the selling price for this firm is above the market equilibrium price.B. new firms will enter this market.C. some existing firms in this market will leave.D. there must be price fixing by the industry's firms. 

35. Long-run competitive equilibrium: A. is realized only in constant-cost industries.B. will never change once it is realized.C. is not economically efficient.D. results in zero economic profits. 

36. A purely competitive firm: A. must earn a normal profit in the short run.B. cannot earn economic profit in the long run.C. may realize either economic profit or losses in the long run.D. cannot earn economic profit in the short run. 

37. When LCD televisions first came on the market, they sold for at least $1,000, and some for much more. Now many units can be purchased for under $400. These facts imply that: A. the LCD television industry was once competitive, but is now monopolistic.B. fewer firms produce LCD televisions than was the case five or ten years ago.C. the demand curve for LCD televisions has shifted leftward.D. the LCD television industry is a decreasing-cost industry. 

38. The MR = MC rule applies: A. in the short run, but not in the long run.B. in the long run, but not in the short run.C. in both the short run and the long run.D. only to a purely competitive firm.

      39. Refer to the above diagrams which pertain to a purely competitive firm producing output q and the industry in which it operates. In the long run we should expect: A. firms to enter the industry, market supply to rise, and product price to fall.B. firms to leave the industry, market supply to rise, and product price to fall.C. firms to leave the industry, market supply to fall, and product price to rise.D. no change in the number of firms in this industry. 

 40. Creative destruction is: A. the process by which large firms buy up small firms.B. the process by which new firms and new products replace existing dominant firms and products.C. a term coined many years ago by Adam Smith.D. is applicable to planned economies, but not to market economies.

Page 81: Microeconomics Study Guide

1. Explicit costs are payments to a. hourly employees.b. insurance companies.c. utility companies.d. all of the above.

ANS:d. Explicit costs are payments to non owners of a firm.

2. Implicit costs are the opportunity costs of using the resources of a. outsiders.b. owners.c. banks.d. retained earnings.

ANS:b. Implicit costs are opportunity costs that a business owner incurs when using resources owned by the firm.

3. Which of the following equalities is true? a. Economic profit = total revenue – accounting profit.b. Economic profit = total revenue - explicit costs - accounting profit. c. Economic profit = total revenue - implicit costs - explicit costs.d. Economic profit = opportunity costs + accounting costs.

ANS:c. The difference between accounting profit and economic profit is that economic profit is total revenue minus both explicit and implicit costs. Accounting profit is total revenue minus explicit costs only.

4. Fixed inputs are factors of production that a. are determined by a firm’s plant size. b. can be increased or decreased quickly as output changes.c. cannot be increased or decreased as output changes.d. none of the above. ANS:c. In the short run, there are two types of inputs, fixed and variable. Because a firm cannot change its plant capacity, some of its inputs are fixed. In the long run, all costs are variable.

5. An example of a variable input is a. raw materials.b. energy.c. hourly labor.d. all of the above.

ANS:d. As a firm produces more, it will use more raw materials, energy, and labor. Therefore, all are variable costs.

Page 82: Microeconomics Study Guide

6. Suppose a car wash has 2 washing stations and 5 workers and is able to wash 100 cars per day. When it adds a third station, but no more workers, it is able to wash 150 cars per day. The marginal product of the third washing station is a. 100 cars per day.b. 150 cars per day.c. 5 cars per day.d. 50 cars per day.

ANS:d. 50 cars is how many extra cars can be washed by adding a new machine, ceteris paribus.

7. If the units of variable input in a production process are 1, 2, 3, 4, and 5 and the corresponding total outputs are 10, 22, 33, 42, and 48, respectively, the marginal product of the fourth unit is a. 2.b. 6.c. 9.d. 42.

ANS:c. The difference between 42 and 33 is 9, the extra output when producing 4 units instead of 3.

8. The total fixed cost curve is a. upward sloping.b. downward sloping.c. upward sloping, and then downward sloping.d. unchanged with the level of output.

ANS:d. Fixed costs never change regardless of the units of output; therefore its curve has to be horizontal at a fixed cost dollar value.

9. Assuming that the marginal cost curve is a smooth J-shaped curve, the corresponding total cost curve has a (an)a. linear shape.b. S-shape.c. U-shape.d. reverse S-shape.

ANS:d. Marginal cost decreases as output increases from zero, and then increases beyond a certain output level. A reverse S-Shape total cost curve corresponds to the changes in its slope (MC) as output expands.

10. If both the marginal cost and the average variable cost curves are J-shaped, at the point of minimum average variable cost, the marginal cost must be a. greater than the average variable cost.b. less than the average variable cost. c. equal to the average variable cost. d. at its minimum.

Page 83: Microeconomics Study Guide

ANS:c. If the margin is above the average, the average will increase. If the margin is less than the average, the average will decrease. If the margin equals the average, average does not change, that is, it is a horizontal curve.

11. Which of the following is true at the point where diminishing returns set in? a. Both marginal product and marginal cost are at a maximum. b. Both marginal product and marginal cost are at a minimum. c. Marginal product is at a maximum, and marginal cost at a minimum. d. Marginal product is at a minimum, and marginal cost at a maximum.

ANS:c. The rising portion of the MP curve corresponds to the declining portion of the MC curve, and vice versa.

12. As shown in Exhibit 10, total fixed cost for the firm is a. zero.b. $250.c. $500.d. $750.e. $1,000.

ANS:b. $250 is the answer because total cost is 0 when output is zero. These are costs that have to be paid even when output is zero.

13. As shown in Exhibit 10, the total cost of producing 100 units of output per day isa. zero.b. $250.c. $500.d. $750.e. $1,000.

ANS:c. A vertical line drawn at 100 units crosses the total cost curve at $500.

Page 84: Microeconomics Study Guide

14. In Exhibit 10, if the total cost of producing 99 units of output per day is $475, the marginal cost of producing the 100th unit of output per day is approximatelya. zero.b. $25.c. $475.d. $500.

ANS:b. When total cost at 99 units is $475 and total cost at 100 units is $500, the cost of producing the 100th unit is $25.

15. Each potential short-run average total cost curve is tangent to the long-run average cost curve at a. the level of output that minimizes short-run average total cost.b. the minimum point of the average total cost curve.c. the minimum point of the long-run average cost curve.d. a single point on the short-run average total cost curve.

ANS:d. As shown in the next slide, the long-run average cost curve is derived from all possible SRAC curves. Geometrically, the only way to draw this is to connect all the curves by a smooth curve; thus, the LRAC curve touches each SRAC curve at only one place.

16. Suppose a typical firm is producing X units of output per day. Using any other plant size, the long-run average cost would increase. The firm is operating at a point which a. its long-run average cost curve is at a minimum.b. its short-run average total cost curve is at a minimum.c. both (a) and (b)are true. d. neither (a) nor (b) is true.

ANS:c. When a firm is producing at the minimum points of the long-run average cost curve, it is operating at the most efficient level possible.

17. The downward-sloping segment of the long-run average cost curve corresponds to a. diseconomies of scale.b. both economies and diseconomies of scale.c. the decrease in average variable cost.d. economies of scale.

ANS:d. As shown in the next slide, economies of scale take place when a firm increases its efficiency by producing more units of output.

18. Long-run diseconomies of scale exist when the a. short-run average total cost curve falls.b. long-run marginal cost curve rises.

Page 85: Microeconomics Study Guide

c. long-run average cost curve falls.d. short-run average cost curve rises.e. long-run average cost curve rises.

ANS:e. Diseconomies of scale are evident when increasing output leads to inefficiencies.

19. Long-run constant returns to scale exist when the a. short-run average total cost curve is constant.b. long-run average cost curve rises.c. long-run average cost curve is flat.d. long-run average cost curve falls.

ANS:c. Constant returns to scale are evident when there is no change in costs as output increases.

20. Which of the following is not a source of economies of scale?a. Division and specialization of laborb. Increase in outputc. More efficient use of capitald. All of the abovee. Centralized marketing

ANS:d. Centralized marketing is not one of the reasons for economies of scale.

Page 86: Microeconomics Study Guide

1. Marginal utility can be: A. positive, but not negative.B. positive or negative, but not zero.C. positive, negative, or zero.D. decreasing, but not negative. 

     2. Refer to the above data. The value for X is: A. 15.B. 5.C. 55.D. 10. 

3. Marginal utility is the: A. sensitivity of consumer purchases of a good to changes in the price of that good.B. change in total utility obtained by consuming one more unit of a good.C. change in total utility obtained by consuming another unit of a good divided by the change in the price of that good.D. total utility associated with the consumption of a certain number of units of a good divided by the number of units consumed. 

     

4. Refer to the above diagram. The marginal utility of the third unit of X is: A. 5.B. 4.C. 2.D. 15. 

5. Where total utility is at a maximum, marginal utility is: A. negative.B. positive and increasing.C. zero.D. positive but decreasing. 

Page 87: Microeconomics Study Guide

6. To maximize utility a consumer should allocate money income so that the: A. elasticity of demand on all products purchased is the same.B. marginal utility obtained from the last dollar spent on each product is the same.C. total utility derived from each product consumed is the same.D. marginal utility of the last unit of each product consumed is the same. 

7. The marginal utility of the last unit of apples consumed is 12 and the marginal utility of the last unit of bananas consumed is 8. What set of prices for apples and bananas, respectively, would be consistent with consumer equilibrium? A. $4 and $6B. $6 and $4C. $8 and $12D. $16 and $9 

8. If a rational consumer is in equilibrium, which of the following conditions will hold true? A. MUa = MUb = MUc = ... = MUn.B. The marginal utility of each good purchased will be zero.C. The marginal utility of the last dollar spent on each good purchased will be the same.D. The total utility obtained from each good purchased will be the same. 

9. A consumer's demand curve for a product is downsloping because: A. total utility falls below marginal utility as more of a product is consumed.B. marginal utility diminishes as more of a product is consumed.C. time becomes less valuable as more of a product is consumed.D. the income and substitution effects precisely offset each other. 

 Answer the question on the basis of the following marginal utility data for products X and Y. Assume that the prices of X and Y are $4 and $2 respectively and that the consumer's income is $18.

    

10. Refer to the above data. What quantities of X and Y should be purchased to maximize utility? A. 2 of X and 1 of YB. 4 of X and 5 of YC. 2 of X and 5 of YD. 2 of X and 6 of Y 

11. An explicit cost is: A. omitted when accounting profits are calculated.B. a money payment made for resources not owned by the firm itself.C. an implicit cost to the resource owner who receives that payment.D. always in excess of a resource's opportunity cost. 

Page 88: Microeconomics Study Guide

 The following is cost information for the Creamy Crisp Donut Company:Entrepreneur's potential earnings as a salaried worker = $50,000Annual lease on building = $22,000Annual revenue from operations = $380,000Payments to workers = $120,000Utilities (electricity, water, disposal) costs = $8,000Value of entrepreneur's talent in the next best entrepreneurial activity = $80,000Entrepreneur's forgone interest on personal funds used to finance the business = $6,000 

12. Refer to the above data. Creamy Crisp's accounting profit is: A. $150,000.B. $380,000.C. $230,000.D. $294,000. 

13. Refer to the above data. Creamy Crisp's economic profit is: A. $150,000.B. $80,000.C. $230,000.D. $94,000. 

14. Refer to the above data. If, other things equal, Creamy Crisp's revenue fell to $286,000: A. its implicit costs, including a normal profit, would exceed its explicit costs.B. it would earn a normal profit but not an economic profit.C. it would suffer an economic loss.D. its accounting profit would fall to zero. 

 Answer the question on the basis of the following output data for a firm. Assume that the amounts of all non-labor resources are fixed.

   

15. Refer to the above data. The marginal product of the sixth worker is: A. 180 units of output.B. 30 units of output.C. 15 units of output.D. negative. 

16. Refer to the above data. Average product is at a maximum when: A. five workers are hired.B. four workers are hired.C. three workers are hired.D. two workers are hired.  

Page 89: Microeconomics Study Guide

17. To economists, the main difference between the short run and the long run is that: A. the law of diminishing returns applies in the long run, but not in the short run.B. in the long run all resources are variable, while in the short run at least one resource is fixed.C. fixed costs are more important to decision making in the long run than they are in the short run.D. in the short run all resources are fixed, while in the long run all resources are variable. 

     

18. In the above diagram the range of diminishing marginal returns is: A. 0Q3.B. 0Q2.C. Q1Q2.D. Q1Q3. 

 Use the following data to answer the question:

   19. Refer to the above data. The average product (AP) when two units of labor are hired is: A. 8.B. 9.C. 10.D. 18. 

20. Refer to the above data. Diminishing returns begin to occur with the hiring of the _________ unit of labor. A. firstB. secondC. thirdD. seventh 

21. Refer to the above data. Marginal product becomes negative with the hiring of the __________ unit of labor. A. thirdB. fourthC. sixthD. seventh 

Page 90: Microeconomics Study Guide

22. Fixed cost is: A. the cost of producing one more unit of capital, for example, machinery.B. any cost which does not change when the firm changes its output.C. average cost multiplied by the firm's output.D. usually zero in the short run. 

23. If a firm decides to produce no output in the short run, its costs will be: A. its marginal costs.B. its variable costs.C. its fixed costs.D. zero. 

24. A purely competitive seller is: A. both a "price maker" and a "price taker."B. neither a "price maker" nor a "price taker."C. a "price taker."D. a "price maker." 

25. The demand curve in a purely competitive industry is _____, while the demand curve to a single firm in that industry is _____. A. perfectly inelastic, perfectly elasticB. downsloping, perfectly elasticC. downsloping, perfectly inelasticD. perfectly elastic, downsloping 

26. A competitive firm in the short run can determine the profit-maximizing (or loss-minimizing) output by equating: A. price and average total cost.B. price and average fixed cost.C. marginal revenue and marginal cost.D. price and marginal revenue. 

27. When a firm is maximizing profit it will necessarily be: A. maximizing profit per unit of output.B. maximizing the difference between total revenue and total cost.C. minimizing total cost.D. maximizing total revenue. 

28. Suppose you find that the price of your product is less than minimum AVC. You should: A. minimize your losses by producing where P = MC.B. maximize your profits by producing where P = MC.C. close down because, by producing, your losses will exceed your total fixed costs.D. close down because total revenue exceeds total variable cost. 

Page 91: Microeconomics Study Guide

 Answer the question on the basis of the following data confronting a firm:

    29. Refer to the above data. If the firm's minimum average variable cost is $10, the firm's profit-maximizing level of output would be: A. 2.B. 3.C. 4.D. 5. 

30. If a firm is confronted with economic losses in the short run, it will decide whether or not to produce by comparing: A. marginal revenue and marginal cost.B. price and minimum average variable cost.C. total revenue and total cost.D. total revenue and total fixed cost. 

31. A firm finds that at its MR = MC output, its TC = $1,000, TVC = $800, TFC = $200, and total revenue is $900. This firm should: A. shut down in the short run.B. produce because the resulting loss is less than its TFC.C. produce because it will realize an economic profit.D. liquidate its assets and go out of business. 

     

 32. Refer to the above diagram for a purely competitive producer. The firm will produce at a loss at all prices: A. above P1.B. above P3.C. above P4.D. between P2 and P3. 

33. The primary force encouraging the entry of new firms into a purely competitive industry is: A. normal profits earned by firms already in the industry.B. economic profits earned by firms already in the industry.C. government subsidies for start-up firms.D. a desire to provide goods for the betterment of society.  

Page 92: Microeconomics Study Guide

34. If a purely competitive firm is producing at the MR = MC output level and earning an economic profit, then: A. the selling price for this firm is above the market equilibrium price.B. new firms will enter this market.C. some existing firms in this market will leave.D. there must be price fixing by the industry's firms. 

35. Long-run competitive equilibrium: A. is realized only in constant-cost industries.B. will never change once it is realized.C. is not economically efficient.D. results in zero economic profits. 

36. A purely competitive firm: A. must earn a normal profit in the short run.B. cannot earn economic profit in the long run.C. may realize either economic profit or losses in the long run.D. cannot earn economic profit in the short run. 

37. When LCD televisions first came on the market, they sold for at least $1,000, and some for much more. Now many units can be purchased for under $400. These facts imply that: A. the LCD television industry was once competitive, but is now monopolistic.B. fewer firms produce LCD televisions than was the case five or ten years ago.C. the demand curve for LCD televisions has shifted leftward.D. the LCD television industry is a decreasing-cost industry. 

38. The MR = MC rule applies: A. in the short run, but not in the long run.B. in the long run, but not in the short run.C. in both the short run and the long run.D. only to a purely competitive firm.

      39. Refer to the above diagrams which pertain to a purely competitive firm producing output q and the industry in which it operates. In the long run we should expect: A. firms to enter the industry, market supply to rise, and product price to fall.B. firms to leave the industry, market supply to rise, and product price to fall.C. firms to leave the industry, market supply to fall, and product price to rise.D. no change in the number of firms in this industry. 

 40. Creative destruction is: A. the process by which large firms buy up small firms.B. the process by which new firms and new products replace existing dominant firms and products.C. a term coined many years ago by Adam Smith.D. is applicable to planned economies, but not to market economies.

Page 93: Microeconomics Study Guide

41. Which of the following is correct? A. Both purely competitive and monopolistic firms are "price takers."B. Both purely competitive and monopolistic firms are "price makers."C. A purely competitive firm is a "price taker," while a monopolist is a "price maker."D. A purely competitive firm is a "price maker," while a monopolist is a "price taker."

42. Which of the following is a characteristic of pure monopoly? A. close substitute productsB. barriers to entryC. the absence of market powerD. "price taking"

43. The nondiscriminating pure monopolist's demand curve: A. is the industry demand curve.B. shows a direct or positive relationship between price and quantity demanded.C. tends to be inelastic at high prices and elastic at low prices.D. is identical to its marginal revenue curve.

44. When a firm is on the inelastic segment of its demand curve, it can: A. increase total revenue by reducing price.B. decrease total costs by decreasing price.C. increase profits by increasing price.D. increase total revenue by more than the increase in total cost by increasing price.Answer the question on the basis of the demand schedule shown below:

    45. Refer to the above data. The marginal revenue obtained from selling the third unit of output is: A. $6.B. $1.C. $3.D. $5.

46. For a pure monopolist marginal revenue is less than price because: A. the monopolist's demand curve is perfectly elastic.B. the monopolist's demand curve is perfectly inelastic.C. when a monopolist lowers price to sell more output, the lower price applies to all units sold.D. the monopolist's total revenue curve is linear and slopes upward to the right.

Page 94: Microeconomics Study Guide

47. A pure monopolist should never produce in the: A. elastic segment of its demand curve because it can increase total revenue and reduce total cost by lowering price.B. inelastic segment of its demand curve because it can increase total revenue and reduce total cost by increasing price.C. inelastic segment of its demand curve because it can always increase total revenue by more than it increases total cost by reducing price.D. segment of its demand curve where the price elasticity coefficient is greater than one.

Answer the question on the basis of the following table showing the demand schedulefacing a nondiscriminating monopolist:

    48. Refer to the above table. The monopolist will select its profit-maximizing level of output somewhere within the: A. 3-5 unit range of output.B. 1-3 unit range of output.C. 1-4 unit range of output.D. 2-4 unit range of output.

      49. Which of the above diagrams correctly portray a nondiscriminating pure monopolist's demand (D) and marginal revenue (MR) curves? A. AB. BC. CD. D

50. Which of the above diagrams correctly portray the demand (D) and marginal revenue (MR) curves of a purely competitive seller? A. AB. BC. CD. D

Page 95: Microeconomics Study Guide

51. Suppose that a pure monopolist can sell 5 units of output at $4 per unit and 6 units at $3.90 per unit. The monopolist will produce and sell the sixth unit if its marginal cost is: A. $4 or less.B. $3.90 or less.C. $3.50 or less.D. $3.40 or less.

 52. Refer to the above data for a nondiscriminating monopolist. This firm will maximize its profit by producing: A. 3 units.B. 4 units.C. 5 units.D. 6 units.

53. Refer to the above data for a nondiscriminating monopolist. At its profit-maximizing output, this firm's price will exceed its marginal cost by ____ and its average total cost by ___. A. $20; $27.33B. $10; $10.40C. $24; $27.33D. $30; $20.50

54. Refer to the above data for a nondiscriminating monopolist. At its profit-maximizing output, this firm's total costs will be: A. $300.B. $248.C. $198.D. $126.

55. Refer to the above data. At its profit-maximizing output, this firm's total revenue will be: A. $300.B. $198.C. $180.D. $280.

56. Refer to the above data for a nondiscriminating monopolist. At its profit-maximizing output, this firm's total profit will be: A. $82.B. zero.C. $54.D. $27. 

Page 96: Microeconomics Study Guide

1. For economists, the word "utility" means: A. versatility and flexibility.B. rationality.C. pleasure or satisfaction.D. purposefulness. 

2. Joe sold gold coins for $1000 that he bought a year ago for $1000. He says, "At least I didn't lose any money on my financial investment." His economist friend points out that in effect he did lose money, because he could have received a 3 percent return on the $1000 if he had bought a bank certificate of deposit instead of the coins. The economist's analysis in this case incorporates the idea of: A. opportunity costs.B. marginal benefits that exceed marginal costs.C. imperfect information.D. normative economics.  

3. The assertion that "There is no free lunch" means that: A. there are always tradeoffs between economic goals.B. all production involves the use of scarce resources and thus the sacrifice of alternative goods.C. marginal analysis is not used in economic reasoning.D. choices need not be made if behavior is rational.  

4. The budget line shows: A. the amount of product A that a consumer is willing to give up to obtain one more unit of product B.B. all possible combinations of two goods that can be purchased, given money income and the prices of the goods.C. the minimum amount of two goods that a consumer can purchase with a given money income.D. all possible combinations of two goods that yield the same level of utility to the consumer.

      5. Refer to the budget line shown in the diagram above. If the consumer's money income is $20, the: A. prices of C and D cannot be determined.B. price of C is $2 and the price of D is $4.C. consumer can obtain a combination of 5 units of both C and D.D. price of C is $4 and the price of D is $2. 

6. Any point inside the production possibilities curve indicates: A. the presence of technological change.B. that resources are imperfectly substitutable among alternative uses.C. the presence of inflationary pressures.D. that more output could be produced with available resources.

Page 97: Microeconomics Study Guide

 7. Command systems are also known as: A. market systems.B. pure capitalism.C. laissez-faire capitalism.D. communism. 

8. The two basic markets shown by the simple circular flow model are: A. capital goods and consumer goods.B. free and controlled.C. product and resource.D. household and business. 

9. The law of demand states that, other things equal: A. price and quantity demanded are inversely related.B. the larger the number of buyers in a market, the lower will be product price.C. price and quantity demanded are directly related.D. consumers will buy more of a product at high prices than at low prices. 

10. The income and substitution effects account for: A. the upward sloping supply curve.B. the downward sloping demand curve.C. movements along a given supply curve.D. shifts in the demand curve. 

11. Which of the following would not shift the demand curve for beef? A. a widely publicized study that indicates beef increases one's cholesterolB. a reduction in the price of cattle feedC. an effective advertising campaign by pork producersD. a change in the incomes of beef consumers 

12. If the demand curve for product B shifts to the right as the price of product A declines, then: A. both A and B are inferior goods.B. A is a superior good and B is an inferior good.C. A is an inferior good and B is a superior good.D. A and B are complementary goods. 

13. If X is a normal good, a rise in money income will shift the: A. supply curve for X to the left.B. supply curve for X to the right.C. demand curve for X to the left.D. demand curve for X to the right. 

14. A decrease in the price of digital cameras will: A. cause the demand curve for memory cards to become vertical.B. shift the demand curve for memory cards to the right.C. shift the demand curve for memory cards to the left.D. not affect the demand for memory cards. 

15. Assume that the demand curve for product C is downsloping. If the price of C falls from $2.00 to $1.75: A. a smaller quantity of C will be demanded.B. a larger quantity of C will be demanded.C. the demand for C will increase.D. the demand for C will decrease. 

Page 98: Microeconomics Study Guide

16. A leftward shift of a product supply curve might be caused by: A. an improvement in the relevant technique of production.B. a decline in the prices of needed inputs.C. an increase in consumer incomes.D. some firms leaving an industry. 

17. A government subsidy to the producers of a product: A. reduces product supply.B. increases product supply.C. reduces product demand.D. increases product demand. 

18. Suppose that corn prices rise significantly. If farmers expect the price of corn to continue rising relative to other crops, then we would expect: A. the supply of ethanol, a corn-based product, to increase.B. consumer demand for wheat to fall.C. the supply to increase as farmers plant more corn.D. the supply to fall as farmers plant more of other crops. 

     

19. Refer to the above table. If demand is represented by columns (3) and (2) and supply is represented by columns (3) and (5), equilibrium price and quantity will be: A. $10 and 60 units.B. $9 and 50 units.C. $8 and 60 units.D. $7 and 50 units. 

20. Refer to the above table. Suppose that demand is represented by columns (3) and (2) and supply is represented by columns (3) and (5). If the price were artificially set at $9, A. the market would clear.B. a surplus of 20 units would occur.C. a shortage of 20 units would occur.D. demand would change from columns (3) and (2) to columns (3) and (1).

      21. Refer to the above diagram. The equilibrium price and quantity in this market will be: A. $1.00 and 200.B. $1.60 and 130.C. $0.50 and 130.D. $1.60 and 290. 

Page 99: Microeconomics Study Guide

22. Refer to the above diagram. A surplus of 160 units would be encountered if the price was: A. $1.10, that is, $1.60 minus $.50.B. $1.60.C. $1.00.D. $0.50. 

23. Refer to the above diagram. A shortage of 160 units would be encountered if price was: A. $1.10, that is, $1.60 minus $.50.B. $1.60.C. $1.00.D. $0.50. 

24. If there is a surplus of a product, its price: A. is below the equilibrium level.B. is above the equilibrium level.C. will rise in the near future.D. is in equilibrium. 

    25. Refer to the above diagram. A price of $60 in this market will result in: A. equilibrium.B. a shortage of 50 units.C. a surplus of 50 units.D. a surplus of 100 units. 

26. Refer to the above diagram. A price of $20 in this market will result in a: A. shortage of 50 units.B. surplus of 50 units.C. surplus of 100 units.D. shortage of 100 units. 

27. Assume in a competitive market that price is initially below the equilibrium level. We can predict that price will: A. decrease, quantity demanded will decrease, and quantity supplied will increase.B. decrease and quantity demanded and quantity supplied will both decrease.C. increase, quantity demanded will increase, and quantity supplied will decrease.D. increase, quantity demanded will decrease, and quantity supplied will increase. 

28. If the price elasticity of demand for a product is 2.5, then a price cut from $2.00 to $1.80 will: A. increase the quantity demanded by about 2.5 percent.B. decrease the quantity demanded by about 2.5 percent.C. increase the quantity demanded by about 25 percent.D. increase the quantity demanded by about 250 percent. 

Page 100: Microeconomics Study Guide

29. Suppose that as the price of Y falls from $2.00 to $1.90 the quantity of Y demanded increases from 110 to 118. Then the price elasticity of demand is: A. 4.00.B. 2.09.C. 1.37.D. 3.94. 

30. The price elasticity of demand of a straight-line demand curve is: A. elastic in high-price ranges and inelastic in low-price ranges.B. elastic, but does not change at various points on the curve.C. inelastic, but does not change at various points on the curve.D. 1 at all points on the curve. 

31. The price elasticity of demand is generally: A. negative, but the minus sign is ignored.B. positive, but the plus sign is ignored.C. positive for normal goods and negative for inferior goods.D. positive because price and quantity demanded are inversely related. 

32. If a demand for a product is elastic, the value of the price elasticity coefficient is: A. zero.B. greater than one.C. equal to one.D. less than one.

      33. Refer to the above diagram. Between prices of $5.70 and $6.30: A. D1 is more elastic than D2.B. D2 is an inferior good and D1 is a normal good.C. D1 and D2 have identical elasticities.D. D2 is more elastic than D1. 

34. Refer to the above diagram and assume a single good. If the price of the good decreases from $6.30 to $5.70, consumer expenditure would: A. decrease if demand were D1 only.B. decrease if demand were D2 only.C. decrease if demand were either D1 or D2.D. increase if demand were either D1 or D2. 

35. Refer to the above diagram and assume a single good. If the price of the good increased from $5.70 to $6.30 along D1, the price elasticity of demand along this portion of the demand curve would be: A. 0.8.B. 1.0.C. 1.2.D. 2.0.

Page 101: Microeconomics Study Guide

 36. Suppose the price of local cable TV service increased from $16.20 to $19.80 and as a result the number of cable subscribers decreased from 224,000 to 176,000. Along this portion of the demand curve, price elasticity of demand is: A. 0.8.B. 1.2.C. 1.6.D. 8.0 

37. A firm can sell as much as it wants at a constant price. Demand is thus: A. perfectly inelastic.B. perfectly elastic.C. relatively inelastic.D. relatively elastic. 

38. Suppose the price elasticity coefficients of demand are 1.43, 0.67, 1.11, and 0.29 for products W, X, Y, and Z respectively. A 1 percent decrease in price will increase total revenue in the case(s) of: A. W and Y.B. Y and Z.C. X and Z.D. Z and W. 

39. If a firm finds that it can sell $13,000 worth of a product when its price is $5 per unit and $11,000 worth of it when its price is $6, then: A. the demand for the product is elastic in the $6-$5 price range.B. the demand for the product must have increased.C. elasticity of demand is 0.74.D. the demand for the product is inelastic in the $6-$5 price range. 

40. Suppose the price elasticity of demand for bread is 0.20. If the price of bread falls by 10 percent, the quantity demanded will increase by: A. 2 percent and total expenditures on bread will rise.B. 2 percent and total expenditures on bread will fall.C. 20 percent and total expenditures on bread will fall.D. 20 percent and total expenditures on bread will rise. 

41. Gigantic State University raises tuition for the purpose of increasing its revenue so that more faculty can be hired. GSU is assuming that the demand for education at GSU is: A. decreasing.B. relatively elastic.C. perfectly elastic.D. relatively inelastic. 

42. Suppose the income elasticity of demand for toys is +2.00. This means that: A. a 10 percent increase in income will increase the purchase of toys by 20 percent.B. a 10 percent increase in income will increase the purchase of toys by 2 percent.C. a 10 percent increase in income will decrease the purchase of toys by 2 percent.D. toys are an inferior good. 

43. We would expect the cross elasticity of demand between dress shirts and ties to be: A. positive, indicating normal goods.B. positive, indicating complementary goods.C. negative, indicating substitute goods.D. negative, indicating complementary goods. 

Page 102: Microeconomics Study Guide

44. We would expect the cross elasticity of demand between Pepsi and Coke to be: A. positive, indicating normal goods.B. positive, indicating inferior goods.C. positive, indicating substitute goods.D. negative, indicating substitute goods. 

45. Assume that a 3 percent increase in income across the economy produces a 1 percent decline in the quantity demanded of good X. The coefficient of income elasticity of demand for good X is: A. negative and therefore X is an inferior good.B. negative and therefore X is a normal good.C. positive and therefore X is an inferior good.D. positive and therefore X is a normal good. 

46. The two main characteristics of a public good are: A. production at constant marginal cost and rising demand.B. nonexcludability and production at rising marginal cost.C. nonrivalry and nonexcludability.D. nonrivalry and large negative externalities. 

47. Unlike a private good, a public good: A. has no opportunity costs.B. has benefits available to all, including nonpayers.C. produces no positive or negative externalities.D. is characterized by rivalry and excludability. 

48. Nonexcludability describes a condition where: A. one person's consumption of a good does not prevent consumption of the good by others.B. there is no effective way to keep people from using a good once it comes into being.C. sellers can withhold the benefits of a good from those unwilling to pay for it.D. there is no potential for free-riding behavior. 

49. At the optimal quantity of a public good: A. marginal benefit exceeds marginal cost by the greatest amount.B. total benefit equals total cost.C. marginal benefit equals marginal cost.D. marginal benefit is zero. 

50. The ability of a good or service to satisfy wants is called: A. utility maximization.B. opportunity cost.C. revenue potential.D. utility. 

51. Marginal utility is the: A. sensitivity of consumer purchases of a good to changes in the price of that good.B. change in total utility obtained by consuming one more unit of a good.C. change in total utility obtained by consuming another unit of a good divided by the change in the price of that good.D. total utility associated with the consumption of a certain number of units of a good divided by the number of units consumed. 

Page 103: Microeconomics Study Guide

52. To maximize utility a consumer should allocate money income so that the: A. elasticity of demand on all products purchased is the same.B. marginal utility obtained from the last dollar spent on each product is the same.C. total utility derived from each product consumed is the same.D. marginal utility of the last unit of each product consumed is the same. 

 Answer the question on the basis of the following total utility data for products L and M. Assume that the prices of L and M are $3 and $4 respectively and that the consumer's income is $18.

    53. Refer to the above data. How many units of the two products will the rational consumer purchase? A. 3 of L and none of MB. 4 of L and 2 of MC. 3 of L and 5 of MD. 2 of L and 3 of M 

54. Refer to the above data. What level of total utility does the rational consumer realize in equilibrium? A. 87 utilsB. 104 utilsC. 51 utilsD. 58 utils 

55. Diminishing marginal utility explains why: A. the income effect exceeds the substitution effect.B. the substitution effect exceeds the income effect.C. supply curves are upsloping.D. demand curves are downsloping. 

56. Why do people tend to eat more at all-you-can-eat buffet restaurants than at restaurants where each item is purchased separately? A. Once the all-you-can-eat meal is purchased, consumers view additional trips back to the buffet as having a price of zero.B. MU/P is greater at all-you-can-eat restaurants.C. People who eat at all-you-can-eat restaurants do not experience diminishing marginal utility.D. Food at all-you-can-eat restaurants tends to have fewer calories, so consumers feel the need to consume a greater volume of food. 

57. To the economist, total cost includes: A. explicit and implicit costs, including a normal profit.B. neither implicit nor explicit costs.C. implicit, but not explicit, costs.D. explicit, but not implicit, costs. 

58. Suppose that a business incurred implicit costs of $500,000 and explicit costs of $5 million in a specific year. If the firm sold 100,000 units of its output at $50 per unit, its accounting: A. profits were $100,000 and its economic profits were zero.B. losses were $500,000 and its economic losses were zero.C. profits were $500,000 and its economic profits were $1 million.D. profits were zero and its economic losses were $500,000. 

Page 104: Microeconomics Study Guide

59. The basic difference between the short run and the long run is that: A. all costs are fixed in the short run, but all costs are variable in the long run.B. the law of diminishing returns applies in the long run, but not in the short run.C. at least one resource is fixed in the short run, while all resources are variable in the long run.D. economies of scale may be present in the short run, but not in the long run. 

 Answer the question on the basis of the following output data for a firm. Assume that the amounts of all non-labor resources are fixed.

    60. Refer to the above data. Diminishing marginal returns become evident with the addition of the: A. sixth worker.B. fourth worker.C. third worker.D. second worker. 

61. Refer to the above data. The marginal product of the sixth worker is: A. 180 units of output.B. 30 units of output.C. 15 units of output.D. negative. 

 Use the following data to answer the question:

    62. Refer to the above data. The average product (AP) when two units of labor are hired is: A. 8.B. 9.C. 10.D. 18. 

63. Refer to the above data. Diminishing returns begin to occur with the hiring of the _________ unit of labor. A. firstB. secondC. thirdD. seventh 

64. Refer to the above data. Marginal product becomes negative with the hiring of the __________ unit of labor. A. thirdB. fourthC. sixthD. seventh

Page 105: Microeconomics Study Guide

  Answer the question on the basis of the following cost data:

    65. Refer to the above data. Total fixed cost is: A. $6.25.B. $100.00.C. $150.00.D. $50.00. 

66. Refer to the above data. The average total cost of five units of output is: A. $69.B. $78.C. $3.D. $10. 

67. Refer to the above data. The total cost of four units of output is: A. $260.B. $77.50.C. $310.D. $215. 

68. Refer to the above data. If the firm closed down in the short run and produced zero units of output, its total cost would be: A. zero.B. $50.C. $150.D. $100. 

69. Refer to the above data. The marginal cost of the fifth unit of output is: A. $3.B. $62.C. $80.D. $78. 

70. In which of the following market structures is there clear-cut mutual interdependence with respect to price-output policies? A. pure monopolyB. oligopolyC. monopolistic competitionD. pure competition 

71. An industry comprised of 40 firms, none of which has more than 3 percent of the total market for a differentiated product is an example of: A. monopolistic competition.B. oligopoly.C. pure monopoly.D. pure competition.

Page 106: Microeconomics Study Guide

 72. An industry comprised of a small number of firms, each of which considers the potential reactions of its rivals in making price-output decisions is called: A. monopolistic competition.B. oligopoly.C. pure monopoly.D. pure competition. 

73. A purely competitive seller is: A. both a "price maker" and a "price taker."B. neither a "price maker" nor a "price taker."C. a "price taker."D. a "price maker."  

74. For a purely competitive seller, price equals: A. average revenue.B. marginal revenue.C. total revenue divided by output.D. all of these.

      75. Refer to the above short-run data. The profit-maximizing output for this firm is: A. above 440 units.B. 440 units.C. 320 units.D. 100 units. 

76. In the short run the individual competitive firm's supply curve is that segment of the: A. average variable cost curve lying below the marginal cost curve.B. marginal cost curve lying above the average variable cost curve.C. marginal revenue curve lying below the demand curve.D. marginal cost curve lying between the average total cost and average variable cost curves. 

77. Suppose you find that the price of your product is less than minimum AVC. You should: A. minimize your losses by producing where P = MC.B. maximize your profits by producing where P = MC.C. close down because, by producing, your losses will exceed your total fixed costs.D. close down because total revenue exceeds total variable cost. 

 Answer the question on the basis of the following data confronting a firm:

   

Page 107: Microeconomics Study Guide

 78. Refer to the above data. This firm is selling its output in a(n): A. monopolistically competitive market.B. monopolistic market.C. purely competitive market.D. oligopolistic market. 

79. Refer to the above data. If the firm's minimum average variable cost is $10, the firm's profit-maximizing level of output would be: A. 2.B. 3.C. 4.D. 5. 

80. Refer to the above data. At the profit-maximizing output the firm's total revenue is: A. $48.B. $32.C. $80.D. $64. 

81. Refer to the above data. Assuming total fixed costs equal to zero, the firm's: A. economic profit is $12.B. economic profit is $16.C. loss is $14.D. economic profit is $3. 

82. A firm finds that at its MR = MC output, its TC = $1,000, TVC = $800, TFC = $200, and total revenue is $900. This firm should: A. shut down in the short run.B. produce because the resulting loss is less than its TFC.C. produce because it will realize an economic profit.D. liquidate its assets and go out of business.

       83. Refer to the above diagram for a purely competitive producer. The lowest price at which the firm should produce (as opposed to shutting down) is: A. P1.B. P2.C. P3.D. P4. 

84. Refer to the above diagram for a purely competitive producer. The firm will produce at a loss at all prices: A. above P1.B. above P3.C. above P4.D. between P2 and P3.

Page 108: Microeconomics Study Guide

85. Refer to the above diagram for a purely competitive producer. If product price is P3: A. the firm will maximize profit at point d.B. the firm will earn an economic profit.C. economic profits will be zero.D. new firms will enter this industry. 

 Answer the question on the basis of the following cost data for a firm that is selling in a purely competitive market:

    86. Refer to the above data. If the market price for the firm's product is $12, the competitive firm will produce: A. 4 units at a loss of $109.B. 4 units at an economic profit of $31.75.C. 8 units at a loss of $48.80.D. zero units at a loss of $100. 

87. Refer to the above data. If the market price for the firm's product is $32, the competitive firm will produce: A. 8 units at an economic profit of $16.B. 6 units at an economic profit of $7.98.C. 10 units at an economic profit of $4.D. 7 units at an economic profit of $41.50. 

88. Refer to the above data. If the market price for the firm's product is $28, the competitive firm will: A. produce 4 units at a loss of $17.40.B. produce 7 units at a loss of $14.00.C. shut down in the short run.D. produce 6 units at a loss of $23.80. 

89. In a purely competitive industry: A. there will be no economic profits in either the short run or the long run.B. economic profits may persist in the long run if consumer demand is strong and stable.C. there may be economic profits in the short run, but not in the long run.D. there may be economic profits in the long run, but not in the short run. 

90. If a purely competitive constant-cost industry is realizing economic profits, we can expect industry supply to: A. increase, output to increase, price to decrease, and profits to decrease.B. increase, output to increase, price to increase, and profits to decrease.C. decrease, output to decrease, price to increase, and profits to increase.D. increase, output to decrease, price to decrease, and profits to decrease. 

Page 109: Microeconomics Study Guide

91. Which of the following conditions is true for a purely competitive firm in long-run equilibrium? A. P > MC = minimum ATC.B. P > MC > minimum ATC.C. P = MC = minimum ATC.D. P < MC < minimum ATC. 

92. Which of the following is correct? A. Both purely competitive and monopolistic firms are "price takers."B. Both purely competitive and monopolistic firms are "price makers."C. A purely competitive firm is a "price taker," while a monopolist is a "price maker."D. A purely competitive firm is a "price maker," while a monopolist is a "price taker." 

93. Which of the following is a characteristic of pure monopoly? A. close substitute productsB. barriers to entryC. the absence of market powerD. "price taking" 

 Answer the question on the basis of the demand schedule shown below:

   94. Refer to the above data. The marginal revenue obtained from selling the third unit of output is: A. $6.B. $1.C. $3.D. $5. 

95. Which of the following is characteristic of a pure monopolist's demand curve? A. Average revenue is less than price.B. Its elasticity coefficient is 1 at all levels of output.C. Price and marginal revenue are equal at all levels of output.D. It is the same as the market demand curve. 

96. Because the monopolist's demand curve is downsloping: A. MR will equal price.B. price must be lowered to sell more output.C. the elasticity coefficient will increase as price is lowered.D. its supply curve will also be downsloping. 

97. An unregulated pure monopolist will maximize profits by producing that output at which: A. P = MC.B. P = ATC.C. MR = MC.D. MC = AC. 

Page 110: Microeconomics Study Guide

98. Suppose that a pure monopolist can sell 5 units of output at $4 per unit and 6 units at $3.90 per unit. The monopolist will produce and sell the sixth unit if its marginal cost is: A. $4 or less.B. $3.90 or less.C. $3.50 or less.D. $3.40 or less. 

99. A pure monopolist is producing an output such that ATC = $4, P = $5, MC = $2, and MR = $3. This firm is realizing: A. a loss that could be reduced by producing more output.B. a loss that could be reduced by producing less output.C. an economic profit that could be increased by producing more output.D. an economic profit that could be increased by producing less output.

      100. Refer to the above diagram. To maximize profits or minimize losses this firm should produce: A. E units and charge price C.B. E units and charge price A.C. M units and charge price N.D. L units and charge price LK. 

101. Refer to the above diagram. At the profit-maximizing level of output, total revenue will be: A. NM times 0M.B. 0AJE.C. 0EGC.D. 0EHB.

102. Refer to the above diagram. At the profit-maximizing level of output, total cost will be: A. NM times 0M.B. 0AJE.C. 0CGC.D. 0BHE. 

103. Refer to the above diagram. At the profit-maximizing level of output, the firm will realize: A. an economic profit of ABHJ.B. an economic profit of ACGJ.C. a loss of GH per unit.D. a loss of JH per unit. 

104. If profits are maximized (or losses minimized), which of the following conditions is common to both unregulated monopoly and to pure competition? A. MC = PB. MC = ATCC. MR = MCD. P = MR 

Page 111: Microeconomics Study Guide

    105. Refer to the above diagram for a pure monopolist. Monopoly price will be: A. e.B. c.C. b.D. a. 

106. Refer to the above diagram for a pure monopolist. Monopoly output will be: A. between f and g.B. h.C. g.D. f. 

     107. Refer to the above diagram. If this industry is comprised of only one seller, the profit-maximizing price and quantity will be: A. P3 and Q3.B. P1 and Q3.C. P2 and Q2.D. indeterminate on the basis of the information given. 

108. Which of the following statements is correct? A. The pure monopolist will maximize profit by producing at that point on the demand curve where elasticity is zero.B. In seeking the profit-maximizing output the pure monopolist underallocates resources to its production.C. The pure monopolist maximizes profits by producing that output at which the differential between price and average cost is the greatest.D. Purely monopolistic sellers earn only normal profits in the long run. 

109. When economists say that the demand for labor is a derived demand, they mean that it is: A. dependent on government expenditures for public goods and services.B. related to the demand for the product or service labor is producing.C. based on the desire of businesses to exploit labor by paying below equilibrium wage rates.D. based on the assumption that workers are trying to maximize their money incomes. 

Page 112: Microeconomics Study Guide

110. Marginal product is: A. the output of the least skilled worker.B. a worker's output multiplied by the price at which each unit can be sold.C. the amount an additional worker adds to the firm's total output.D. the amount any given worker contributes to the firm's total revenue. 

111. Assume labor is the only variable input and that an additional input of labor increases total output from 72 to 78 units. If the product sells for $6 per unit in a purely competitive market, the MRP of this additional worker is: A. $6.B. $12.C. $36.D. $72. 

112. If one worker can pick $30 worth of grapes and two workers together can pick $50 worth of grapes, the: A. marginal revenue product of each worker is $25.B. marginal revenue product of the first worker is $20.C. marginal revenue product of the second worker is $20.D. data given do not permit the determination of the marginal revenue product of either worker.

Answer the question on the basis of the following information for Manfred's Shoe Shine Parlor. Assume Manfred hires labor, its only variable input, under purely competitive conditions. Shoe shines are also sold competitively.

    113. Refer to the above data. How many units of output are produced when 2 workers are employed? A. 4B. 16C. 24D. 10 

114. Refer to the above data. What is the marginal product of the sixth worker? A. 2 unitsB. 3 unitsC. 4 unitsD. 5 units 

115. Refer to the above data. At what price does each shoe shine sell? A. $1B. $2C. $3D. $2.50 

Page 113: Microeconomics Study Guide

116. Refer to the above data. If the wage rate is $11, how many workers will Manfred hire to maximize profits? A. 1B. 2C. 3D. 5 

117. Refer to the above data. If the wage rate is $11 and Manfred's only fixed input is capital, the total cost of which is $30, then what will be his economic profit? A. $62B. $42C. $28D. $32 

118. Assume that a restaurant is hiring labor in an amount such that the MRC of the last worker is $16 and her MRP is $12. On the basis of this information we can say that: A. profits will be increased by hiring additional workers.B. profits will be increased by hiring fewer workers.C. marginal revenue product must exceed average revenue product.D. the restaurant is maximizing profits. 

 Answer the question on the basis of the data contained in the following table. Assume that the firm is hiring labor in a purely competitive market.

    119. Refer to the above data. If the wage rate is $20, how many workers will the firm choose to employ? A. 5B. 4C. 3D. 2 

120. Refer to the above data. If the wage rate is $11, how many workers will the firm choose to employ? A. 5B. 4C. 3D. 2 

121. If the nominal wages of carpenters rose by 5 percent in 2010 and the price level increased by 3 percent, then the real wages of carpenters: A. decreased by 2 percent.B. increased by 2 percent.C. increased by 3 percent.D. increased by 8 percent. 

122. If the nominal wage rises by 4 percent, and the price level rises by 7 percent, the real wage will: A. be unaffected.B. rise by 3 percent.C. fall by 3 percent.D. rise by 11 percent.

Page 114: Microeconomics Study Guide

     123. Refer to the above diagram. Assuming no union or relevant minimum wage, the firm represented will hire: A. Q2 workers and pay a W4 wage rate.B. Q2 workers and pay a W1 wage rate.C. Q3 workers and pay a W2 wage rate.D. Q4 workers and pay a W1 wage rate. 

124. A progressive tax is such that: A. tax rates are higher the greater one's income.B. the same tax rate applies to all income receivers, so that the rich pay absolutely more taxes than the poor.C. entrepreneurial income is exempt from taxation.D. the revenues it yields are spent on transfer payments. 

 The following data represent a personal income tax schedule. Answer the question on the basis of this information.

    125. Refer to the above table. If your taxable income is $8,000, your average tax rate is: A. 25 percent and the marginal rate on additional income is also 25 percent.B. 25 percent and the marginal rate on additional income is 40 percent.C. 25 percent and the marginal rate on additional income cannot be determined from the information given.D. 20 percent and the marginal rate on additional income is 30 percent. 

126. Which of the following best reflects the ability-to-pay philosophy of taxation? A. a tax on residential propertyB. a progressive income taxC. an excise tax on gasolineD. an excise tax on coffee 

127. The incidence of a tax pertains to: A. the degree to which it alters the distribution of income.B. how easy it is to evade the tax.C. who actually bears the burden of a tax.D. the progressiveness or regressiveness of tax rates.

Page 115: Microeconomics Study Guide

Suppose that a 10 percent increase in the price of normal good Y causes a 20% increase in the quantity demanded of normal good X. The coefficient of cross elasticity of demand is:A. positive and therefore complementsB. negative and therefore complementsC. negative and therefore these goods are substitutesD. positive and therefore substitutes

D

Which of the following statements is correct?A. economic profits induce firms to enter an industry; losses encourage firms to leaveB. Normal profits will cause an industry to expandC. Economic profits induce firms to leave an industry profits encourage firms to leaveD. Economic profits and losses have no significant impact on the growth or decline of an industry

A

A firm can sell as much as it wants at constant price. Demand is thus:A. perfectly elasticB. relatively elasticC. perfectly inelasticD. relatively inelastic

A

Economists would describe the US automobile industry as:A. a pure monopolyB. monopolistically competitiveC. an oligopolyD. purely competitive

C

Implicit and explicit costs are different in that:A. the latter refer to non-expenditure costs and the former to monetary paymentsB. the former refer to non-expenditure costs and the latter to monetary paymentsC. explicit costs are opportunity costs; implicit costs are notD. implicit costs are opportunity costs; explicit costs are not

B

Quasi-Public goods all have negative externalities because the government provides them. (T/F)

F

In efficiently functioning markets, the supply curve must reflect all the costs of production.

Page 116: Microeconomics Study Guide

T

Which of the following industries most closely approximates pure competition?A. clothingB. steelC. farm implementsD. agriculture

D

In the short run, fixed costs are irrelevant in determining a firm's optimal level of output. T/F

T

From an economist's perspective, an important consideration for policies to address global warming is:A. a lawsuit that can arise from the enactment of the policiesB. the market for recyclable inputsC. the marginal cost and marginal benefit of policiesD. the supply and demand for recycled products

C

A good with a price-elasticity coefficient of ).75 has a demand that is price-inelastic. T/F

T

In the market system, prices tend to guide resources from less important to more important uses as the market system accommodates change T/F

T

The utility of a good or service:A. is easy to quantifyB. rarely varies from person to personC. is synonymous with usefulnessD. is the satisfaction or pleasure one gets from consuming it

D

Creative destruction:A. is applicable to planned economies, but not to market economiesB. the process by which large firms buy up small firmsC. the process by which new firms and new products replace existing dominant firms and productsD. a term coined many years ago by Adam Smith

C

Page 117: Microeconomics Study Guide

A person should consume more of something when its marginal benefit exceeds its marginal cost. T/F

T

By free enterprise, we mean that products are provided free to those who can't afford to buy them.

F

The four factors of production are land, labor, capital, and government services. T/F

F

Which of the following distinguishes the short run from the long run in pure competition?A. firms can enter and exit the market in the long run, but not in the short runB. firms use the MR=MC rule to maximize profits in the short run, but not in the long runC. firms attempt to maximize profits in the long run, but not in the short runD. the quantity of labor hired can vary in the long run, but not in the short run

A

the law of diminishing returns explains diseconomies of scale. T/F

F

Government-set price floors and price ceilings interfere with rationing function of price in a fee market. T/F

T

Characteristics of public goods include non-rivalry, non-excludability, and the free-rider problem. T/F

T

What do wages paid to factory workers, interest paid on a bank loan, forgone interest, and the purchase of component parts have in common?A. All are opportunity costsB. All are implicit costsC. all are explicit costsD. None are either implicit or explicit costs

A

The concept of consumer sovereignty refers to the situation where consumers have the right to vote for the board of directors of large corporations. T/F

F

Page 118: Microeconomics Study Guide

In which of the following instances will total revenue decline?A. price rises and demand is elasticB. price rises and demand is inelasticC. price rises and supply is elasticD. price falls and demand is elastic

A

Price is constant or given to the individual firm selling in a purely competitive market because:A. of product differentiation reinforced by extensive advertisingB. the firm's demand curve is downslopingC. there are no good substitutes for its productD. each seller supplies a negligible fraction of total supply

D

Which of the following is an example of creative destruction?A. automobile production causes the wagon industry to shut downB. Apple earns more economic profits than other manufacturers of MP3 playersC. Starbucks shuts down stores to crate greater demand for its remaining outletsD. an economic recession forces firms out of business

A

All costs are variable in the long run but not the short run. T/F

T

Which of the following is not a characteristic of pure competition?A. a standardized productB. price strategies by firmsC. no barriers to entryD. a large number of sellers

B

If a market is allocatively efficient, it means that at the equilibrium price, the maximum price consumers are willing to pay is equal to the minimum acceptable price for producers. T/F

T

Extenalities are unforeseen costs or benefits accruing to a company producing a product. T/F

F

The short run is a period of time during which all costs are fixed costs. T/F

Page 119: Microeconomics Study Guide

F

Th economy of the United States can best be described as laissez-faire capitalism. T/F

F

At zero units of output a firm's variable costs are zero. T/F

T

A firm's economic profit is usually higher than its accounting profit.

F

Which of the following types of firms are least likely to have their MC, AVC, and ATC curves affected by fluctuations in gasoline prices?A. taxi cab and limousine companiesB. firms like UPS that use a fleet of gasoline-powered vehiclesC. firms like iTunes that distributed their products over the internetD. companies that operate bus tours to popular vacation destinations

C

An effective anti pollution policy from the economic perspective requires that all pollution be eliminated and banned.

F

The primary force encouraging the entry of new firms into a purely competitive industry is:A. economic profits earned by firms already in the industryB. a desire to provide goods for the betterment of societyC. government subsidies for start-up firmsD. normal profits earned by firms already in the industry

A

If there are many firms in an industry, then it must be a purely competitive market. T/F

F

A constant-cost industry is one in which:A. a higher price per unit wil not result in an increased outputB. if 100 units canbe produced for $100, then 150 can be produced for $150, 200 for $200, and so forthC. the total cost of producing 200 or 300 units is no greater than the costs of producing 100 unitsD. the demand curve and therefore the unit price and quantity sold seldom change

Page 120: Microeconomics Study Guide

B

The budget line shows all the combinations of two products which the consumer can buy, given money income and product prices. T/F

T

The Illinois Central Railroad once asked the Illinois Commerce Commission for permission to increase its commuter rates by 20%. The railroad argued that declining revenues made this rate increase essential. Opponents of the rate increase contended that the railroad's revenues would fall because of the rate hike. It can be concluded that:A. the railroad felt that the demand for passenger service was inelastic and opponents of the rate increase felt it was elasticB. both groups felt that the demand was inelastic but for different reasonsC. both groups felt that the demand was elastic but for different reasonsD. the railroad felt that the demand for passenger service was elastic and opponents of the rate increase felt it was inelastic.

A

The types and quantities of public goods produced are ultimately determined through the political process. T/F

T

Economic profits are calculated by subtracting:A. explicit costs from total revenueB. implicit costs from normal profitsC. implicit costs from total revenueD. explicit and implicit costs from total revenue

D

The demand curve for a purely competitive industry is perfectly elastic, but the demand curves faced by individual firms in such an industry are downsloping. T/F

F

If the price of ground beef increases, the demand for hamburger buns is predicted to increase as well. T/F

F

The supply of tickets to a major sporting event held in a stadium is perfect inelastic. T/F

T

Page 121: Microeconomics Study Guide

Gigantic State University raises tuition for the purpose of increasing its revenue so that more faculty can be hired. GSU is assuming that the demand for education at GSU is:A. relatively elasticB. decreasingC. relatively inelasticD. perfectly elastic

C

Micro economics is concerned with the establishing of an overall view of the operation of the economic system. T/F

F

A surplus indicates that the quantity demanded is greater than the quantity supplied at the price. T/F

F

In the price range where demand is elastic, if the seller of the good raises its price, then total revenues will increase. T/F

F

Page 122: Microeconomics Study Guide

Costs: explicit and implicit

Type: D Topic: 1 E: 392 MI: 148

1. Economic cost can best be defined as:

A) any contractual obligation that results in a flow of money expenditures from an enterprise to resource suppliers.

B) any contractual obligation to labor or material suppliers.

C) compensations that must be received by resource owners to insure their continued supply.

D) all costs exclusive of payments to fixed factors of production.

Answer: C

Type: A Topic: 1 E: 393 MI: 149

2. Which of the following constitutes an implicit cost to the Johnston Manufacturing Company?

A) payments of wages to its office workers

B) rent paid for the use of equipment owned by the Schultz Machinery Company

C) depreciation charges on company-owned equipment

D) economic profits resulting from current production

Answer: C

Type: A Topic: 1 E: 393 MI: 149

3. Which of the following is most likely to be an implicit cost for Company X?

A) depreciation charges on company-owned equipment

B) rental payments on IBM equipment

C) payments for raw materials purchased from Company Y

D) transportation costs paid to a nearby trucking firm

Answer: A

Page 123: Microeconomics Study Guide

Type: A Topic: 1 E: 393 MI: 149

4. Costs to an economist:

A) consist only of explicit costs. C) never reflect monetary outlays.

B) may or may not involve monetary outlays. D) always reflect monetary outlays.

Answer: B

Type: A Topic: 1 E: 393 MI: 149

5. What do wages paid to blue-collar workers, interest paid on a bank loan, forgone interest, and the purchase of component parts have in common?

A) None are either implicit or explicit costs. C) All are implicit costs.

B) All are opportunity costs. D) All are explicit costs.

Answer: B

Type: A Topic: 1 E: 393 MI: 149

6. To the economist total cost includes:

A) explicit and implicit costs, including a normal profit.

B) neither implicit nor explicit costs.

C) implicit, but not explicit, costs.

D) explicit, but not implicit, costs.

Answer: A

Type: D Topic: 1 E: 393 MI: 149

7. Implicit and explicit costs are different in that:

A) explicit costs are relevant only in the short run.

B) implicit costs are relevant only in the short run.

C) the latter refer to nonexpenditure costs and the former to out-of-pocket costs.

D) the former refer to nonexpenditure costs and the latter to out-of-pocket costs.

Answer: D

Type: D Topic: 1 E: 393 MI: 149

Page 124: Microeconomics Study Guide

8. Implicit costs are:

A) regarded as costs by accountants but not by economists.

B) payments that a firm makes to other firms or individuals who supply resources to it.

C) nonexpenditure costs.

D) costs that vary proportionately with output.

Answer: C

Type: D Topic: 1 E: 393 MI: 149

9. An explicit cost is:

A) omitted when accounting profits are calculated.

B) a money payment made for resources not owned by the firm itself.

C) an implicit cost to the resource owner who receives that payment.

D) always in excess of a resource's opportunity cost.

Answer: B

Page 125: Microeconomics Study Guide

Profits

Type: A Topic: 2 E: 393 MI: 149

10. Accounting profits are typically:

A) greater than economic profits because the former do not take explicit costs into account.

B) equal to economic profits because accounting costs include all opportunity costs.

C) smaller than economic profits because the former do not take implicit costs into account.

D) greater than economic profits because the former do not take implicit costs into account.

Answer: D

Type: A Topic: 2 E: 393 MI: 149

11. Economic profits are calculated by subtracting:

A) explicit costs from total revenue. C) implicit costs from normal profits.

B) implicit costs from total revenue. D) explicit and implicit costs from total revenue.

Answer: D

Type: D Topic: 2 E: 393 MI: 149

12. Normal profit is:

A) determined by subtracting implicit costs from total revenue.

B) determined by subtracting explicit costs from total revenue.

C) the return to the entrepreneur when economic profits are zero.

D) the average profitability of an industry over the preceding 10 years.

Answer: C

Type: A Topic: 2 E: 393 MI: 149

13. Which of the following definitions is correct?

A) Accounting profit + economic profit = normal profit.

B) Economic profit - accounting profit = explicit costs.

C) Economic profit = accounting profit - implicit costs.

D) Economic profit - implicit costs = accounting profits.

Page 126: Microeconomics Study Guide

Answer: C

Type: A Topic: 2 E: 393 MI: 149

14. Suppose that a business incurred implicit costs of $200,000 and explicit costs of $1 million in a specific year. If the firm sold 4,000 units of its output at $300 per unit, its accounting profits were:

A) $100,000 and its economic profits were zero.

B) $200,000 and its economic profits were zero.

C) $100,000 and its economic profits were $100,000.

D) zero and its economic loss was $200,000.

Answer: B

Type: A Topic: 2 E: 393 MI: 149

15. Suppose that a business incurred implicit costs of $500,000 and explicit costs of $5 million in a specific year. If the firm sold 100,000 units of its output at $50 per unit, its accounting:

A) profits were $100,000 and its economic profits were zero.

B) losses were $500,000 and its economic losses were zero.

C) profits were $500,000 and its economic profits were $1 million.

D) profits were zero and its economic losses were $500,000.

Answer: D

Page 127: Microeconomics Study Guide

Use the following to answer questions 16-23:

Use the following cost information for the Creamy Crisp Donut Company to answer questions 16-23:

Entrepreneur's potential earnings as a salaried worker = $50,000

Annual lease on building = $22,000

Annual revenue from operations = $380,000

Payments to workers = $120,000

Utilities (electricity, water, disposal) costs = $8,000

Entrepreneur's potential economic profit from the next best entrepreneurial activity = $80,000

Entrepreneur's forgone interest on personal funds used to finance the business = $6,000

Type: A Topic: 2 E: 393 MI: 149

16. Refer to the above data. Creamy Crisp's explicit costs are:

A) $286,000. B) $150,000. C) $94,000. D) $156,000.

Answer: B

Type: A Topic: 2 E: 393 MI: 149

17. Refer to the above data. Creamy Crisp's implicit costs, including a normal profit are:

A) $136,000. B) $150,000. C) $94,000. D) $156,000.

Answer: A

Type: A Topic: 2 E: 393 MI: 149

18. Refer to the above data. Creamy Crisp's total economic costs (explicit + implicit costs, including a normal profit) are:

A) $286,000. B) $150,000. C) $94,000. D) $156,000.

Answer: A

Type: A Topic: 2 E: 393 MI: 149

19. Refer to the above data. Creamy Crisp's accounting profit is:

A) $150,000. B) $380,000. C) $230,000. D) $294,000.

Page 128: Microeconomics Study Guide

Answer: C

Type: A Topic: 2 E: 393 MI: 149

20. Refer to the above data. Creamy Crisp's economic profit is:

A) $150,000. B) $80,000. C) $230,000. D) $94,000.

Answer: D

Type: A Topic: 2 E: 393 MI: 149

21. Refer to the above data. Creamy Crisp's total revenues exceed its total costs, including a normal profit, by:

A) $150,000. B) $94,000. C) $80,000. D) $230,000.

Answer: B

Page 129: Microeconomics Study Guide

Type: A Topic: 2 E: 393 MI: 149

22. Refer to the above data. Creamy Crisp:

A) has lower implicit costs, including a normal profit, than its explicit costs.

B) is earning a normal profit but not an economic profit.

C) is earning an economic profit.

D) is suffering an economic loss, when implicit costs are considered.

Answer: C

Type: A Topic: 2 E: 393 MI: 149

23. Refer to the above data. If, other things equal, Creamy Crisp's revenue fell to $286,000:

A) its implicit costs, including a normal profit, would exceed its explicit costs.

B) it would earn a normal profit but not an economic profit.

C) it would suffer an economic loss.

D) its accounting profit would fall to zero.

Answer: B

Short run versus long run

Type: A Topic: 3 E: 394 MI: 150

24. The basic characteristic of the short run is that:

A) barriers to entry prevent new firms from entering the industry.

B) the firm does not have sufficient time to change the size of its plant.

C) the firm does not have sufficient time to cut its rate of output to zero.

D) a firm does not have sufficient time to change the amounts of any of the resources it employs.

Answer: B

Type: A Topic: 3 E: 394 MI: 150

25. Which of the following represents a long-run adjustment?

A) a farmer uses an extra dose of fertilizer on his corn crop

B) unable to meet foreign competition, a U.S. watch manufacturer sells one of its branch plants

Page 130: Microeconomics Study Guide

C) a steel manufacturer cuts back on its purchases of coke and iron ore

D) a supermarket hires four additional clerks

Answer: B

Type: A Topic: 3 E: 394 MI: 150

26. Which of the following is a short-run adjustment?

A) A local bakery hires two additional bakers.

B) Six new firms enter the plastics industry.

C) The number of farms in the United States declines by 5 percent.

D) BMW constructs a new assembly plant in South Carolina.

Answer: A

Type: D Topic: 3 E: 394 MI: 150

27. To economists the main difference between the short run and the long run is that:

A) the law of diminishing returns applies in the long run, but not in the short run.

B) in the long run all resources are variable, while in the short run at least one resource is fixed.

C) fixed costs are more important to decision making in the long run than they are in the short run.

D) in the short run all resources are fixed, while in the long run all resources are variable.

Answer: B

Page 131: Microeconomics Study Guide

Type: A Topic: 3 E: 394 MI: 150

28. The amount of calendar time associated with the long run:

A) is less than that associated with the immediate market period.

B) varies from industry to industry.

C) is the same for all firms.

D) is one year by definition.

Answer: B

Type: A Topic: 3 E: 394 MI: 150

29. The basic difference between the short run and the long run is that:

A) all costs are fixed in the short run, but all costs are variable in the long run.

B) the law of diminishing returns applies in the long run, but not in the short run.

C) at least one resource is fixed in the short run, while all resources are variable in the long run.

D) economies of scale may be present in the short run, but not in the long run.

Answer: C

Type: A Topic: 3 E: 394 MI: 150

30. The short run is characterized by:

A) plenty of time for firms to either enter or leave the industry.

B) increasing, but not diminishing returns.

C) at least one fixed resource.

D) zero fixed costs.

Answer: C

Type: A Topic: 3 E: 394 MI: 150

31. The long run is characterized by:

A) the relevance of the law of diminishing returns.

B) at least one fixed input.

C) insufficient time for firms to enter or leave the industry.

D) the ability of the firm to change its plant size.

Answer: D

Page 132: Microeconomics Study Guide

Law of diminishing returns

Type: D Topic: 4 E: 395 MI: 151

32. Marginal product is:

A) the increase in total output attributable to the employment of one more worker.

B) the increase in total revenue attributable to the employment of one more worker.

C) the increase in total cost attributable to the employment of one more worker.

D) total product divided by the number of workers employed.

Answer: A

Page 133: Microeconomics Study Guide

Type: D Topic: 4 E: 395 MI: 151

33. The law of diminishing returns indicates that:

A) as extra units of a variable resource are added to a fixed resource, marginal product will decline beyond some point.

B) because of economies and diseconomies of scale a competitive firm's long-run average total cost curve will be U-shaped.

C) the demand for goods produced by purely competitive industries is downsloping.

D) beyond some point the extra utility derived from additional units of a product will yield the consumer smaller and smaller extra amounts of satisfaction.

Answer: A

Type: A Topic: 4 E: 395-396 MI: 151-152

34. Which of the following statements concerning the relationships between total product (TP), average product (AP), and marginal product (MP) is not correct?

A) AP continues to rise so long as TP is rising.

B) AP reaches a maximum before TP reaches a maximum.

C) TP reaches a maximum when the MP of the variable input becomes zero.

D) MP cuts AP at the maximum AP.

Answer: A

Type: D Topic: 4 E: 396 MI: 152

35. Which of the following best expresses the law of diminishing returns?

A) Because large-scale production allows the realization of economies of scale, the real costs of production vary directly with the level of output.

B) Population growth automatically adjusts to that level at which the average product per worker will be at a maximum.

C) As successive amounts of one resource (labor) are added to fixed amounts of other resources (property), beyond some point the resulting extra output will decline.

D) Proportionate increases in the inputs of all resources will result in a less-than-proportionate increase in total output.

Answer: C

Use the following to answer questions 36-38:

Page 134: Microeconomics Study Guide

Answer the next question(s) on the basis of the following output data for a firm. Assume that the amounts of all nonlabor resources are fixed.

N u m b er U n its o f o f w o rk ers o u tp u t

0 0 1 4 0 2 9 0 3 1 2 6 4 1 5 0 5 1 6 5 6 1 8 0

Type: T Topic: 4 E: 396 MI: 152

36. Refer to the above data. Diminishing marginal returns become evident with the addition of the:

A) sixth worker. B) fourth worker. C) third worker. D) second worker.

Answer: C

Page 135: Microeconomics Study Guide

Type: T Topic: 4 E: 396 MI: 152

37. Refer to the above data. The marginal product of the sixth worker is:

A) 180 units of output. B) 30 units of output. C) 15 units of output. D) negative.

Answer: C

Type: T Topic: 4 E: 396 MI: 152

38. Refer to the above data. Average product is at a maximum when:

A) five workers are hired. C) three workers are hired.

B) four workers are hired. D) two workers are hired.

Answer: D

Type: A Topic: 4 E: 395 MI: 151

39. Marginal product:

A) diminishes at all levels of production.

B) may initially increase, then diminish, but never become negative.

C) may initially increase, then diminish, and ultimately become negative.

D) is always less than average product.

Answer: C

Type: A Topic: 4 E: 395-396 MI: 151-152

40. The first, second, and third workers employed by a firm add 24, 18, and 9 units to total product respectively. Therefore, the:

A) marginal product of the third worker is 9. C) average product of the three workers is 18.

B) total product of the three workers is 54. D) marginal product of the second worker is 18.

Answer: A

Type: D Topic: 4 E: 395 MI: 151

41. If a variable input is added to some fixed input, beyond some point the resulting extra output will decline. This statement describes:

A) economies and diseconomies of scale. C) the law of diminishing returns.

B) X-inefficiency. D) the law of diminishing marginal utility.

Page 136: Microeconomics Study Guide

Answer: C

Type: A Topic: 4 E: 395 MI: 151

42. If in the short run a firm's total product is increasing, then its:

A) marginal product must also be increasing.

B) marginal product must be decreasing.

C) marginal product could be either increasing or decreasing.

D) average product must also be increasing.

Answer: C

Type: A Topic: 4 E: 396-397 MI: 152-153

43. The law of diminishing returns results in:

A) an eventually rising marginal product curve.

B) a total product curve that eventually increases at a decreasing rate.

C) an eventually falling marginal cost curve.

D) a total product curve that rises indefinitely.

Answer: B

Page 137: Microeconomics Study Guide

Type: A Topic: 4 E: 395 MI: 151

44. The law of diminishing returns describes the:

A) relationship between total costs and total revenues.

B) profit-maximizing position of a firm.

C) relationship between resource inputs and product outputs in the short run.

D) relationship between resource inputs and product outputs in the long run.

Answer: C

Type: A Topic: 4 E: 396 MI: 152

45. Which of the following is correct?

A) When total product is rising, both average product and marginal product must also be rising.

B) When marginal product is falling, total product must be falling.

C) When marginal product is falling, average product must also be falling.

D) Marginal product rises faster than average product and also falls faster than average product.

Answer: D

Type: A Topic: 4 E: 396 MI: 152

46. Which of the following is not correct?

A) Where marginal product is greater than average product, average product is rising.

B) Where total product is at a maximum, average product is also at a maximum.

C) Where marginal product is zero, total product is at a maximum.

D) Marginal product becomes negative before average product becomes negative.

Answer: B

Use the following to answer questions 47-48:

Page 138: Microeconomics Study Guide

Pro

du

ct

Va r ia b le in p u t1

2

3

Type: G Topic: 4 E: 397 MI: 153

47. In the above diagram curves 1, 2, and 3 represent the:

A) average, marginal, and total product curves respectively.

B) marginal, average, and total product curves respectively.

C) total, average, and marginal product curves respectively.

D) total, marginal, and average product curves respectively.

Answer: B

Page 139: Microeconomics Study Guide

Type: G Topic: 4 E: 397 MI: 153

48. The above diagram suggests that:

A) when marginal product is zero, total product is at a minimum.

B) when marginal product lies above average product, average product is rising.

C) when marginal product lies below average product, average product is rising.

D) when total product is at a maximum, so is marginal product and average product.

Answer: B

Type: G Topic: 4 E: 397 MI: 153

49. The total output of a firm will be at a maximum where:

A) MP is at a maximum. B) AP is at a minimum. C) MP is zero. D) AP is at a maximum.

Answer: C

Use the following to answer questions 50-51:

Answer the next question(s) on the basis of the following information:

N u m b er o f To ta l M arg in a l w o rk ers p rod u ct p rod u ct

0 0 -- 1 8 8 2 1 0 3 2 5 4 3 0 5 3 6 3 4

Type: T Topic: 4 E: 396 MI: 152

50. Refer to the above data. When two workers are employed:

A) total product is 20.

B) total product is 18.

C) average product is 10.

D) total product cannot be determined from the information given.

Answer: B

Page 140: Microeconomics Study Guide

Type: T Topic: 4 E: 396 MI: 152

51. Refer to the above data. The marginal product of the fourth worker:

A) is 5. B) is 7. C) is 71/2. D) cannot be calculated from the information given.

Answer: A

Page 141: Microeconomics Study Guide

Type: G Topic: 4 E: 397 MI: 153

52.

M a rg in a l p ro d u ct

In p u ts o f la b or

Av era g e p rod u ct

Q 1 Q 2 Q 30M

argi

nal

and

aver

age

pro

du

ct

In the above diagram the range of diminishing marginal returns is:

A) 0Q3. B) 0Q2. C) Q1Q2. D) Q1Q3.

Answer: D

Use the following to answer questions 53-55:

Use the following data to answer the next question(s):

In p u ts To ta l o f lab o r p ro d u ct

0 0 1 8 2 1 8 3 2 5 4 3 0 5 3 3 6 3 4 7 3 2

Type: T Topic: 4 E: 397 MI: 153

53. Refer to the above data. When total product is increasing at an increasing rate, marginal product is:

A) positive and increasing. B) positive and decreasing. C) constant. D) negative.

Answer: A

Type: T Topic: 4 E: 397 MI: 153

54. Refer to the above data. When total product is increasing at a decreasing rate, marginal product is:

Page 142: Microeconomics Study Guide

A) positive and increasing. B) positive and decreasing. C) constant. D) negative.

Answer: B

Type: T Topic: 4 E: 397 MI: 153

55. Refer to the above data. When total product is diminishing, marginal product is:

A) positive and increasing. B) positive and decreasing. C) constant. D) negative.

Answer: D

Page 143: Microeconomics Study Guide

Short-run costs

Type: D Topic: 5 E: 398 MI: 154

56. Fixed cost is:

A) the cost of producing one more unit of capital, say, machinery.

B) any cost which does not change when the firm changes its output.

C) average cost multiplied by the firm's output.

D) usually zero in the short run

Answer: B

Type: A Topic: 5 E: 398 MI: 154

57. Which of the following is most likely to be a fixed cost?

A) shipping charges C) wages for unskilled labor

B) property insurance premiums D) expenditures for raw materials

Answer: B

Type: A Topic: 5 E: 398 MI: 154

58. If you owned a small farm, which of the following would be a fixed cost?

A) harvest labor B) hail insurance C) fertilizer D) seed

Answer: B

Type: A Topic: 5 E: 398 MI: 154

59. Which of the following is most likely to be a variable cost?

A) fuel and power payments C) rental payments on IBM equipment

B) interest on business loans. D) real estate taxes

Answer: A

Type: A Topic: 5 E: 398 MI: 154

60. If you operated a small bakery, which of the following would be a variable cost in the short run?

Page 144: Microeconomics Study Guide

A) baking ovens C) annual lease payment for use of the building

B) interest on business loans D) baking supplies (flour, salt, etc.)

Answer: D

Type: D Topic: 5 E: 400 MI: 156

61. Marginal cost is the:

A) rate of change in total fixed cost that results from producing one more unit of output.

B) change in total cost that results from producing one more unit of output.

C) change in average variable cost that results from producing one more unit of output.

D) change in average total cost that results from producing one more unit of output.

Answer: B

Type: A Topic: 5 E: 400 MI: 156

62. For most producing firms:

A) marginal cost rises as output is carried to a certain level, and then begins to decline.

B) total costs rise as output is carried to a certain level, and then begin to decline.

C) average total costs decline as output is carried to a certain level, and then begin to rise.

D) average total costs rise as output is carried to a certain level, and then begin to decline.

Answer: C

Page 145: Microeconomics Study Guide

Type: A Topic: 5 E: 399 MI: 155

63. Average fixed cost:

A) equals marginal cost when average total cost is at its minimum.

B) may be found for any output by adding average variable cost and average total cost.

C) graphs as a U-shaped curve.

D) declines continually as output increases.

Answer: D

Type: A Topic: 5 E: 402 MI: 158

64. Which of the following is correct as it relates to cost curves?

A) Average variable cost intersects marginal cost at the latter's minimum point.

B) Marginal cost intersects average total cost at the latter's minimum point.

C) Average fixed cost intersects marginal cost at the latter's minimum point.

D) Marginal cost intersects average fixed cost at the latter's minimum point.

Answer: B

Use the following to answer questions 65-70:

Type: G Topic: 5 E: 399-400 MI: 155-156

65. Refer to the above diagram. At output level Q total variable cost is:

A) 0BEQ. B) BCDE. C) 0CDQ. D) 0AFQ.

Answer: A

Page 146: Microeconomics Study Guide

Type: G Topic: 5 E: 399-400 MI: 155-156

66. Refer to the above diagram. At output level Q total fixed cost is:

A) 0BEQ. B) BCDE. C) 0BEQ-0AFQ. D) 0CDQ.

Answer: B

Type: G Topic: 5 E: 399-400 MI: 155-156

67. Refer to the above diagram. At output level Q total cost is:

A) 0BEQ. B) BCDE. C) 0BEQ plus BCDE. D) 0AFQ plus BCDE.

Answer: C

Page 147: Microeconomics Study Guide

Type: G Topic: 5 E: 399 MI: 155

68. Refer to the above diagram. At output level Q average fixed cost:

A) is equal to EF.

B) is equal to QE.

C) is measured by both QF and ED.

D) cannot be determined from the information given.

Answer: C

Type: G Topic: 5 E: 401 MI: 157

69. Refer to the above diagram. At output level Q:

A) marginal product is falling.

B) marginal product is rising.

C) marginal product is negative.

D) one cannot determine whether marginal product is falling or rising.

Answer: A

Type: G Topic: 5 E: 399-400 MI: 155-156

70. Refer to the above diagram. The vertical distance between ATC and AVC reflects:

A) the law of diminishing returns. C) marginal cost at each level of output.

B) the average fixed cost at each level of output. D) the presence of economies of scale.

Answer: B

Type: A Topic: 5 E: 402 MI: 158

71. Marginal cost:

A) equals both average variable cost and average total cost at their respective minimums.

B) is the difference between total cost and total variable cost.

C) rises for a time, but then begins to decline when diminishing returns set in.

D) declines continuously as output increases.

Answer: A

Page 148: Microeconomics Study Guide

Type: A Topic: 5 E: 399-400 MI: 155-156

72. When average fixed costs are falling:

A) average total cost must be falling.

B) average variable cost may be either rising or falling.

C) marginal cost must be falling.

D) average variable costs must be rising.

Answer: B

Type: A Topic: 5 E: 400 MI: 156

73. Which of the following statements is correct?

A) Average total cost is the difference between average variable cost and average fixed cost.

B) Marginal cost measures the cost per unit of output associated with any level of production.

C) When marginal product rises, marginal cost must also rise.

D) Marginal cost is the price or cost of an extra variable input (for example, an additional worker) divided by its marginal product.

Answer: D

Page 149: Microeconomics Study Guide

Type: A Topic: 5 E: 400 MI: 156

74. Assume that in the short run a firm is producing 100 units of output, has average total costs of $200, and average variable costs of $150. The firm's total fixed costs are:

A) $5,000. B) $500. C) $.50. D) $50.

Answer: A

Type: A Topic: 5 E: 400 MI: 156

75. If average total cost is declining, then:

A) marginal cost must be greater than average total cost.

B) the average fixed cost curve must lie above the average variable cost curve.

C) marginal cost must be less than average total cost.

D) total cost must also be declining.

Answer: C

Type: A Topic: 5 E: 401 MI: 157

76. The relationship between the marginal cost and the average total cost schedule is such that:

A) the behavior of one schedule does not affect the other.

B) if ATC exceeds MC, MC must be rising.

C) if MC is declining, ATC may be either declining or rising.

D) if MC is declining, ATC must also be declining.

Answer: D

Type: A Topic: 5 E: 402 MI: 158

77. Other things equal, if the prices of a firm's variable inputs were to fall:

A) one could not predict how unit costs of production would be affected.

B) marginal cost, average variable cost, and average fixed cost would all fall.

C) marginal cost, average variable cost, and average total cost would all fall.

D) average variable cost would fall, but marginal cost would be unchanged.

Answer: C

Page 150: Microeconomics Study Guide

Type: A Topic: 5 E: 398-400 MI: 154-156

78. Other things equal, if the fixed costs of a firm were to increase by $100,000 per year, which of the following would happen?

A) Marginal costs and average variable costs would both rise.

B) Average fixed costs and average variable costs would rise.

C) Average fixed costs and average total costs would rise.

D) Average fixed costs would rise, but marginal costs would fall.

Answer: C

Type: A Topic: 5 E: 398 MI: 154

79. If a firm decides to produce no output in the short run, its costs will be:

A) its marginal costs. B) its fixed plus its variable costs. C) its fixed costs. D) zero.

Answer: C

Page 151: Microeconomics Study Guide

Use the following to answer questions 80-84:

Answer the next question(s) on the basis of the following cost data:

To ta l O u tp u t cost

0 $ 2 4 1 3 3 2 4 1 3 4 8 4 5 4 5 6 1 6 6 9

Type: T Topic: 5 E: 398 MI: 154

80. Refer to the above data. The total variable cost of producing 5 units is:

A) $61. B) $48. C) $37. D) $24.

Answer: C

Type: T Topic: 5 E: 400 MI: 156

81. Refer to the above data. The average total cost of producing 3 units of output is:

A) $14. B) $12. C) $13.50. D) $16.

Answer: D

Type: T Topic: 5 E: 399 MI: 155

82. Refer to the above data. The average fixed cost of producing 3 units of output is:

A) $8. B) $7.40. C) $5.50. D) $6.

Answer: A

Type: T Topic: 5 E: 400 MI: 156

83. Refer to the above data. The marginal cost of producing the sixth unit of output is:

A) $24. B) $12. C) $16. D) $8.

Answer: D

Page 152: Microeconomics Study Guide

Type: T Topic: 5 E: 400 MI: 156

84. Refer to the above data. The profit-maximizing output for this firm:

A) is 3. B) is 4. C) is 5. D) cannot be determined from the information given.

Answer: D

Type: A Topic: 5 E: 398 MI: 154

85. In comparing the changes in TVC and TC associated with an additional unit of output, we find that:

A) no generalization about the changes in TC and TVC can be made.

B) the changes in TC and TVC are equal.

C) the change in TC is greater than the change in TVC.

D) the change in TVC is greater than the change in TC.

Answer: B

Page 153: Microeconomics Study Guide

Use the following to answer questions 86-89:

Answer the next question(s) on the basis of the following information:

TFC = total fixed cost Q = quantity of output

MC = marginal cost P = product price

TVC = total variable cost

Type: E Topic: 5 E: 399 MI: 155

86. Refer to the above information. Average fixed cost is:

A) TVC - MC

B) MC

Q

C) TFC

Q

D) TVC

Q

Answer: C

Type: E Topic: 5 E: 400 MI: 156

87. Refer to the above information. Average total cost is:

A) TVC - MC

B) TVC - TFC

Q

C) TVC

Q

D) TFC + TVC

Q

Answer: D

Type: E Topic: 5 E: 400 MI: 156

Page 154: Microeconomics Study Guide

88. Refer to the above information. Marginal cost is:

A) change in TVC

Q

B) change in TVC

change in Q

C) P•Q

change in Q

D) change in TFC

change in Q

Answer: D

Type: E Topic: 5 E: 398 MI: 154

89. Refer to the above information. Total cost is:

A) the change in marginal cost.

B) TVC - TFC

C) TFC + TVC

D) TFC + TVC

Q

Answer: C

Page 155: Microeconomics Study Guide

Use the following to answer questions 90-92:

M a rg in a l p ro d u ct

In p u ts o f la b o r

Av era g e p ro d u ct

Q 1 Q 2 Q 30M

argi

nal

and

aver

age

pro

du

ct

Type: G Topic: 5 E: 402 MI: 158

90. Refer to the above diagram, where variable inputs of labor are being added to a constant amount of property resources. The total output of this firm will cease to expand:

A) if a labor force in excess of Q1 is employed.

B) if a labor force in excess of Q2 is employed.

C) if a labor force in excess of Q3 is employed.

D) only if the marginal product curve becomes negative at all levels of output.

Answer: C

Type: G Topic: 5 E: 402 MI: 158

91. Refer to the above diagram, where variable inputs of labor are being added to a constant amount of property resources. Marginal cost will be at a minimum for this firm when it is hiring:

A) Q3 workers. B) Q2 workers. C) Q1 workers. D) more than Q3 workers.

Answer: C

Type: G Topic: 5 E: 402 MI: 158

92. Refer to the above diagram, where variable inputs of labor are being added to a constant amount of property resources. Average variable cost will be at a minimum when the firm is hiring:

A) Q3 workers. B) Q2 workers. C) Q1 workers. D) more than Q3 workers.

Answer: B

Page 156: Microeconomics Study Guide

Type: G Topic: 5 E: 402 MI: 158

93.

In the above figure, curves 1, 2, 3, and 4 represent the:

A) ATC, MC, AFC, and AVC curves respectively. C) MC, ATC, AVC, and AFC curves respectively.

B) AFC, MC, AVC, and ATC curves respectively. D) ATC, AVC, AFC, and MC curves respectively.

Answer: C

Type: A Topic: 5 E: 403 MI: 159

94. If a technological advance reduces the amount of variable resources needed to produce any level of output, then the:

A) AVC curve will shift upward. C) ATC curve will shift upward.

B) MC curve will shift downward. D) AFC curve will shift downward.

Answer: B

Type: A Topic: 5 E: 398-399 MI: 154-155

95. In the short run which of the following statements is correct?

A) The marginal cost curve intersects the average variable and average fixed cost curves at their minimum points.

B) Average variable cost declines continuously as total output is expanded.

C) Total cost will exceed variable cost.

D) If the inputs of all resources are increased by equal amounts, total output will expand by diminishing amounts.

Answer: C

Type: A Topic: 5 E: 398 MI: 154

Page 157: Microeconomics Study Guide

96. Total fixed cost (TFC):

A) falls as the firm expands output from zero, but eventually rises.

B) falls continuously as total output expands.

C) varies directly with total output.

D) does not change as total output increases or decreases.

Answer: D

Page 158: Microeconomics Study Guide

Type: A Topic: 5 E: 398 MI: 154

97. Fixed costs are associated with:

A) highly adjustable inputs such as labor. C) the short run only.

B) both the short run and the long run. D) the long run only.

Answer: C

Type: G Topic: 5 E: 399 MI: 155

98.

In the above diagram curves 1, 2, and 3 represent:

A) average variable cost, marginal cost, and average fixed cost respectively.

B) total variable cost, total fixed cost, and total cost respectively.

C) total fixed cost, total variable cost, and total cost respectively.

D) marginal product, average variable cost, and average total cost respectively.

Answer: C

Type: A Topic: 5 E: 401-402 MI: 157-158

99. Which of the following is correct?.

A) There is no relationship between MP and MC.

B) When AP is rising MC is falling, and when AP is falling MC is rising.

C) When MP is rising MC is rising, and when MP is falling MC is falling.

D) When MP is rising MC is falling, and when MP is falling MC is rising.

Answer: D

Page 159: Microeconomics Study Guide

Type: A Topic: 5 E: 399 MI: 155

100. Which of the following curves is not U-shaped?

A) MC B) AFC C) AVC D) ATC

Answer: B

Type: A Topic: 5 E: 400 MI: 156

101. If a firm wanted to know how much it would save by producing one less unit of output, it would look to:

A) MC. B) ATC. C) AVC. D) AFC.

Answer: A

Page 160: Microeconomics Study Guide

Type: A Topic: 5 E: 402 MI: 158

102. Which of the following holds true?

A) There is no relationship between AP and AVC.

B) When MP is rising AVC is falling, and when MP is falling AVC is rising.

C) When AP is rising AVC is falling, and when AP is falling AVC is rising.

D) When AP is rising AVC is rising, and when AP is falling AVC is falling.

Answer: C

Type: A Topic: 5 E: 399 MI: 155

103. In the short run it is impossible for an expansion of output to increase:

A) average total cost. B) average fixed cost. C) marginal cost. D) average variable cost.

Answer: B

Type: A Topic: 5 E: 400 MI: 156

104. Average fixed costs can be determined graphically by:

A) summing the marginal costs of any number of units of output and dividing the sum by that output.

B) the vertical distance between TC and TVC.

C) the vertical distance between AVC and MC.

D) the vertical distance between ATC and AVC.

Answer: D

Type: A Topic: 5 E: 398-399 MI: 154-155

105. The vertical distance between the total cost and the total variable cost curves differs by an amount which:

A) initially increases, but then decreases, as output increases.

B) is constant as output changes.

C) decreases as output increases.

D) increases as output increases.

Answer: B

Type: A Topic: 5 E: 400 MI: 156

Page 161: Microeconomics Study Guide

106. The vertical distance between a firm's ATC and AVC curves represents:

A) AFC, which increases as output increases.

B) AFC, which decreases as output increases.

C) marginal costs, which decrease as output decreases.

D) marginal costs, which increase as output increases.

Answer: B

Type: A Topic: 5 E: 400 MI: 156

107. If a profitable firm's fixed costs somehow were zero:

A) MC and ATC would be equal at all levels of output.

B) AFC would become negative as output increases.

C) AVC and ATC would coincide.

D) ATC would be zero at all output levels.

Answer: C

Page 162: Microeconomics Study Guide

Use the following to answer questions 108-111:

Type: G Topic: 5 E: 402 MI: 158

108. Refer to the above short-run production and cost data. In Figure A curve (1) is:

A) total product and curve (2) is average product.

B) total product and curve (2) is marginal product.

C) average product and curve (2) is marginal product.

D) marginal product and curve (2) is average product.

Answer: C

Type: G Topic: 5 E: 402 MI: 158

109. Refer to the above short-run production and cost data. In Figure B curve (3) is:

A) AVC and curve (4) is MC. C) MC and curve (4) is AFC.

B) MC and curve (4) is AVC. D) AFC and curve (4) is MC.

Answer: B

Type: G Topic: 5 E: 402 MI: 158

Page 163: Microeconomics Study Guide

110. Refer to the above short-run production and cost data. The curves of Figures A and B suggest that:

A) marginal product and marginal cost reach their maximum points at the same output.

B) marginal cost reaches a minimum where marginal product is at its maximum.

C) marginal cost and marginal product reach their minimum points at the same output.

D) AVC cuts MC at the latter's minimum point.

Answer: B

Page 164: Microeconomics Study Guide

Type: G Topic: 5 E: 402 MI: 158

111. Refer to the above short-run production and cost data. The curves of Figures A and B suggest that:

A) average product and average variable cost reach their maximum points at the same output.

B) AVC cuts MC at the latter's maximum point.

C) AVC reaches a minimum where AP is at its maximum.

D) AFC declines so long as output increases.

Answer: C

Type: A Topic: 5 E: 398-399 MI: 154-155

112. In the short run:

A) TVC will increase for a time at a diminishing rate, but then beyond some point will increase at an increasing rate.

B) TVC will increase for a time at an increasing rate, but then beyond some point will increase at a diminishing rate.

C) TVC will increase by the same absolute amount for each additional unit of output produced.

D) one cannot generalize concerning the behavior of TVC as output increases.

Answer: A

Type: A Topic: 5 E: 398-399 MI: 154-155

113. Total cost minus total variable cost equals:

A) average fixed cost. B) total fixed cost. C) average variable cost. D) marginal cost.

Answer: B

Type: A Topic: 5 E: 398-399 MI: 154-155

114. As output increases, total variable cost:

A) increases more rapidly than does total cost.

B) increases continuously at a decreasing rate.

C) increases at a decreasing rate and then at an increasing rate.

D) increases at a constant rate.

Answer: C

Type: A Topic: 5 E: 399 MI: 155

Page 165: Microeconomics Study Guide

115. In the short run the Sure-Screen T-Shirt Company is producing 500 units of output. Its average variable costs are $2.00 and its average fixed costs are $.50. The firm's total costs:

A) are $2.50. B) are $1250. C) are $750. D) are $1100.

Answer: B

Type: A Topic: 5 E: 402 MI: 158

116. Because the marginal product of a variable resource at first increases and then decreases as the output of the firm is increased:

A) total cost at first increases at a decreasing rate and then increases at an increasing rate.

B) total variable cost at first increases at an increasing rate and then increases at a decreasing rate.

C) average total cost at first increases and then diminishes.

D) average fixed cost will rise beyond the point of diminishing returns.

Answer: A

Page 166: Microeconomics Study Guide

Type: A Topic: 5 E: 399 MI: 155

117. Suppose that, when producing 10 units of output, a firm's AVC is $22, its AFC is $5, and its MC is $30. This:

A) firm's ATC is $35. C) firm's total cost is $270.

B) firm's ATC is $57. D) firm's total cost is $30.

Answer: C

Type: A Topic: 5 E: 399-400 MI: 155-156

118. The relationship between marginal cost and average fixed cost is such that:

A) declines in MC cause AFC to decline as output increases.

B) increases in MC cause AFC to increase as output increases.

C) MC intersects AFC at that output where AFC is at a minimum.

D) MC may either rise or fall as AFC declines.

Answer: D

Type: A Topic: 5 E: 400 MI: 156

119. In comparing the changes in TC and TVC associated with an additional unit of output, we find that:

A) the change in TVC is equal to MC, while the change in TC is equal to TFC.

B) the change in TC exceeds the change in TVC.

C) the change in TVC exceeds the change in TC.

D) both are equal to MC.

Answer: D

Type: A Topic: 5 E: 402 MI: 158

120. Which of the following is correct?

A) When AP is rising, AVC is rising. C) When AP is rising, AP exceeds MP.

B) When AP is rising, AVC is falling. D) There is no relationship between AP and AVC.

Answer: B

Use the following to answer questions 121-127:

Page 167: Microeconomics Study Guide

Answer the next question(s) on the basis of the following cost data:

Average Average fixed var iab le

O u tp u t cost cost 1 $ 5 0 .0 0 $ 1 0 0 .0 0 2 2 5 .0 0 8 0 .0 0 3 1 6 .6 7 6 6 .6 7 4 1 2 .5 0 6 5 .0 0 5 1 0 .0 0 6 8 .0 0 6 8 .3 7 7 3 .3 3 7 7 .1 4 8 0 .0 0 8 6 .2 5 8 7 .5 0

Type: T Topic: 5 E: 399 MI: 155

121. Refer to the above data. Total fixed cost is:

A) $6.25. B) $100.00. C) $150.00. D) $50.00.

Answer: D

Page 168: Microeconomics Study Guide

Type: T Topic: 5 E: 400 MI: 156

122. Refer to the above data. The average total cost of five units of output is:

A) $69. B) $78. C) $3. D) $10. .

Answer: B

Type: T Topic: 5 E: 399-400 MI: 155-156

123. Refer to the above data. The total cost of four units of output is:

A) $260. B) $77.50. C) $310. D) $215.

Answer: C

Type: T Topic: 5 E: 398-399 MI: 154-155

124. Refer to the above data. If the firm closed down and produced zero units of output, its total cost would be:

A) zero. B) $50. C) $150. D) $100.

Answer: B

Type: T Topic: 5 E: 399-400 MI: 155-156

125. Refer to the above data. The marginal cost of the fifth unit of output is:

A) $3. B) $62. C) $80. D) $78.

Answer: C

Type: T Topic: 5 E: 401-402 MI: 157-158

126. Refer to the above data. The marginal cost curve would intersect the average variable cost curve at about:

A) 2 units of output. B) 4 units of output. C) 6 units of output. D) 7 units of output.

Answer: B

Type: T Topic: 5 E: 398-399 MI: 154-155

127. Refer to the above data. If the firm decided to increase its output from 6 to 7 units, its total costs would rise by:

A) $87.14. B) $80.00. C) $6.67. D) $120.00.

Answer: D

Page 169: Microeconomics Study Guide

Type: D Topic: 5 E: 398 MI: 154

128. A fixed cost is:

A) associated with any productive resource whose price is fixed.

B) any cost which increases proportionately with output.

C) any cost which a firm would incur even if output was zero.

D) associated with all inputs whose short-run supply is perfectly inelastic.

Answer: C

Type: A Topic: 5 E: 402 MI: 158

129. Other things equal, if the wage rates paid to a firm's labor inputs were to rise, we would expect the:

A) AFC, AVC, ATC, and MC curves all to rise. C) AFC and ATC curves to fall.

B) AVC, ATC, and MC curves all to rise. D) MP curve to fall.

Answer: B

Page 170: Microeconomics Study Guide

Type: A Topic: 5 E: 403 MI: 159

130. If a technological advance increases a firm's labor productivity, we would expect its:

A) average total cost curve to rise. C) total cost curve to rise.

B) average total cost curve to fall. D) average total cost curve to be unaffected.

Answer: B

Type: A Topic: 5 E: 398 MI: 154

131. As a firm produces successive units of output in the short run we would expect:

A) TVC to increase initially by declining amounts, but eventually increase by increasing amounts.

B) TVC to increase initially by increasing amounts, but eventually by decreasing amounts.

C) TFC to increase by constant amounts.

D) the sum of TVC and TFC to increase initially by increasing amounts, but eventually by decreasing amounts.

Answer: A

Type: A Topic: 5 E: 398 MI: 154

132. Assume a firm closes down in the short run and produces no output. Under these conditions:

A) TVC is positive, but TFC and TC are zero. C) TFC and TC are positive, but TVC is zero.

B) TFC is positive, but TVC and TC are zero. D) TFC, TVC, and TC will all be positive.

Answer: C

Type: A Topic: 5 E: 400-401 MI: 156-157

133. If marginal cost is:

A) falling, then average total cost must also be falling.

B) rising, then average total cost must also be rising.

C) rising, then average total cost could be either falling or rising.

D) falling, then average total cost could be either falling or rising.

Answer: C

Type: A Topic: 5 E: 398-399 MI: 154-155

134. Which of the following is incorrect?

Page 171: Microeconomics Study Guide

A) Total fixed cost does not change with output in the short run.

B) Fixed costs exist only in the short run.

C) Total fixed cost must be added to total variable cost to determine total cost.

D) Total fixed cost equals total variable cost in the long run.

Answer: D

Type: A Topic: 5 E: 399-400 MI: 155-156

135. If the total variable cost of 9 units of output is $90 and the total variable cost of 10 units of output is $120, then:

A) the average variable cost of 10 units is $10.

B) the average variable cost of 9 units is $10.

C) the marginal cost of the tenth unit is $90.

D) the firm is operating in the range of increasing marginal returns.

Answer: B

Page 172: Microeconomics Study Guide

Type: A Topic: 5 E: 398 MI: 154

136. A firm's total variable cost will depend on:

A) the prices of variable resources. C) the level of output.

B) the production techniques that are used. D) all of the above.

Answer: D

Type: A Topic: 5 E: 400 MI: 156

137. The short-run average total cost curve is U-shaped because:

A) average fixed costs decline continuously as output increases.

B) of increasing and diminishing returns.

C) of economies and diseconomies of scale.

D) minimum efficient scale is encountered.

Answer: B

Use the following to answer questions 138-142:

The Sunshine Corporation finds that its costs are $40 when it produces no output. Its total variable costs (TVC) change with output as shown in the accompanying table. Use this information to answer the following

question(s).

O u tp u t T V C 1 $ 3 0 2 5 0 3 6 5 4 8 5 5 11 0

Type: T Topic: 5 E: 398 MI: 154

138. Refer to the above information. The total cost of producing 3 units of output is:

A) $65. B) $105. C) $145. D) $185.

Answer: B

Type: T Topic: 5 E: 400 MI: 156

Page 173: Microeconomics Study Guide

139. Refer to the above information. The average total cost of 3 units of output is:

A) $65. B) $21.67. C) $40. D) $35.

Answer: D

Type: T Topic: 5 E: 399 MI: 155

140. Refer to the above information. The average fixed cost of 3 units of output is:

A) $13.33. B) $12.50. C) $40. D) $18.50.

Answer: A

Type: T Topic: 5 E: 400 MI: 156

141. Refer to the above information. The marginal cost of the third unit of output is:

A) $105. B) $25. C) $15. D) $20.

Answer: C

Page 174: Microeconomics Study Guide

Type: T Topic: 5 E: 393-394 MI: 149-150

142. Refer to the above information. This firm:

A) is making an economic profit of $260. C) may be either realizing a profit or a loss.

B) is realizing a loss of $125. D) is selling its output in a competitive market.

Answer: C

Use the following to answer questions 143-146:

Type: G Topic: 5 E: 399-400 MI: 155-156

143. Refer to the above diagram. This firm's average fixed costs are:

A) not shown. C) the vertical distance between AVC and ATC.

B) the vertical distance between AVC and MC. D) equal to the per unit change in MC.

Answer: C

Type: G Topic: 5 E: 402 MI: 158

144. Refer to the above diagram. If labor is the only variable input, the marginal product of labor is at a:

A) maximum at point a. B) minimum at point a. C) maximum at point b. D) maximum at point c.

Answer: A

Type: G Topic: 5 E: 402 MI: 158

145. Refer to the above diagram. If labor is the only variable input, the average product of labor is at a:

Page 175: Microeconomics Study Guide

A) minimum at point b. B) maximum at point b. C) maximum at point a. D) maximum at point c.

Answer: B

Type: G Topic: 5 E: 393-394 MI: 149-150

146. Refer to the above diagram. The profit-maximizing level of output for this firm:

A) is at point a.

B) is at point b.

C) is at point c.

D) cannot be determined from the information given.

Answer: D

Page 176: Microeconomics Study Guide

Use the following to answer questions 147-150:

Answer the next question(s) on the basis of the accompanying table that shows average total costs (ATC) for a manufacturing firm whose total fixed costs are $10:

O u tp u t AT C 1 $ 4 0 2 2 7 3 2 9 4 3 1 5 3 8

Type: T Topic: 5 E: 398-399 MI: 154-155

147. Refer to the above data. The total cost of producing 4 units of output is:

A) $31. B) $87. C) $124. D) $108.

Answer: C

Type: T Topic: 5 E: 399-400 MI: 155-156

148. Refer to the above data. The average variable cost of 4 units of output is:

A) $33.50. B) $28.50. C) $19.00. D) $21.00.

Answer: B

Type: T Topic: 5 E: 399-400 MI: 155-156

149. Refer to the above data. The marginal cost of the fourth unit of output is:

A) $2. B) $12. C) $37. D) $16.

Answer: C

Type: T Topic: 5 E: 393-394 MI: 149-150

150. Refer to the above data. The profit-maximizing level of output for this firm:

A) is 3. B) is 4. C) is 5. D) cannot be determined from the information given.

Answer: D

Page 177: Microeconomics Study Guide

Use the following to answer questions 151-157:

Q u an tity

Dol

lars

0

ab

c

de

f

AT C 1

Type: G Topic: 5 E: 400 MI: 156 Status: New

151. Refer to the above graph. Which one of the following would cause a move from point b to point c along short-run average total cost curve ATC1?

A) diminishing marginal returns C) a decrease in the wage rate

B) an increase in the wage rate D) increasing marginal returns

Answer: A

Type: G Topic: 5 E: 400 MI: 156 Status: New

152. Refer to the above graph. Which one of the following would cause a move from point d to point e along short-run average total cost curve ATC2?

A) diminishing marginal returns C) a decrease in the wage rate

B) an increase in the wage rate D) increasing marginal returns

Answer: D

Type: G Topic: 5 E: 403 MI: 159 Status: New

153. Refer to the above graph. Which one of the following would cause a move from point b on short-run average total cost curve ATC1 to point e on short-run average cost curve ATC2?

A) diminishing marginal returns C) a decrease in the wage rate

B) an increase in the wage rate D) increasing marginal returns

Answer: B

Type: G Topic: 5 E: 403 MI: 159 Status: New

Page 178: Microeconomics Study Guide

154. Refer to the above graph. Which one of the following would cause a move from point e on short-run average total cost curve ATC2 to point b on short-run average cost curve ATC1?

A) diminishing marginal returns C) a decrease in the wage rate

B) an increase in the wage rate D) increasing marginal returns

Answer: C

Type: G Topic: 5 E: 403 MI: 159 Status: New

155. Refer to the above graph. Which one of the following would cause a move from point e on short-run average total cost curve ATC2 to point b on short-run average cost curve ATC1?

A) a decline in total fixed costs C) an increase in the wage rate

B) diminishing marginal returns D) increasing marginal returns

Answer: A

Page 179: Microeconomics Study Guide

Type: G Topic: 5 E: 400 MI: 156 Status: New

156. Refer to the above graph. Diminishing marginal returns are reflected in:

A) the shift of the of short-run average total cost curve from ATC2 to ATC1.

B) a move along short-run average total cost curve ATC2 from point e to point f.

C) a move along short-run average total cost curve ATC1 from point a to point b.

D) the shift of the short-run average total cost curve from ATC1 to ATC2.

Answer: B

Type: G Topic: 5 E: 403 MI: 159 Status: New

157. Refer to the above graph. A decrease in fixed costs is shown by:

A) a move along short-run average total cost curve ATC2 from point e to point f.

B) a move along short-run average total cost curve ATC1 from point a to point b.

C) the shift of the short-run average total cost curve from ATC1 to ATC2.

D) the shift of the of short-run average total cost curve from ATC2 to ATC1.

Answer: D

Long-run costs

Type: A Topic: 6 E: 404 MI: 160

158. Economies and diseconomies of scale explain:

A) the profit-maximizing level of production.

B) why the firm's long-run average total cost curve is U-shaped.

C) why the firm's short-run marginal cost curve cuts the short-run average variable cost curve at its minimum point.

D) the distinction between fixed and variable costs.

Answer: B

Type: D Topic: 6 E: 403 MI: 159

159. In the long run:

A) all costs are variable costs. C) variable costs equal fixed costs.

Page 180: Microeconomics Study Guide

B) all costs are fixed costs. D) fixed costs are greater than variable costs.

Answer: A

Page 181: Microeconomics Study Guide

Use the following to answer questions 160-164:

Type: G Topic: 6 E: 400 MI: 156 Status: New

160. The above diagram shows the short-run average total cost curves for five different plant sizes of a firm. The shape of each individual curve reflects:

A) increasing returns, followed by diminishing returns.

B) economies of scale, followed by diseconomies of scale.

C) constant costs.

D) increasing costs, followed by decreasing costs.

Answer: A

Type: G Topic: 6 E: 404 MI: 160 Status: New

161. As the firm in the above diagram expands from plant size #1 to plant size #3, it experiences:

A) diminishing returns. B) economies of scale. C) diseconomies of scale. D) constant costs.

Answer: B

Type: G Topic: 6 E: 407 MI: 163 Status: New

162. As the firm in the above diagram expands from plant size #3 to plant size #5, it experiences:

A) increasing returns. B) economies of scale. C) diseconomies of scale. D) constant costs.

Answer: C

Type: G Topic: 6 E: 404 MI: 160

163. The above diagram shows the short-run average total cost curves for five different plant sizes of a firm. The position of these five curves in relation to one another reflects:

Page 182: Microeconomics Study Guide

A) economies and diseconomies of scale.

B) the effect of fixed costs on ATC as output increases.

C) the law of constant costs.

D) the law of diminishing returns.

Answer: A

Type: G Topic: 6 E: 403 MI: 159

164. The above diagram shows the short-run average total cost curves for five different plant sizes of a firm. In the long run the firm should produce output 0x with a plant of size:

A) #4 B) #3. C) #2. D) #1.

Answer: C

Page 183: Microeconomics Study Guide

Type: A Topic: 6 E: 407 MI: 163

165. When diseconomies of scale occur:

A) the long-run average total cost curve falls. C) the long-run average total cost curve rises.

B) marginal cost intersects average total cost. D) average fixed costs will rise.

Answer: C

Type: A Topic: 6 E: 404-406 MI: 160-162

166. Which of the following is not a source of economies of scale?

A) learning-by-doing. C) use of larger machines.

B) labor specialization. D) inelastic resource supply curves.

Answer: D

Type: A Topic: 6 E: 406 MI: 162

167. When a firm does more of something, it gets better at it. This learning-by-doing is:

A) a source of diseconomies of scale. C) called the principle of natural progression.

B) a source of economies of scale. D) called "spreading the overhead."

Answer: B

Use the following to answer questions 168-171:

Use the following data to answer the next question(s). The letters A, B, and C designate three successively larger plant sizes.

O u tp u t AT C -A AT C -B AT C -C 1 0 $ 6 $ 1 3 $ 4 4 2 0 5 9 3 5 3 0 4 6 2 7 4 0 5 4 2 0 5 0 7 3 1 4 6 0 1 0 4 11 7 0 1 4 5 8 8 0 1 9 7 6 9 0 2 5 1 0 5 1 0 0 3 2 1 6 7

Page 184: Microeconomics Study Guide

Type: T Topic: 6 E: 403-404 MI: 159-160

168. Refer to the above data. In the long run the firm should use plant size "A" for:

A) all possible levels of output. C) 30 to 60 units of output.

B) 10 to 30 units of output. D) all outputs greater than 40.

Answer: B

Type: T Topic: 6 E: 403-404 MI: 159-160

169. Refer to the above data. In the long run the firm should use plant size "C" for:

A) all possible levels of output. C) 40 to 70 units of output.

B) 10 to 30 units of output. D) all units of output greater than 80.

Answer: D

Page 185: Microeconomics Study Guide

Type: T Topic: 6 E: 404-405 MI: 160-161

170. Refer to the above data. Economies of scale are realized over the ___ to ___ levels of output; diseconomies of scale exist over the ___ to ___ levels of output.

A) 10, 30; 40, 100 B) 10, 40; 80, 100 C) 10, 50; 60, 100 D) 10, 70; 80, 100

Answer: C

Type: T Topic: 6 E: 407 MI: 163

171. Refer to the above data. At what level of output is minimum efficient scale realized?

A) 30 B) 40 C) 50 D) 60

Answer: C

Type: A Topic: 6 E: 404 MI: 160

172. Economies of scale are indicated by:

A) the rising segment of the average variable cost curve.

B) the declining segment of the long-run average total cost curve.

C) the difference between total revenue and total cost.

D) a rising marginal cost curve.

Answer: B

Type: A Topic: 6 E: 407 MI: 163

173. Diseconomies of scale:

A) pertain to the long run. C) are synonymous with diminishing returns.

B) pertain to the short run. D) are synonymous with increasing returns.

Answer: A

Type: A Topic: 6 E: 403 MI: 159

174. The long-run average total cost curve:

A) will rise if diminishing returns are encountered.

B) will fall if diminishing returns are encountered.

C) will rise if economies of scale are incurred.

Page 186: Microeconomics Study Guide

D) is based on the assumption that all resources are variable.

Answer: D

Type: A Topic: 6 E: 404 MI: 160

175. If a firm doubles its output in the long run and its unit costs of production decline, we can conclude that:

A) technological progress has occurred. C) the firm is encountering diminishing returns.

B) economies of scale are being realized. D) diseconomies of scale are being encountered.

Answer: B

Page 187: Microeconomics Study Guide

Use the following to answer questions 176-178:

Type: G Topic: 6 E: 404 MI: 160

176. Refer to the above diagram. For output level Q, per unit costs of C are:

A) unobtainable and imply the inefficient use of resources.

B) unobtainable, given resource prices and the current state of technology.

C) obtainable, but imply the inefficient use of resources.

D) obtainable and imply that resources are being combined efficiently.

Answer: C

Type: G Topic: 6 E: 404 MI: 160

177. Refer to the above diagram. For output level Q, per unit costs of B are:

A) unobtainable and imply the inefficient use of resources.

B) unobtainable, given resource prices and the current state of technology.

C) obtainable, but imply the inefficient use of resources.

D) obtainable and imply least-cost production of this output.

Answer: D

Type: G Topic: 6 E: 404 MI: 160

178. Refer to the above diagram. For output level Q, per unit costs of A are:

A) unobtainable and imply the inefficient use of resources.

B) unobtainable, given resource prices and the current state of technology.

C) obtainable, but imply the inefficient use of resources.

D) obtainable and imply least-cost production of this output.

Page 188: Microeconomics Study Guide

Answer: B

Type: D Topic: 6 E: 407 MI: 163

179. The minimum efficient scale of a firm:

A) is realized somewhere in the range of diseconomies of scale.

B) occurs where marginal product becomes zero.

C) is in the middle of the range of constant returns to scale.

D) is the smallest level of output at which long-run average total cost is minimized.

Answer: D

Page 189: Microeconomics Study Guide

Type: A Topic: 6 E: 406-407 MI: 162-163

180. If an industry's long-run average total cost curve has an extended range of constant returns to scale, this implies that:

A) technology precludes both economies and diseconomies of scale.

B) the industry will be a natural monopoly.

C) both relatively small and relatively large firms can be viable in the industry.

D) the industry will be comprised of a very large number of small firms.

Answer: C

Type: D Topic: 6 E: 407 MI: 163

181. A natural monopoly exists when:

A) unit costs are minimized by having one firm produce an industry's entire output.

B) several formerly competing producers merge to become the only firm in an industry.

C) short-run average total cost curves are tangent to long-run average total cost curves.

D) minimum efficient scale is attained at a small level of output.

Answer: A

Type: A Topic: 6 E: 407 MI: 163

182. Diseconomies of scale arise primarily because:

A) the short-run average total cost curve rises when marginal product is increasing.

B) of the difficulties involved in managing and coordinating a large business enterprise.

C) firms must be large both absolutely and relative to the market to employ the most efficient productive techniques available.

D) beyond some point marginal product declines as additional units of a variable resource (labor) are added to a fixed resource (capital).

Answer: B

Use the following to answer questions 183-186:

Page 190: Microeconomics Study Guide

Type: G Topic: 6 E: 403 MI: 159

183. In the above diagram it is assumed that:

A) some costs are fixed and other costs are variable.

B) all costs are variable.

C) the law of diminishing returns determines the shape of the cost curve.

D) marginal product first falls, but ultimately rises as output is increased.

Answer: B

Page 191: Microeconomics Study Guide

Type: G Topic: 6 E: 407 MI: 163

184. Refer to the above diagram. Economies of scale:

A) are evident over the entire range of output. C) begin at output Q3.

B) occur over the 0Q1 range of output. D) occur only over the Q1Q3 range of output.

Answer: B

Type: G Topic: 6 E: 407 MI: 163

185. Refer to the above diagram. Diseconomies of scale:

A) begin at output Q1. C) begin at output Q3.

B) occur over the Q1Q3 range of output. D) are in evidence at all output levels.

Answer: C

Type: G Topic: 6 E: 407 MI: 163

186. Refer to the above diagram. Minimum efficient scale:

A) occurs at some output greater than Q3. C) is achieved at Q3.

B) is achieved at Q1. D) cannot be identified in this diagram.

Answer: B

Type: G Topic: 6 E: 407 MI: 163

187.

In the above long-run average total cost curve the:

A) movement from A to B reflects diseconomies of scale.

B) movement from B to C reflects diseconomies of scale.

Page 192: Microeconomics Study Guide

C) realization of economies of scale would shift the entire curve downward.

D) movement from B to C reflects the law of diminishing returns.

Answer: B

Type: D Topic: 6 E: 403-404 MI: 159-160

188. The long-run average total cost curve:

A) displays declining unit costs so long as output is increasing.

B) indicates the lowest unit costs achievable when a firm has had sufficient time to alter plant size.

C) has a shape which is the inverse of the law of diminishing returns.

D) can be derived by summing horizontally the average total cost curves of all firms in an industry.

Answer: B

Page 193: Microeconomics Study Guide

Type: A Topic: 6 E: 406 MI: 162

189. If a firm increases all of its inputs by 10 percent and its output increases by 15 percent, then:

A) it is encountering diseconomies of scale. C) the law of diminishing returns is taking hold.

B) it is encountering economies of scale. D) the firm's long-run ATC curve will be rising.

Answer: B

Type: A Topic: 6 E: 407 MI: 163

190. If a firm increases all of its inputs by 10 percent and its output increases by 10 percent, then:

A) it is encountering diseconomies of scale. C) it is encountering constant returns to scale.

B) it is encountering economies of scale. D) the marginal products of all inputs are falling.

Answer: C

Type: A Topic: 6 E: 407 MI: 163

191. The ABC Corporation decreases all of its inputs by 12 percent and finds that its output falls by only 8 percent. This means that initially it was producing:

A) in the range of diseconomies of scale. C) where AP is less than MP.

B) in the range of economies of scale. D) at the point of minimum efficient scale.

Answer: A

Type: A Topic: 6 E: 407 MI: 163

192. Diseconomies of scale means that:

A) a firm's long-run average total cost curve is declining.

B) a firm's long-run average total cost curve is rising.

C) the advantages of specialization are being more fully realized.

D) a given increase in inputs results in a more-than-proportionate increase in output.

Answer: B

Type: A Topic: 6 E: 406 MI: 162

193. Suppose a firm is in a range of production where it is experiencing economies of scale. Knowing this, we can predict that:

A) the long-run average total cost curve is upsloping.

Page 194: Microeconomics Study Guide

B) a 10 percent increase in all inputs will increase output by less than 10 percent.

C) a 10 percent increase in all inputs will increase output by more than 10 percent.

D) the firm is encountering problems of managerial bureaucracy because of its size.

Answer: C

Last Word Questions

Type: D E: 409 MI: 165

194. (Last Word) A cost that cannot be partly or fully recovered through any subsequent action is known as a:

A) variable cost. B) fixed cost. C) marginal cost. D) sunk cost.

Answer: D

Page 195: Microeconomics Study Guide

Type: B E: 409 MI: 165

195. (Last Word) Which of the following is an example of a sunk cost, as it relates to a firm?

A) an expenditure on raw materials used in the production process.

B) an expenditure on a nonrefundable, nontransferable airline ticket.

C) an expenditure to buy a delivery van.

D) an expenditure for a new factory.

Answer: B

Type: A E: 409 MI: 165

196. (Last Word) Which of the following sayings relates most closely to the idea of sunk costs:

A) Don't cry over spilt milk. C) He who hesitates is lost.

B) A bird in the hand is worth two in the bush. D) Show me the money.

Answer: A

True/False Questions

Type: A E: 392 MI: 148

197. The real opportunity cost of producing product X is the amounts of products Y, Z, and so forth, that might have been produced if resources had not been used to produce X.

Answer: True

Type: D E: 394 MI: 150

198. The short run is a period of time during which all costs are fixed costs.

Answer: False

Type: D E: 398 MI: 154

199. Variable costs are costs that vary directly with output.

Answer: True

Page 196: Microeconomics Study Guide

Type: A E: 395 MI: 151

200. The law of diminishing returns explains why the long-run average total cost curve is U-shaped.

Answer: False

Type: A E: 407 MI: 163

201. Diseconomies of scale stem primarily from the difficulties in managing and coordinating a large-scale business enterprise.

Answer: True

Type: A E: 398 MI: 154

202. At zero units of output a firm's variable costs are zero.

Answer: True

Type: A E: 399 MI: 155

203. Average fixed costs diminish continuously as output increases.

Answer: True

Page 197: Microeconomics Study Guide

Type: A E: 401 MI: 157

204. If the marginal-cost curve lies below the average-variable-cost curve, the average-variable-cost curve must be falling.

Answer: True

Type: A E: 393 MI: 149

205. Economic profit is found by subtracting accounting costs from total revenue.

Answer: False

Type: A E: 393 MI: 149

206. A firm's economic profit is usually higher than its accounting profit.

Answer: False

Type: D E: 393 MI: 149

207. In economics, a firm earns a normal profit when its total revenue equals its total economic costs.

Answer: True

Type: C E: 401 MI: 157

208. The law of diminishing returns explains why short-run marginal cost curves are upward sloping.

Answer: True

Type: C E: 404 MI: 160

209. The law of diminishing returns explains diseconomies of scale.

Answer: False

Type: C E: 407 MI: 163

210. Minimum efficient scale varies by industry.

Answer: True

Page 198: Microeconomics Study Guide

1. Average fixed costs can be determined graphically by:A. summing the marginal costs of any number of units of output and dividing the sum by that

output.B. the vertical distance between TC and TVC.C. the vertical distance between AVC and MC.D. the vertical distance between ATC and AVC.

2. Diseconomies of scale stem primarily from the difficulties in managing and coordinating a large-scale business enterprise.

A. TrueB. False

3. Which of the following statements is correct?A. Average total cost is the difference between average variable cost and average fixed cost.B. Marginal cost measures the cost per unit of output associated with any level of production.C. When marginal product rises, marginal cost must also rise.D. Marginal cost is the price or cost of an extra variable input (for example, an additional

worker) divided by its marginal product.

4. The long-run average total cost curve:A. displays declining unit costs so long as output is increasing.B. indicates the lowest unit costs achievable when a firm has had sufficient time to alter plant

size.C. has a shape which is the inverse of the law of diminishing returns.D. can be derived by summing horizontally the average total cost curves of all firms in an

industry.

5. Marginal product is:A. the increase in total output attributable to the employment of one more worker.B. the increase in total revenue attributable to the employment of one more worker.C. the increase in total cost attributable to the employment of one more worker.D. total product divided by the number of workers employed.

6. Accounting profits are typically:A. greater than economic profits because the former do not take explicit costs into account.B. equal to economic profits because accounting costs include all opportunity costs.

Page 199: Microeconomics Study Guide

C. smaller than economic profits because the former do not take implicit costs into account.D. greater than economic profits because the former do not take implicit costs into account.

7. To the economist total cost includes:A. explicit and implicit costs, including a normal profit.B. neither implicit nor explicit costs.C. implicit, but not explicit, costs.D. explicit, but not implicit, costs.

8. Suppose that a business incurred implicit costs of $200,000 and explicit costs of $1 million in a specific year. If the firm sold 4,000 units of its output at $300 per unit, its accounting profits were:

A. $100,000 and its economic profits were zero.B. $200,000 and its economic profits were zero.C. $100,000 and its economic profits were $100,000.D. zero and its economic loss was $200,000.

9. The amount of calendar time associated with the long run:A. is less than that associated with the immediate market period.B. varies from industry to industry.C. is the same for all firms.D. is one year by definition.

10. Answer the next question(s) on the basis of the following information:

R-1 REF22050

Refer to the above data. When two workers are employed:

A. total product is 20.B. total product is 18.C. average product is 10.D. total product cannot be determined from the information given.

Page 200: Microeconomics Study Guide

11. For most producing firms:A. marginal cost rises as output is carried to a certain level, and then begins to decline.B. total costs rise as output is carried to a certain level, and then begin to decline.C. average total costs decline as output is carried to a certain level, and then begin to rise.D. average total costs rise as output is carried to a certain level, and then begin to decline.

12. Total cost minus total variable cost equals:A. average fixed cost.B. total fixed cost.C. average variable cost.D. marginal cost.

13. The law of diminishing returns describes the:A. relationship between total costs and total revenues.B. profit-maximizing position of a firm.C. relationship between resource inputs and product outputs in the short run.D. relationship between resource inputs and product outputs in the long run.

14. Diseconomies of scale arise primarily because:A. the short-run average total cost curve rises when marginal product is increasing.B. of the difficulties involved in managing and coordinating a large business enterprise.C. firms must be large both absolutely and relative to the market to employ the most efficient

productive techniques available.D. beyond some point marginal product declines as additional units of a variable resource

(labor) are added to a fixed resource (capital).

15. Use the following data to answer the next question(s). The letters A, B, and C designate three successively larger plant sizes.

Page 201: Microeconomics Study Guide

R-2 REF22158

Refer to the above data. At what level of output is minimum efficient scale realized?

A. 30B. 40C. 50D. 60

16. Use the following data to answer the next question(s). The letters A, B, and C designate three successively larger plant sizes.

R-2 REF22158

Refer to the above data. Economies of scale are realized over the ___ to ___ levels of output; diseconomies of scale exist over the ___ to ___ levels of output.

A. 10, 30; 40, 100B. 10, 40; 80, 100C. 10, 50; 60, 100D. 10, 70; 80, 100

17.

Page 202: Microeconomics Study Guide

In the above diagram curves 1, 2, and 3 represent:A. average variable cost, marginal cost, and average fixed cost respectively.B. total variable cost, total fixed cost, and total cost respectively.C. total fixed cost, total variable cost, and total cost respectively.D. marginal product, average variable cost, and average total cost respectively.

18. "If a variable input is added to some fixed input, beyond some point the resulting extra output will decline." This statement describes:

A. economies and diseconomies of scale.B. X-inefficiency.C. the law of diminishing returns.D. the law of diminishing marginal utility.

19. Use the following cost information for the Creamy Crisp Donut Company to answer questions 16-23:

Entrepreneur's potential earnings as a salaried worker = $50,000

Page 203: Microeconomics Study Guide

Annual lease on building = $22,000Annual revenue from operations = $380,000Payments to workers = $120,000Utilities (electricity, water, disposal) costs = $8,000Entrepreneur's potential economic profit from the next best entrepreneurial activity = $80,000Entrepreneur's forgone interest on personal funds used to finance the business = $6,000R-3 REF22016

Refer to the above data. Creamy Crisp's accounting profit is:

A. $150,000.B. $380,000.C. $230,000.D. $294,000.

20.

R-4 F22065

Refer to the above diagram. At output level Q total cost is:

A. 0BEQ.B. BCDE.C. 0BEQ plus BCDE.D. 0AFQ plus BCDE.

Page 204: Microeconomics Study Guide

1. Normal profit is:

A.determined by subtracting implicit costs from total revenue.

B.determined by subtracting explicit costs from total revenue.

C.the return to the entrepreneur when economic profits are zero.

D.the average profitability of an industry over the preceding 10 years.

2. Suppose that a business incurred implicit costs of $200,000 and explicit costs of $1 million in a specific year. If the firm sold 4,000 units of its output at $300 per unit, its accounting profits were:

A.$100,000 and its economic profits were zero.

B.$200,000 and its economic profits were zero.

C.$100,000 and its economic profits were $100,000.

D.zero and its economic loss was $200,000.

3. The basic characteristic of the short run is that:

A.barriers to entry prevent new firms from entering the industry.

Page 205: Microeconomics Study Guide

B.the firm does not have sufficient time to change the size of its plant.

C.the firm does not have sufficient time to cut its rate of output to zero.

D.a firm does not have sufficient time to change the amounts of any of the resources it employs.

4. Marginal product is:

A.the increase in total output attributable to the employment of one more worker.

B.the increase in total revenue attributable to the employment of one more worker.

C.the increase in total cost attributable to the employment of one more worker.

D.total product divided by the number of workers employed.

5. The first, second, and third workers employed by a firm add 24, 18, and 9 units to total product respectively. We can conclude that:

A.the marginal product of the third worker is 9.

B.the total product of the three workers is 54.

C.the average product of the three workers is 18.

D.the average product of the first two workers is 18.

6.

Page 206: Microeconomics Study Guide

R-1 F22039

In the above diagram curves 1, 2, and 3 represent the:

A.average, marginal, and total product curves respectively.

B.marginal, average, and total product curves respectively.

C.total, average, and marginal product curves respectively.

D.total, marginal, and average product curves respectively.

7.

Page 207: Microeconomics Study Guide

R-1 F22039

The above diagram suggests that:

A.when marginal product is zero, total product is at a maximum.

B.when marginal product lies above average product, average product is rising.

C.when marginal product lies below average product, average product is falling.

D.all of the above hold true.

8. If you owned a small farm, which of the following would be a fixed cost?

A.harvest labor

B.hail insurance

C.fertilizer

Page 208: Microeconomics Study Guide

D.seed

9. Marginal cost is the:

A.rate of change in total fixed cost which results from producing one more unit of output.

B.change in total cost which results from producing one more unit of output.

C.change in average variable cost which results from producing one more unit of output.

D.change in average total cost which results from producing one more unit of output.

10. Assume that in the short run a firm is producing 100 units of output, has average total costs of $200, and average variable costs of $150. The firm's total fixed costs are:

A.$5,000.

B.$500.

C.$.50.

D.$50.

11. If a firm decides to produce no output in the short run, its costs will be:

A.its marginal costs.

B.its fixed plus its variable costs.

C.its fixed costs.

D.zero.

Page 209: Microeconomics Study Guide

12. Answer the next question(s) on the basis of the following cost data:

Output Total Cost

0 $ 24

1 33

2 41

3 48

4 54

5 61

6 69

R-2 REF22070

Refer to the above data. The total variable cost of producing 5 units:

A.is $61.

B.is $48.

C.is $37.

D.is $24.

13. Answer the next question(s) on the basis of the following cost data:

Output Total Cost

0 $ 24

1 33

2 41

3 48

4 54

5 61

6 69

R-2 REF22070

Refer to the above data. The average total cost of producing 3 units of output:

A.is $14.

B.is $12.

Page 210: Microeconomics Study Guide

C.is $13.50.

D.is $16.

14. Answer the next question(s) on the basis of the following cost data:

Output Total Cost

0 $ 24

1 33

2 41

3 48

4 54

5 61

6 69

R-2 REF22070

Refer to the above data. The average fixed cost of producing 3 units of output:

A.is $8.

B.is $7.40.

C.is $5.50.

D.is $6.

15. Answer the next question(s) on the basis of the following cost data:

Output Total Cost

0 $ 24

1 33

2 41

3 48

4 54

5 61

6 69

R-2 REF22070

Refer to the above data. The marginal cost of producing the sixth unit of output:

Page 211: Microeconomics Study Guide

A.is $24.

B.is $12.

C.is $16.

D.is $8.

16.

In the above figure, curves 1, 2, 3, and 4 represent the:

A.ATC, MC, AFC, and AVC curves respectively.

B.AFC, MC, AVC, and ATC curves respectively.

C.MC, ATC, AVC, and AFC curves respectively.

D.ATC, AVC, AFC, and MC curves respectively.

Page 212: Microeconomics Study Guide

17. Economies and diseconomies of scale explain:

A.the profit-maximizing level of production.

B.why the firm's long-run average total cost curve is U-shaped.

C.

why the firm's short-run marginal cost curve cuts the short-run average variable cost curve at its minimum point.

D.the distinction between fixed and variable costs.

18.

R-3 F22144

The above diagram shows the short-run average total cost curves for five different plant sizes of a firm. In the long run the firm should produce output 0*B x with a plant of size:

A.#4

B.#3.

C.#2.

D.#1.

Page 213: Microeconomics Study Guide

19. Use the following data to answer the next question(s). The letters A, B, and C designate three successively larger plant sizes.

Output ATC-A ATC-B ATC-C

10 $6 $13 $44

20 5 9 35

30 4 6 27

40 5 4 20

50 7 3 14

60 10 4 11

70 14 5 8

80 19 7 6

90 25 10 5

100 32 16 7

R-4 REF22147

Refer to the above data. At what level of output is minimum efficient scale realized?

A.30

B.40

C.50

D.60

E.70

20.

Page 214: Microeconomics Study Guide

R-5 F22055

Refer to the above diagram. At output level Q:

A.marginal product is falling.

B.marginal product is rising.

C.marginal product is negative.

D.one cannot determine whether marginal product is falling or rising.

21.

Page 215: Microeconomics Study Guide

R-5 F22055

Refer to the above diagram. At output level Q average fixed cost:

A.is equal to EF.

B.is equal to QE.

C.is measured by both QF and ED.

D.cannot be determined from the information given.

22.

Page 216: Microeconomics Study Guide

R-5 F22055

Refer to the above diagram. At output level Q total cost is:

A.0BEQ.

B.BCDE.

C.0BEQ plus BCDE.

D.0AFQ plus BCDE.

23.

Page 217: Microeconomics Study Guide

R-5 F22055

Refer to the above diagram. At output level Q total fixed cost is:

A.0BEQ.

B.BCDE.

C.0BEQ-0AFQ.

D.0CDQ.

24.

Page 218: Microeconomics Study Guide

R-5 F22055

Refer to the above diagram. At output level Q total variable cost is:

A.0BEQ.

B.BCDE.

C.0CDQ.

D.0AFQ.

Page 219: Microeconomics Study Guide

Practice Exam - Final Key

1. For economists, the word "utility" means: A. versatility and flexibility.B. rationality.C. pleasure or satisfaction.D. purposefulness. 

2. Joe sold gold coins for $1000 that he bought a year ago for $1000. He says, "At least I didn't lose any money on my financial investment." His economist friend points out that in effect he did lose money, because he could have received a 3 percent return on the $1000 if he had bought a bank certificate of deposit instead of the coins. The economist's analysis in this case incorporates the idea of: A. opportunity costs.B. marginal benefits that exceed marginal costs.C. imperfect information.D. normative economics.  

3. The assertion that "There is no free lunch" means that: A. there are always tradeoffs between economic goals.B. all production involves the use of scarce resources and thus the sacrifice of alternative goods.C. marginal analysis is not used in economic reasoning.D. choices need not be made if behavior is rational.  

4. The budget line shows: A. the amount of product A that a consumer is willing to give up to obtain one more unit of product B.B. all possible combinations of two goods that can be purchased, given money income and the prices of the goods.C. the minimum amount of two goods that a consumer can purchase with a given money income.D. all possible combinations of two goods that yield the same level of utility to the consumer.

      5. Refer to the budget line shown in the diagram above. If the consumer's money income is $20, the: A. prices of C and D cannot be determined.B. price of C is $2 and the price of D is $4.C. consumer can obtain a combination of 5 units of both C and D.D. price of C is $4 and the price of D is $2. 

6. Any point inside the production possibilities curve indicates: A. the presence of technological change.B. that resources are imperfectly substitutable among alternative uses.C. the presence of inflationary pressures.D. that more output could be produced with available resources.

Page 220: Microeconomics Study Guide

 7. Command systems are also known as: A. market systems.B. pure capitalism.C. laissez-faire capitalism.D. communism. 

8. The two basic markets shown by the simple circular flow model are: A. capital goods and consumer goods.B. free and controlled.C. product and resource.D. household and business. 

9. The law of demand states that, other things equal: A. price and quantity demanded are inversely related.B. the larger the number of buyers in a market, the lower will be product price.C. price and quantity demanded are directly related.D. consumers will buy more of a product at high prices than at low prices. 

10. The income and substitution effects account for: A. the upward sloping supply curve.B. the downward sloping demand curve.C. movements along a given supply curve.D. shifts in the demand curve. 

11. Which of the following would not shift the demand curve for beef? A. a widely publicized study that indicates beef increases one's cholesterolB. a reduction in the price of cattle feedC. an effective advertising campaign by pork producersD. a change in the incomes of beef consumers 

12. If the demand curve for product B shifts to the right as the price of product A declines, then: A. both A and B are inferior goods.B. A is a superior good and B is an inferior good.C. A is an inferior good and B is a superior good.D. A and B are complementary goods. 

13. If X is a normal good, a rise in money income will shift the: A. supply curve for X to the left.B. supply curve for X to the right.C. demand curve for X to the left.D. demand curve for X to the right. 

14. A decrease in the price of digital cameras will: A. cause the demand curve for memory cards to become vertical.B. shift the demand curve for memory cards to the right.C. shift the demand curve for memory cards to the left.D. not affect the demand for memory cards. 

15. Assume that the demand curve for product C is downsloping. If the price of C falls from $2.00 to $1.75: A. a smaller quantity of C will be demanded.B. a larger quantity of C will be demanded.C. the demand for C will increase.D. the demand for C will decrease. 

Page 221: Microeconomics Study Guide

16. A leftward shift of a product supply curve might be caused by: A. an improvement in the relevant technique of production.B. a decline in the prices of needed inputs.C. an increase in consumer incomes.D. some firms leaving an industry. 

17. A government subsidy to the producers of a product: A. reduces product supply.B. increases product supply.C. reduces product demand.D. increases product demand. 

18. Suppose that corn prices rise significantly. If farmers expect the price of corn to continue rising relative to other crops, then we would expect: A. the supply of ethanol, a corn-based product, to increase.B. consumer demand for wheat to fall.C. the supply to increase as farmers plant more corn.D. the supply to fall as farmers plant more of other crops. 

     

19. Refer to the above table. If demand is represented by columns (3) and (2) and supply is represented by columns (3) and (5), equilibrium price and quantity will be: A. $10 and 60 units.B. $9 and 50 units.C. $8 and 60 units.D. $7 and 50 units. 

20. Refer to the above table. Suppose that demand is represented by columns (3) and (2) and supply is represented by columns (3) and (5). If the price were artificially set at $9, A. the market would clear.B. a surplus of 20 units would occur.C. a shortage of 20 units would occur.D. demand would change from columns (3) and (2) to columns (3) and (1).

      21. Refer to the above diagram. The equilibrium price and quantity in this market will be: A. $1.00 and 200.B. $1.60 and 130.C. $0.50 and 130.D. $1.60 and 290. 

Page 222: Microeconomics Study Guide

22. Refer to the above diagram. A surplus of 160 units would be encountered if the price was: A. $1.10, that is, $1.60 minus $.50.B. $1.60.C. $1.00.D. $0.50. 

23. Refer to the above diagram. A shortage of 160 units would be encountered if price was: A. $1.10, that is, $1.60 minus $.50.B. $1.60.C. $1.00.D. $0.50. 

24. If there is a surplus of a product, its price: A. is below the equilibrium level.B. is above the equilibrium level.C. will rise in the near future.D. is in equilibrium. 

    25. Refer to the above diagram. A price of $60 in this market will result in: A. equilibrium.B. a shortage of 50 units.C. a surplus of 50 units.D. a surplus of 100 units. 

26. Refer to the above diagram. A price of $20 in this market will result in a: A. shortage of 50 units.B. surplus of 50 units.C. surplus of 100 units.D. shortage of 100 units. 

27. Assume in a competitive market that price is initially below the equilibrium level. We can predict that price will: A. decrease, quantity demanded will decrease, and quantity supplied will increase.B. decrease and quantity demanded and quantity supplied will both decrease.C. increase, quantity demanded will increase, and quantity supplied will decrease.D. increase, quantity demanded will decrease, and quantity supplied will increase. 

28. If the price elasticity of demand for a product is 2.5, then a price cut from $2.00 to $1.80 will: A. increase the quantity demanded by about 2.5 percent.B. decrease the quantity demanded by about 2.5 percent.C. increase the quantity demanded by about 25 percent.D. increase the quantity demanded by about 250 percent. 

Page 223: Microeconomics Study Guide

29. Suppose that as the price of Y falls from $2.00 to $1.90 the quantity of Y demanded increases from 110 to 118. Then the price elasticity of demand is: A. 4.00.B. 2.09.C. 1.37.D. 3.94. 

30. The price elasticity of demand of a straight-line demand curve is: A. elastic in high-price ranges and inelastic in low-price ranges.B. elastic, but does not change at various points on the curve.C. inelastic, but does not change at various points on the curve.D. 1 at all points on the curve. 

31. The price elasticity of demand is generally: A. negative, but the minus sign is ignored.B. positive, but the plus sign is ignored.C. positive for normal goods and negative for inferior goods.D. positive because price and quantity demanded are inversely related. 

32. If a demand for a product is elastic, the value of the price elasticity coefficient is: A. zero.B. greater than one.C. equal to one.D. less than one.

      33. Refer to the above diagram. Between prices of $5.70 and $6.30: A. D1 is more elastic than D2.B. D2 is an inferior good and D1 is a normal good.C. D1 and D2 have identical elasticities.D. D2 is more elastic than D1. 

34. Refer to the above diagram and assume a single good. If the price of the good decreases from $6.30 to $5.70, consumer expenditure would: A. decrease if demand were D1 only.B. decrease if demand were D2 only.C. decrease if demand were either D1 or D2.D. increase if demand were either D1 or D2. 

35. Refer to the above diagram and assume a single good. If the price of the good increased from $5.70 to $6.30 along D1, the price elasticity of demand along this portion of the demand curve would be: A. 0.8.B. 1.0.C. 1.2.D. 2.0.

Page 224: Microeconomics Study Guide

 36. Suppose the price of local cable TV service increased from $16.20 to $19.80 and as a result the number of cable subscribers decreased from 224,000 to 176,000. Along this portion of the demand curve, price elasticity of demand is: A. 0.8.B. 1.2.C. 1.6.D. 8.0 

37. A firm can sell as much as it wants at a constant price. Demand is thus: A. perfectly inelastic.B. perfectly elastic.C. relatively inelastic.D. relatively elastic. 

38. Suppose the price elasticity coefficients of demand are 1.43, 0.67, 1.11, and 0.29 for products W, X, Y, and Z respectively. A 1 percent decrease in price will increase total revenue in the case(s) of: A. W and Y.B. Y and Z.C. X and Z.D. Z and W. 

39. If a firm finds that it can sell $13,000 worth of a product when its price is $5 per unit and $11,000 worth of it when its price is $6, then: A. the demand for the product is elastic in the $6-$5 price range.B. the demand for the product must have increased.C. elasticity of demand is 0.74.D. the demand for the product is inelastic in the $6-$5 price range. 

40. Suppose the price elasticity of demand for bread is 0.20. If the price of bread falls by 10 percent, the quantity demanded will increase by: A. 2 percent and total expenditures on bread will rise.B. 2 percent and total expenditures on bread will fall.C. 20 percent and total expenditures on bread will fall.D. 20 percent and total expenditures on bread will rise. 

41. Gigantic State University raises tuition for the purpose of increasing its revenue so that more faculty can be hired. GSU is assuming that the demand for education at GSU is: A. decreasing.B. relatively elastic.C. perfectly elastic.D. relatively inelastic. 

42. Suppose the income elasticity of demand for toys is +2.00. This means that: A. a 10 percent increase in income will increase the purchase of toys by 20 percent.B. a 10 percent increase in income will increase the purchase of toys by 2 percent.C. a 10 percent increase in income will decrease the purchase of toys by 2 percent.D. toys are an inferior good. 

43. We would expect the cross elasticity of demand between dress shirts and ties to be: A. positive, indicating normal goods.B. positive, indicating complementary goods.C. negative, indicating substitute goods.D. negative, indicating complementary goods. 

Page 225: Microeconomics Study Guide

44. We would expect the cross elasticity of demand between Pepsi and Coke to be: A. positive, indicating normal goods.B. positive, indicating inferior goods.C. positive, indicating substitute goods.D. negative, indicating substitute goods. 

45. Assume that a 3 percent increase in income across the economy produces a 1 percent decline in the quantity demanded of good X. The coefficient of income elasticity of demand for good X is: A. negative and therefore X is an inferior good.B. negative and therefore X is a normal good.C. positive and therefore X is an inferior good.D. positive and therefore X is a normal good. 

46. The two main characteristics of a public good are: A. production at constant marginal cost and rising demand.B. nonexcludability and production at rising marginal cost.C. nonrivalry and nonexcludability.D. nonrivalry and large negative externalities. 

47. Unlike a private good, a public good: A. has no opportunity costs.B. has benefits available to all, including nonpayers.C. produces no positive or negative externalities.D. is characterized by rivalry and excludability. 

48. Nonexcludability describes a condition where: A. one person's consumption of a good does not prevent consumption of the good by others.B. there is no effective way to keep people from using a good once it comes into being.C. sellers can withhold the benefits of a good from those unwilling to pay for it.D. there is no potential for free-riding behavior. 

49. At the optimal quantity of a public good: A. marginal benefit exceeds marginal cost by the greatest amount.B. total benefit equals total cost.C. marginal benefit equals marginal cost.D. marginal benefit is zero. 

50. The ability of a good or service to satisfy wants is called: A. utility maximization.B. opportunity cost.C. revenue potential.D. utility. 

51. Marginal utility is the: A. sensitivity of consumer purchases of a good to changes in the price of that good.B. change in total utility obtained by consuming one more unit of a good.C. change in total utility obtained by consuming another unit of a good divided by the change in the price of that good.D. total utility associated with the consumption of a certain number of units of a good divided by the number of units consumed. 

Page 226: Microeconomics Study Guide

52. To maximize utility a consumer should allocate money income so that the: A. elasticity of demand on all products purchased is the same.B. marginal utility obtained from the last dollar spent on each product is the same.C. total utility derived from each product consumed is the same.D. marginal utility of the last unit of each product consumed is the same. 

 Answer the question on the basis of the following total utility data for products L and M. Assume that the prices of L and M are $3 and $4 respectively and that the consumer's income is $18.

    53. Refer to the above data. How many units of the two products will the rational consumer purchase? A. 3 of L and none of MB. 4 of L and 2 of MC. 3 of L and 5 of MD. 2 of L and 3 of M 

54. Refer to the above data. What level of total utility does the rational consumer realize in equilibrium? A. 87 utilsB. 104 utilsC. 51 utilsD. 58 utils 

55. Diminishing marginal utility explains why: A. the income effect exceeds the substitution effect.B. the substitution effect exceeds the income effect.C. supply curves are upsloping.D. demand curves are downsloping. 

56. Why do people tend to eat more at all-you-can-eat buffet restaurants than at restaurants where each item is purchased separately? A. Once the all-you-can-eat meal is purchased, consumers view additional trips back to the buffet as having a price of zero.B. MU/P is greater at all-you-can-eat restaurants.C. People who eat at all-you-can-eat restaurants do not experience diminishing marginal utility.D. Food at all-you-can-eat restaurants tends to have fewer calories, so consumers feel the need to consume a greater volume of food. 

57. To the economist, total cost includes: A. explicit and implicit costs, including a normal profit.B. neither implicit nor explicit costs.C. implicit, but not explicit, costs.D. explicit, but not implicit, costs. 

58. Suppose that a business incurred implicit costs of $500,000 and explicit costs of $5 million in a specific year. If the firm sold 100,000 units of its output at $50 per unit, its accounting: A. profits were $100,000 and its economic profits were zero.B. losses were $500,000 and its economic losses were zero.C. profits were $500,000 and its economic profits were $1 million.D. profits were zero and its economic losses were $500,000. 

Page 227: Microeconomics Study Guide

59. The basic difference between the short run and the long run is that: A. all costs are fixed in the short run, but all costs are variable in the long run.B. the law of diminishing returns applies in the long run, but not in the short run.C. at least one resource is fixed in the short run, while all resources are variable in the long run.D. economies of scale may be present in the short run, but not in the long run. 

 Answer the question on the basis of the following output data for a firm. Assume that the amounts of all non-labor resources are fixed.

    60. Refer to the above data. Diminishing marginal returns become evident with the addition of the: A. sixth worker.B. fourth worker.C. third worker.D. second worker. 

61. Refer to the above data. The marginal product of the sixth worker is: A. 180 units of output.B. 30 units of output.C. 15 units of output.D. negative. 

 Use the following data to answer the question:

    62. Refer to the above data. The average product (AP) when two units of labor are hired is: A. 8.B. 9.C. 10.D. 18. 

63. Refer to the above data. Diminishing returns begin to occur with the hiring of the _________ unit of labor. A. firstB. secondC. thirdD. seventh 

64. Refer to the above data. Marginal product becomes negative with the hiring of the __________ unit of labor. A. thirdB. fourthC. sixthD. seventh

Page 228: Microeconomics Study Guide

  Answer the question on the basis of the following cost data:

    65. Refer to the above data. Total fixed cost is: A. $6.25.B. $100.00.C. $150.00.D. $50.00. 

66. Refer to the above data. The average total cost of five units of output is: A. $69.B. $78.C. $3.D. $10. 

67. Refer to the above data. The total cost of four units of output is: A. $260.B. $77.50.C. $310.D. $215. 

68. Refer to the above data. If the firm closed down in the short run and produced zero units of output, its total cost would be: A. zero.B. $50.C. $150.D. $100. 

69. Refer to the above data. The marginal cost of the fifth unit of output is: A. $3.B. $62.C. $80.D. $78. 

70. In which of the following market structures is there clear-cut mutual interdependence with respect to price-output policies? A. pure monopolyB. oligopolyC. monopolistic competitionD. pure competition 

71. An industry comprised of 40 firms, none of which has more than 3 percent of the total market for a differentiated product is an example of: A. monopolistic competition.B. oligopoly.C. pure monopoly.D. pure competition.

Page 229: Microeconomics Study Guide

 72. An industry comprised of a small number of firms, each of which considers the potential reactions of its rivals in making price-output decisions is called: A. monopolistic competition.B. oligopoly.C. pure monopoly.D. pure competition. 

73. A purely competitive seller is: A. both a "price maker" and a "price taker."B. neither a "price maker" nor a "price taker."C. a "price taker."D. a "price maker."  

74. For a purely competitive seller, price equals: A. average revenue.B. marginal revenue.C. total revenue divided by output.D. all of these.

      75. Refer to the above short-run data. The profit-maximizing output for this firm is: A. above 440 units.B. 440 units.C. 320 units.D. 100 units. 

76. In the short run the individual competitive firm's supply curve is that segment of the: A. average variable cost curve lying below the marginal cost curve.B. marginal cost curve lying above the average variable cost curve.C. marginal revenue curve lying below the demand curve.D. marginal cost curve lying between the average total cost and average variable cost curves. 

77. Suppose you find that the price of your product is less than minimum AVC. You should: A. minimize your losses by producing where P = MC.B. maximize your profits by producing where P = MC.C. close down because, by producing, your losses will exceed your total fixed costs.D. close down because total revenue exceeds total variable cost. 

 Answer the question on the basis of the following data confronting a firm:

   

Page 230: Microeconomics Study Guide

 78. Refer to the above data. This firm is selling its output in a(n): A. monopolistically competitive market.B. monopolistic market.C. purely competitive market.D. oligopolistic market. 

79. Refer to the above data. If the firm's minimum average variable cost is $10, the firm's profit-maximizing level of output would be: A. 2.B. 3.C. 4.D. 5. 

80. Refer to the above data. At the profit-maximizing output the firm's total revenue is: A. $48.B. $32.C. $80.D. $64. 

81. Refer to the above data. Assuming total fixed costs equal to zero, the firm's: A. economic profit is $12.B. economic profit is $16.C. loss is $14.D. economic profit is $3. 

82. A firm finds that at its MR = MC output, its TC = $1,000, TVC = $800, TFC = $200, and total revenue is $900. This firm should: A. shut down in the short run.B. produce because the resulting loss is less than its TFC.C. produce because it will realize an economic profit.D. liquidate its assets and go out of business.

       83. Refer to the above diagram for a purely competitive producer. The lowest price at which the firm should produce (as opposed to shutting down) is: A. P1.B. P2.C. P3.D. P4. 

84. Refer to the above diagram for a purely competitive producer. The firm will produce at a loss at all prices: A. above P1.B. above P3.C. above P4.D. between P2 and P3.

Page 231: Microeconomics Study Guide

85. Refer to the above diagram for a purely competitive producer. If product price is P3: A. the firm will maximize profit at point d.B. the firm will earn an economic profit.C. economic profits will be zero.D. new firms will enter this industry. 

 Answer the question on the basis of the following cost data for a firm that is selling in a purely competitive market:

    86. Refer to the above data. If the market price for the firm's product is $12, the competitive firm will produce: A. 4 units at a loss of $109.B. 4 units at an economic profit of $31.75.C. 8 units at a loss of $48.80.D. zero units at a loss of $100. 

87. Refer to the above data. If the market price for the firm's product is $32, the competitive firm will produce: A. 8 units at an economic profit of $16.B. 6 units at an economic profit of $7.98.C. 10 units at an economic profit of $4.D. 7 units at an economic profit of $41.50. 

88. Refer to the above data. If the market price for the firm's product is $28, the competitive firm will: A. produce 4 units at a loss of $17.40.B. produce 7 units at a loss of $14.00.C. shut down in the short run.D. produce 6 units at a loss of $23.80. 

89. In a purely competitive industry: A. there will be no economic profits in either the short run or the long run.B. economic profits may persist in the long run if consumer demand is strong and stable.C. there may be economic profits in the short run, but not in the long run.D. there may be economic profits in the long run, but not in the short run. 

90. If a purely competitive constant-cost industry is realizing economic profits, we can expect industry supply to: A. increase, output to increase, price to decrease, and profits to decrease.B. increase, output to increase, price to increase, and profits to decrease.C. decrease, output to decrease, price to increase, and profits to increase.D. increase, output to decrease, price to decrease, and profits to decrease. 

Page 232: Microeconomics Study Guide

91. Which of the following conditions is true for a purely competitive firm in long-run equilibrium? A. P > MC = minimum ATC.B. P > MC > minimum ATC.C. P = MC = minimum ATC.D. P < MC < minimum ATC. 

92. Which of the following is correct? A. Both purely competitive and monopolistic firms are "price takers."B. Both purely competitive and monopolistic firms are "price makers."C. A purely competitive firm is a "price taker," while a monopolist is a "price maker."D. A purely competitive firm is a "price maker," while a monopolist is a "price taker." 

93. Which of the following is a characteristic of pure monopoly? A. close substitute productsB. barriers to entryC. the absence of market powerD. "price taking" 

 Answer the question on the basis of the demand schedule shown below:

   94. Refer to the above data. The marginal revenue obtained from selling the third unit of output is: A. $6.B. $1.C. $3.D. $5. 

95. Which of the following is characteristic of a pure monopolist's demand curve? A. Average revenue is less than price.B. Its elasticity coefficient is 1 at all levels of output.C. Price and marginal revenue are equal at all levels of output.D. It is the same as the market demand curve. 

96. Because the monopolist's demand curve is downsloping: A. MR will equal price.B. price must be lowered to sell more output.C. the elasticity coefficient will increase as price is lowered.D. its supply curve will also be downsloping. 

97. An unregulated pure monopolist will maximize profits by producing that output at which: A. P = MC.B. P = ATC.C. MR = MC.D. MC = AC. 

Page 233: Microeconomics Study Guide

98. Suppose that a pure monopolist can sell 5 units of output at $4 per unit and 6 units at $3.90 per unit. The monopolist will produce and sell the sixth unit if its marginal cost is: A. $4 or less.B. $3.90 or less.C. $3.50 or less.D. $3.40 or less. 

99. A pure monopolist is producing an output such that ATC = $4, P = $5, MC = $2, and MR = $3. This firm is realizing: A. a loss that could be reduced by producing more output.B. a loss that could be reduced by producing less output.C. an economic profit that could be increased by producing more output.D. an economic profit that could be increased by producing less output.

      100. Refer to the above diagram. To maximize profits or minimize losses this firm should produce: A. E units and charge price C.B. E units and charge price A.C. M units and charge price N.D. L units and charge price LK. 

101. Refer to the above diagram. At the profit-maximizing level of output, total revenue will be: A. NM times 0M.B. 0AJE.C. 0EGC.D. 0EHB.

102. Refer to the above diagram. At the profit-maximizing level of output, total cost will be: A. NM times 0M.B. 0AJE.C. 0CGC.D. 0BHE. 

103. Refer to the above diagram. At the profit-maximizing level of output, the firm will realize: A. an economic profit of ABHJ.B. an economic profit of ACGJ.C. a loss of GH per unit.D. a loss of JH per unit. 

104. If profits are maximized (or losses minimized), which of the following conditions is common to both unregulated monopoly and to pure competition? A. MC = PB. MC = ATCC. MR = MCD. P = MR 

Page 234: Microeconomics Study Guide

    105. Refer to the above diagram for a pure monopolist. Monopoly price will be: A. e.B. c.C. b.D. a. 

106. Refer to the above diagram for a pure monopolist. Monopoly output will be: A. between f and g.B. h.C. g.D. f. 

     107. Refer to the above diagram. If this industry is comprised of only one seller, the profit-maximizing price and quantity will be: A. P3 and Q3.B. P1 and Q3.C. P2 and Q2.D. indeterminate on the basis of the information given. 

108. Which of the following statements is correct? A. The pure monopolist will maximize profit by producing at that point on the demand curve where elasticity is zero.B. In seeking the profit-maximizing output the pure monopolist underallocates resources to its production.C. The pure monopolist maximizes profits by producing that output at which the differential between price and average cost is the greatest.D. Purely monopolistic sellers earn only normal profits in the long run. 

109. When economists say that the demand for labor is a derived demand, they mean that it is: A. dependent on government expenditures for public goods and services.B. related to the demand for the product or service labor is producing.C. based on the desire of businesses to exploit labor by paying below equilibrium wage rates.D. based on the assumption that workers are trying to maximize their money incomes. 

Page 235: Microeconomics Study Guide

110. Marginal product is: A. the output of the least skilled worker.B. a worker's output multiplied by the price at which each unit can be sold.C. the amount an additional worker adds to the firm's total output.D. the amount any given worker contributes to the firm's total revenue. 

111. Assume labor is the only variable input and that an additional input of labor increases total output from 72 to 78 units. If the product sells for $6 per unit in a purely competitive market, the MRP of this additional worker is: A. $6.B. $12.C. $36.D. $72. 

112. If one worker can pick $30 worth of grapes and two workers together can pick $50 worth of grapes, the: A. marginal revenue product of each worker is $25.B. marginal revenue product of the first worker is $20.C. marginal revenue product of the second worker is $20.D. data given do not permit the determination of the marginal revenue product of either worker.

Answer the question on the basis of the following information for Manfred's Shoe Shine Parlor. Assume Manfred hires labor, its only variable input, under purely competitive conditions. Shoe shines are also sold competitively.

    113. Refer to the above data. How many units of output are produced when 2 workers are employed? A. 4B. 16C. 24D. 10 

114. Refer to the above data. What is the marginal product of the sixth worker? A. 2 unitsB. 3 unitsC. 4 unitsD. 5 units 

115. Refer to the above data. At what price does each shoe shine sell? A. $1B. $2C. $3D. $2.50 

Page 236: Microeconomics Study Guide

116. Refer to the above data. If the wage rate is $11, how many workers will Manfred hire to maximize profits? A. 1B. 2C. 3D. 5 

117. Refer to the above data. If the wage rate is $11 and Manfred's only fixed input is capital, the total cost of which is $30, then what will be his economic profit? A. $62B. $42C. $28D. $32 

118. Assume that a restaurant is hiring labor in an amount such that the MRC of the last worker is $16 and her MRP is $12. On the basis of this information we can say that: A. profits will be increased by hiring additional workers.B. profits will be increased by hiring fewer workers.C. marginal revenue product must exceed average revenue product.D. the restaurant is maximizing profits. 

 Answer the question on the basis of the data contained in the following table. Assume that the firm is hiring labor in a purely competitive market.

    119. Refer to the above data. If the wage rate is $20, how many workers will the firm choose to employ? A. 5B. 4C. 3D. 2 

120. Refer to the above data. If the wage rate is $11, how many workers will the firm choose to employ? A. 5B. 4C. 3D. 2 

121. If the nominal wages of carpenters rose by 5 percent in 2010 and the price level increased by 3 percent, then the real wages of carpenters: A. decreased by 2 percent.B. increased by 2 percent.C. increased by 3 percent.D. increased by 8 percent. 

122. If the nominal wage rises by 4 percent, and the price level rises by 7 percent, the real wage will: A. be unaffected.B. rise by 3 percent.C. fall by 3 percent.D. rise by 11 percent.

Page 237: Microeconomics Study Guide

     123. Refer to the above diagram. Assuming no union or relevant minimum wage, the firm represented will hire: A. Q2 workers and pay a W4 wage rate.B. Q2 workers and pay a W1 wage rate.C. Q3 workers and pay a W2 wage rate.D. Q4 workers and pay a W1 wage rate. 

124. A progressive tax is such that: A. tax rates are higher the greater one's income.B. the same tax rate applies to all income receivers, so that the rich pay absolutely more taxes than the poor.C. entrepreneurial income is exempt from taxation.D. the revenues it yields are spent on transfer payments. 

 The following data represent a personal income tax schedule. Answer the question on the basis of this information.

    125. Refer to the above table. If your taxable income is $8,000, your average tax rate is: A. 25 percent and the marginal rate on additional income is also 25 percent.B. 25 percent and the marginal rate on additional income is 40 percent.C. 25 percent and the marginal rate on additional income cannot be determined from the information given.D. 20 percent and the marginal rate on additional income is 30 percent. 

126. Which of the following best reflects the ability-to-pay philosophy of taxation? A. a tax on residential propertyB. a progressive income taxC. an excise tax on gasolineD. an excise tax on coffee 

127. The incidence of a tax pertains to: A. the degree to which it alters the distribution of income.B. how easy it is to evade the tax.C. who actually bears the burden of a tax.D. the progressiveness or regressiveness of tax rates.

Page 238: Microeconomics Study Guide

Multiple Choice Questions

Four market models

Type: A Topic: 1 E: 414 MI: 1701. Economists would describe the U.S. automobile industry as:A) purely competitive. B)an oligopoly. C) monopolistically competitive. D) a puremonopoly.  Answer: B  

Type: A Topic: 1 E: 414 MI: 170

2.In which of the following market structures is there clear-cut mutual interdependence with respect to price-output policies?

A) pure monopolyB) oligopoly C) monopolistic competition D) pure competitionAnswer: B  Type: A Topic: 1 E: 414 MI: 170

3.Which of the following industries most closely approximates pure competition?

A)agriculture B) farm implements C) clothing D) steel

Chapter 23: Pure Competition

Answer: A

Type: D Topic: 1 E: 414 MI: 170

4.Economists use the term imperfect competition to describe:

A)all industries which produce standardized products.

B)any industry in which there is no nonprice competition.

C)a pure monopoly only.

D)those markets which are not purely competitive. Answer: D

McConnell/Brue: Economics, 16/e Page 688

Chapter 23: Pure Competition

Type: A Topic: 1 E: 414 MI: 170

5. In which of the following industry structures is the entry of new firms the most difficult? A) pure monopoly B) oligopoly C) monopolistic competition D) pure competition Answer: A

Type: A Topic: 1 E: 414 MI: 170

Page 239: Microeconomics Study Guide

6.An industry comprised of 40 firms, none of which has more than 3 percent of the total market for a differentiated product is an example of:

A) monopolistic competition B) oligopoly C)pure monopolyD)pure competitionAnswer: A            Type: A Topic: 1 E: 413-414 MI: 169-170        7. A one-firm industry is known as:         A) monopolistic competition B) oligopoly C)pure monopolyD)pure competitionAnswer: C            

Type: A Topic: 1 E: 414 MI: 170

8.An industry comprised of four firms, each with about 25 percent of the total market for a  product is an example of:        A) monopolistic competition B) oligopoly C) pure monopoly D) pure competition  Answer: B        Type: A Topic: 1 E: 413-414 MI: 169-170    9.An industry comprised of a very large number of sellers producing a standardized product  is known as:        A) monopolistic competition B) oligopoly C) pure monopoly D) pure competition  Answer: D      

Type: A Topic: 1 E: 414 MI: 170

10.An industry comprised of a small number of firms, each of which considers the potential reactions of its rivals in making price-output decisions is called:

A) monopolistic competition B) oligopoly C) pure monopoly D) pure competition Answer: B

Pure competition defined; demand curve

Type: A Topic: 2 E: 414 MI: 170

11.Which of the following statements applies to a purely competitive producer?

A)It will not advertise its product.

McConnell/Brue: Economics, 16/e Page 689

Chapter 23: Pure Competition

B)In long-run equilibrium it will earn an economic profit.

C)Its product will have a brand name.

D)Its product is slightly different from those of its competitors. Answer: A

McConnell/Brue: Economics, 16/e Page 690

Chapter 23: Pure Competition

Page 240: Microeconomics Study Guide

Type: A Topic: 2 E: 414 MI: 170 12. A purely competitive seller is:

A) both a "price maker" and a "price taker." C)a "price taker."B) neither a "price maker" nor a "price taker." D) a "price maker."Answer: C        Type: ATopic: 2 E: 414 MI: 170    13. Which of the following is not characteristic of pure competition?A) price strategies by firms   C) no barriers to entryB) a standardized product   D) a larger number of sellersAnswer: A        Type: ATopic: 2 E: 414 MI: 170    14. Which of the following is not a basic characteristic of pure competition?A) considerable nonprice competition C) a standardized or homogeneous product

B) no barriers to the entry or exodus of firms D) a large number of buyers and sellers Answer: A

Type: A Topic: 2 E: 415 MI: 171

15.The demand schedule or curve confronted by the individual purely competitive firm is:

A)relatively elastic, that is, the elasticity coefficient is greater than unity.

B)perfectly elastic.

C)relatively inelastic, that is, the elasticity coefficient is less than unity.

D)perfectly inelastic.

Answer: B

Type: A Topic: 2 E: 415-416 MI: 171-172

16.Which of the following is characteristic of a purely competitive seller's demand curve?

A)Price and marginal revenue are equal at all levels of output.

B)Average revenue is less than price.

C)Its elasticity coefficient is 1 at all levels of output.

D)It is the same as the market demand curve.

Answer: A

Use the following to answer questions 17-19:

In answering the next question(s), assume a graph in which dollars are measured on the vertical axis and output on the horizontal axis.

Type: G Topic: 2 E: 415-416 MI: 171-172

Page 241: Microeconomics Study Guide

McConnell/Brue: Economics, 16/e Page 691

Chapter 23: Pure Competition

17. Refer to the above information. For a purely competitive firm total revenue:

A) graphs as a straight, upsloping line.C) is a straight line, parallel to the horizontalaxis.  

B)is a straight line, parallel to the vertical axis. D) graphs as a straight, downsloping line.

Answer: A

McConnell/Brue: Economics, 16/e Page 692

  Chapter 23: Pure CompetitionType: G Topic: 2 E: 416 MI: 172  18. Refer to the above information. For a purely competitive firm marginal revenue:A) graphs as a straight, upsloping line.C) is a straight line, parallel to the horizontalaxis.    

B)is a straight line, parallel to the vertical axis. D) graphs as a straight, downsloping line.

Answer: C

Type: G Topic: 2 E: 416 MI: 172

19.Refer to the above information. For a purely competitive firm:

A)marginal revenue will graph as an upsloping line.

B)the demand curve will lie above the marginal revenue curve.

C)the marginal revenue curve will lie above the demand curve.

D)the demand and marginal revenue curves will coincide. Answer: D

Type: A Topic: 2 E: 415-416 MI: 171-172

20.If a firm in a purely competitive industry is confronted with an equilibrium price of $5, its marginal revenue:

A)may be either greater or less than $5.

B)will also be $5.

Answer: B

C)will be less than $5.

D)will be greater than $5.

Page 242: Microeconomics Study Guide

Type: A Topic: 2 E: 414 MI: 170

21.Price is constant or given to the individual firm selling in a purely competitive market because:

A)the firm's demand curve is downsloping.

B)of product differentiation reinforced by extensive advertising.

C)each seller supplies a negligible fraction of total supply.

D)there are no good substitutes for its product.

Answer: C

Type: A Topic: 2 E: 415-416 MI: 171-17222. For a purely competitive seller, price equals:A) average revenue.B) marginal revenue. C) total revenue divided by output. D) allof the above.  Answer: D  

Type: A Topic: 2 E: 415 MI: 171

23.For a purely competitive firm total revenue:

A)is price times quantity sold.

McConnell/Brue: Economics, 16/e Page 693

Chapter 23: Pure Competition

B)increases by a constant absolute amount as output expands.

C)graphs as a straight upsloping line from the origin.

D)has all of the above characteristics.

Answer: D

Type: A Topic: 2 E: 416 MI: 172

24.The marginal revenue curve of a purely competitive firm:

A)lies below the firm's demand curve.

B)increases at an increasing rate as output expands.

C)is horizontal at the market price.

D)is downsloping because price must be reduced to sell more output. Answer: C

McConnell/Brue: Economics, 16/e Page 694

Page 243: Microeconomics Study Guide

Chapter 23: Pure Competition

Type: A Topic: 2 E: 415 MI: 171

25. The demand curve in a purely competitive industry is ______, while the demand curve to a

single firm in that industry is ______.    A) perfectly inelastic, perfectly elastic

C)downsloping, perfectly inelastic

B) downsloping, perfectly elastic D)perfectly elastic, downslopingAnswer: B    

Type: A Topic: 2 E: 415 MI: 171

26.A perfectly elastic demand curve implies that the firm:

A)must lower price to sell more output.

B)can sell as much output as it chooses at the existing price.

C)realizes an increase in total revenue which is less than product price when it sells an extra unit.

D)is selling a differentiated (heterogeneous) product.

Answer: B

Type: A Topic: 2 E: 415-416 MI: 171-172

27.The vertical distance between the horizontal axis and any point on a pure competitor's demand curve measures:

A)total revenue.

B)total cost.

C)product price, marginal revenue, and average revenue.

D)the quantity demanded.

Answer: C

Type: A Topic: 2 E: 416 MI: 172

28.The fact that a purely competitive firm's total revenue curve is linear and upsloping to the right implies that:

A)product price increases as output increases.

B)product price decreases as output increases.

C)product price is constant at all levels of output.

Page 244: Microeconomics Study Guide

D)marginal revenue declines as more output is produced.

Answer: C

Type: A Topic: 2 E: 415 MI: 171

29.Which of the following statements is correct?

A)The demand curve for a purely competitive firm is perfectly elastic, but the demand curve for a purely competitive industry is downsloping.

B)The demand curve for a purely competitive firm is downsloping, but the demand curve for a purely competitive industry is perfectly elastic.

C)The demand curves are downsloping for both a purely competitive firm and a purely competitive industry.

McConnell/Brue: Economics, 16/e Page 695

Chapter 23: Pure Competition

D)The demand curves are perfectly elastic for both a purely competitive firm and a purely competitive industry.

Answer: A

McConnell/Brue: Economics, 16/e Page 696

Chapter 23: Pure Competition

Use the following to answer questions 30-31:

Type: G Topic: 2 E: 416 MI: 172

30. Refer to the above diagram, which pertains to a purely competitive firm. Curve A

represents:    A) total revenue and marginal revenue.C)total revenue and average revenue.B) marginal revenue only. D)total revenue only.Answer: D    

Type: G Topic: 2 E: 415-416 MI: 171-172

Page 245: Microeconomics Study Guide

31.Refer to the above diagram, which pertains to a purely competitive firm. Curve C represents:

A)total revenue and marginal revenue.

B)marginal revenue only.

Answer: D

C)total revenue and average revenue.

D)average revenue and marginal revenue.

Type: A Topic: 2 E: 416 MI: 172

32.A purely competitive seller's average revenue curve coincides with:

A)its marginal revenue curve only.

B)its demand curve only.

C)both its demand and marginal revenue curves.

D)neither its demand nor its marginal revenue curve.

Answer: C

Type: D Topic: 2 E: 416 MI: 172

33.Marginal revenue is the:

A)change in product price associated with the sale of one more unit of output.

B)change in average revenue associated with the sale of one more unit of output.

C)difference between product price and average total cost.

D)change in total revenue associated with the sale of one more unit of output. Answer: D

McConnell/Brue: Economics, 16/e Page 697

Chapter 23: Pure Competition

Type: A Topic: 2 E: 416 MI: 172  34. Marginal revenue for a purely competitive firm: A) is greater than price. C) is equal to price.B) is less than price. D) may be either greater or less than price.Answer: C    

McConnell/Brue: Economics, 16/e Page 698

Page 246: Microeconomics Study Guide

Chapter 23: Pure Competition

Profit maximizing in short run

Type: A Topic: 3 E: 416 MI: 172

35.Firms seek to maximize:

A)per unit profit. B) total revenue. C) total profit. D) market share. Answer: C

Type: A Topic: 3 E: 418 MI: 174

36.A competitive firm in the short run can determine the profit-maximizing (or loss-minimizing) output by equating:

A)price and average total cost.

B)price and average fixed cost. Answer: C

C)marginal revenue and marginal cost.

D)price and marginal revenue.

Type: A Topic: 3 E: 417 MI: 173

37.In the short run a purely competitive firm that seeks to maximize profit will produce:

A)where the demand and the ATC curves intersect.

B)where total revenue exceeds total cost by the maximum amount.

C)that output where economic profits are zero.

D)at any point where the total revenue and total cost curves intersect.

Answer: B

Use the following to answer questions 38-41:

Page 247: Microeconomics Study Guide

Type: G Topic: 3 E: 417-418 MI: 173-174

38.Refer to the above short-run data. Total fixed cost for this firm is:

A)about $67. B) $300. C) $200. D) $100.

McConnell/Brue: Economics, 16/e Page 699

Chapter 23: Pure Competition

Answer: C

McConnell/Brue: Economics, 16/e Page 700

Chapter 23: Pure Competition

Type: G Topic: 3 E: 417 MI: 173

39. Refer to the above short-run data. The shape of the total cost curve reflects:

A)diminishing opportunity costs.C)increasing and diminishing returns.B)the law of rising fixed costs. D)economies and diseconomies of scale.Answer: C    

Type: G Topic: 3 E: 418 MI: 174

40.Refer to the above short-run data. The profit-maximizing output for this firm is:

A)above 440 units. B) 440 units. C) 320 units. D) 100 units.

Answer: C

Type: G Topic: 3 E: 418 MI: 174

41.Refer to the above short-run data. Which of the following is correct?

A)This firm will maximize its profit at 440 untis of output.

B)Any level of output between 100 and 440 units will yield an economic profit.

Page 248: Microeconomics Study Guide

C)This firm's marginal revenue rises with output.

D)Any level of output less than 100 units or greater than 440 units is profitable. Answer: B

Type: A Topic: 3 E: 417 MI: 173

42.A competitive firm will maximize profits at that output at which:

A)total revenue exceeds total cost by the greatest amount.

B)total revenue and total cost are equal.

C)price exceeds average total cost by the largest amount.

D)the difference between marginal revenue and price is at a maximum. Answer: A

Use the following to answer questions 43-48:

Type: G Topic: 3 E: 418 MI: 174

McConnell/Brue: Economics, 16/e Page 701

Chapter 23: Pure Competition

43. Curve (1) in the above diagram is a purely competitive firm's:

A)total cost curve. C)marginal revenue curveB)total revenue curve.D)total economic profit curve.Answer: D    

McConnell/Brue: Economics, 16/e Page 702

      Chapter 23: Pure CompetitionType: G Topic: 3 E: 416 MI: 17244. Curve (2) in the above diagram is a purely competitive firm'sA) total cost curve.   C) marginal revenue curveB) total revenue curve. D) total economic profit curve.Answer: C    Type: G Topic: 3 E: 418 MI: 17445. Curve (3) in the above diagram is a purely competitive firm'sA) total cost curve.   C) marginal revenue curve.B) total revenue curve. D) total economic profit curve.Answer: B    Type: G Topic: 3 E: 418 MI: 17446. Curve (4) in the above diagram is a purely competitive firm's:A) total cost curve. B) total revenue curve. C) marginal revenue curve. D) total profitcurve.      Answer: A    Type: G Topic: 3 E: 418 MI: 174

47.Refer to the above diagram. Other things equal, an increase of product price would be shown as:

Page 249: Microeconomics Study Guide

A)an increase in the steepness of curve (3), an upward shift in curve (2), and upward shift in curve (1).

B)a decrease in the steepness of curve (3), a downward shift in curve (2), and an upward shift in curve (1).

C)an downward shift in curve (4) and an upward shift in curve (1), with no changes in lines (2) and (3).

D)an upward shift in line (2) only.

Answer: A

Type: G Topic: 3 E: 418 MI: 174

48.The firm represented by the above diagram would maximize its profit where:

A)curves (2) and (1) intersect.

B)curve (1) touches the horizontal axis for the second time.

C)the vertical distance between curves (3) and (4) is the greatest.

D)curves (3) and (4) intersect.

Answer: C

Type: D Topic: 3 E: 417-418 MI: 173-174

49.A firm reaches a break-even point (normal profit position) where:

A)marginal revenue cuts the horizontal axis.

McConnell/Brue: Economics, 16/e Page 703

Chapter 23: Pure Competition

B)marginal cost intersects the average variable cost curve.

C)total revenue equals total variable cost.

D)total revenue and total cost are equal.

Answer: D

Type: A Topic: 3 E: 419 MI: 175    50. The MR = MC rule applies:    A) to firms in all types of industries. C)only to monopolies.B) only when the firm is a "price taker."D)only to purely competitive firms.Answer: A    

McConnell/Brue: Economics, 16/e Page 704

Chapter 23: Pure Competition

Page 250: Microeconomics Study Guide

Type: A Topic: 3 E: 418 MI: 174

51.When a firm is maximizing profit it will necessarily be:

A)maximizing profit per unit of output.

B)maximizing the difference between total revenue and total cost.

C)minimizing total cost.

D)maximizing total revenue.

Answer: B

Type: A Topic: 3 E: 419 MI: 175

52.The MR = MC rule can be restated for a purely competitive seller as P = MC because:

A)each additional unit of output adds exactly its price to total revenue.

B)the firm's average revenue curve is downsloping.

C)the market demand curve is downsloping.

D)the firm's marginal revenue and total revenue curves will coincide.

Answer: A

Type: A Topic: 3 E: 424 MI: 180

53.In the short run the individual competitive firm's supply curve is that segment of the:

A)average variable cost curve lying below the marginal cost curve.

B)marginal cost curve lying above the average variable cost curve.

C)marginal revenue curve lying below the demand curve.

D)marginal cost curve lying between the average total cost and average variable cost curves.

Answer: B

Type: A Topic: 3 E: 424 MI: 180

54.Which of the following is not a valid generalization concerning the relationship between price and costs for a purely competitive seller in the short run?

A)Price must be at least equal to average total cost.

B)Price times quantity produced must be equal to or greater than total variable cost for some level of output or the firm will close down in the short run.

Page 251: Microeconomics Study Guide

C)Price may be equal to, greater than, or less than average total cost.

D)Price must be equal to or greater than minimum average variable cost for the firm to

continue producing. Answer: A

Type: A Topic: 3 E: 418 MI: 174

55.Assume the XYZ Corporation is producing 20 units of output. It is selling this output in a purely competitive market at $10 per unit. Its total fixed costs are $100 and its average variable cost is $3 at 20 units of output. This corporation:

A)should close down in the short run.

B)is maximizing its profits.

C)is realizing a loss of $60.

D)is realizing an economic profit of $40.

McConnell/Brue: Economics, 16/e Page 705

Chapter 23: Pure Competition

Answer: D

McConnell/Brue: Economics, 16/e Page 706

Chapter 23: Pure Competition

Type: D Topic: 3 E: 424 MI: 180

56.A purely competitive firm's short-run supply curve is:

A)perfectly elastic at the minimum average total cost.

B)upsloping and equal to the portion of the marginal cost curve that lies above the average variable cost curve.

C)upsloping and equal to the portion of the marginal cost curve that lies above the average total cost curve.

D)upsloping only when the industry has constant costs.

Answer: B

Type: A Topic: 3 E: 424 MI: 180

57.Suppose you find that the price of your product is less than minimum AVC. You should:

A)minimize your losses by producing where P = MC.

B)maximize your profits by producing where P = MC.

Page 252: Microeconomics Study Guide

C)close down because, by producing, your losses will exceed your total fixed costs.

D)close down because total revenue exceeds total variable cost.

Answer: C

Type: A Topic: 3 E: 422 MI: 178

58.If a purely competitive firm shuts down in the short run:

A)its loss will be zero.

B)it will realize a loss equal to its total variable costs.

C)it will realize a loss equal to its total fixed costs.

D)it will realize a loss equal to its total costs.

Answer: C

Type: A Topic: 3 E: 422 MI: 178

59.A purely competitive firm should produce in the short run if its total revenue is sufficient to cover its:

A)total variable costs. B) total costs. C) total fixed costs. D) marginal costs. Answer: A

Use the following to answer questions 60-64:

Answer the next question(s) on the basis of the following data confronting a firm:

McConnell/Brue: Economics, 16/e Page 707

Chapter 23: Pure Competition

  MarginalMarginalOutput

revenue cost

0 -- --1 $16 $102 16 93 16 134 16 175 16 21

Type: T Topic: 3 E: 416 MI: 172

60. Refer to the above data. This firm is selling its output in a(n):

A)imperfectly competitive market.

C)purely competitive market.

B)monopolistic market. D)oligopolistic market.

Page 253: Microeconomics Study Guide

Answer: C    

McConnell/Brue: Economics, 16/e Page 708

Chapter 23: Pure Competition

Type: T Topic: 3 E: 424 MI: 180

61. Refer to the above data. If the firm's minimum average variable cost is $10, the firm's profit-maximizing level of output would be:

A) 2. B) 3. C) 4. D) 5. Answer: B

Type: T Topic: 3 E: 417 MI: 173

62.Refer to the above data. At the profit-maximizing output the firm's total revenue is:

A)$48. B) $32. C) $80. D) $64.

Answer: A

Type: T Topic: 3 E: 417 MI: 173

63.Refer to the above data. At the profit-maximizing output the firm's total cost is:

A)$48. B) $32. C) $80. D) $64.

Answer: B

Type: T Topic: 3 E: 417 MI: 173 64. Refer to the above data. The firm's:

A)economic profit is $12. B) economic profit is $16. C) loss is $14. D) economic profit is $3.

Answer: B

Type: A Topic: 3 E: 424 MI: 180

65. In the short run a purely competitive firm will always make an economic profit if: A) P = ATC. B) P > AVC. C) P = MC. D) P > ATC.

Answer: D

Type: A Topic: 3 E: 424 MI: 180

66.Suppose that at 500 units of output marginal revenue is equal to marginal cost. The firm is selling its output at $5 per unit and average total cost at 500 units of output is $6. On the basis of this information we:

A)can say that the firm should close down in the short run.

B)can say that the firm can produce and realize an economic profit in the short run.

Page 254: Microeconomics Study Guide

C)cannot determine whether the firm should produce or shut down in the short run.

D)can assume the firm is not using the most efficient technology.

Answer: C

Type: A Topic: 3 E: 424 MI: 180

67.If a firm is confronted with economic losses in the short run, it will decide whether or not to produce by comparing:

McConnell/Brue: Economics, 16/e Page 709

Chapter 23: Pure Competition

A)marginal revenue and marginal cost. C) total revenue and total cost.B)price and minimum average variable cost.D) total revenue and total fixed cost.Answer: B  

McConnell/Brue: Economics, 16/e Page 710

Chapter 23: Pure Competition

Type: A Topic: 3 E: 424 MI: 180

68.A firm finds that at its MR = MC output, its TC = $1000, TVC = $800, TFC = $200, and total revenue is $900. This firm should:

A)shut down in the short run.

B)produce because the resulting loss is less than its TFC.

C)produce because it will realize an economic profit.

D)liquidate its assets and go out of business.

Answer: B

Page 255: Microeconomics Study Guide

Type: A Topic: 3 E: 424 MI: 180

69.The lowest point on a purely competitive firm's short-run supply curve corresponds to:

A)the minimum point on its ATC curve. C) the minimum point on its AFC curve.

B)the minimum point on its AVC curve. D) the minimum point on its MC curve. Answer: B

Use the following to answer questions 70-73:

Type: G Topic: 3 E: 423-424 MI: 179-180

70.Refer to the above diagram for a purely competitive producer. The lowest price at which the firm should produce (as opposed to shutting down) is:

A) P1. B) P2. C) P3. D) P4.

Answer: B

Type: G Topic: 3 E: 423-424 MI: 179-180

71.Refer to the above diagram for a purely competitive producer. The firm will produce at a loss at all prices:

A)above P1. B) above P3. C) above P4. D) between P2 and P 3.

Answer: D

McConnell/Brue: Economics, 16/e Page 711

Chapter 23: Pure Competition

Type: G Topic: 3 E: 424 MI: 180

72. Refer to the above diagram for a purely competitive producer. If product price is P3:

A)the firm will maximize profit at point d.C)economic profits will be zero.

B)the firm will earn an economic profit.D)

new firms will enter this industry.

Answer: C    

McConnell/Brue: Economics, 16/e Page 712

Page 256: Microeconomics Study Guide

Chapter 23: Pure Competition

Type: G Topic: 3 E: 424 MI: 180

73. Refer to the above diagram for a purely competitive producer. The firm's short-run supply

curve is:    A) the abcd segment of the MC curve.

C)the cd segment of the MC curve.

B) the bcd segment of the MC curve. D)not shown.Answer: B    

Type: A Topic: 3 E: 424 MI: 180

74.The short-run supply curve of a purely competitive producer is based on its: A) AVC curve. B) ATC curve. C) AFC curve. D) MC curve. Answer: D

Type: A Topic: 3 E: 424 MI: 180

75. On a per unit basis economic profit can be determined as the difference between:

A) marginal revenue and product price.C)marginal revenue and marginal cost.B) product price and average total cost.D)average fixed cost and product price.Answer: B    

Type: A Topic: 3 E: 423-424 MI: 179-180

76.In the short run a purely competitive seller will shut down if:

A)it cannot produce at an economic profit.

B)price is less than average variable cost at all outputs.

C)price is less than average fixed cost at all outputs.

D)there is no point at which marginal revenue and marginal cost are equal. Answer: B

Use the following to answer questions 77-81:

Page 257: Microeconomics Study Guide

Type: G Topic: 3 E: 424 MI: 180

McConnell/Brue: Economics, 16/e Page 713

Chapter 23: Pure Competition

77. Refer to the above diagram. To maximize profit or minimize losses this firm will produce: A) K units at price C. B) D units at price J. C) E units at price A. D) E units at price

B.

Answer: C

McConnell/Brue: Economics, 16/e Page 714

Chapter 23: Pure Competition

Type: G Topic: 3 E: 420 MI: 176

78. Refer to the above diagram. At the profit-maximizing output, total revenue will be: A) 0AHE. B) 0BGE. C) 0CFE. D) ABGE.

Answer: A

Type: G Topic: 3 E: 420 MI: 176

79.Refer to the above diagram. At the profit-maximizing output, total fixed cost is equal to:

A)0AHE. B) 0BGE. C) 0CFE. D) BCFG.

Answer: D

Type: G Topic: 3 E: 420 MI: 176

80.Refer to the above diagram. At the profit-maximizing output, total variable cost is equal to:

A)0AHE. B) 0CFE. C) 0BGE. D) ABGH.

Answer: B

Type: G Topic: 3 E: 420 MI: 176

81. Refer to the above diagram. At the profit-maximizing output, the firm will realize:

A) a loss equal to BCFG.C)an economic profit of ACFH.

B) a loss equal to ACFH.D)

an economic profit of ABGH.

Answer: D    

Type: A Topic: 3 E: 420 MI: 176

Page 258: Microeconomics Study Guide

82.If a purely competitive firm is producing at some level less than the profit-maximizing output, then:

A)price is necessarily greater than average total cost.

B)fixed costs are large relative to variable costs.

C)price exceeds marginal revenue.

D)marginal revenue exceeds marginal cost.

Answer: D

McConnell/Brue: Economics, 16/e Page 715

Chapter 23: Pure Competition

Use the following to answer questions 83-87:

Answer the next question(s) on the basis of the following cost data for a firm that is selling in a purely competitive market:

  AverageAverageAverage Total fixed variable total Marginalproductcost cost cost cost1 $100.00 $17.00 $117.00 $172 50.00 16.00 66.00 153 33.33 15.00 48.33 134 25.00 14.25 39.25 125 20.00 14.00 34.00 136 16.67 14.00 30.67 147 14.29 15.71 30.00 268 12.50 17.50 30.00 309 11.11 19.44 30.55 3510 10.00 21.60 31.60 4111 9.09 24.00 33.09 4812 8.33 26.67 35.00 56

Type: T Topic: 3 E: 421 MI: 177

83.Refer to the above data. If the market price for the firm's product is $12, the competitive firm will produce:

A)

4 units at a loss of $109. C) 8 units at a loss of $48.80.

B)4 units at an economic profit of $31.75. D)zero units at a loss of $100.Answer: D    Type: T Topic: 3 E: 420 MI: 176  

84.Refer to the above data. If the market price for the firm's product is $32, the competitive firm will produce:

A)8 units at an economic profit of $16.

Page 259: Microeconomics Study Guide

B)5 units at a loss of $10.

C)8 units at a loss equal to the firm's total fixed cost.

D)7 units at an economic profit of $41.50.

Answer: A

Type: T Topic: 3 E: 421 MI: 177

85.Refer to the above data. If the market price for the firm's product is $28, the competitive firm will:

A)produce 4 units at a loss of $17.40.

B)produce 7 units at a loss of $14.00. Answer: B

C)close down in the short run.

D)produce 6 units at a loss of $23.80.

McConnell/Brue: Economics, 16/e Page 716

Chapter 23: Pure Competition

Type: T Topic: 3 E: 423 MI: 179

86. Refer to the above data. Which of the following is the firm's short-run supply schedule?

(a)   (b)   (c)   (d)  PriceQsPriceQsPriceQsPriceQs$50 12$50 12$50 11$50 1142 1042 1142 1042 1036 8 36 9 36 9 36 932 8 32 8 32 8 32 820 6 20 6 20 6 20 613 0 13 5 13 0 13 5

Answer: C

Type: T Topic: 3 E: 425 MI: 181

87.Refer to the above data. If there were 1,000 identical firms in this industry and total or market demand is as shown below, equilibrium price will be:

PriceQuantity demanded$50 3,00042 6,00036 9,00032 11,00020 14,000

Page 260: Microeconomics Study Guide

1319,500

A)$32. B) $42. C) $36. D) $13. Answer: C

Type: A Topic: 3 E: 423-424 MI: 179-180

88.If at the MC = MR output, AVC exceeds price:

A)new firms will enter this industry.

B)the firm should produce the MC = MR output and realize an economic profit.

C)the firm should shut down in the short run.

D)the firm should expand its plant.

Answer: C

McConnell/Brue: Economics, 16/e Page 717

Chapter 23: Pure Competition

Use the following to answer questions 89-94:

Type: G Topic: 3 E: 424 MI: 180

89.Refer to the above diagram. The profit-maximizing output:

A)is n. B) is k. C) is h. D) cannot be determined from the information given. Answer: A

Type: G Topic: 3 E: 424 MI: 180

90.Refer to the above diagram. At the profit-maximizing output, average variable cost is:

A)ef. B) fg. C) na. D) ac.

Answer: C

Page 261: Microeconomics Study Guide

Type: G Topic: 3 E: 424 MI: 180

91. Refer to the above diagram. At the profit-maximizing output, total profit is: A) efbc. B) fgab. C) egac. D) 0fbn.

Answer: A

Type: G Topic: 3 E: 424 MI: 180

92.Refer to the above diagram. For any level of output, total fixed cost:

A)is fgab. B) is 0gan. C) is ba. D) is efbc.

Answer: A

Type: G Topic: 3 E: 424 MI: 180

93.Refer to the above diagram. The short-run supply curve for this firm is the:

A)entire MC curve.

B)segment of the AVC curve lying to the right of the MC curve.

C)segment of the MC curve lying above the ATC curve.

McConnell/Brue: Economics, 16/e Page 718

Chapter 23: Pure Competition

D) segment of the MC curve lying above the AVC curve. Answer: D

McConnell/Brue: Economics, 16/e Page 719

Chapter 23: Pure Competition

Type: G Topic: 3 E: 416 MI: 172

94. Refer to the above diagram. This firm is selling its product in a(n):

A)

purely competitive market. C)monopsonistic market.

B)imperfectly competitive market.D)monopolistic market.Answer: A    

Type: A Topic: 3 E: 423-424 MI: 179-180

95.In the short run a purely competitive seller will shut down if product price:

A)equals average revenue. B) is greater than MC. C) is less than AVC. D) is less than ATC.

Answer: C

Page 262: Microeconomics Study Guide

Type: A Topic: 3 E: 423-424 MI: 179-180

96.The short-run shut-down point for a purely competitive firm occurs:

A)at any point where price is less than the minimum AVC.

B)between the two break-even points.

C)at any point where total revenue is less than total cost.

D)at any point where the firm is not making an economic profit. Answer: A

Type: A Topic: 3 E: 427 MI: 183

97.In a purely competitive industry:

A)there will be no economic profits in either the short run or the long run.

B)economic profits may persist in the long run if consumer demand is strong and stable.

C)there may be economic profits in the short run, but not in the long run.

D)there may be economic profits in the long run, but not in the short run.

Answer: C

Type: A Topic: 3 E: 424 MI: 180

98.The short-run supply curve for a purely competitive industry can be found by:

A)multiplying the AVC curve of the representative firm by the number of firms in the industry.

B)adding horizontally the AVC curves of all firms.

C)summing horizontally the segments of the MC curves lying above the AVC curve for all firms.

D)adding horizontally the immediate market period supply curves of each firm. Answer: C

Type: A Topic: 3 E: 419 MI: 175

99.DASH Airlines is considering the addition of a flight from Red Cloud to David City. The total cost of the flight would be $1100 of which fixed costs are $800. Expected revenues

McConnell/Brue: Economics, 16/e Page 720

Chapter 23: Pure Competition

from the flight are $600. DASH should:

A)not add this flight because only flights which cover their full costs are profitable.

Page 263: Microeconomics Study Guide

B)not add this flight because it is not profitable at the margin.

C)add this flight because marginal revenue exceeds marginal costs.

D)not add this flight because total costs exceed total revenue.

Answer: C

McConnell/Brue: Economics, 16/e Page 721

Chapter 23: Pure Competition

Type: A Topic: 3 E: 421 MI: 177

100.In contrast to American firms, Japanese firms frequently make lifetime employment commitments to their workers and agree not to lay them off when product demand is weak. Other things being equal, we would expect Japanese firms to:

A)face more elastic product demand curves than American firms.

B)have relatively greater variable costs than American firms.

C)discontinue production at higher product prices than would American firms.

D)continue to produce in the short run at lower prices than would American firms. Answer: D

Type: A Topic: 3 E: 421 MI: 177

101.Assume for a competitive firm that MC = AVC at $12, MC = ATC at $20, and MC = MR at $16. This firm will:

A)realize a profit of $4 per unit of output.

B)maximize its profit by producing in the short run.

C)minimize its losses by producing in the short run.

D)shut down in the short run.

Answer: C

Type: A Topic: 3 E: 418 MI: 174

102.The principle that a firm should produce up to the point where the marginal revenue from the sale of an extra unit of output is equal to the marginal cost of producing it is known as the:

A)output-maximizing rule. B) profit-maximizing rule. C) shut-down rule. D)

break-even rule. Answer: B

Type: A Topic: 3 E: 419 MI: 175

Page 264: Microeconomics Study Guide

103.If a purely competitive firm is producing at the P = MC output and realizing an economic profit, at that output:

A)marginal revenue is less than price.

B)marginal revenue exceeds ATC. Answer: B

C)ATC is being minimized.

D)total revenue equals total cost.

Type: A Topic: 3 E: 418 MI: 174

104.If a profit-seeking competitive firm is producing its profit-maximizing output and its total fixed costs fall by 25 percent, the firm should:

A)use more labor and less capital to produce a larger output.

B)not change its output.

C)reduce its output.

D)increase its output.

Answer: B

McConnell/Brue: Economics, 16/e Page 722

Chapter 23: Pure Competition

McConnell/Brue: Economics, 16/e Page 723

Chapter 23: Pure Competition

Use the following to answer questions 105-108:

Type: G Topic: 3 E: 424 MI: 180

Page 265: Microeconomics Study Guide

105.Refer to the above diagram. At P2, this firm will:

A)produce 44 units and realize an economic profit.

B)produce 44 units and earn only a normal profit.

C)produce 66 units and earn only a normal profit.

D)shut down in the short run.

Answer: B

Type: G Topic: 3 E: 424 MI: 180    106. Refer to the above diagram. At P1, this firm will produce:A)

47 units and break even. C) 66 units and earn only a normal profit.

B)47 units and realize an economic profit. D) 24 units and earn only a normal profit.Answer: B      Type: G Topic: 3 E: 423-424 MI: 179-180 107. Refer to the above diagram. At P4, this firm will:A) shut down in the short run. C) produce 30 units and earn only a normal  profit.        B) produce 30 units and incur a loss. D) produce 10 units and earn only a normal  profit.        Answer: A      

Type: G Topic: 3 E: 423-424 MI: 179-180

108.Refer to the above diagram. At P3, this firm will:

A)produce 14 units and realize an economic profit.

B)produce 62 units and earn only a normal profit.

C)produce 40 units and incur a loss.

D)shut down in the short run.

McConnell/Brue: Economics, 16/e Page 724

Chapter 23: Pure Competition

Answer: C

Type: A Topic: 3 E: 423-424 MI: 179-180

109. The loss of a purely competitive firm which shuts down in the short run:

A)

is equal to its total variable costs.C)is equal to its total fixed costs.

B)is zero. D)cannot be determined.

Page 266: Microeconomics Study Guide

Answer: C    

McConnell/Brue: Economics, 16/e Page 725

Chapter 23: Pure Competition

Type: A Topic: 3 E: 423-424 MI: 179-180

110. The Ajax Manufacturing Company is selling in a purely competitive market. Its output is 100 units which sell at $4 each. At this level of output total cost is $600, total fixed cost is

$100, and marginal cost is $4. The firm should: A) reduce output to about 80 units. C) continue to produce 100 units.B) expand its production. D) produce zero units of output.Answer: D    

Type: A Topic: 3 E: 419 MI: 175

111.The MR = MC rule can be restated for a purely competitive seller as P = MC because:

A)each additional unit of output adds exactly its constant price to total revenue.

B)the firm's average revenue curve is downsloping.

C)the market demand curve is downsloping.

D)the firm's marginal revenue and total revenue curves will coincide.

Answer: A

Type: A Topic: 3 E: 421 MI: 177

112.If a purely competitive firm is maximizing economic profit:

A)it is necessarily maximizing per-unit profit.

B)it may or may not be maximizing per unit profit.

C)then per-unit profit will be minimized.

D)it is necessarily overallocating resources to its product. Answer: B

Use the following to answer questions 113-115:

Answer the next question(s) on the basis of the following cost data for a purely competitive seller:

Total

Output cost

0$ 50

Page 267: Microeconomics Study Guide

190

2120

3140

4170

5210

6260

7330

Type: T Topic: 3 E: 417 MI: 173

113.Refer to the above data. If product price is $60, the firm will:

A)shut down.

B)produce 4 units and realize a $120 economic profit.

C)produce 6 units and realize a $100 economic profit.

D)produce 3 units and incur a $40 loss.

McConnell/Brue: Economics, 16/e Page 726

Chapter 23: Pure Competition

Answer: C

McConnell/Brue: Economics, 16/e Page 727

Chapter 23: Pure Competition

Type: T Topic: 3 E: 417 MI: 173

114.Refer to the above data. If product price is $45, the firm will:

A)shut down.

B)produce 4 units and realize a $120 economic profit.

C)produce 5 units and realize a $15 economic profit.

D)produce 6 units and realize a $100 economic profit. Answer: C

Type: T Topic: 3 E: 422 MI: 178

115.Refer to the above data. If product price is $25, the firm will:

Page 268: Microeconomics Study Guide

A)shut down and incur a $90 loss.

B)shut down and incur a $50 loss.

C)produce 3 units and incur a $65 loss.

D)produce 4 units and realize a $10 economic profit. Answer: B

Type: A Topic: 3 E: 422 MI: 178

116.If total revenue is less than total variable costs at the MR = MC output, a purely competitive firm should:

A) shut down. C) produce and may or may not realize aprofit.  B) produce, but will necessarily realize a loss. D) increase its output.Answer: A  Type: A Topic: 3 E: 417 MI: 173

117.Assume a purely competitive firm is selling 200 units of output at $3 each. At this output its total fixed cost is $100 and its total variable cost is $350. This firm:

A)is maximizing its profit.

B)is making a profit, but not necessarily the maximum profit.

C)is incurring losses.

D)should shut down in the short run.

Answer: B

McConnell/Brue: Economics, 16/e Page 728

Chapter 23: Pure Competition

Use the following to answer questions 118-122:

Page 269: Microeconomics Study Guide

Type: G Topic: 3 E: 424 MI: 180

118. Refer to the above diagram. This firm will earn only a normal profit if product price is: A) P1. B) P2. C) P3. D) P4.

Answer: C

Type: G Topic: 3 E: 424 MI: 180

119. Refer to the above diagram. The firm will realize an economic profit if price is: A) P1. B) P2. C) P3. D) P4.

Answer: D

Type: G Topic: 3 E: 423-424 MI: 179-180

120. Refer to the above diagram. The firm will produce at a loss if price is: A) P1. B) P2. C) P3. D) P4.

Answer: B

Type: G Topic: 3 E: 423-424 MI: 179-180

121. Refer to the above diagram. The firm will shut down at any price less than: A) P1. B) P2. C) P3. D) P4.

Answer: A

Type: G Topic: 3 E: 424 MI: 180 Status: New

122.Refer to the above diagram. The firm's supply curve is the segment of the:

A)MC curve above its intersection with the AVC curve.

B)MC curve above its intersection with the ATC curve.

C)AVC curve above its intersection with the MC curve.

McConnell/Brue: Economics, 16/e Page 729

Chapter 23: Pure Competition

D) ATC curve above its intersection with the MC curve. Answer: A

McConnell/Brue: Economics, 16/e Page 730

Chapter 23: Pure Competition

Use the following to answer questions 123-132:

Answer the next question(s) on the basis of the following cost data for a firm that is selling in a purely competitive market.

Page 270: Microeconomics Study Guide

  AverageAverageAverage Total fixed variable total Marginaloutput

cost cost cost cost

1 $150.00 $25.00 $175.00 $ 25.002 75.00 23.00 98.00 21.003 50.00 20.00 70.00 14.004 37.50 21.00 58.50 24.005 30.00 23.00 53.00 31.006 25.00 25.00 50.00 35.007 21.43 28.00 49.43 46.018 18.75 33.00 51.76 68.079 16.67 39.00 55.67 86.9510 15.00 48.00 63.00 128.97

Type: G Topic: 3 E: 402 MI: 158 Status: New

123. Refer to the above data. The marginal cost column reflects:

A)the law of diminishing returns. C)diseconomies of scale.

B)the law of diminishing marginal utility.

D)economies of scale.

Answer: A    

Type: G Topic: 3 E: 398-399 MI: 154-155 Status: New

124.Refer to the above data. At 6 units of output, total fixed cost is ____ and total cost is ____:

A)$25; $50. B) $50; $300. C) $100, $200. D) $150; $300.

Answer: D

Type: G Topic: 3 E: 398-399 MI: 154-155 Status: New

125. Refer to the above data. At 3 units of output, total variable cost is ____ and total cost is

____:

A) $20; $70. B) $60; $210. C) $20, $210. D) $60; $350. Answer: B

Type: G Topic: 3 E: 398-399 MI: 154-155 Status: New

126.Refer to the above data. We can infer that, at zero output, this firm's total fixed, total variable, and total costs are:

A)zero, zero, and zero, respectively.

B)zero, $25, and $175, respectively. Answer: D

C)$150, $25, and $175, respectively.

Page 271: Microeconomics Study Guide

D)$150, zero, and $150, respectively.

Type: G Topic: 3 E: 419-420 MI: 175-176 Status: New

McConnell/Brue: Economics, 16/e Page 731

Chapter 23: Pure Competition

127.Refer to the above data. If the market price for this firm's product is $86.95, it will produce:

A)9 units at an economic profit of zero.

B)6 units at a loss of $90.

Answer: C

C)9 units at an economic profit of $281.52.

D)8 units at an economic profit of $130.48.

McConnell/Brue: Economics, 16/e Page 732

      Chapter 23: Pure CompetitionType: G Topic: 3 E: 419-420 MI: 175-176 Status: New128. Refer to the above data. If the market price for this firm's product is $68.07, it willproduce:          A) 8 units at an economic profit of zero.C) 9 units at an economic profit of $281.52.B) 6 units at a loss of $90.   D) 8 units at an economic profit of $130.48.Answer: D        Type: G Topic: 3 E: 421-422 MI: 177-178 Status: New129.Refer to the above data. If the market price for this firm's product is $35, it will produce:  A) 6 units at a loss of $150.C) 9 units at an economic profit of $281.52.  B) 6 units at a loss of $90. D) 8 units at an economic profit of $130.48.  Answer: B        Type: GTopic: 3 E: 421-422 MI: 177-178 Status: New130.Refer to the above data. If the market price for this firm's product is $24, it will produce:  A) 4 units at a loss of $150.C) 3 units at an economic profit of zero.  B) 6 units at a loss of $90. D) 4 units at a loss of $138.  Answer: D        Type: GTopic: 3 E: 422-423 MI: 178-179 Status: New131.Refer to the above data. If the market price for this firm's product is $14, it will produce:  A) 0 units at a loss of $150.C) 3 units at an economic profit of zero.  B) 3 units at a loss of $168.D) 4 units at a loss of $138.

Answer: A

Type: G Topic: 3 E: 424 MI: 180 Status: New

132.Refer to the above data. The firm's supply schedule is reflected in the:

A)marginal cost data for the fourth through tenth units of output.

Page 272: Microeconomics Study Guide

B)marginal cost data for the first through tenth units of output.

C)average total cost data for the seventh through tenth units of output.

D)average variable cost data for fifth through tenth units of output. Answer: A

Type: A Topic: 3 E: 417 MI: 173

133.A purely competitive seller should produce (rather than shut down) in the short run:

A)only if total revenue exceeds total cost.

B)only if total cost exceeds total revenue.

C)if total revenue exceeds total cost or if total cost exceeds total revenue by some amount less than total fixed cost.

D)if total cost exceeds total revenue by some amount greater than total fixed cost. Answer: C

McConnell/Brue: Economics, 16/e Page 733

Chapter 23: Pure Competition

Type: A Topic: 3 E: 417-418 MI: 173-174

134.In the short run a purely competitive firm will maximize profit by producing that output at which:

A)total revenue exceeds total cost by a maximum amount.

B)total revenue exceeds total cost by a minimum amount.

C)total revenue and total cost are equal.

D)total fixed cost equals total variable cost.

Answer: A

McConnell/Brue: Economics, 16/e Page 734

Chapter 23: Pure Competition

Use the following to answer questions 135-139:

Answer the next question(s) on the basis of the following cost data for a purely competitive seller:

  TotalTotal  Total fixedvariableTotalproductcost cost cost0 $50 $ 0 $ 501 50 70 1202 50 120 170

Page 273: Microeconomics Study Guide

3 50 150 2004 50 220 2705 50 300 3506 50 390 440

Type: T Topic: 3 E: 417 MI: 173

135.The above data are for:

A)the long run.

B)the short run. Answer: B

C)both the short run and the long run.

D)the intermediate market period only.

Type: T Topic: 3 E: 419-420 MI: 175-176

136.Refer to the above data. At 5 units of output average fixed cost, average variable cost, and average total cost are:

A) $10, $60, and $70 respectively.C)$10, $70, and $80 respectively.B) $50, $40, and $90 respectively.D)$5, $25, and $30 respectively.Answer: A    Type: T Topic: 3 E: 419 MI: 175    

137.Refer to the above data. The marginal cost of the fifth unit of output is:

A)$80. B) $90. C) $50. D) $20.

Answer: A

Type: T Topic: 3 E: 419-420 MI: 175-176

138.Refer to the above data. If product price is $75, the firm will produce:

A)3 units of output. B) 4 units of output. C) 5 units of output. D) 6 units of output. Answer: B

Type: T Topic: 3 E: 419-420 MI: 175-176

139. Refer to the above data. Given the $75 product price, at its optimal output the firm will:

A) realize a $25 economic profit.

C)incur a $25 loss.

B) realize a $30 economic profit.D)realize a $30 loss.Answer: B    

McConnell/Brue: Economics, 16/e Page 735

Chapter 23: Pure Competition

Page 274: Microeconomics Study Guide

Type: A Topic: 3 E: 424 MI: 180

140.A purely competitive firm's short-run supply curve is:

A)the upward sloping portion of its marginal cost curve.

B)the upward sloping portion of its average variable cost curve.

C)its marginal cost curve above average variable cost.

D)its average total cost curve.

Answer: C

Type: A Topic: 3 E: 424 MI: 180

141.In the short run, a purely competitive firm will earn a normal profit when:

A) P = AVC. B) P > MC. C) that firm's MR = market equilibrium price. D) P = ATC. Answer: D

Use the following to answer questions 142-147:

The following table applies to a purely competitive industry composed of 100 identical firms.

Quantity   QuantityDemanded

PriceSupplied

400,000 $5 800,000500,000 4 700,000600,000 3 600,000700,000 2 500,000800,000 1 400,000

Type: T Topic: 3 E: 425 MI: 181

142.Refer to the above table. The equilibrium price in this purely competitive market is:

A)$5. B) $4. C) $3. D) $2.

Answer: C

Type: T Topic: 3 E: 425 MI: 181

143.Refer to the above table. At the equilibrium price, each of the 100 firms in this industry will produce:

A)600,000 units of output.

B)60,000 units of output. Answer: C

C)6,000 units of output

Page 275: Microeconomics Study Guide

D)600 units of output.

Type: T Topic: 3 E: 425-426 MI: 181-182

144.Refer to the above table. For each of the 100 firms in this industry, marginal revenue and total revenue will be:

A)

$4 and $400, respectively. C)$4 and $20,000, respectively.

B)$3 and $30,000, respectively.D)

$3 and $18,000, respectively.

Answer: D    

McConnell/Brue: Economics, 16/e Page 736

Chapter 23: Pure Competition

Type: T Topic: 3 E: 425-426 MI: 181-182

145.Refer to the above table. If each of the 100 firms in the industry is maximizing its profit, each must have a marginal cost of:

A)$5. B) $4. C) $3. D) $2.

Answer: C

McConnell/Brue: Economics, 16/e Page 737

Chapter 23: Pure Competition

Type: T Topic: 3 E: 425-426 MI: 181-182

146. Refer to the above table. If each of the 100 firms in the industry is maximizing its profit and earning only a normal profit, each must have a total cost of:

A) $18,000. B) $20,000. C) $22,000. D) $24,000. Answer: A

Type: T Topic: 3 E: 425-426 MI: 181-182

147.Refer to the above table. If each of the 100 firms in the industry is maximizing its profit and earning only a normal profit, each must have an average total cost of:

A)$2. B) $3. C) $4. D) $5.

Answer: B

Profit maximizing in long run

Type: A Topic: 4 E: 427 MI: 183

Page 276: Microeconomics Study Guide

148.Suppose a firm in a purely competitive market discovers that the price of its product is above its minimum AVC point but everywhere below ATC. Given this, the firm:

A)minimizes losses by producing at the minimum point of its AVC curve.

B)maximizes profits by producing where MR = ATC.

C)should close down immediately.

D)should continue producing in the short run, but leave the industry in the long run. Answer: D

Type: A Topic: 4 E: 427 MI: 183

149.Which of the following is true concerning purely competitive industries?

A)There will be economic losses in the long run because of cut-throat competition.

B)Economic profits will persist in the long run if consumer demand is strong and stable.

C)In the short run, firms may incur economic losses or earn economic profits, but in the long run they earn normal profits.

D)There are economic profits in the long run, but not in the short run.

Answer: C

Type: A Topic: 4 E: 428 MI: 184

150.If a purely competitive firm is producing at the MR = MC output level and earning an economic profit, then:

A)the selling price for this firm is above the market equilibrium price.

B)new firms will enter this market.

C)some existing firms in this market will leave.

D)there must be price fixing by the industry's firms.

Answer: B

McConnell/Brue: Economics, 16/e Page 738

Chapter 23: Pure Competition

Type: A Topic: 4 E: 427 MI: 183 151. Long-run competitive equilibrium:

A)

is realized only in constant-cost industries. C) is not economically efficient.

B)will never change once it is realized. D) results in zero economic profits.Answer: D  

Page 277: Microeconomics Study Guide

McConnell/Brue: Economics, 16/e Page 739

Chapter 23: Pure Competition

Type: A Topic: 4 E: 428 MI: 184

152. We would expect an industry to expand if firms in that industry are:

A)earning normal profits. C)incurring economic losses.B)earning economic profits.D)earning accounting profits.Answer: B    

Type: A Topic: 4 E: 422-423 MI: 178-179

153.Which of the following statements is correct?

A)Economic profits induce firms to enter an industry; losses encourage firms to leave.

B)Economic profits induce firms to leave an industry; profits encourage firms to leave.

C)Economic profits and losses have no significant impact on the growth or decline of an industry.

D)Normal profits will cause an industry to expand.

Answer: A

Type: A Topic: 4 E: 430 MI: 186

154.Suppose a purely competitive increasing-cost industry is in long-run equilibrium. Now assume that a decrease in consumer demand occurs. After all resulting adjustments have been completed, the new equilibrium price:

A)and industry output will be less than the initial price and output.

B)will be greater than the initial price, but the new industry output will be less than the original output.

C)will be less than the initial price, but the new industry output will be greater than the original output.

D)and industry output will be greater than the initial price and output.

Answer: A

Type: A Topic: 4 E: 430 MI: 186

155.Which of the following statements is correct?

A)The long-run supply curve for a purely competitive increasing-cost industry will be upsloping.

B)The long-run supply curve for a purely competitive increasing-cost industry will be perfectly elastic.

Page 278: Microeconomics Study Guide

C)The long-run supply curve for a purely competitive industry will be less elastic than the industry's short-run supply curve.

D)The long-run supply curve for a purely competitive decreasing-cost industry will be upsloping.

Answer: A

Type: A Topic: 4 E: 429 MI: 185

156.A constant-cost industry is one in which:

A)a higher price per unit will not result in an increased output.

McConnell/Brue: Economics, 16/e Page 740

Chapter 23: Pure Competition

B)if 100 units can be produced for $100, then 150 can be produced for $150, 200 for $200, and so forth.

C)the demand curve and therefore the unit price and quantity sold seldom change.

D)the total cost of producing 200 or 300 units is no greater than the cost of producing 100 units.

Answer: B

Type: A Topic: 4 E: 432 MI: 188

157. Which of the following will not hold true for a competitive firm in long-run equilibrium? A) P equals AFC B) P equals minimum ATC C) MC equals minimum ATC D) P

equals MC Answer: A

McConnell/Brue: Economics, 16/e Page 741

Chapter 23: Pure Competition

Type: A Topic: 4 E: 430 MI: 186

158.Assume a purely competitive increasing-cost industry is initially in long-run equilibrium and that an increase in consumer demand occurs. After all economic adjustments have been completed product price will be:

A)lower, but total output will be larger than originally.

B)higher and total output will be larger than originally.

C)lower and total output will be smaller than originally.

D)higher, but total output will be smaller than originally.

Answer: B

Page 279: Microeconomics Study Guide

Type: A Topic: 4 E: 430 MI: 186

159.Assume a purely competitive, increasing-cost industry is in long-run equilibrium. If a decline in demand occurs, firms will:

A)leave the industry, price will decrease, and quantity produced will increase.

B)enter the industry and price and quantity will both increase.

C)leave the industry and price and output will both increase.

D)leave the industry and price and output will both decline.

Answer: D

Type: A Topic: 4 E: 427 MI: 183

160.When a purely competitive firm is in long-run equilibrium:

A)marginal revenue exceeds marginal cost.

B)price equals marginal cost.

C)total revenue exceeds total cost.

D)minimum average total cost is less than the product price. Answer: B

Type: A Topic: 4 E: 427 MI: 183

161.A purely competitive firm:

A)must earn a normal profit in the short run.

B)cannot earn economic profit in the long run.

C)may realize either economic profit or losses in the long run.

D)cannot earn economic profit in the short run.

Answer: B

Type: A Topic: 4 E: 429 MI: 185

162.A constant-cost industry is one in which:

A)resource prices fall as output is increased.

B)resource prices rise as output is increased.

C)resource prices remain unchanged as output is increased.

Page 280: Microeconomics Study Guide

D)small and large levels of output entail the same total costs. Answer: C

McConnell/Brue: Economics, 16/e Page 742

Chapter 23: Pure Competition

Type: A Topic: 4 E: 430 MI: 186

163.An increasing-cost industry is associated with:

A)a perfectly elastic long-run supply curve. C) a perfectly inelastic long-run supply curve.

B) an upsloping long-run supply curve.

D) an upsloping long-run demand curve.

Answer: B  

McConnell/Brue: Economics, 16/e Page 743

Chapter 23: Pure Competition

Use the following to answer questions 164-166:

Type: G Topic: 4 E: 426 MI: 182

164.Refer to the above diagrams, which pertain to a purely competitive firm producing output q and the industry in which it operates. Which of the following is correct?

A)The diagrams portray neither long-run nor short-run equilibrium.

B)The diagrams portray both long-run and short-run equilibrium.

C)The diagrams portray short-run equilibrium, but not long-run equilibrium.

D)The diagrams portray long-run equilibrium, but not short-run equilibrium.

Answer: C

Type: G Topic: 4 E: 427-428 MI: 183-184

165.Refer to the above diagrams, which pertain to a purely competitive firm producing output q and the industry in which it operates. In the long run we should expect:

Page 281: Microeconomics Study Guide

A)firms to enter the industry, market supply to rise, and product price to fall.

B)firms to leave the industry, market supply to rise, and product price to fall.

C)firms to leave the industry, market supply to fall, and product price to rise.

D)no change in the number of firms in this industry.

Answer: C

Type: G Topic: 4 E: 432 MI: 188

166.Refer to the above diagrams, which pertain to a purely competitive firm producing output q and the industry in which it operates. The predicted long-run adjustments in this industry might be offset by:

A)a decline in product demand.

B)an increase in resource prices.

C)a technological improvement in production methods.

D)entry of new firms into the industry.

Answer: C

Type: A Topic: 4 E: 427 MI: 183

167.Assume a purely competitive firm is maximizing profit at some output at which long-run average total cost is at a minimum. Then:

McConnell/Brue: Economics, 16/e Page 744

Chapter 23: Pure Competition

A)the firm is earning an economic profit.

B)there is no tendency for the firm's industry to expand or contract.

C)allocative but not productive efficiency is being achieved.

D)other firms will enter this industry.

Answer: B

McConnell/Brue: Economics, 16/e Page 745

Chapter 23: Pure Competition

Type: A Topic: 4 E: 430 MI: 186

168.An increasing-cost industry is the result of:

Page 282: Microeconomics Study Guide

A)higher resource prices which occur as the industry expands.

B)a change in the industry's minimum efficient scale.

C)X-inefficiency.

D)the law of diminishing returns.

Answer: A

Type: A Topic: 4 E: 427 MI: 183

169.A purely competitive firm is precluded from making economic profit in the long run because:

A)it is a "price taker."

B)its demand curve is perfectly elastic. Answer: C

C)of unimpeded entry to the industry.

D)it produces a differentiated product.

Type: A Topic: 4 E: 428-429 MI: 184-185

170.If a purely competitive constant-cost industry is realizing economic profits, we can expect industry supply to:

A)increase, output to increase, price to decrease, and profits to decrease.

B)increase, output to increase, price to increase, and profits to decrease.

C)decrease, output to decrease, price to increase, and profits to increase.

D)increase, output to decrease, price to decrease, and profits to decrease.

Answer: A

Type: A Topic: 4 E: 427 MI: 183

171.Assume that a decline in consumer demand occurs in a purely competitive industry which is initially in long-run equilibrium. We can:

A)predict that the new price will be greater than the original price.

B)predict that the new price will be less than the original price.

C)predict that the new price will be the same as the original price.

D)not compare the original and the new price without knowing about cost conditions in the industry.

Answer: D

Page 283: Microeconomics Study Guide

Type: D Topic: 4 E: 430-431 MI: 186-187

172.A decreasing-cost industry is one in which:

A)contraction of the industry will decrease unit costs.

B)input prices fall or technology improves as the industry expands.

C)the long-run supply curve is perfectly elastic.

D)the long-run supply curve is upsloping.

Answer: B

McConnell/Brue: Economics, 16/e Page 746

Chapter 23: Pure Competition

Type: A Topic: 4 E: 430-431 MI: 186-187

173.When compact disc (CD) players first came on the market, they sold for over $1,000. Now they cost only $100. These facts imply that:

A)the CD industry was once competitive, but is now monopolistic.

B)fewer firms produce CD players than was the case five or ten years ago.

C)the demand curve for CD players has shifted leftward.

D)the CD player industry is a decreasing-cost industry.

Answer: D

McConnell/Brue: Economics, 16/e Page 747

Chapter 23: Pure Competition

Type: A Topic: 4 E: 431 MI: 187

174.Suppose that an industry's long-run supply curve is downsloping. This suggests that:

A)it is an increasing-cost industry.

B)relevant inputs have become more expensive as the industry has expanded.

C)technology has become less efficient as a result of the industry's expansion.

D)it is a decreasing-cost industry.

Answer: D

Page 284: Microeconomics Study Guide

Type: A Topic: 4 E: 430-431 MI: 186-187

175.Suppose an increase in product demand occurs in a decreasing-cost industry. As a result:

A)the new long-run equilibrium price will be lower than the original long-run equilibrium price.

B)equilibrium quantity will decline.

C)firms will eventually leave the industry.

D)the new long-run equilibrium price will be higher than the original price.

Answer: A

Type: C Topic: 4 E: 429-430 MI: 185-186

176.Purely competitive industry X has constant costs and its product is an inferior good. The industry is currently in long-run equilibrium. The economy now goes into a recession and average incomes decline. The result will be:

A)an increase in output and in the price of the product.

B)an increase in output, but not in the price, of the product.

C)a decrease in the output, but not in the price, of the product.

D)a decrease in output and in the price of the product.

Answer: B

Type: A Topic: 4 E: 430 MI: 186

177.Suppose losses cause industry X to contract and, as a result, the prices of relevant inputs decline. Industry X is:

A)

a constant-cost industry. C)an increasing-cost industry.

B)a decreasing-cost industry.D)

encountering X-inefficiency.

Answer: C    

McConnell/Brue: Economics, 16/e Page 748

Page 285: Microeconomics Study Guide

Chapter 23: Pure Competition

Use the following to answer questions 178-183:

Type: G Topic: 4 E: 420 MI: 176

178.Refer to the above diagram showing the average total cost curve for a purely competitive firm. Suppose this firm is maximizing its total profit and the market price is $15. The firm's per unit profit is:

A)$5. B) $200. C) a positive amount less than $5. D) a positive amount more than $200.

Answer: C

Type: G Topic: 4 E: 420 MI: 176

179.Refer to the above diagram showing the average total cost curve for a purely competitive firm. Suppose that total variable cost is $300 at 40 units of output. At that level of output, average fixed cost:

A)is $2.50. B) is $4. C) is $100. D) cannot be determined from the information

provided. Answer: A

Type: G Topic: 4 E: 420 MI: 176

180.Refer to the above diagram showing the average total cost curve for a purely competitive firm. Suppose that average variable cost is $8 at 40 units of output. At that level of output,

total fixed cost:

A)is $2. B) is $40. C) is $80. D) cannot be determined from the information provided.

Answer: C

Type: G Topic: 4 E: 415-416, 427 MI: 171-172, 183

181.Refer to the above diagram showing the average total cost curve for a purely competitive firm. At the long-run equilibrium level of output, this firm's total revenue:

A)is $10. B) is $40. C) is $400. D) cannot be determined from the information provided.

Answer: C

Page 286: Microeconomics Study Guide

McConnell/Brue: Economics, 16/e Page 749

Chapter 23: Pure Competition

Type: G Topic: 4 E: 420, 427 MI: 176, 183

182.Refer to the above diagram showing the average total cost curve for a purely competitive firm. At the long-run equilibrium level of output, this firm's total cost:

A)is $10. B) is $40. C) is $400. D) cannot be determined from the information provided.

Answer: C

Type: G Topic: 4 E: 427 MI: 183

183. Refer to the above diagram showing the average total cost curve for a purely competitive firm. At the long-run equilibrium level of output, this firm's economic profit:

A) is zero. B) is $400. C) is $200. D) cannot be determined from the information provided.

Answer: A

McConnell/Brue: Economics, 16/e Page 750

Chapter 23: Pure Competition

Type: A Topic: 4 E: 427 MI: 183    184. The MR = MC rule applies:    A) in the short run, but not in the long run.

C)in both the short run and the long run.

B) in the long run, but not in the short run.D)only to a purely competitive firm.Answer: C    

Type: A Topic: 4 E: 430 MI: 186

185.If the long-run supply curve of a purely competitive industry slopes upward, this implies that the prices of relevant resources:

A)will fall as the industry expands.

B)are constant as the industry expands. Answer: D

Page 287: Microeconomics Study Guide

C)rise as the industry contracts.

D)rise as the industry expands.

Use the following to answer questions 186-187:

Type: G Topic: 4 E: 430 MI: 186    186. Refer to the above diagram. Line (1) reflects the long-run supply curve for:A) a constant-cost industry.  C) an increasing-cost industry.B) a decreasing-cost industry. D) technologically progressive industry.Answer: C        Type: G Topic: 4 E: 429 MI: 185    187. Refer to the above diagram. Line (2) reflects the long-run supply curve for:A) a constant-cost industry.  C) an increasing-cost industry.B) a decreasing-cost industry. D) technologically progressive industry.Answer: A        

Pure competition and efficiency

Type: A Topic: 5 E: 432 MI: 188

McConnell/Brue: Economics, 16/e Page 751

Chapter 23: Pure Competition

188. Allocative efficiency is achieved when the production of a good occurs where:

A) P = minimum ATC. B) P = MC. C) P = minimum AVC. D) total revenue is equal to TFC.

Answer: B

McConnell/Brue: Economics, 16/e Page 752

Chapter 23: Pure Competition

Type: A Topic: 5 E: 432 MI: 188

189.A firm is producing an output such that the benefit from one more unit is more than the cost of producing that additional unit. This means the firm is:

A)producing more output than allocative efficiency requires.

B)producing less output than allocative efficiency requires.

C)achieving productive efficiency.

D)producing an inefficient output, but we cannot say whether output should be increased

or decreased. Answer: B

Type: A Topic: 5 E: 431 MI: 187

Page 288: Microeconomics Study Guide

190. Resources are efficiently allocated when production occurs where:

A)marginal cost equals average variable cost. C) price is equal to marginal cost.B)price is equal to average revenue. D) price is equal to average variable cost.Answer: C  

Type: D Topic: 5 E: 432 MI: 188

191.The term productive efficiency refers to:

A)any short-run equilibrium position of a competitive firm.

B)the production of the product-mix most desired by consumers.

C)the production of a good at the lowest average total cost.

D)fulfilling the condition P = MC.

Answer: C

Type: A Topic: 5 E: 432 MI: 188

192.If the price of product Y is $25 and its marginal cost is $18:

A)Y is being produced with the least-cost combination of resources.

B)society will realize a net gain if less of Y is produced.

C)resources are being underallocated to Y.

D)resources are being overallocated to Y.

Answer: C

Type: D Topic: 5 E: 432 MI: 188

193.The term allocative efficiency refers to:

A)the level of output that coincides with the intersection of the MC and AVC curves.

B)minimization of the AFC in the production of any good.

C)the production of the product-mix most desired by consumers.

D)the production of a good at the lowest average total cost.

Answer: C

Type: A Topic: 5 E: 433 MI: 189

McConnell/Brue: Economics, 16/e Page 753

Page 289: Microeconomics Study Guide

Chapter 23: Pure Competition

194.Under pure competition in the long run:

A)neither allocative efficiency nor productive efficiency are achieved.

B)both allocative efficiency and productive efficiency are achieved.

C)productive efficiency is achieved, but allocative efficiency is not.

D)allocative efficiency is achieved, but productive efficiency is not. Answer: B

McConnell/Brue: Economics, 16/e Page 754

Chapter 23: Pure Competition

Type: A Topic: 5 E: 432 MI: 188

195.If for a firm P = minimum ATC = MC, then:

A)neither allocative efficiency nor productive efficiency is being achieved.

B)productive efficiency is being achieved, but allocative efficiency is not.

C)both allocative efficiency and productive efficiency are being achieved.

D)allocative efficiency is being achieved, but productive efficiency is not. Answer: C

Use the following to answer questions 196-201:

Type: G Topic: 5 E: 431 MI: 187

196.The above diagram portrays:

A)a competitive firm that should shut down in the short run.

B)the equilibrium position of a competitive firm in the long run.

C)a competitive firm that is realizing an economic profit.

Page 290: Microeconomics Study Guide

D)the loss-minimizing position of a competitive firm in the short run. Answer: B

Type: G Topic: 5 E: 431 MI: 187 Status: New

197.Refer to the above diagram. If this competitive firm produces output Q, it will:

A)suffer an economic loss.

B)earn a normal profit.

C)earn an economic profit.

D)achieve productive efficiency, but not allocative efficiency.

Answer: B

Type: G Topic: 5 E: 433 MI: 189

198.Refer to the above diagram. By producing output level Q:

A)neither productive nor allocative efficiency are achieved.

B)both productive and allocative efficiency are achieved.

C)allocative efficiency is achieved, but productive efficiency is not.

McConnell/Brue: Economics, 16/e Page 755

Chapter 23: Pure Competition

D) productive efficiency is achieved, but allocative efficiency is not. Answer: B

McConnell/Brue: Economics, 16/e Page 756

Chapter 23: Pure Competition

Type: G Topic: 5 E: 432 MI: 188

199.Refer to the above diagram. At output level Q1:

A)neither productive nor allocative efficiency are achieved.

B)both productive and allocative efficiency are achieved.

C)allocative efficiency is achieved, but productive efficiency is not.

D)productive efficiency is achieved, but allocative efficiency is not. Answer: A

Type: G Topic: 5 E: 432 MI: 188

200.Refer to the above diagram. At output level Q1:

Page 291: Microeconomics Study Guide

A)resources are overallocated to this product and productive efficiency is not realized.

B)resources are underallocated to this product and productive efficiency is not realized.

C)productive efficiency is achieved, but resources are underallocated to this product.

D)productive efficiency is achieved, but resources are overallocated to this product. Answer: B

Type: G Topic: 5 E: 432-433 MI: 188-189

201.Refer to the above diagram. At output level Q2:

A)resources are overallocated to this product and productive efficiency is not realized.

B)resources are underallocated to this product and productive efficiency is not realized.

C)productive efficiency is achieved, but resources are underallocated to this product.

D)productive efficiency is achieved, but resources are overallocated to this product. Answer: A

Type: A Topic: 5 E: 432 MI: 188

202.Assume that society places a higher value on the last unit of X produced than the value of the resources used to produce that unit. With no spillovers, this information means that:

A)total cost is greater than total revenue.

B)price is greater than marginal cost. Answer: B

C)marginal cost is greater than price.

D)resources are being overallocated to X.

Type: A Topic: 5 E: 432-433 MI: 188-189

203.If production is occurring where marginal cost exceeds price, the purely competitive firm will:

A)maximize profit, but resources will be underallocated to the product.

B)maximize profit, but resources will be overallocated to the product.

C)fail to maximize profit and resources will be overallocated to the product.

D)fail to maximize profit and resources will be underallocated to the product.

Answer: C

Type: A Topic: 5 E: 432 MI: 188

McConnell/Brue: Economics, 16/e Page 757

Page 292: Microeconomics Study Guide

Chapter 23: Pure Competition

204.If a purely competitive firm is producing where price exceeds marginal cost, then:

A)the firm will fail to maximize profit, but resources will be efficiently allocated.

B)the firm will fail to maximize profit and resources will be overallocated to the product.

C)the firm will fail to maximize profit and resources will be underallocated to the product.

D)resources will be underallocated to the product, but the firm will maximize profit. Answer: C

McConnell/Brue: Economics, 16/e Page 758

Chapter 23: Pure Competition

Type: A Topic: 5 E: 432 MI: 188 Status: New

205. Which of the following conditions is true for a purely competitive firm in long-run

equilibrium?    A) P > MC = minimum ATC.C)P = MC = minimum ATC.B) P > MC > minimum ATC.D)P < MC < minimum ATC.Answer: C    

Consider This Questions

Type: A E: 423 MI: 179 Status: New206. (Consider This)

An unprofitable motel will stay open in the short-run if:

A)price (average nightly room rate) exceeds average variable cost.

B)marginal revenue exceeds marginal cost.

C)price (average nightly room rate) exceeds average fixed cost.

D)marginal revenue exceeds price.

Answer: A

Page 293: Microeconomics Study Guide

Type: A E: 423 MI: 179 Status: New207. (Consider This)An otherwise unprofitable motel located on a largely abandoned roadway

might be able to stay open for several years by:

A)increasing its nightly room rates.

B)reducing or eliminating its annual maintenance expenses.

C)charging room rates that exceed marginal revenue.

D)eliminating its fixed costs, including its opportunity costs. Answer: B

Last Word Questions

Use the following to answer questions 208-210:

McConnell/Brue: Economics, 16/e Page 759

Chapter 23: Pure Competition

Type: G E: 434 MI: 190

208.(Last Word) Refer to the above graph of the market for asparagus. At the market price of $2, area A + B represents:

A)

total consumer utility. C)consumer surplus.

B)total revenue to sellers.D)returns to capital and to labor.Answer: A    

McConnell/Brue: Economics, 16/e Page 760

Chapter 23: Pure Competition

Type: G E: 434 MI: 190

209. (Last Word) Refer to the above graph of the market for asparagus. At the market price of $2, area A represents:

A) total consumer utility. B) total revenue to sellers. C) consumer surplus. D) economic profit.

Answer: C

Type: G E: 434 MI: 190

210.(Last Word) In long-run equilibrium, purely competitive markets:

A)minimize total cost.

B)maximize consumer surplus.

Page 294: Microeconomics Study Guide

C)yield economic profits to most sellers.

D)inevitably degenerate into monopoly in increasing cost industries. Answer: B

True/False Questions

Type: A E: 416 MI: 172

211.In maximizing profit a firm will always produce that output where total revenues are at a maximum.

Answer: False

Type: A E: 422 MI: 178

212.In the short run a competitive firm will always choose to shut down if product price is less than the lowest attainable average total cost.

Answer: False

Type: A E: 427 MI: 183

213.After all long-run adjustments have been completed, a firm in a competitive industry will produce that level of output where average total cost is at a minimum.

Answer: True

Type: A E: 430-431 MI: 186-187

214.The long-run supply curve for a decreasing-cost industry is downsloping. Answer: True

Type: A E: 420 MI: 176

215.A competitive firm will produce in the short run so long as its price exceeds its average fixed cost.

McConnell/Brue: Economics, 16/e Page 761

Chapter 23: Pure Competition

Answer: False

Type: A E: 432 MI: 188

216.Marginal cost is a measure of the alternative goods which society forgoes in using resources to produce an additional unit of some specific product.

Answer: True

McConnell/Brue: Economics, 16/e Page 762

Chapter 23: Pure Competition

Page 295: Microeconomics Study Guide

Type: A E: 415-416 MI: 171-172

217.Price and marginal revenue are identical for an individual purely competitive seller. Answer: True

Type: A E: 432 MI: 188

218.Because the equilibrium position of a purely competitive seller entails an equality of price and marginal costs, competition produces up to an efficient allocation of economic resources.

Answer: True

Type: A E: 423-424 MI: 179-180

219.The short-run supply curve slopes upward because producers must be compensated for rising marginal costs.

Answer: True

Type: A E: 425 MI: 181

220.The demand curve for a purely competitive industry is perfectly elastic, but the demand curves faced by individual firms in such an industry are downsloping.

Answer: False

Type: A E: 417-418 MI: 173-174

221.The total revenue curve of a competitive seller graphs as a straight, upsloping line. Answer: True

Type: D E: 416 MI: 172

222.Marginal revenue is the addition to total revenue resulting from the sale of one more unit of output.

Answer: True

Use the following to answer questions 223-231:

McConnell/Brue: Economics, 16/e Page 763

Chapter 23: Pure Competition

Page 296: Microeconomics Study Guide

Type: G E: 419-420 MI: 175-176

223.Refer to the above diagram. This firm will maximize profits by producing output D. Answer: False

McConnell/Brue: Economics, 16/e Page 764

Chapter 23: Pure Competition

Type: G E: 419-420 MI: 175-176

224.Refer to the above diagram. At the profit-maximizing output total revenue will be 0GLD. Answer: False

Type: G E: 420 MI: 176

225.Refer to the above diagram. At output C production will result in an economic profit. Answer: True

Type: G E: 420 MI: 176

226.Refer to the above diagram. At output C total variable cost is FGKJ. Answer: False

Type: G E: 420 MI: 176

227.Refer to the above diagram. At output C average fixed cost is GF. Answer: False

Type: G E: 424 MI: 180

228.Refer to the above diagram. At any price below R the firm will shut down in the short run. Answer: True

Type: G E: 424 MI: 180

229.Refer to the above diagram. If demand fell to the level of FNJ, there would be no output at which the firm could realize an economic profit.

Answer: False

Type: G E: 424 MI: 180

230.Refer to the above diagram. If the firm produced D units of output at price G, it would earn a normal profit.

Answer: True

Type: G E: 417 MI: 173

231.Refer to the above diagram. Total costs are minimized at output level B. Answer: False

Type: A E: 425 MI: 181

232.Although individual purely competitive firms can influence the price of their product, these firms as a group cannot influence market price.

Answer: False

Page 297: Microeconomics Study Guide

McConnell/Brue: Economics, 16/e Page 765

Chapter 23: Pure Competition

Type: A E: 414 MI: 170

233.In a purely competitive industry competition centers more on advertising and sales promotion than on price.

Answer: False

For an imperfectly competitive firm:

the marginal revenue curve lies below the demand curve because any reduction in price applies to all units sold.

Refer to the above two diagrams for individual firms. Figure 1 pertains to:

a purely competitive firm.

Refer to the above diagram. Demand is relatively inelastic:

at any price below P2.

The MR = MC rule:

applies both to pure monopoly and pure competition.

Refer to the above data for a nondiscriminating monopolist. At its profit-maximizing output, this firm's total profit will be:

$82.

The supply curve of a pure monopolist:

does not exist because prices are not "given" to a monopolist.

In which one of the following market models is X-inefficiency least likely to be present?

pure competition

Which of the following is not a precondition for price discrimination?

The commodity involved must be a durable good.

A price discriminating pure monopolist will attempt to charge each buyer (or group of buyers):

the maximum price each would be willing to pay.

If a regulatory commission wants to provide a natural monopoly with a fair return, it should establish a price that is equal to:

average total cost.

Page 298: Microeconomics Study Guide

A monopolistically competitive industry combines elements of both competition and monopoly. It is correct to say that the competitive element results from:

a relatively large number of firms and the monopolistic element from product differentiation.

Refer to the above diagram for a monopolistically competitive firm. If more firms were to enter the industry and product differentiation were to weaken, then:

the demand curve would become more elastic.

Refer to the above data. If columns (1) and (3) of the demand data shown above are this firm's demand schedule, the profit-maximizing level of output will be:

8 units

A significant benefit of monopolistic competition compared with pure competition is:

greater product variety.

In monopolistically competitive markets, resources are:

underallocated because long-run equilibrium occurs where price exceeds marginal cost.

In long-run equilibrium a monopolistically competitive producer achieves:

neither productive efficiency nor allocative efficiency.

If an oligopolist is faced with a marginal revenue curve that has a gap in it, we may assume that:

its demand curve is kinked.

The economic profits earned by monopolistically competitive sellers are zero in the long run.

True

Refer to the above payoff matrix. Bob's Burgers and Sam's Sandwiches are competing restaurants in a small town. Both are considering adding pizza to their line of products. If this is a one-time simultaneous game:

neither firm has a dominant strategy.

Which of the following statements concerning a monopolistically competitive industry is correct?

If there are short-run losses, firms will leave the industry and the demand curves of the remaining firms will shift to the right.

Kodak introduced to the marketplace a digital camera which uses no film, but which takes photos that can be shown on personal computers. This is an example of:

product innovation.

As it relates to R&D, the imitation problem is that:

a firm's rivals may be able to copy its new product or process innovation, reducing the original firm's return on R&D.

All of the following increase the expected rate of return on R&D expenditures, except:

Page 299: Microeconomics Study Guide

imitation by others.

Legal protections against competitors producing and selling a product identical to the one you invented are called _________; legal protections against competitors using your product's name are called __________.

patents; trademarks

Suppose that Marlen Fisher has legal protection against anyone producing and selling a fishing lure identical to his unique-action "MarFish" lure, whatever the competitor might name the lure. This legal protection is most likely to be a:

patent.

Those who contend that oligopolists are less likely than more competitive firms to engage in R&D say that:

Oligopolists have little incentive to introduce costly new technology and produce new products when they currently are earning large economic profit using existing technology and selling existing products.

Technological advance improves allocative efficiency by:

giving society a more-preferred mix of goods and services.

Creative destruction is:

the process by which new firms and new products replace existing dominant firms and products.

The marginal cost to a firm of R&D expenditures is the market interest rate the firm must pay to obtain the needed financing.

True

The interest-rate cost-of-funds curve is perfectly elastic because firms can borrow as much or as little as they want at market interest rates.

True

The quantity demanded of a good is the amount that buyers

are willing and able to purchase.

The movement from point A to point B on the graph shows

an increase in quantity demanded.

The following table contains a supply schedule for a good.

If the law of supply applies to this good, then "?" could be

0

New cars are normal goods. What will happen to the equilibrium price of new cars if the price of gasoline rises, the price of steel falls, public transportation becomes cheaper and more comfortable, auto-workers accept lower wages, and automobile insurance becomes more expensive?

Page 300: Microeconomics Study Guide

Price will stay exactly the same.

The price elasticity of demand measures how much

quantity demanded responds to a change in price.

Consumer surplus is

the amount a buyer is willing to pay for a good minus the amount the buyer actually pays for it.

The theory of consumer choice provides the foundation for understanding the

demand for a firm's product. ???

In a market economy, government intervention

may improve market outcomes in the presence of externalities.

Assume that a college student spends her income on books and pizza. The price of a pizza is $8, and the price of a book is $15. If she has $100 of income, she could choose to consume

4 pizzas and 3 books.

An indifference curve illustrates

a consumer's preferences.

Large minimum efficient scale of plant combined with limited market demand may lead to:

natural monopoly.

Answer the question on the basis of the following table showing the demand schedulefacing a nondiscriminating monopolist:

Refer to the above table. The profit-maximizing monopolist will sell at a price:

that cannot be determined with the information provided.

If the variable costs of a profit-maximizing pure monopolist decline, the firm should:

produce more output and charge a lower price.

An important economic problem associated with pure monopoly is that, at the profit maximizing outputs, resources are:

underallocated because price exceeds marginal cost.

Refer to the above diagrams. The price will be _______ and the quantity will be _______ with the industry structure represented by diagram (B) compared to the one represented in (A).

higher; lower

Refer to the above long-run cost diagram for a firm. If the firm produces output Q2 at an average cost of ATC2, then the firm is:

Page 301: Microeconomics Study Guide

incurring X-inefficiency, but is producing that output at which all existing economies of scale might be realized.

If a pure monopolist can price discriminate by separating buyers into two or more groups:

the firm will face multiple marginal revenue curves.

Refer to the figure above. Suppose the graphs represent the demand for use of a local golf course for which there is no significant competition (it has a local monopoly); P denotes the price of a round of golf; Q is the quantity of rounds "sold" each day. If the left graph represents the demand during weekdays, and the right graph the weekend demand, this profit-maximizing golf course will earn how much economic profit over the course of a full seven-day week?

$4,200

Refer to the above diagram for a pure monopolist. Suppose a regulatory commission is created to determine a legal price for the monopoly. If the commission seeks to provide the monopolist with a "fair return," it will set price at:

P1.

Refer to the above diagram for a natural monopolist. If a regulatory commission set a maximum price of P1, the monopolist would produce output:

Q4 and realize a loss.

Use your basic knowledge and your understanding of market structures to answer this question. Which of the following companies most closely approximates a monopolistic competitor?

Subway Sandwiches

In short-run equilibrium, the monopolistically competitive firm shown above will set its price:

below ATC.

Refer to the above diagram for a monopolistically competitive firm. Long-run equilibrium price will be:

A.

The more elastic a monopolistic competitor's long-run demand curve, the:

lower its average total cost at its profit maximizing level of output.

Refer to the above data. The Herfindahl Index for the above industry is:

1,800.

Refer to the above diagram where the numerical data show profits in millions of dollars. Beta's profits are shown in the northeast corner and Alpha's profits in the southwest corner of each cell. If both firms follow a high-price policy:

each will realize a $20 million profit.

Refer to the above game theory matrix where the numerical data show the profits resulting from alternative combinations of advertising strategies for Ajax and Acme. Ajax's profits are shown in the upper right part of each cell; Acme's profits are shown in the lower left. With collusion and no cheating, the outcome of the game is cell:

D.

Page 302: Microeconomics Study Guide

Refer to the above diagram. Equilibrium price is:

d

Refer to the above payoff matrix. Bob's Burgers and Sam's Sandwiches are competing restaurants in a small town. Both are considering adding pizza to their line of products. If this is a sequential game but we don't know who moves first, what can we say about the final outcome?

Cells B and C both represent possible Nash equilibrium outcomes for this game

Refer to the above payoff matrix. Suppose that Speedy Bike and Power Bike are the only two bicycle manufacturing firms serving the market. Both can choose large or small advertising budgets. If this is a repeated game with no cooperation or reciprocity, cell A:

is the expected outcome of this game, and it is both a Nash equilibrium and a prisoner's dilemma.

The quantity demanded of a good is the amount that buyers

are willing and able to purchase.

The movement from point A to point B on the graph shows

an increase in quantity demanded.

The following table contains a supply schedule for a good.

Price

Quantity Supplied$10

100$20

?

If the law of supply applies to this good, then "?" could be

0

New cars are normal goods. What will happen to the equilibrium price of new cars if the price of gasoline rises, the price of steel falls, public transportation becomes cheaper and more comfortable, auto-workers accept lower wages, and automobile insurance becomes more expensive?

Price will stay exactly the same.

The price elasticity of demand measures how much

quantity demanded responds to a change in price.

Consumer surplus is

Page 303: Microeconomics Study Guide

the amount a buyer is willing to pay for a good minus the amount the buyer actually pays for it.

In a market economy, government intervention

may improve market outcomes in the presence of externalities.

The theory of consumer choice provides the foundation for understanding the

demand for a firm's product.

Assume that a college student spends her income on books and pizza. The price of a pizza is $8, and the price of a book is $15. If she has $100 of income, she could choose to consume

4 pizzas and 3 books.

An indifference curve illustrates

a consumer's preferences.

Given the budget constraint depicted in the graph, the consumer will choose bundle

C

Economists normally assume that the goal of a firm is to

maximize its profit.

Profit is defined as

total revenue minus total cost.

Those things that must be forgone to acquire a good are called

opportunity costs.

When a factory is operating in the short run,

it cannot adjust the quantity of fixed inputs.

Economies of scale occur when a firm's

long-run average total costs are decreasing as output increases.

A firm has market power if it can

maximize profits.

A key characteristic of a competitive market is that

producers sell nearly identical products.

Which of the following are necessary characteristics of a monopoly?(i)The firm is the sole seller of its product.(ii)

Page 304: Microeconomics Study Guide

The firm's product does not have close substitutes.(iii)The firm generates a large economic profit.(iv)The firm is located in a small geographic market.

(i) and (ii) only

Deadweight loss

equals monopoly revenues minus profits.

Which of the following is not a characteristic of a perfectly competitive market?

Goods offered for sale are largely the same.

Price discrimination

can occur in both perfectly competitive and monopoly markets.

Monopolistic competition is a type of

oligopoly.

What price will the monopolistically competitive firm charge in this market?

$80

Firms in a monopolistically competitive market

produce an output level that minimizes average total cost in the long run.

A group of firms that act in unison to maximize collective profits is called a

cartel.

An equilibrium in which each firm in an oligopoly maximizes profit, given the actions of its rivals, is called

a dominant equilibrium.

Game theory is important for the understanding of

competitive markets.

The prisoners' dilemma game

provides insight into why cooperation is individually rational.

Two companies, ABC and XYZ, each decide whether to produce a high level of output or a low level of output. In the figure, the dollar amounts are payoffs and they represent annual profits for the two companies.The dominant strategy for ABC is to

produce high output, and the dominant strategy for XYZ is to produce high output.

Which of the following statements concerning a monopolistically competitive industry is correct?

Page 305: Microeconomics Study Guide

If there are short-run losses, firms will leave the industry and the demand curves of the remaining firms will shift to the right.

The quantity demanded of a good is the amount that buyers

are willing and able to purchase.

The movement from point A to point B on the graph shows

an increase in quantity demanded.

he following table contains a supply schedule for a good.

Price

Quantity Supplied$10

100$20

?

If the law of supply applies to this good, then "?" could be

0

New cars are normal goods. What will happen to the equilibrium price of new cars if the price of gasoline rises, the price of steel falls, public transportation becomes cheaper and more comfortable, auto-workers accept lower wages, and automobile insurance becomes more expensive?

Price will fall

The price elasticity of demand measures how much

quantity demanded responds to a change in price.

Consumer surplus is

the amount a buyer is willing to pay for a good minus the amount the buyer actually pays for it.

In a market economy, government intervention

may improve market outcomes in the presence of externalities.

The theory of consumer choice provides the foundation for understanding the

demand for a firm's product.

Assume that a college student spends her income on books and pizza. The price of a pizza is $8, and the price of a book is $15. If she has $100 of income, she could choose to consume

Page 306: Microeconomics Study Guide

4 pizzas and 3 books.

An indifference curve illustrates

a consumer's preferences.

Given the budget constraint depicted in the graph, the consumer will choose bundle

C

Economists normally assume that the goal of a firm is to

maximize its profit.

Profit is defined as

total revenue minus total cost.

Those things that must be forgone to acquire a good are called

opportunity costs.

When a factory is operating in the short run,

it cannot adjust the quantity of fixed inputs.

Economies of scale occur when a firm's

long-run average total costs are decreasing as output increases.

A firm has market power if it can

influence the market price of the good it sells.

A key characteristic of a competitive market is that

producers sell nearly identical products.

Which of the following is not a characteristic of a perfectly competitive market?

Firms have difficulty entering the market.

Which of the following are necessary characteristics of a monopoly?(i)The firm is the sole seller of its product.(ii)The firm's product does not have close substitutes.(iii)The firm generates a large economic profit.(iv)The firm is located in a small geographic market.

(i) and (ii) only

Page 307: Microeconomics Study Guide

Deadweight loss

measures monopoly inefficiency.

Price discrimination

can maximize profits if the seller can prevent the resale of goods between customers.

Monopolistic competition is a type of

market structure.

What price will the monopolistically competitive firm charge in this market?

$80

Firms in a monopolistically competitive market

cannot earn economic profits in the long run.

A group of firms that act in unison to maximize collective profits is called a

cartel.

An equilibrium in which each firm in an oligopoly maximizes profit, given the actions of its rivals, is called

a Nash equilibrium.

Game theory is important for the understanding of

oligopolies.

The prisoners' dilemma game

provides insight into why cooperation is difficult.

Two companies, ABC and XYZ, each decide whether to produce a high level of output or a low level of output. In the figure, the dollar amounts are payoffs and they represent annual profits for the two companies.

produce high output, and the dominant strategy for XYZ is to produce high output.

Page 308: Microeconomics Study Guide

13. Refer to the above information. For a purely competitive firm:

D. the demand and marginal revenue curves will coincide.

14. If a firm in a purely competitive industry is confronted with an equilibrium price of $5, its marginal revenue:

B. will also be $5.

15. Price is constant or given to the individual firm selling in a purely competitive market because:

C. each seller supplies a negligible fraction of total supply.

16. For a purely competitive firm total revenue: D. has all of these characteristics.

17. The demand curve in a purely competitive industry is ______, while the demand curve to a single firm in that industry is ______.

B. downsloping, perfectly elastic

18. A perfectly elastic demand curve implies that the firm: B. can sell as much output as it chooses at the existing price.

19. Refer to the above diagram, which pertains to a purely competitive firm. Curve A represents:

D. total revenue only.

20. Refer to the above diagram, which pertains to a purely competitive firm. Curve C represents:

D. average revenue and marginal revenue.

21. Marginal revenue is the: D. change in total revenue associated with the sale of one more unit of output.

22. Marginal revenue for a purely competitive firm: C. is equal to price.

23. Firms seek to maximize: C. total profit.

24. A competitive firm in the short run can determine the profit-maximizing (or loss-minimizing) output by equating:

C. marginal revenue and marginal cost.

25. The MR = MC rule applies: A. to firms in all types of industries.

26. The MR = MC rule can be restated for a purely competitive seller as P = MC because:

A. each additional unit of output adds exactly its price to total revenue.

27. In the short run the individual competitive firm's supply curve is that segment of the:

B. marginal cost curve lying above the average variable cost curve.

Page 309: Microeconomics Study Guide

28. Assume the XYZ Corporation is producing 20 units of output. It is selling this output in a purely competitive market at $10 per unit. Its total fixed costs are $100 and its average variable cost is $3 at 20 units of output. This corporation:

D. is realizing an economic profit of $40.

29. A purely competitive firm's short-run supply curve is: B. upsloping and equal to the portion of the marginal cost curve that lies above the average variable cost curve.

30. Suppose you find that the price of your product is less than minimum AVC. You should:

C. close down because, by producing, your losses will exceed your total fixed costs.

31. If a purely competitive firm shuts down in the short run: C. it will realize a loss equal to its total fixed costs.

32. A purely competitive firm should produce in the short run if its total revenue is sufficient to cover its:

A. total variable costs.

33. Refer to the above data. This firm is selling its output in a(n): C. purely competitive market.

34. Refer to the above data. If the firm's minimum average variable cost is $10, the firm's profit-maximizing level of output would be:

B. 3.

35. Refer to the above data. At the profit-maximizing output the firm's total revenue is:

A. $48.

36. Refer to the above data. At the profit-maximizing output the firm's total cost is:

B. $32.

37. Refer to the above data. The firm's: B. economic profit is $16.

38. If a firm is confronted with economic losses in the short run, it will decide whether or not to produce by comparing:

B. price and minimum average variable cost.

39. The lowest point on a purely competitive firm's short-run supply curve corresponds to:

B. the minimum point on its AVC curve.

40. Refer to the above diagram for a purely competitive producer. The lowest price at which the firm should produce (as opposed to shutting down) is:

B. P2.

41. Refer to the above diagram for a purely competitive producer. The firm will produce at a loss at all prices:

D. between P2 and P3.

42. Refer to the above diagram for a purely competitive producer. If product price is P3:

C. economic profits will be zero.

Page 310: Microeconomics Study Guide

43. Refer to the above diagram for a purely competitive producer. The firm's short-run supply curve is:

B. the bcd segment and above on the MC curve.

44. Refer to the above diagram. To maximize profit or minimize losses this firm will produce:

C. E units at price A.

45. Refer to the above diagram. At the profit-maximizing output, total revenue will be:

A. 0AHE.

46. Refer to the above diagram. At the profit-maximizing output, total fixed cost is equal to:

D. BCFG.

47. Refer to the above diagram. At the profit-maximizing output, total variable cost is equal to:

B. 0CFE.

48. Refer to the above diagram. At the profit-maximizing output, the firm will realize:

D. an economic profit of ABGH.

49. In a purely competitive industry: C. there may be economic profits in the short run, but not in the long run.

50. The principle that a firm should produce up to the point where the marginal revenue from the sale of an extra unit of output is equal to the marginal cost of producing it is known as the:

B. profit-maximizing rule.

51. Which of the following is true concerning purely competitive industries?

C. In the short run, firms may incur economic losses or earn economic profits, but in the long run they earn normal profits.

52. Which of the following statements is correct? A. Economic profits induce firms to enter an industry; losses encourage firms to leave.

53. The MR = MC rule applies: C. in both the short run and the long run.

54. The term allocative efficiency refers to: C. the production of the product-mix most desired by consumers.

55. Under pure competition in the long run: B. both allocative efficiency and productive efficiency are achieved.

56. The above diagram portrays: B. the equilibrium position of a competitive firm in the long run.

57. Refer to the above diagram. If this competitive firm produces output Q, it will:

B. earn a normal profit.

Page 311: Microeconomics Study Guide

58. Which of the following conditions is true for a purely competitive firm in long-run equilibrium?

C. P = MC = minimum ATC.

59. If profits are made in purely competitive market in the long run:

d. all of the above.

60. In pure competition in the long run when firms are in an increasing cost industry, the entry of additional firms causes:

b. cost per unit to rise, prices to fall and profits to decrease.

1CORRECT Which of the following is not a characteristic of "pure monopoly"

A)

There is only one firm.

B)

Its products are standardized and have many close substitutes

C)There is considerable control over the price

D)It is very difficult to enter the industry

Feedback: Pure monopoly produces a unique product with no close substitutes.

2CORRECTA market structure, which is dominated by a few large firms, each producing a similar product is called...

A)Monopolistic competition

B)Pure competition

C)Oligopoly

D)Pure monopoly

Feedback: Choice 3 by definition is correct.

3CORRECT In pure competition, firms are "price takers" because...

Page 312: Microeconomics Study Guide

A)Prices are fixed in this market structure

B)

They exert much control over the product price in this market structure

C)They must rely on government to set market product prices

D)There are many firms offering a standardized product and each firm has little control over the product price.

4CORRECT Which of the following is not true of the purely competition market structure?

A)The market demand curve will be perfectly elastic in a purely competition market

B)An individual firm's demand curve will be perfectly elastic in a purely competition market

C)An individual firm's demand curve will coincide with its marginal revenue (MR) curve

D)None of the other choices

Feedback: Choice 1 is incorrect. The market demand curve (including all producers) will be downward sloping and may be inelastic. As a whole, individual firms may change output and thus simultaneously change price. But the individual producer still faces an elastic demand curve for any given good.

5CORRECT For a purely competitive market structure

A)D = MR = TR

B)D = TR = AR

C)D = MR = AR

D)

P = TR = AR

6CORRECT A firm's demand schedule for good X is given below:PX QX $180 0$180 1$180 2$180 3$180 4

Page 313: Microeconomics Study Guide

$180 5If the quantity demanded increases from 3 to 4 units, what is the firm's marginal revenue (MR)?

A)$180

B)$250

C)$300

D)$350

Feedback:

MR = TR QD

=$720 – $540

4 units – 3 units= $180

7CORRECT At break even point...

A)TR – TC is maximized

B)

An economic profit would be earned

C)TR = TC

D)TR is maximized

8CORRECT

Using the chart in question 6, and given the following figures for TFC and TVC,Q TFC TVC0 $200 01 $200 1002 $200 1803 $200 2504 $200 3105 $200 3806 $200 4607 $200 5508 $200 6509 $200 76010 $200 880For output level Q = 3, TC =

A)$330

Page 314: Microeconomics Study Guide

B)$380

C)$450

D)$475

Feedback: TC = TVC + TFC

9CORRECTWhat is the profit maximizing output level for this perfectly competitive firm? Assume that the price remains the same for quantities above 5 units, and for quantities greater than 11 units, TVC continues to increase in the same pattern as shown.

A)

Q = 10

B)Q = 16

C)

Q = 17

D)

Q = 18

Feedback: Using the total revenue – total cost approach, the profit-maximizing output level will occur where TR – TC is maximized. Calculate TR – TC for each of the given values of Q. Q=17 units will maximize the firm's profits as it yields the highest value ($860).

10CORRECT

As long as producing some output is preferable to shutting down, a purely competitive firm will maximize its output when...

A)MR = MC

B)MR > MC

C)MC > MR

D)P > MC

Feedback: By definition, a perfectly competitive firm will maximize profits when MR = MC, or P = MC, since P = MR for this market structure.

11CORRECT

As long as there is MR>MC...

A)A firm will find it profitable to produce less output

B)A firm is producing at its output – maximizing level

C)A firm will find it profitable to product more output

Page 315: Microeconomics Study Guide

D)None of the other choices

Feedback: If MR>MC additional revenue earned from producing an additional unit of output will exceed the additional cost

12CORRECT

If the price of a good is $100, the output is 10 units and the average total cost is $40, what is the firm's economic profit?

A)$300

B)$400

C)$500

D)$600

Feedback: TR – TC = = P x Q – TCATC = TC/Q = P x Q – (ATC x Q)=$100 x 10 units – ($40 x 10 units)=$1000 - $400=$600

13CORRECT

If AVC<P<ATC, then

A)The firm should shut down and produce nothing

B)The firm should try to minimize the loss it will incur

C)The firm will make a profit of (P – ATC)*profit-maximizing output

D)None of the other choices

14CORRECT

If P<AVC then

A)The firm will incur a loss

B)

The firm will be unable to cover any of its fixed cost per unit

C)The firm should shut down and produce no output

D All of the other choices

Page 316: Microeconomics Study Guide

)

Feedback: Choice 1 is correct since P<ATC. Choice 2 is correct since P< (ATC_AVC) or P<AFC, so the firm will not be able to cover its fixed costs per unit. Since choice 2 occurs then choice 3 becomes inevitable – shut down becomes the best strategy. Choice 4 is correct.

15CORRECT

The supply curve of a competitive firm is equal to...

A)

The marginal cost curve above the point where it cuts the ATC curve.

B)

The marginal cost curve at all prices

C)The marginal cost curve above the point where it cuts the AVC curve.

D)

None of the other choices.

16CORRECT

As the price of a good increases, which of the following describes the adjustment along the competitive firm's supply curve?

A)As P increases, MR>MC at the original output level. Therefore, the firm will produce less of the good.

B)As P increases, MR<MC at the original output level, then the firm will produce more of the good.

C)As P increases, MR>MC at the original output level, then the firm will produce more of the good

D)

None of the above.

17CORRECT

Thinking of the long-run equilibrium, if a decrease in tastes and preferences for a good causes the price of the good to fall below ATC, then...

A)In the long run, firms will enter the industry due to an economic profit. This will cause short-run supply to increase and the price to rise back up to ATC.

B)In the long run, firms will exit the industry due to an economic loss. This will cause short-run supply to decrease and price to rise back up to ATC

C)In the long run, firms will exit the industry due to an economic profit. This will cause

Page 317: Microeconomics Study Guide

short-run supply to increase and price to fall further below ATC

D)None of the other choices

18CORRECT

The long-run supply curve for the constant cost industry is...

A)Upward sloping

B)Downward sloping

C)Horizontal

D)Vertical

19CORRECT

For a competitive firm to exhibit productive efficiency,

A)P = AVC

B)P = minimum ATC

C)P = TR – TC

D)None of the other choices

20CORRECT

If P>MC then...

A)MC>MB and too much of the good is being produced (over allocation)

B)

MC = MB and allocation efficiency is realized

C)MB>MC and not enough of the good is being produced

D)None of the other choices

21CORRECT

In the computer industry,

Page 318: Microeconomics Study Guide

A)The long-run aggregate supply curve is horizontal

B)The long-run aggregate supply curve is downward sloping

C)

The long-run aggregate supply is upward sloping

D)None of the other choices

22CORRECT

Which of the following is true?

A)Finding profit – maximizing output using MR = MC works only for the pure competition market structure

B)

If MR>MC, a firm will produce more of a good and output will increase

C)A firm that incurs losses must shut down in a perfectly competitive market structure

D)

A firm earns normal profits if P = Minimum ATC

Feedback: A firm can have losses and still not shut down if it is covering at least some of its fixed costs. But if P<AVC then the firm will shut down.

23CORRECT

Which of the following is not true?

A)If P>ATC, then firms will exit the industry due to reduced demand

B)In the long run, P = minimum ATC due to exit and entry of firms and the effect on the supply curve of the firm

C)If P>AVC, but less than minimum ATC, a firm will minimize its losses but not shut down

D)None of the other choices

24CORRECT

consider two industries – the widget industry and the woggle industry. Suppose that tastes and preferences increase for woggles all else equal. If widgets and woggles are substitutes in production

A)In the long run, the price of woggles would increase causing firms to enter the woggle

Page 319: Microeconomics Study Guide

industry

B)In the long run, the supply of woggles would increase

C)In the long run price of woggles would fall back to P = minimum ATC and all firms would make a normal profit.

D)All of the other choices.

1CORRECTAn organization that uses factors of production to produce goods and services to maximize profits is called

A)A plant

B)A firm

C)An industry

D)None of the above

Feedback: Choice 2 is correct by definition

2CORRECT The main disadvantage of a partnership is...

A)It is difficult to set up or organize

B)Less specialization in management can occur

C)Since two or more people may be in management, disagreements my result

D)

There is limited liability

Feedback: By definition, choice 3 is true.

3CORRECT The main advantage of the corporate form of business organization is...

A)Corporations are relatively easy to set up

B)Corporations have a relatively easy time raising financial capital

Page 320: Microeconomics Study Guide

C)

Corporations are large in size

D)

Corporations enjoy limited liability. Since shareholder debts are limited to what they have invested in the company.

4CORRECT Accountants tend to be concerned about...

A)

Implicit cost

B)

Opportunity costs

C)Explicit costs

D)None of the other choices

Feedback: Explicit costs are those costs, which involve an out-of-pocket cash payment – e.g. wages, utilities.

5CORRECT

Jim has decided to quit his $35,000 per year teaching job in favor of opening up a small restaurant. He has invested $10 000 of his own money, and the interest rate on savings is 5%. During his final year in business, Jim had salaries costs of $100 000, utilities costs of $15 000, materials costs of $40 000, and total revenue of $170 000. Jim owns the store space, and has been renting it out for $10 000 per year. His entrepreneurial talent is worth $8 000 in a similar business. Jim’s accounting profit is…

A)$10,000

B)$20,000

C)$12,000

D)$15,000

Feedback: Acc. Profit = total Revenue - total costs explicit

6CORRECT Jim's total implicit costs are...

A)$53,500

B)$43,000

C)$43,500

D)$18,500

Page 321: Microeconomics Study Guide

Feedback: Total implicit costs

7CORRECT Which of the following is true about Jim's restaurant business?

A)Both his accounting profit and economic profit are positive

B)

His accounting profit is negative, but his economic profit is positive

C)His accounting profit is positive but his economic profit is negative

D)Both is accounting profit and economic profit are negative

Feedback: Economic profit= Accounting profit - implicit costs=$15,000 – $53,500=($38,500) or –$38,500

8CORRECT Economic costs equal

A)Explicit costs only

B)Implicit costs only

C)Implicit costs plus explicit costs

D)Opportunity costs only

Feedback: Choice 3 is correct by definition

9CORRECTNortel is thinking of re-hiring some of its workers due to an increase in demand for its products. However, its plant capacity remains fixed. This means...

A)The period of time being referred to is the short run

B)The period of time being referred to is the long run

C)Labour is a fixed resource

D)None of the other choices

Feedback: Choice 1 is correct since in the short urn only the variable factors of production, such as labour can be changed. The capacity of the plant is fixed.

10CORRECT

Which of the following is true?

Page 322: Microeconomics Study Guide

A)The short and long run of a firm are measured by calendar time

B)In the long run, capital remains fixed, but all other factors of production are variable.

C)

The size of a firm can be changed in the short run, if labour increases a lot.

D)

The time needed for two different firms to reach their long run adjustment will vary depending on the nature of the firms.

Feedback: Choice 4 is mentioned in the text.

11CORRECT

The total product (TP) of labour measures...

A)The output per unit of labour

B)The total output produced by a given quantity of labour

C)

The additional output produced by an additional unit of labour

D)None of the other choices

12CORRECT

Kevin is studying for his Canadian securities exam (CSC). He can learn the following numbers of concepts in the given no. of hours:

Q(hrs)Concept

lnd0 01 32 73 124 165 196 21What is Kevin's total product (TP) after 4 hours?

A)3 concepts

B)12 concepts

C)16 concepts

D)19 concepts

Feedback: TP (4) = 16 concepts

Page 323: Microeconomics Study Guide

13CORRECT

What is Kevin's marginal product when his study time increases from 2 – 3 hours?

A)3 concepts

B)4 concepts

C)5 concepts

D)9 concepts

Feedback:

MPL = TPL L

=12 – 73 – 2

=51

= 5 concepts

14CORRECT

When does Kevin begin to experience diminishing returns to studying for the exam?

A)After 3 hours

B)After 2 hours

C)

After 1 hour

D)After 4 hours

Feedback: MPL after 3 hours from 2 – 3 hours,MPL = 5 conceptsMPL = 4 concepts from 3 – 4 hours. So MPL decreases after 3 hours.

15CORRECT

If MP < AP, then...

A)AP is rising

B)AP is falling

C)MP is rising

D)None of the above

Feedback: If marginal product < average product, additional units of output generated by increasing amounts of labour will be less than the average product (output per unit). So, the

Page 324: Microeconomics Study Guide

average product must decrease, and the AP curve will fall.

16CORRECT

Fixed costs...

A)Increase as a firm's output changes

B)Include cost of raw materials, labour and fuel

C)

Must be paid by firm even if it doesn't produce anything

D)None of the other choices

Feedback: Even if a firm's output is zero, it must still pay fixed costs, such as rent, office salaries and insurance.

17CORRECT

The TVC curve begins to increase at an significant rate after some output level due to...

A)

Higher fixed costs

B)Increasing marginal product of labour

C)Decreasing total product of labour

D)Diminishing returns

Feedback: As the marginal product of labour begins to decrease, "larger amounts of additional variable resources (such as labour) must be used to produce extra units of output." Thus, total variable cost will increase at a significant rate.

18CORRECT

At an output of 10 units, a firm's TFC = $80/unit and its TC = $200. What is the firm's AVC at this output level?

A)$120

B)$50

C)

$12

D)

$20

Feedback:

AVC =TVC

Q

Page 325: Microeconomics Study Guide

=TC – TFC

Q

=        $120         10 units

= $12/unit

19CORRECT

Suppose that this firm has total costs of $170 when its output is 9 units. The marginal cost of increasing output from 9 to 10 units is...

A)

$30

B)$40

C)

$50

D)

$60

Feedback:

M Cost =Change in total cost

Change in output

=$200 – 170

10 units – 9 units= $30

20CORRECT

Which of the following is true?

A)The marginal product and marginal cost curves are mirror images of one another

B)When MP is increasing, MC is decreasing

C)When MP is at maximum, MC is at a minimum

D)

All of the other choices

Feedback: Since MP and MC are mirror images of one another, choice 2 and 3 are also correct. Please see the text.

21CORRECT

Suppose that firm X increases the wages offered to its factors workers. Which of the following will occur?

A)TFC would shift upward

B AFC would shift upward

Page 326: Microeconomics Study Guide

)

C)TVC would shift upward

D)AVC would shift downward

Feedback: Since labour is a variable input (resource) an increase in wages affects only variable costs. In this case, total variable costs would increase, so TVC shifts upward.

22CORRECT

In the long run,

A)

All resources are variable

B)

All costs are variable

C)The firm can alter the size of its plant (plant capacity)

D)

All of the other choices

Feedback: All of these statements are true of long run.

23CORRECT

A firm's long-run average cost curve is derived by joining the minimum points on its _____ curve.

A)Average variable cost (AVC

B)

Average total cost (ATC)

C)Marginal cost (MC)

D)None of the other choices

24CORRECT

If a firm increases its resources by 5%, and its output increases by 20%, then...

A)The firm exhibits constant returns to scale

B)The firm exhibits decreasing returns to scale

C)The firm exhibits increasing returns to scale

D)None of the other choices

Page 327: Microeconomics Study Guide

Feedback: Since a more than proportionate increase in output occurs, we have increasing returns to scale

25CORRECT

In the above question, what would the long-run average cost curve look like?

A)It would be flat (horizontal)

B)It would be downward sloping

C)It would be upward sloping

D)None of the other choices

Feedback: When increasing returns to scale, LRAC would be falling since slower than output total costs are increasing.

1CORRECT The pure monopolist's product...

A)Has many substitutes offered by different companies

B)Is differentiated

C)Is standardized and similar to that in different companies

D)None of the other choices

Feedback: The pure monopolist's product has no substitutes. If the consumer doesn't buy the monopolist's product, he/she must do without it.

2CORRECT Which of the following is not a characteristic of a pure monopoly?

A)A pure monopoly is the single seller of a particular good

B)

The monopolist is a price maker and has great control over the price

C)There is relatively easy entry for firms into the monopolistic industry

D)The monopolist has a downward sloping demand curve

Feedback: There are barriers, which prevent competitors from entering the industry (e.g. patents).

3CORRECT Which of the following market structures has the strongest barriers to entry in an industry?

A)Pure monopoly

Page 328: Microeconomics Study Guide

B)

Oligopoly

C)Monopolistic competition

D)Pure competition

Feedback: Choice 1 is correct. Pure monopoly use many barriers to entry, including economics of scale, patents and licenses.

4CORRECTPfizer is a large Pharmaceutical Company that produces Prozac, an anti-depressant. Which of the following barriers to entry would have been used by Pfizer to prevent other firms from imitating their drug?

A)Economics of scale

B)Patents

C)Licenses

D)

Ownership of resources

5CORRECT Which of the following is true of pure monopoly?

A)Its demand curve slopes downward

B)Since there is only one seller, the demand curve for the firm is the same as that for the industry

C)As a monopolist lowers the price, the lower price applies to all prior units of output, not just the additional ones sold.

D)All of the other choices.

6CORRECT

Consider the following table showing a monopolist's output and price:Q P=AR0 $2001 $1702 $1403 $1104 $805 $50

Page 329: Microeconomics Study Guide

6 $20Also given is total cost for different levels of output:Q TC0 $1001 $1702 $2303 $2804 $3205 $3506 $370Considering the information given, if the monopolist increases its output from 2 – 3 units, its marginal revenue will be...

A)

$30

B)$40

C)

$50

D)

$60

Feedback:

MR – change in total revenue

change in output= P(3) x 3 units – 

P(2) x 2 units1

=$330 – $280

1= $50

7CORRECT Under what condition will a monopolist maximize its profit

A)If MR>MC

B)If MR = MC

C)If MR<MC

D)

If P = MC

8CORRECTthe information given above suggests this monopolist will provide ____ units of output of maximum profits

A)1

Page 330: Microeconomics Study Guide

B)2

C)5

D)3

Feedback: MR = MC for the third output unit produced therefore, 3 units of output maximizes profit.

9CORRECT A monopolist will never operate in the inelastic region of the demand curve because...

A)

Reducing the output and increasing the price in this region will reduce total revenue.

B)Reducing the output and increasing the price in this region will have no effect on total revenue.

C)Increasing the output and reducing price in this region will reduce total revenue

D)Increasing output and reducing price in this region will increase total revenue.

Feedback: On the inelastic region of a monopolist's demand curve, a reduction in price will result in only a small increase in quantity demand, thus TR decreases.

10CORRECT

Which of the following is true? A monopolist will

A)

Increase output in the elastic region of the demand curve.

B)Decrease output in the inelastic region of the demand curve

C)Not change output if MR = MC

D)All of the other choices are correct.

11CORRECT

Which of the following formulas represents a monopolist's economic profit

A)

(Pm – ATC) x QM

B)Pm + ATC

C)Pm x Qm

D)None of the other choices

Page 331: Microeconomics Study Guide

Feedback: Choice 1 is correct since total economic profit is measured by the difference between the monopolist's price per unit and Average total cost multiplied by the number of units produced.

12CORRECT

Suppose a monopolist is producing at a level of output such that MR<MC. Which of the following happens?

A)

The monopolist will increase her output because she can profit by doing so.

B)The monopolist will decrease her output because she can profit by doing so.

C)The monopolist has no incentive to change the level of her output

D)The monopolist will move rightward along her marginal cost curve until MR = MC

Feedback: Since MR<MC by increasing total output the monopolist will experience a greater increase in total cost than in total revenue, so total profit will decrease. Thus, the monopolist should reduce output, because she can increase MR and decrease MC.

13CORRECT

Which of the following is true of the monopolist?

A)He will always charge the highest priced he can get

B)He will maximize profit per unit

C)He may suffer a loss if Pm<Average total cost

D)None of the other choices

Feedback: Choice 1 is wrong because the monopolist maximizing total profit, not total price. Choice 2 is incorrect since again the monopolist maximizes total profit rather than profit per unit.

14CORRECT

pure monopoly will...

A)Produce a greater output at a higher price than a purely competitive firm

B)Produce a greater output at a lower price than a purely competitive firm

C)

Produce a smaller output at a lower price than a purely competitive firm.

D)Produce a smaller output at a higher price than a purely competitive firm

Page 332: Microeconomics Study Guide

15CORRECT

If a monopolist charges different prices at different times of the day (as in the case of a movie theatre)...

A)This is called price fixing

B)This is called price discrimination

C)This is call price disruption

D)None of the other choices

Feedback: Choice 2 is correct by definition.

16CORRECT

Which of the following is true of a pure monopolist?

A)P = MC and P = minimum ATC

B)P>MC and P = minimum ATC

C)P<MC and P<minimum ATC

D)P>MC and P>minimum ATC

Feedback: 4 is correct by definition of a monopoly.

17CORRECT

17- Which of the following is true of X – inefficiency

A)

For a given level of output, ATC is greater than minimum ATC

B)X – inefficiency occurs because business managers may have goals such as an easier work life, avoiding business risk, or providing jobs to inefficient relatives who interfere with cost minimization.

C)

X – inefficiency affects monopolistic firms more than purely competitive firms

D)All of the other choices.

Feedback: Each of these statements correctly describes X – inefficiency.

18CORRECT

Suppose a monopolist spends years in court trying to gain a license to secure its monopoly...

A)This is an example of profit maximizing behaviour

Page 333: Microeconomics Study Guide

B)This is an example of rent-seeking behaviour

C)

This is an example of cost minimizing behaviour

D)None of the other choices

Feedback: By definition, choice 2 is correct.

19CORRECT

Which of the following explains why the monopoly structure is not widespread in the economy?

A)Changes in technology may create substitutes to the good or service produced by the monopoly.

B)Patents may expire allowing other firms to copy the good produced by the monopoly.

C)Fear of the future competition may cause monopolies to keep their prices low, providing consumers with lower prices

D)

All of the other choices

Feedback: All three statements are reasons why monopoly prices and monopolists may not be that widespread in the economy.

20CORRECT

Which of the following is not a condition for price discrimination?

A)All sellers must have the same willingness to purchase the product

B)

Monopoly power

C)

The original purchaser can't resell the product

D)None of the other choices

Feedback: Choice 1 is correct. The monopolist must be able to separate buyers into different classes, each with a different willingness to purchase the product.

21CORRECT

A perfectly discriminating monopolist...

A)Charges a price for each unit of the good equal to the person's willingness to pay for it.

B)Realizes a lower profit than a single price monopolist.

Page 334: Microeconomics Study Guide

C)Will produce less output than a single price monopolist

D)

None of the other choices.

Feedback: Choice 1 is correct. Choice 2 should read a higher profit and choice 3 should read more profit.

22CORRECT

A regulated monopoly produces where D = MC will charge

A)The monopoly price (Pm)

B)The firm return price

C)The socially optimal price

D)

None of the other choices.

23CORRECT

If a monopoly charges a price P = ATC, then...

A)It will earn monopoly profit

B)

It will earn a normal profit and break even

C)It may produce losses for the monopolist

D)None of the other choices

Feedback: Only choice 2 is correct. Choice 1 implies to Pm, which choice 3 applies to the socially optimal price.

24CORRECT

Which of the following is true of the monopolist?

A)He will operate on the inelastic part of the demand curve

B)He will continue to increase output and decrease price as long as demand is elastic

C)He will reduce his output as long as MR>MC

D)

None of the other choices.

Page 335: Microeconomics Study Guide

Feedback: Choice 2 is correct. When demand is elastic, a price reduction will increase total revenue. However, if profit continues to decrease to the inelastic portion of the demand curve, then TR decreases. So as long as demand is elastic output increases.

1CORRECTThe main difference between the perfectly competitive market structure and that of monopolistic competition is that...

A)Monopolistic Competitive firms number in the thousands whereas, perfectly competitive firms number in the millions.

B)Monopolistic competitive firms have difficult entry to and exit from industry. Whereas, perfectly competitive firms have easy entry and exit.

C)There is price setting in monopolistically competitive firms but not in perfectly competitive ones.

D)Monopolistically competitive firms engage in products differentiation, while perfectly competitive firms have a standardized product.

Feedback: Only choice 4 is correct.

2CORRECTThe fact that Pepsi uses a predominantly blue can, while Coke uses a red and white one is an example of...

A)Collusion

B)Product choice

C)Product differentiation

D)

Price competition

Feedback: By definition, choice 3 is correct.

3CORRECT Which of the following is not a characteristic of monopolistically competitive firms?

A)They employ non-price competition such as products differentiation or advertising

B)They have a lot of control over the market price of their product

C)

They have relatively easy entry into and exit from their industry

Page 336: Microeconomics Study Guide

D)None of the other choices

Feedback: Choice 2 is not true since there are a lot of substitutes for each monopolistically competitive firm's output. The individual firm will have little control over the price of its product.

4CORRECT Which of the following is true of the demand curve of a monopolistically competitive firm?

A)It is more elastic than the demand curve of a perfectly competitive firm.

B)

It is more inelastic than the demand curve of a pure monopoly.

C)It is perfectly elastic due to the existence of many competitors producing similar goods.

D)None of the other choices

Feedback: Choice 1 should read less elastic. Choice 2 should read less inelastic and choice 3 should read elastic – remove "perfectly" from the sentence.

5CORRECT A monopolistically competitive firm's demand curve is...

A)

Elastic because it has several competitors producing similar, but not identical goods

B)

Perfectly elastic because it has many competitors producing identical goods.

C)Inelastic because it has at most no competitors producing the same good.

D)None of the other choices

Feedback: By definition, choice 1 is correct.

6CORRECT A monopolistically competitive firm will maximize profit in the short run

A)Where P =MC

B)Where MR>MC

C)Where MR = MC

D)

Where MC>MR

Feedback: Choice 2 is correct by definition

7CORRECT In the long run, a monopolistically competitive firm will earn...

Page 337: Microeconomics Study Guide

A)An economic profit

B)Only a normal profit

C)A definite loss as entry of new firms will reduce demand for a given firm's products

D)None of the other choices

Feedback: Choice 2 is correct P = ATC implies the firm is earning a normal profit.

8CORRECT As new firms enter the industry, in the long run, due to economic profits, then...

A)Each monopolistically competitive firm will see a higher demand for its products, causing the individual firm's demand curve to shift right.

B)All the other monopolistically competitive firms will suffer losses and firms will leave the industry.

C)All the other monopolistically competitive firms will earn higher profits, causing even more firms to enter the industry.

D)Each monopolistically competitive firm will see a lower demand for its products, causing the individual firm's demand curve to shift left.

Feedback: Choice 4 is correct. As new firms enter the industry, all else equal, they will eat into the demand (sales) of the other firms, since they produce similar, substitutable products.

9CORRECTSuppose that in the short run, a monopolistically competitive industry faces P<ATC. This will cause...

A)Firms to enter the industry and each firm's demand curve to shift to the right until profits are normal.

B)Firms to enter the industry and each firm's demand curve to shift left, until profits are normal.

C)Firms to leave the industry and each firm's demand curve to shift right until profits are restored.

D)

None of the other choices.

Feedback: Firms would leave the industry since P<ATC constitutes an economic loss.

10CORRECT

Why might a monopolistically competitive firm earn better than normal profits in the long run as theory suggests?

A)Product differentiation may prevent other firms from duplicating a given firm's product

Page 338: Microeconomics Study Guide

over time.

B)Entry into monopolistically competitive industry may be more difficult than theory suggests

C)Some monopolistically competitive firms attain a certain amount of monopoly power due to choice 1 and 2

D)

All of the other choices

Feedback: All these answers are correct.

11CORRECT

Thinking of monopolistic competition and efficiency...

A)Monopolistic competition exhibits both productive and allocative efficiency.

B)Monopolistic competition exhibits neither productive nor allocative efficiency.

C)Monopolistic competition exhibits production, but not allocative efficiency

D)Monopolistic competition exhibits allocative but not productive efficiency.

Feedback: Choice 2 is correct according to the text.

12CORRECT

Which of the following is not true of an oligopoly?

A)It consists of a few large firms.

B)There is a mutual interdependence between firms in the oligopoly.

C)

Firms that are part of the oligopoly produce only a differentiated product.

D)None of the above

Feedback: Firms in the oligopoly may produce either a homogeneous (standard) product or a differentiated product.

13CORRECT

Which of the following firms belong to an oligopolistic market?

A)Mc Donald's

B)Kellogg's food products

C)Stelco steel

Page 339: Microeconomics Study Guide

D)All of the above

Feedback: All of these firms are oligopolists

14CORRECT

Statistically, an oligopoly is defined as...

A)When the largest firms dominate 50% or more of the market.

B)

When one firm controls 20% or more of the market.

C)

When the largest four firms in an industry control 40% or more of the market.

D)When 5 or more firms control the entire market.

Feedback: By definition choice 3 is correct.

15CORRECT

Suppose that there are six firms in an industry. Firm A has a 10% market share, firm B has a 20% market share, Firm C has a 30% market share, while firms D, E and F have market shares of 15%, 20% and 5% respectively. The Herfindahl index for this industry is

A)1500

B)2050

C)2500

D)1650

Feedback: To get the HI, add up the squares of each market share, and you get 2050.

16CORRECT

A perfectly competitive industry would have a Herfindahl index of...

A)Close to 0

B)

10,000

C)

100

D)5000

Feedback: Since each firm would have almost a regular market share, HI would be very small – almost zero.

Page 340: Microeconomics Study Guide

17CORRECT

David and his economics professor were thrown in jail for insider trading. The jailer has approached them with an individual offer. If one confesses and the other doesn't, the one who doesn't confess is let free while the other serves 20 years. If both confess, each will serve 10 years. If neither confesses, then both will serve 15 years. The payoffs are shown in the following chart:

DavidC/T NC N/O C10 0 15 20

 Professor

10 0 15 20

A)David has an incentive to confess

B)

The professor has an incentive to confess

C)Both men have an incentive not to confess because neither one knows what the other will do.

D)Both men will confess because this will make both of them better off.

Feedback: Because each person's answer depends on that the other says, there is an incentive not to confess. But note that by working together, David and the Professor could confess and both be better off, each serving only 10 years.

18CORRECT

Suppose there are two oligopolists A and B. If A thinks that B is going to price its product low...

A)Then A should price his product high to earn a higher profit.

B)Then A should price his products low as well to capture some of the market

C)

Then A shouldn't Change the price of his product.

D)

None of the other choices.

19CORRECT

If two oligopolistic firms agree to collusion,

A)One will agree to set the product price high, while another will keep it low.

B)

Both firms will agree to keep the product price low.

Page 341: Microeconomics Study Guide

C)Both firms will agree to keep their product price high so as to maximize profits.

D)Both firms will agree to try and force other firms out of business.

Feedback: Choice 3 is correct by definition.

20CORRECT

What is the main difficulty with collusion?

A)The oligopolist may have an incentive to cheat and lower his price

B)The oligopolist has an incentive to cheat and raise prices.

C)Neither firm can profit under collusion

D)

None of the other choices.

Feedback: If is very difficult to enforce a collusive agreement, thus, choice 1 is correct.

21CORRECT

The kinked demand curve suggests...

A)Price changes will occur less frequently under oligopoly than under most of the other market structures.

B)That if firm X raises its price, but firms Y and Z didn't, then firm X will lose a lot of customers and have an elastic demand curve in this region.

C)That if firm X lowers its price and Y and Z follow, then, firm X will have only small gains in sales and thus its demand curve will be inelastic in this region.

D)

All of the other choices

Feedback: All of these statements are correct.

22CORRECT

Which of the following is not true of the kinked demand curve?

A)It assures that rival firms will follow a price decrease but not follow a priced increase.

B)If applies to collusive oligopoly behaviour

C)

It explains why prices in oligopolistic firms tend to stay kinked.

D None of the other choices.

Page 342: Microeconomics Study Guide

)

Feedback: Choice 2 is the correct answer. The kinked demand curve explains non-collusive behaviour.

23CORRECT

Which of the following is not true of cartels?

A)They represent a formal agreement among firms to set prices.

B)

They are illegal in Canada

C)

They are often hidden from legal authorities

D)OPEC is a common example of a cartel.

Feedback: They are not hidden, but "overt" or open as the text points out.

24CORRECT

Which of the following is an obstacle to collusion?

A)Economic slowdowns often cause the oligopoly to reduce its price.

B)It is often difficult to block the entry of new firms.

C)There is a temptation for firms to cheat and lower their price to increase profits.

D)All of the other choices.

Feedback: All three of these statements illustrate obstacles to collusion.

25CORRECT

Non-price competition such as advertising is often more attractive to oligopolists than price wars because...

A)Developing new products and advertising makes it difficult for competing firms to imitate them.

B)Firms may have a difficult time imitating price changes.

C)Price changes usually provide more lasting gains than non-price competing.

D)

None of the other choices.

Feedback: Only choice 1 is correct. It is a manor reason why oligopolists prefer non-price competition over price wars.

Page 343: Microeconomics Study Guide

1CORRECT Which of the following involve a technological advance?

A)Intel creating its new Pentium 4 processor

B)Microsoft comes out with a new version of windows.

C)Henry Ford develops the assembly line in 1920

D)

All of the other choices

Feedback: Each of these examples shows the development of new and better goods.

2CORRECT According to economists, technological advance occurs over a period of time called...

A)The short run

B)

The long run

C)

The very long run

D)The very short run

Feedback: By definition, choice 3 is correct.

3CORRECT What is the main difference between innovation and invention?

A)

Invention refers to the initial product discovery; while innovation refers to the first successful introduction of the product.

B)

Innovations can be patented, but inventions cannot.

C)

Inventions can enable a firm to "render other firms' products or processes obsolete."

D)They are both the same in meaning

Feedback: Choice 1 is correct by definition.

4CORRECTSuppose that one pharmaceutical company creates a new drug that helps combat AIDS, and other companies copy the drug and pass it on to consumers. This is an example of...

A)Process innovation

B)Negative externalities

Page 344: Microeconomics Study Guide

C)Diffusion

D)

None of the other choices.

Feedback: By definition, choice 3 is correct.

5CORRECT Which of the following is true of R & D?

A)

It is an "umbrella" term that includes invention, innovation and diffusion.

B)Canada's R & D expenditure is a large percentage of its GDP.

C)

Among the seven leading industrial nations, Canada is a leading spender on R & D.

D)None of the other choices

Feedback: Choice 1 is correct but choice 2 is incorrect. Since Canada's R & D expenditure is a small percentage of its GDP. Choice 3 should read "not a leading spender".

6CORRECT Companies which focus on creating and introducing a new product are called

A)

Conglomerates

B)Monopolies

C)Start-ups

D)Established companies

Feedback: By definition choice 3 is correct.

7CORRECTOnly a small percentage of government R& D spending is allocated to basic scientific research since...

A)The funding is not readily available

B)Scientific principles cannot be immediately applied to commercial uses

C)

University scientists and innovators rarely work together.

D)

None of the other choices.

Feedback: Choice 2 is correct. However, recently scientists have found that more of their work

Page 345: Microeconomics Study Guide

can have commercial uses.

8CORRECT Which of the following is an example of an entrepreneur?

A)Bill Gates

B)Henry Ford

C)

Dave Thomas – founder of Wendy's

D)All of the other choices.

Feedback: All 3 of these choices are correct. Since they are innovators and risk-takers who have combined resources to produce goods and services.

9CORRECTA technology firm is considering spending an extra $5 million on R & D to product a new product. It will continue to spend funds on R & D until

A)The marginal benefit (MB) of investing in R & D exceeds the marginal cost (MC).

B)

The marginal cost of investing in R & D exceeds the marginal benefit.

C)The marginal benefit of investing in R & D just equals the marginal cost.

D)None of the other choices

Feedback: As in basic economics, a firm will continue to increase its R & D spending until MB = MC.

10CORRECT

Deciding how much to spend on R & D can be tricky since...

A)

The marginal costs of R & D spending are not always apparent

B)

The marginal benefits are realized in future and are often uncertain.

C)It is often difficult for a firm to get financing.

D)Interest rates are often too high

Feedback: Choice 2 is correct. The expected benefits will be realized in the future long after the product has been purchased. Thus, there is some risk attached.

1CORRECT Which of the following is not a common method of financing R& D activities?

Page 346: Microeconomics Study Guide

1

A)

Bonds

B)

Bank loans

C)Venture capital

D)

Dividends

Feedback: Choice 4 is not a common method of financing since they are given out to shareholders while retained earnings are used to finance R & D.

12CORRECT

Suppose that firm X faces a constant marginal cost of 6% for its borrowing. Its expected rate of return curve in millions is:R & D ERR %

15 1030 845 660 475 2

In this case, firm X will spend ____ on R & D.

A)$15 million

B)$30 million

C)$45 million

D)$60 million

Feedback: Optimal amount of R & D spending occurs where the marginal cost of borrowing just equals the expected rate of return. This occurs where r = I = 6% spending = $45 million.

13CORRECT

If at some level of R & D spending, r > i, then...

A)The company would expand R & D spending until r = i

B)The company would contract R & D spending until r = i

C)

The company would continue spending on R & D at the same level

D)

None of the other choices.

Page 347: Microeconomics Study Guide

Feedback: Choice 1 is correct. If r > i, the expected rate of return exceeds the marginal borrowing (the interest rate). Thus, a firm should expand its R & D spending as MB > MC. The firm should keep spending until r = i.

14CORRECT

The expected rate of return curve is negatively sloped because...

A)As i falls, it becomes less attractive for firms to borrow money.

B)

A firm will allocate its first R & D expenditures to the activities it believes will yield the highest rates of return.

C)

As the firm increases R & D spending, it will finance projects with successively lower rates of return

D)Choices 2 and 3 are correct.

Feedback: The text explains that choices 2 and 3 are correct.

15CORRECT

A decrease in the interest rate (i),

A)Will lower the marginal cost of R & D spending

B)

Will raise the marginal cost of R & D spending

C)Will increase the marginal benefit of R & D spending

D)Will lower the marginal benefit of R & D spending.

Feedback: As i decreases, borrowing costs fall, so the marginal cost of expanding R & D spending falls as well.

16CORRECT

In the model "optimal R & D expenditures" the opportunity cost of expanding R & D spending is...

A)The expected rate of return on the project undertaken.

B)The interest rate given up on the funds used.

C)Profits must be given up now in exchange for the future.

D)

None of the other choices.

Feedback: Choice 2 is correct. As i increases, the opportunity cost of R & D spending increases. More interest must be forgiven on the money spent.

Page 348: Microeconomics Study Guide

17CORRECT

If a new product is introduced into the market

A)Consumers will base their decision on whether to buy the product on pricing.

B)

Consumers will base their decision on whether to buy the product on marginal utility only.

C)

Consumers will purchase the new product only if it increases their marginal utility obtained.

D)

None of the other choices.

Feedback: Choice 4 is correct. When deciding whether to purchase the new product, consumers will evaluate both the marginal utility they desire and its price.

18CORRECT

Process innovation will...

A)Increase total product and reduce average total cost.

B)Increase both total product and average total cost.

C)Decrease total product and increase average total cost.

D)Decrease both total product and average total cost.

Feedback: Choice 1 is correct as explained in the text.

19CORRECT

What incentive is there for firms to first produce a good, in spite of the higher expenses and risks of innovation?

A)By being first, firms can get patents that will protect them against imitation.

B)

Trade marks protect the original innovations of a product by "giving the exclusive rights to use a given product name."

C)

The firm that first introduces a product may receive brand-name recognition form consumers

D)All of the other choices.

Feedback: Each of these reasons acts as an incentive.

20CORRECT

Which of the following market structures has the strongest incentive to under take R and D spending?

Page 349: Microeconomics Study Guide

A)Pure competition

B)Monopolistic competition

C)Oligopoly

D)Pure monopoly

Feedback: Choice 3 has the most incentive to engage in R and D spending.

21CORRECT

According to the inverted-u theory of R & D expenditures,

A)

A single firm with 100% market share has more incentive to innovate than a firm with 1% market share.

B)

A single firm with a 70% market share will have less incentive to innovate that a single firm with a 5% market share.

C)

R & D expenditures are maximized when four larger firms share about 50% of the industry sales.

D)

None of the other choices.

Feedback: Choice 3 is shown to be correct in the text.

22CORRECT

Bob Canuck is an avid tennis player who likes to play at the local tennis club. Bob is willing to pay $25 for the first hour of tennis, $20 for the second hour, $15 for the third hour, $10 for the fourth hour and nothing ($0) for the fifth hour (he is to tired to play anymore!) If the tennis club charges a flat rate of $12/hour, what is the total consumer surplus realized for four hours of tennis?

A)

$10

B)$13

C)

$21

D)

$24

Feedback: Total consumers' surplus:= ($25 - $12)+ ($20 - $12)+ ($15 - $12)+ $0 + $0

Page 350: Microeconomics Study Guide

=$24

23CORRECT

The demand and supply schedule for Canadian flats in a small city are shown below (in thousands):

P Qs Qd30 10 125 8 220 6 315 4 410 2 55 1 6

If the market price is $15, total producer surplus is

A)$5

B)$8

C)

$15

D)

$25

Feedback: Since the supply curve = total marginal cost, then total surplus = ($15 - $5)+ ($15 - $10)= $15

24CORRECT

Which of the following is true if the price in the previous question were set at P = $20

A)Both consumer's and producer's surplus would be reduced.

B)

A dead-weight loss would occur for society.

C)

The marginal cost of producing Canadian flags would be less than the marginal benefit.

D)

All of the other choices

Feedback: All of these are true.

The Principles of Microeconomics 1 of 11 Practice Final Exam - VERSION 1 Answers

Name______________________________

Page 351: Microeconomics Study Guide

Framingham State College

Department of Economics and Business

Principles of Microeconomics Section D

Practice Final Exam – Spring 2006 VERSION 1 WITH ANSWERS

This exam provides questions that are representative of those contained on your exam. This test

should in no way be considered a facsimile of your exam. All topics, problems, key words,

and class discussion are fair game for the exam whether or not they are contained on this

practice exam.

-----------------------------------------------------------------------------------------------------------

Directions:

• Provide the best answer to the following:

• 25 Multiple Choice Questions each worth 8 points for a total of 200 points.

• You will be given 110 minutes to complete the exam.

• If you finish early, you may quietly turn in your exam and leave.

• The exam is closed book and closed note, please place everything other than your calcu-

lator and writing tools at the front of the class.

• You may not share a calculator.

------------------------------------------------------------------------------------------------------------

Multiple Choice: Identify the letter of the choice that best completes the statement or answers the

question.

____ 1a. The word economy comes from the Greek word for

a. "environment."

b. "one who manages a household."

c. "one who participates in a market."

d. "conservation."

ANS: B "one who manages a household." The Principles of Microeconomics 2 of 11 Practice Final Exam - VERSION 1 Answers

Page 352: Microeconomics Study Guide

____ 1b. Trade can benefit a family

a. by allowing the family to buy a greater variety of goods and services at a lower cost.

b. by allowing each person to specialize in the activities he or she does best.

c. only if the family is not in economic competition with other families.

d. All of the above are correct.

e. Both a and b are correct.

ANS: E Both a and b are correct.

____ 1c. A budget deficit

a. occurs when government receipts are less than spending.

b. occurs when government spending is less than receipts.

c. occurs when government receipts are equal to spending.

d. is the accumulation of years of government overspending.

ANS: A occurs when government receipts are less than spending.

____ 2. Total revenue equals

a. total output multiplied by price per unit of output.

b. total output divided by profit.

c. (total output multiplied by sales price) - inventory surplus.

d. (total output multiplied by sales price) - inventory shortage.

ANS: A total output multiplied by price per unit of output.

____ 3. When adding another unit of labor leads to an increase in output that is smaller than increases in

output that resulted from adding previous units of labor, we have the property of

a. diminishing labor.

b. diminishing output.

c. diminishing marginal product.

d. negative marginal product.

ANS: C diminishing marginal product. The Principles of Microeconomics 3 of 11 Practice Final Exam - VERSION 1 Answers

____ 4. Assume that a given firm experiences decreasing marginal product of labor with the addition of each

worker regardless of the current output level: average fixed cost will be

Page 353: Microeconomics Study Guide

a. always rising.

b. always falling.

c. U-shaped.

d. constant.

ANS: B always falling.

____ 5. Economies of scale occur when

a. long-run average total costs rise as output increases.

b. long-run average total costs fall as output increases.

c. average fixed costs are falling.

d. average fixed costs are constant.

ANS: B long-run average total costs fall as output increases.

____ 6. When a firm in a competitive market receives $500 in total revenue, it has a marginal revenue of $10.

What is the average revenue, and how many units were sold?

a. $5 and 100

b. $10 and 50

c. $10 and 100

d. The answer cannot be determined from the information given.

ANS: B $10 and 50

____ 6. If marginal cost exceeds marginal revenue, the firm

a. is most likely to be at a profit-maximizing level of output.

b. should increase the level of production to maximize its profit.

c. must be experiencing losses.

d. may still be earning a profit.

AN S: D may still be earning a profit. The Principles of Microeconomics 4 of 11 Practice Final Exam - VERSION 1 Answers

____ 7. As part of an estate settlement Mary received $1 million. She decided to use the money to purchase a

small business in Anywhere, USA. If Mary would have invested the $1 million in a risk-free bond

fund she could have made $100,000 each year. She also quit her job with Lucky.Com Inc. to devote

all of her time to her new business; her salary at Lucky.Com Inc. was $75,000 per year.

Page 354: Microeconomics Study Guide

At the end of the first year of operating her new business, Mary's accountant reported an accounting

profit of $150,000. What was Mary's economic profit?

a. $25,000 loss

b. $50,000 loss

c. $25,000 profit

d. $150,000 profit

ANS: A $25,000 loss

___ 8. Which of the following statements is correct?

a. A competitive firm is a price maker and a monopoly is a price taker.

b. A competitive firm is a price taker and a monopoly is a price maker.

c. Both competitive firms and monopolies are price takers.

d. Both competitive firms and monopolies are price makers.

ANS: B A competitive firm is a price taker and a monopoly is a price maker.

____ 9. Competitive firms have

a. downward-sloping demand curves and they can sell as much output as they desire at the market

price.

b. downward-sloping demand curves and they can sell only a limited quantity of output at each

price.

c. horizontal demand curves and they can sell as much output as they desire at the market price.

d. horizontal demand curves and they can sell only a limited quantity of output at each price.

ANS: C horizontal demand curves and they can sell as much output as they desire at the market

price. The Principles of Microeconomics 5 of 11 Practice Final Exam - VERSION 1 Answers

Figure 1

____ 10. Refer to Figure 1. The monopolist's maximum profit

a. is $800.

b. is $1,000.

Page 355: Microeconomics Study Guide

c. is $1,250.

d. cannot be determined from the diagram.

ANS: D cannot be determined from the diagram.

____ 11. If a monopolist is able to perfectly price discriminate,

a. consumer surplus is always increased.

b. total surplus is always decreased.

c. consumer surplus and deadweight losses are transformed into monopoly profits.

d. the price effect dominates the output effect on monopoly revenue.

ANS: C consumer surplus and deadweight losses are transformed into monopoly profits.

____ 12. Which of the following may eliminate some or all of the inefficiency that results from monopoly

pricing?

a. The government can regulate the monopoly.

b. The monopoly can be prohibited from price discriminating.

c. The monopoly can be forced to operate at a point where its marginal revenue is equal to its

marginal cost.

d. All of the above are correct. The Principles of Microeconomics 6 of 11 Practice Final Exam - VERSION 1 Answers

ANS: A The government can regulate the monopoly.

____ 13. In a typical cartel agreement, the cartel maximizes profit when it

a. behaves as a monopolist.

b. behaves as a duopolist.

c. is flexible in enforcing production targets.

d. behaves as a perfectly competitive firm.

ANS: A behaves as a monopolist.

Use the following information to answer question 14. Two cigarette manufacturers (Firm A and

Firm B) are faced with lawsuits from states to recover the health care related expenses associated

with cigarette smoking. Both cigarette firms have evidence that indicates that cigarette smoke

causes lung cancer (and other related illness). State prosecutors do not have access to the same

data used by cigarette manufacturers and thus will have difficulty recovering full costs without

Page 356: Microeconomics Study Guide

the help of at least one cigarette firm study. Each firm has been presented with an opportunity to

lower their liability in the suit if they cooperate with attorneys representing the states.

Table 1

Firm A

Concede that

cigarette smoke

causes lung cancer

Argue that there is no

evidence that smoke

causes cancer

Firm B Concede that

cigarette smoke

causes lung cancer

Firm A profit = $–20 b

Firm B profit = $–15 b

Firm A profit = $–50 b

Firm B profit = $–5 b

Argue that there

is no evidence that

smoke causes can-

cer

Firm A profit = $–5 b

Firm B profit = $–50 b

Firm A profit = $–10 b

Page 357: Microeconomics Study Guide

Firm B profit = $–10 b

____ 14. Refer to Table 1. Pursuing its own best interests, Firm A will concede that cigarette smoke causes

lung cancer

a. only if Firm B concedes that cigarette smoke causes lung cancer.

b. only if Firm B does not concede that cigarette smoke causes lung cancer.

c. regardless of whether Firm B concedes that cigarette smoke causes lung cancer.

d. none of the above; in pursuing its own best interests, Firm A will in no case concede that cigarette

smoke causes lung cancer.

ANS: C regardless of whether Firm B concedes that cigarette smoke causes lung cancer. The Principles of Microeconomics 7 of 11 Practice Final Exam - VERSION 1 Answers

____ 15. Assume that Peach Computers has entered into a resale price maintenance agreement with Computer

Super Stores Inc. (CSS Inc.) but not with CompuMart. In this case,

a. The wholesale price of Peach computers will be different for CSS Inc. than it is for CompuMart.

b. Peach computers will never increase profits by having a resale price maintenance agreement with

all retail outlets that sell its products.

c. CompuMart will benefit from customers who go to CSS Inc. for information about different

computers.

d. CSS Inc. will sell Peach computers at a lower price than CompuMart.

ANS: C CompuMart will benefit from customers who go to CSS Inc. for information about

different computers.

Figure 2

____ 16. Refer to Figure 2. The firm depicted in panel b faces a horizontal demand curve. If panel b depicts a

profit-maximizing firm,

a. it could be operating in either a perfectly competitive market or in a monopolistically competitive

market.

b. it would not have excess capacity in its production as long as it is earning zero economic profit.

c. it is able to choose the price at which it sells its product.

d. All of the above are correct.

Page 358: Microeconomics Study Guide

ANS: B it would not have excess capacity in its production as long as it is earning zero

economic profit. The Principles of Microeconomics 8 of 11 Practice Final Exam - VERSION 1 Answers

____ 17. Since a firm in a monopolistically competitive market faces a

a. downward-sloping demand curve, it will always operate with excess capacity.

b. downward-sloping demand curve, it will always operate at efficient scale.

c. perfectly elastic demand curve, it will always operate with excess capacity.

d. perfectly inelastic demand curve, it will always operate at efficient scale.

ANS: A downward-sloping demand curve, it will always operate with excess capacity.

____ 18a. Critics of vacation advertisements that depict "beautiful" human bodies on sun-drenched beaches as

the primary focus of the message are likely to claim that the purpose of the advertisement is to

a. provide helpful information about a tourist destination.

b. manipulate people's tastes through psychological messages.

c. inflame carnal desire.

d. encourage sun-bathing in general.

ANS: B manipulate people's tastes through psychological messages.

____ 18b. On a vacation to Cancun, Mexico, you find yourself eating every meal at the local McDonald's rather

than having a hamburger from one of the street vendors. Your traveling companion claims that you

are irrational, since you never eat McDonald's hamburgers when you are home and McDonald's

hamburgers cost more than those prepared and sold by Cancun's street vendors. An economist would

most likely explain your behavior by suggesting that

a. your behavior is rational, but your friend's behavior is clearly irrational.

b. you are clearly irrational.

c. the McDonald's brand name guarantees consistent quality.

d. the advertising by McDonalds in Cancun is more persuasive than the advertising by McDonalds

in your home town.

ANS: C the McDonald's brand name guarantees consistent quality. The Principles of Microeconomics 9 of 11 Practice Final Exam - VERSION 1 Answers

Table 2

Number of

Page 359: Microeconomics Study Guide

Workers

Output Marginal Product

of Labor

Value of Marginal

Product of Labor

Wage Marginal-

Profit

0 0

1 100 $1,000 $500 $500

2 80 $ 800 $500

3 60 $500 $100

4 280 $ 400 $500

5 20 $500

____ 19. Refer to Table 2. What is the marginal profit of the fourth worker?

a. $400

b. $200

c. $0

d. $-100

ANS: D $-100

____ 20. The opportunity cost of an hour of leisure is

a. 60 minutes.

b. an hour of sleep.

c. an hour's worth of wear and tear on your TV, exercise equipment, etc.

d. your hourly wage.

ANS: D your hourly wage. The Principles of Microeconomics 10 of 11 Practice Final Exam - VERSION 1 Answers

Figure 3

____ 21. Refer to Figure 3. Assume W1 = $20 and W2 = $18 and the market is always in equilibrium. Then the

Page 360: Microeconomics Study Guide

shift of the labor supply curve from S1 to S2

a. increases the value of the marginal product of labor by $2.

b. decreases the value of the marginal product of labor by $2.

c. decreases the value of the marginal product of labor by more than $2.

d. does not change the value of the marginal product of labor.

ANS: B decreases the value of the marginal product of labor by $2.

____ 22. Because of diminishing returns, a factor in abundant supply has

a. a high marginal product and a high rental price.

b. a high marginal product and a low rental price.

c. a low marginal product and a high rental price.

d. a low marginal product and a low rental price.

ANS: D a low marginal product and a low rental price.

____ 23. A recent law school graduate is considering two offers to practice law, one in California and one in

Alabama. Rumor has it that it is very difficult to pass the Alabama bar exam and meet other

certification requirements required by the Alabama Bar Association. Assuming all other things equal,

the attorney would expect

a. to be unable to predict the wage difference between Alabama and California.

b. to make a higher wage in California.

c. to make a higher wage in Alabama.

d. wages in California and Alabama to be identical. ANS: C to make a higher wage in Alabama.

____ 24. Workers with more human capital on average earn substantially higher pay than workers with less

human capital in

a. most countries, but not in the United States.

b. the United States, but not in most other countries.

c. the United States and in most other countries.

d. None of the above are correct; the evidence fails to indicate that human capital is a significant

factor in earnings anywhere in the world.

ANS: C the United States and in most other countries.

Page 361: Microeconomics Study Guide

____ 25. If employers are profit-maximizers, then

a. competition will always eventually eliminate employment discrimination.

b. employment discrimination may persist if consumers discriminate.

c. employment discrimination will persist because it is always profitable.

d. compensating differentials will not exist.

ANS: B employment discrimination may persist if consumers discriminate.

Questions that still need thought attention are 11 (the new graph in

the answer key is not showing u), 21 (the answer is in yellow)

Use the following to answer question 1.

Figure: Demand and Supply of Gasoline

1. (Figure: Demand and Supply of Gasoline) The supply curve shifts from S 1 to S 2 .

Which statement is most correct?

A) at the old equilibrium price of $2.50, there will be pressure for the price to fall.

B) the new price will be $2.00.

C) the new quantity will be 300.

D) all of the above are true.

A: A) is correct, B and C are not

Page 362: Microeconomics Study Guide

Use the following to answer question 2.

Figure: Aggregate Demand for Shirts

2. (Figure: Demand for Shirts) The price elasticity of demand for the segment BC using

the midpoint method is:

A) greater than 3.33 (absolute value).

B) 3.33.

C) 3.

D) 0.33.

C: the price elasticity of demand =( change in demand/average demand)/(change in price/average

price) = (100/150) /(10/45) =45/15=3

The answer from my point of view is C.

The KW says, the price elasticity of demand – the percent change in the quantity demanded divided

by the percent change in the price.

3. Assume you consume two commodities, pollution, which is a bad, and other stuff. Do

your indifference curves for pollution and other stuff look the same as the indifference

curves discussed in Krugman and Wells?

A) Yes

B) No

C) Maybe

B: If the other stuff are goods then the indifference curves are upward sloping, so not like

in KW. If other stuff is bad, so two bads, the indifference curves are downward sloping

Page 363: Microeconomics Study Guide

but bend the opposite way from those in KW.

Use the following figure to answer questions 4-5.

Figure: A Perfectly Competitive Firm in Short Run

4. (Figure: A Perfectly Competitive Firm in the Short Run) At a price of G, the firm's

total cost is represented by the area _____, and it’s profits are _________. A) FGLK, negative.

B) FGLK, positive.

C) 0FKD, negative.

D) 0FKD, positive.

D: Total costs are average costs at quantity D, F multiplied by D, so OFKD. Total

revenues are greater, OGLD, so profits are positive.

5. (Figure: A Perfectly Competitive Firm in the Short Run) The price at which profits are

zero is:

A) G.

B) F.

C) E.

D) N.

C: at a price of E at the profit maximizing quantity, price equals average costs

Page 364: Microeconomics Study Guide

6. The slope of a(n) _______ curve shows the rate at which two goods can be exchanged

_______ the consumer's ________.

A) marginal utility; which increases; marginal utility

B) indifference; without affecting; total utility

C) iso-utility; without affecting; budget

D) indifference; without affecting; budget

B: The indifference curve has nothing to do with the budget, B is correct, and A makes

no sense to me.

7. Which of the following is not true for indifference curves of ordinary goods?

A) They never cross.

B) They slope downward.

C) They are convex from the origin.

D) Indifference curves farther away from the origin have lower levels of utility.

D: Since goods are goods, as one increases the amount of both goods (moves to the right),

one moves to a more preferred bundle, one with higher utility, not lower utility.

8. If the price of a cookie is $1 and the price of a brownie is $2, the relative price of

cookies in terms of brownies is:

A) 0.50.

B) 1.00.

C) 2.00.

D) undefined.

A: The question asks how many brownies one has to give up to get one more cookie. One

has to give up ½ of a brownie.

Page 365: Microeconomics Study Guide

9. If the marginal rate of substitution of cookies in terms of brownies equals the relative

price of cookies in terms of brownies, the consumer:

A) is maximizing her utility

B) should consume more cookies and fewer brownies to maximize total utility.

C) should consume fewer cookies and more brownies to maximize total utility.

D) may or may not be maximizing total utility.

D: since the question says nothing about budget exhaustion, we do not know whether

the respondent is maximizing utility.

10. If Joseph chooses a combination of apples and oranges along his budget line where

the marginal rate of substitution of apples in place of oranges is 2 and the price of an

apple is $0.50 and the price of an orange is $0.50, then Joseph:

A) is maximizing total utility.

B) should consume more apples and fewer oranges to maximize total utility.

C) should consume fewer apples and more oranges to maximize total utility.

D) may or may not be maximizing total utility.

B: Joe would give up two oranges to get one more apple (if I interpret correctly), but only

has to give up one orange to get another apple, so should consume more apples

and fewer oranges. From Yuchen-- The MRS(apple to orange) is 2, larger than

the price ratio = 1. So we need to decrease the MU(Apple) and increase

MU(orange). Since utility function is diminishing return to scale, we will

consume more apples to get its MU down.

Use the following to answer questions 11-12.

Figure: Indifference Map for Soda and Pizza

Page 366: Microeconomics Study Guide

The figure shows three of Owen's indifference curves for pizza and soda per week.

Owen has $180 per week to spend on the two goods; the price of a pizza is $20 and

the price of a soda is $1.50.

11. (Figure: Indifference Map for Soda and Pizza) If Owen is consuming 1.5 pizzas and

100 sodas, which of the following is true? A) Owen is maximizing his total utility, given his budget line.

B) Owen should consume more pizza and less soda to maximize his total utility,

given his budget line.

C) Owen should consume less pizza and more soda to maximize his total utility,

given his budget line.

D) Owen should consume more pizza and more soda to maximize his total utility,

given his budget line.

B: His budget line is S=120 – 13.33P; so if he spends all of his money on soda he can

consume 120 cans; if he spends all of his money on pizza he can by 9 pizza. So

at current bundle he is spending $150 on soda and 30 on pizza, so is on his

budget lines. The question is whether the slope of his budget line equals the slope

of his indifference curve at this point.

Draw in the budget line on the above graph. The slope of the budget line is -13.33

Here is the graph with the budget line drawn in. You can see that it crosses the first

indifference curve at the point given in the question and is tangent to the second

indifference curve at 6 pizzas and 40 sodas, which shows he must increase pizza

consumption to maximize utility.

Page 367: Microeconomics Study Guide

12. (Figure: Indifference Map for Soda and Pizza) What is Owen's marginal rate of

substitution of pizza in place of soda when he is consuming his optimal consumption

bundle?

A) 0.75

B) 1.5

C) 13.3

D) 20

I would have said 60. To get another .5 sodas he will give up soda, so 60? I am terribly

confused her. Slope of the budget line (and indiff curve at that point) is 120/9=-

13.333

13. Which of the following statements is true about Average Fixed Cost (AFC)?

A) AFC is always less than Average Variable Cost

B) AFC is always less than Marginal Cost

C) AFC is always decreasing D) AFC is constant across all quantities of output

C: Dividing a positive constant (fixed cost) by larger and larger numbers will make

the ratio decline.

14. Assume an increase in the tax that oil and gas producers pay to the state of Colorado

when they extract gas from the state of Colorado. Will the tax translate to higher gasoline

and fuel prices for consumers?

A) Yes, because demand for gas is inelastic in Colorado.

B) Yes, because demand for gas is elastic everywhere.

Page 368: Microeconomics Study Guide

C) No, because the price of gas is determined by world demand and supply, and how

much is supplied by Colorado is too small of the total to affect the price.

D) No, because Colorado supply is inelastic

E) Cannot determine because we don’t know the elasticity of demand.

C:

Use the following figure to answer questions 15-16.

Figure: Indifference Map for Goods X and Y

15. Figure (Indifference Map for goods X and Y)

The indifference map shown above shows the consumer’s preferences for:

A) Two goods

B) Three goods

C) One good and one bad

D) Two bads or two goods

A: if they were a good and a bad the indifference curves would be upward sloping. If

both bads they would be downward sloping but curve the other way.

16. (Figure: Indifference Map for goods X and Y) Which of the following statements about the points on the graph above is not true?

A) G is feasible but not optimal

B) I is feasible and optimal

C) H is optimal and feasible

D) J is not feasible

Page 369: Microeconomics Study Guide

C) H does not exhaust the budget constraint, so it ot optimal.

17. Marginal revenue of a profit-maximizing firm:

A) is the slope of the average revenue curve.

B) equals the market price in perfect competition.

C) is the change in quantity divided by the change in total revenue.

D) is the price divided by the changes in quantity.

B: a competitive firm cans sell as much as it want at p, so p is its mr.

18. Price elasticity of demand measures the responsiveness of the change in:

A) quantity demanded to a change in price.

B) price to a change in quantity demanded.

C) slope of the demand curve to a change in price.

D) slope of the demand curve to a change in quantity demanded.

A: in percentage terms.

19. Assume that Brazil gives up 3 automobiles for each ton of coffee it produces, while

Peru gives up 7 automobiles for each ton of coffee it produces.

A) Brazil has a comparative advantage in automobile production and should

specialize in coffee.

B) Brazil has a comparative advantage in coffee production and should specialize in

the production of automobiles.

C) Brazil has a comparative advantage in coffee production and should specialize in

coffee production.

Page 370: Microeconomics Study Guide

D) Brazil has a comparative advantage in automobile production and should

specialize in automobile production.

C: to produce another ton of coffee, Brazil gives up 3 cars, Peru gives up 7.

20. Rent controls set a price ceiling below the equilibrium price. Which is the best

explanation of what will happen?

A) quantity supplied exceeds the quantity demanded.

B) quantity demanded exceeds the quantity supplied.

C) a surplus of rental units will result.

D) poor people will be helped.

B: B is correct. A and C are incorrect. While some poor people might be helped they will

not all be helped.

21. True or False: As a general rule, a perfectly competitive firm should not produce

where MC > ATC.

A) True

B) False

C) It depends on the fixed costs

D) It depends on average fixed costs

False: It should produce where MC>ATC. Prices are maximized where P=MC. If at this

point MC<ATC, the firm would be losing money.

22. Assume that aquarium fish and dog biscuits are the only two goods that Wilma

consumes – what a diet. How would Wilma’s indifference curves change if she woke up

one morning having developed a stronger preference for dog biscuits? Assume that dog

Page 371: Microeconomics Study Guide

biscuits are on the x-axis and goldfish are on the y-axis.

A) Her indifference curves would become less steep

B) Her indifference curves would not change

C) Her indifference curves would become steeper

D) Her indifference curves will bend backwards

C: Consider a point on the original indifference curve, and how many more biscuits she

would need to give up a fish. After her preference for biscuits increases she would need

fewer biscuits to make up for the lost fish; her indifference curve is steeper.

23. Which of the following statements is true about the short run?

A) The firm can go out of business

B) The firm can produce zero output

C) None of the above

D) A & B

B: The firm can produce zero output in the short run, but they cannot rid of their fixed

costs, so must stay in business, keep paying the bills.

24. When the benefit to you of doing something are lower than the benefits to society,

from society’s perspective

A) you do too much of it

B) you do too little of it

C) you don’t do it at all

D) you do just the right amount

B: You do too little of it. At your equilibrium level of consumption the marginal benefits

to society are greater than the marginal costs to society.

Page 372: Microeconomics Study Guide

25. Why are Ford, GM and Chrysler in more precarious positions than the foreign

automakers operating in the U.S.?

A) Higher fixed costs

B) Higher variable costs

C) Both of the above

C: They have higher fixed costs in terms of stuff like obligations to their retired workers.

They have higher variables costs because how many hours there workers work can be

varied in the SR and the big three face higher per-hour labor costs (union determined).

26. Your preferences are determined by __________.

A) Market prices

B) Your constraints

C) Market supply

D) Only what is in your head

D:

27. How does one indifference curve represent a change in utility?

A) There is no change in utility between indifference curves

B) The utility taken away by giving up one unit of a good must be offset by the gain

in utility from acquiring some amount of the other good.

C) A single indifference curve represents two levels of utility

D) The utility taken away by giving up one unit of a good must be offset by the gain

in utility from giving up one unit of the other good.

B: along the indifference curve utility is a constant.

Page 373: Microeconomics Study Guide

28. Which of the following equations for indifference curves puts more weight on

sporting goods (s) than cleaning supplies (c)?

A) U(s,c)=s

.8

c

.4

B) U(s,c)=s

1.6

c

.1

C) U(s,c)=s

.7

c

.5

D) U(s,c)=s

.6

c

.6

A: look at the ratio of the two coefficients.

29. Lars and Ingrid have the same preferences for Bibles and Jose Cuervo Gold Tequila.

Lars consumes 3 Bibles and 4 bottles of tequila and Ingrid consumes 3 Bibles and 6

Page 374: Microeconomics Study Guide

bottles of tequila. Which of the following must be true?

A) Ingrid and Lars face different prices

B) Ingrid and Lars have different levels of income

C) Ingrid likes tequila more than Lars

D) A or B or both

E) A, B or C could be true

D: It could be because they have different incomes or because they face different prices,

or both. The question said they have the same preferences, so it cannot be that.

30. Which of the following represents a condition that needs to be met for efficiency to

be achieved in a market economy?

A) Well-defined property rights

B) Excess market power

C) Negative externalities

D) Perfect equity

E) All of the above

A: D has nothing to do with efficiency. B and C are reasons that efficiency would not be

achieved.

31. Which of the following are examples of external effects?

A) I smoke a cigarette you are subjected to second-hand smoke

B) I eat pizza for lunch and am happy because I like pizza

C) You put Snow White and the Seven Dwarfs figurines in your yard and make your

neighbors happy when they walk by.

D) A&C

E) A&B

Page 375: Microeconomics Study Guide

D: A is a negative external effect, C is a positive external effect. In B there are no

external effects mentioned.

False: many external effects are not efficiently taxes or regulated.

32. Since the beginning of the semester: Why did the price of gas go up? Why did the

price of gas go down?

A) Demand growth was outstripping supply growth; a worldwide recession

decreased the demand

B) Supply growth was outstripping demand; a worldwide recession decreased the

price

C) OPEC was refusing to produce more oil; Hugo Chavez decided to produce tons

and tons of oil when the price was high

D) OPEC was pumping more and more oil each month; Saudi Arabia decided to

flood the market to punish OPEC for cheating

A:

33. An increase in the consumer's income will do all of the following, except:

A) shift the budget line away from the origin.

B) increase the horizontal intercept.

C) increase the vertical intercept.

D) change the slope of the budget line.

D: relative prices remain the same, so the slope of the budget line does not change.

Use the following to answer question 34-35.

Page 376: Microeconomics Study Guide

Table: Utility of Ice Cream Cones

Cones 0 1 2 3 4 5 6 7

Total utility 0 20 35 45 50 50 45 35

34. (Table: Utility of Ice Cream Cones) The marginal utility for the second ice cream

cone is:

A) 35.

B) 15.

C) 10.

D) 5.

B: utility increases from 20 to 35 when you eat the second cone.

35. (Table: Utility of Ice Cream Cones) True or False: Ice cream cones are a good.

A) True

B) False

B: They are neither a good or a bad. They start off as a good, but go bad at high levels of

consumption.

36. When you do stuff where the cost to you is less than the cost to society, from

society’s perspective

A) you do too little of it

B) you do too much of it

C) you do just the right amount

D) you don’t do it at all

Page 377: Microeconomics Study Guide

B: You do too much. At your level of consumption the marginal benefit to society is less

than the marginal cost.

37. True or False. There is an efficient amount of each external effect.

A) True

B) False

38. Consider the George tattoo example from class. Which of the following are necessary

conditions for George’s tattoo consumption to constitute a negative externality market

failure? A) George doesn’t internalize the costs his tattoo consumption imposes on society

B) George’s tattoo consumption directly affects other people (not through prices)

C) George is consuming more tattoos than is efficient

D) George’s tattoo consumption causes prices to rise

E) A, B, & C

E:

39. Consider the George tattoo example from class. Which of the following, from an

economist’s perspective, is the best way for the government to entice George to put fewer

tattoos on himself?

A) The government could pass a law saying that George can only have 4.2 tattoos

(the efficient amount)

B) The government could tax George for each tattoo he has.

C) The government could ban tattoos on George

D) A & B

Page 378: Microeconomics Study Guide

D: C would leave George with too few tattoos from society’s perspective.

40. What is a Pigouvian tax?

A) A tax that attempts to drive up demand

B) A tax that internalizes the inefficiency associated with an external effect

C) A tax that attempts to address the inefficiency associated with price floors

D) A tax on property rights

B: by definition

41. True or False: There is a cost associated with reducing pollution

A) True

B) False

True: if there was not, we could get rid of all pollution at zero cost, and would.

42. True or False: There is an efficient amount of pollution from society’s perspective

that is greater than zero

A) True

B) False

True: it would cost too much to eliminate all pollution: you would have to stop breathing.

43. Gary Hart, a former Senator from Colorado and former candidate for the Democratic

nomination for President, said, “Pollution is a bad that we must get rid of.” Your best

response as an economist is

Page 379: Microeconomics Study Guide

A) Maybe

B) Yes

C) No

No: there is an efficient amount of pollution

Use the following information to answer questions 44-45.

The government gave out 300 tradable pollution permits to a number of firms that

produce steel. Each permit allows the producer to emit 1 ton of pollution per year; if a

firm pollutes without a permit, the owners are shot. Without regulations or permits you

produce 4 tons of pollution per year. If you reduced your pollution by 1 ton if would cost

you $100; if you reduced your pollution by 2 tons it would cost you a total of $300; if

you reduced your pollution by 3 tons it would cost you a total of $600; and if your

pollution is reduced by 4 tons the total cost would be $1000.

44. Then, you are given 3 tradable permits. Your neighbor, whose plant is much older

than yours, offers to pay you $250 for each permit you are willing to sell him. You

should _________.

A) Sell him two permits and reduce your pollution by three tons

B) Sell him three permits and reduce your pollution by four tons

C) Sell him one permit and reduce your pollution by two tons

D) Don’t sell him any of the permits and reduce your pollution by one ton

C: You can increase your profits by selling the other guy one permit. This requires that

you reduce your pollution by two permits. Your costs go up by $300 and your revenue

goes up by $250. So you lose $50 in profits. If you reduced you pollution by only one

unit (no sales to him), your profits would go down by $100. If you reduced your pollution

Page 380: Microeconomics Study Guide

by 3 units (selling two to him) your costs would increase by $600 and your revenues

would increase by $500, so your profits would go down by $100. If you reduced you

pollution by four tons, selling 3 to him. You profits would decline by $250 ($750-$1000).

45. Now, assume you are given zero permits. Which of the following statements is

correct?

A) All of the following

B) If the going price of a permit is $100, you should pollute three or four tons

C) If the going price of a permit is $200, you should buy one or two permits

D) You should buy two or three permits if the going price of a permit is $300

A: if the price of a permit is $100, since it cost $100 to reduce pollution from 4 to 3 units,

you are indifferent between producing 3 or 4 units of pollution (buying or not buying one

permit). If the price of a permit is $200, reduce as long as the cost of reducing is less than

$200, so reduce at least one unit (buy at least one permit). You are indifferent to buying

the second permit because you can reduce pollution by a second unit for a cost $200 or

pollute it and pay $200 for the permit. If the going price is $300, it is profitable to buy

two permits, but you are indifferent about buying a third permit. 46. Because they bother other users, the City of Boulder has banned mountain bikers

from most of the trails in the City of Boulder. True or False: The ban likely has achieved

the efficient use of our trails?

A) True

B) False

B: the efficient number of bikers is likely not zero.

47. Consider the George’s tattoo example from class. Assuming others do not like

George’s tattoos, which answer best describes the vertical distance between the benefit to

George from adding a tattoo to his body, and the benefit to society from George adding

Page 381: Microeconomics Study Guide

the tattoo

A) The benefit to society from George adding the tattoo.

B) The willingness-to-pay of others members to stop the tattoo being added

C) The cost of producing the tattoo.

D) None of the above

B: the marginal benefit curve for society lies below the marginal benefit curve for George

because his tattoos make others worse off. The vertical distance is their loss, so what they

would pay to get rid of it.

48. True or False: An externality has occurred if a bunch of people move to Boulder and

drive up the cost of housing.

A) True

B) False

False: this is an example of the market working, not a direct external effect.

49. Of the following, choose the best answer to the question: What causes market

failures?

A) Government price controls

B) The lack of fairness in the distribution of wealth

C) Market prices sending the wrong signals

D) A & C

C: Government price controls might cause inefficiency but are not the fault of the market,

so are not “market” failures.

50. True or False: There can be too little pollution from an efficiency point of view.

Page 382: Microeconomics Study Guide

A) True

B) False

C) Probably true

D) Probably false

C: The sense is that we have under rather than over corrected for the negative external

effects from pollution. But it is difficult to say with 100% certaintly

51. True or False: Assume that the following conditions hold: the government enforces

property rights, there are no market failures and society approves of how wealth is

distributed. Thus, the government should do nothing more (build no roads, build no

schools, and produce no national defense, nothing).

A) True

B) False

True the allocation is efficient and the distribution is fair, so there is nothing for the

government to do besides enforce property rights.

52. What determines the rate you are willing to substitute guns for Obama posters

A) Market prices

B) Your preferences

C) Your preference and market prices

D) The slopes of your budget line and the slope of you indifference curve.

E) More than one of the above.

B: Your MRS is only determined by your preferences, not by prices.

53. Candy bars and cookies are substitutes. The government puts an excise tax of $1 on

candy bars. The burden of the tax falls partly on producers and partly on consumers. How

Page 383: Microeconomics Study Guide

will this tax on candy bars affect the demand for cookies?

A) Demand for cookies will increase.

B) Demand for cookies will decrease.

C) Demand for cookies will stay the same.

D) It is impossible to tell from the given information.

A: the price of candy bars goes up increasing the demand for substitutes of candy bars.

Use the figure to below to answer questions 54-55.

54. The concave shape of the Production Possibilities Frontier shown above reflects A) Increased opportunity cost

B) Constant returns to scale

C) The Law of Diminishing Marginal Utility

D) The Law of Diminishing Marginal Cost

A: As the quantity of grain increases, you must give up more and more wine in order to

increase your production of grain by one unit.

55. In the figure above, point “a” represents:

A) Efficiency in production

B) Inefficiency in production

C) Production beyond technological capabilities

D) Not enough information to tell

Page 384: Microeconomics Study Guide

B: point “a” is inside the PPF

Use the information below for questions 56-58.

Joe used to make $40,000/year and rented 150 movies to watch by himself each year. In

2005, Joe was given a promotion and his salary was increased to $50,000. Subsequently,

Joe rented 30 more movies per year.

56. What is Joe’s income elasticity of demand? (Hint: Use the midpoint formula)

A) 1.25

B) 1

C) .818

D) It is impossible to tell from the information given.

C: Income elasticity of demand is:

% change in quantity demanded

% change in income

= [(180-150)/165]/ [(50,000-40,000)/45,000]

57. Which is Joe’s price elasticity of demand? (Hint: Use the midpoint formula)

A) 1.25

B) 1

C) .8

D) It is impossible to tell from the information given.

D: No information on prices is given.

Page 385: Microeconomics Study Guide

58. For Joe, movies are a

A) Normal good B) Inferior good

C) Giffen good

D) Luxury good

A: demand increased when income increased by the elasticity is less than one, so it is a

normal good.

Use the following figure to answer questions 59-61.

Figure: Indifference Maps of Goods and Bads

Indifference Map A

0

1

2

3

4

5

6

7

8

9

10

0 2 4 6 8 10

X

Indifference Map B

Page 386: Microeconomics Study Guide

0

1

2

3

4

5

6

0 1 2 3 4

X

Indifference Map C

0

0.2

0.4

0.6

0.8

1

1.2

1.4

1.6

1.8

0 2 4 6 8

X

59. (Figure: Indifference maps of goods and bads) Of the maps above, which represents

indifference curves for two bads?

A) Map A

Page 387: Microeconomics Study Guide

B) Map B

C) Map C

D) Map A and Map C

C: Map A represents one good and one bad. For two bads the indifference curve must by

downward sloping: to accept more of one bad you had to have less of the other bad. The

question comes down to shape. Explaining in a way beyond the students, the upper level

sets have to be convex sets; this is true for C but not for B.

60. (Figure: Indifference maps of goods and bads) Which of the following reasons best

explains one reason why your answer to the question above is correct? A) The indifference curves are increasing in utility as you move away from the

origin.

B) The indifference curves are downward sloping, which they must be for two bads.

C) The indifference curves are downward-sloping and increasing in utility as one

moves towards the origin.

D) With two bad, the indifference curves must have a positive slope.

C: see my answer to 59

61. (Figure: Indifference maps of goods and bads) Of the maps above, which represents

indifference curves for one bad and one good?

A) Map A

B) Map B

C) Map C

D) Maps A and Map C

A: if you get more of the good commodity to hold utility constant you need to get more

of the bad commodity.

Page 388: Microeconomics Study Guide

62. True or False: Myrna can produce an efficient number of tattoos for herself without it

being efficient from a societal perspective.

A) True

B) False

True: If Myrna’s tattoos affect other people and she does not have the correct incentive to

take this into account, she will have too many or too few from society’s perspective.

Figure: Ellen’s indifference curve map for bananas and peaches.

Answer questions 63-65 based on Ellen’s indifference curve map for bananas and

peaches given above.

63. (Figure: Ellen’s indifference curve map for bananas and peaches) Ellen has $10 to

spend on bananas and peaches and the prices of each is $1. What is her optimal bundle?

A) 2 bananas, 2 peaches

B) 4 bananas, 6 peaches

C) 7 bananas, 3 peaches

D) 4 bananas, 2 peaches

B: Ellen will maximize utility given the budget line when she consumes 4 banana and 6

peach. He also exhausts his budget when he consume bundle C, but that is not optimal.

Once you draw the budget line, you can check that.

64. (Figure: Ellen’s indifference curve map for bananas and peaches) What is the

Page 389: Microeconomics Study Guide

marginal rate of substitution of peaches for bananas when she is consuming her optimal

bundle?

A) 1/10

B) 1/5

C) 1

D) 5

C: At the optimal bundle, MRS equals the ratio of prices, and the prices are equal

65. (Figure: Ellen’s indifference curve map for bananas and peaches) Ellen nowhas $20

to spend on banana and peach and the prices of both are $2 each. What’s the optimal

bundle?

A) 2 bananas, 2 peaches

B) 4 bananas, 6 peaches

C) 5 bananas, 5 peaches

D) 4 bananas, 2 peaches

B:His budget line does not change from the original. So his optimal bundle would be

same.

66. The marginal rate of substitution of peaches for apples, in absolute terms, is

represented by ________.

A) How many apples one would give up to get another peach

B) How many peaches one would give up to get another apple

A: Note that word “for – “of peaches for apples”. So this is the change in apples divided

by the increase in peaches, holding utility constant, and converted to a positive number.

Page 390: Microeconomics Study Guide

For example if it is 4 it says to get one more peach you would give up four apples.

67. Imagine a wilderness park in the middle of nowhere, filled with exotic wildlife and

fauna. No one visits the park; it is too far to go. Which of the following statements would

an economist most agree with: A) The park is a good example of a public good

B) The park is a bad example of a public good.

A: The answer is good example. The benefits one get from the part are non-use values

and non-congestible. In addition once the park is created for you it is also there for me.

68. True or False: A congestible resource is one where one person’s use of a unit of the

resource precludes the use of that unit by others.

A) True

B) False

True: This is the definition of congestible resource.

69. Imagine that the three firms that produce Guber gas all have the same marginal cost

of abatement curves. Further assume that in the unregulated state they all emit the same

amount of Guber gas. Which statement is incorrect?

A) Guber gas emissions can be efficiently reduced by 100 units by splitting the 100

unit reduction among the three firms such that each is reducing emissions the

same amount.

B) Efficiently reducing Guber gas emissions by 100 units requires different amounts

of abatement for the three firms.

C) Guber gas emissions can be efficiently reduced by 100 units by splitting the 100

unit reduction among the three firms such that each after the split the marginal

cost of abatement is equal for all three firms.

Page 391: Microeconomics Study Guide

B: The second one is incorrect. They all have the same marginal cost of abatement

curves, so the same marginal costs of abatement. So, efficiency dictates that they all

reduce their pollution by the same number of units. A and C are correct statements.

70. Will widening I-70 between Denver and Summit County, by itself, eliminate all

congestion externalities on the highway?

A) Yes

B) No

C) Maybe

D) Can’t tell

B: The answer is no. There will still be un-internalized negative effects.

71. True or False: I would enjoy another cigar more than another cigarette. I should

therefore smoke another cigar rather than another cigarette.

A) True

B) False

False: The fact that I would enjoy it more does not mean I should choose it. Which I

should choose also depends on the price.

72. Which statement best describes microeconomics?

A) It develops models of how individual and firms behave assuming that individuals

behave rationally.

B) It is a course in how to run a business and get more utility from life.

C) It is science devoted to understanding and explaining the choices made by

individuals, firms and other economic agents, and the outcomes of those choices.

Page 392: Microeconomics Study Guide

D) It helps to explain the current recession.

C:

73. True or False: Basic consumer theory is consistent with the idea that one’s

preferences are a function of what they consume?

C) True

D) False

False: Preferences are assumed stable. In basic consumer theory, what you consume

affects your utility level but not your preferences

74. Which of the following is the best definition of property rights?

A) Having property rights to a commodity means one owns it. For example, one can

build a house on your land because you own it.

B) Having property right to a commodity means one can sell it to someone else

C) Having property rights to a commodity means that one can control access to its

use and consumption. One can, if they choose to, relinquish these controls

D) All of the above

C: Saying that having property rights means you “own it” is not very informative. B is

true by incomplete. C gives the most detailed definition.

75. True or False: Everybody does everything better than Joe. If he works, he should

work producing what he can produce the most of per hour.

A) True

B) False

Page 393: Microeconomics Study Guide

False, he should produce the commodity for which he has a comparative advantage,

which might be the commodity he can produce the most quickly (has an absolute

advantage), but not necessarily.

76. True or False: The U.S. should do everything it can to eliminate violent crime in the

U.S.

A) True

B) False

False: have to look at the benefits and costs of eliminating additional crime. The marginal

cost of or eliminating crime likely rises steeply rises.

77. The basic economic models of consumer behavior does not assume:

A) Five utils of utility is better than 2 utils.

B) Five utils of utility is 2.5 times better than 2 utils

C) Transitive preferences.

B: the model only assumes the individual can rank bundles. They won’t know what

transitive preferences are but that is ok.

78. In the mind of a economist, which statement is least correct?

A) There is some efficient amount of global warming

B) There is a cost to reducing global warming

C) The benefits of reducing global warming are always greater than the costs.

D) It might be better to help now poor people in low-lying tropical countries rather

than allocating resources to reducing global warming.

Page 394: Microeconomics Study Guide

C:

79. True or False: City, State and National parks are pubic goods:

A) True

B) False

False: While they are provided by the government, they are congestible.

80. True of False: If a purely competitive firm took it upon itself to reduce its pollution it

would go out of business in the longrun.

A) True

B) False

True.

State whether the following statements are true or false and explain why you say so. I will only grade

what is written in the space provided (10 points total).

1. (6 points) Using a Solow model it is possible to show that the saving rate that achieves the maximal

long-run GDP level also achieves the highest long-run consumption level.

FALSE, HOLDING OTHER FACTORS CONSTANT, THE SAVING RATE THAT GEN-

Page 395: Microeconomics Study Guide

ERATES THE HIGHEST SOLOW CURVE IS S=1. THUS, THE SAVING RATE THAT

GENERATES THE HIGHEST LONG RUN CAPITAL, AND THE HIGHEST GDP IS

S=1. HOWEVER, AT S=1 CONSUMPTION IS ZERO.

2. (4 points) Consider an economy where the labor market is always in equilibrium (and goes back to

equilibrium immediately after any possible shock). If the central bank of such an economy performs an

open market purchase whenever there is a recession then GDP will necessarily increase, which results

in a milder recession. It is thus clear that central banks should always play an active role in smoothing

the business cycle.

FALSE. IF THE LABOR MARKET IS ALWAYS IN EQUILIBRIUM THEN AGGRE-

GATE SUPPLY IS PERFECTLY VERTICAL. CHANGES IN THE INTEREST RATE

(WHICH RESULT FROM OPEN MARKET OPERATIONS) WOULD HAVE NO IM-

PACT ON GDP, ONLY ON PRICES.

Multiple choice section

Circle what you consider is the best answer to each of the following questions (50 points total, equally

weighted).

Use the following data to answer questions 3-5

Consumption in Snowville

2002 2003

Price Quantity Price Quantity

Rutabaga $0.50 200 $0.70 110

Parka $50.00 2 $75.00 1

Book $40.00 5 $30.00 10

In 2002, consumers in Snowville consumed only rutabagas, parkas, and books. The prices and quantities for

2002 and 2003 are listed in the above table. The base year for Snowville’s CPI is 2002

3. The CPI for Snowville in 2002 equaled

(a) 1.1

(b) 1

B, BY DEFINITION OF CPI

Page 396: Microeconomics Study Guide

4. The CPI for Snowville in 2003 equaled

(a) 1.1

1(b) 1

(c) none of the above

CPI 2003 (BASE = 2002) =

200 × $0.7 + 2 × $75 + 5 × $30

200 × $0.5 + 2 × $50 + 5 × $40

=

140 + 150 + 150

100 + 100 + 200

=

440

400

= 1.1

5. The 2003 real GDP of Snowville equaled (assume the base year is also 2002)

(a) 452

(b) 505

(c) 400

Useful information: The price index of year t when the base year is year ˆt equals the cost of the base

year basket at period t prices divided by the cost of the base year basket at period ˆt (the base) prices.

REALGDP 2003 (BASE2002) = $0.5 ∗ 110 + $50 ∗ 1 + $40 ∗ 10 = 55 + 50 + 400 = 505

6. Gross domestic product (GDP) equals the ______ of final _______ produced within a country

during a given period of time

(a) market value; goods

(b) market value; services

(c) market value; goods and services

(d) quantity; goods

(e) quantity; goods and services

Page 397: Microeconomics Study Guide

Answer: C

7. You face two job offers. The first is in London and pays £50,000 per year. The second is in the United

Status and pays $84,440 U.S. dollars per year. Assume that the exchange rate between British pounds

and American dollars is £1 = $1.8 U.S. In PPP terms £1 = $1.2 U.S. PPP. The offer that provides

you with the highest purchasing power is

(a) The first

(b) The second

Answer: B. THE OFFER IN LONDON, IN PURCHASING POWER, IS EQUIVA-

LENT TO 1.2*50000=70,000<84,440.

8. Which of the following transactions would be included in the GDP of the United States?

(a) Coca Cola produces soft drinks in England.

(b) Honda produces cars in Ohio.

(c) McDonalds produces and sells hamburgers in Russia.

(d) Ford Motors produces cars in Mexico.

(e) None of the above.

Answer: B.

9. During 1970, total compensation of employees (i.e. the sum of wage income + all other benefits) per

worker in the United States equaled $6,085. Total compensation of employees per worker in 2005

equaled $50,107 dollars. The price level during 1970 was 27 and it equaled 112 during 2005. Using

these numbers you can conclude

2(a) That the purchasing power of the average worker in the United Sates roughly doubled from 1970

to 2005.

(b) That the purchasing power of the average worker in the United States increased by a factor of 8

in one generation.

(c) That workers are clearly exploited and have obtained no benefit from economic growth.

Answer: A.

Real wage 1970 =

6085

Page 398: Microeconomics Study Guide

27

= 225;Real wage 2005 =

50107

112

= 447;

447

225

= 1.98

10. Growth in Total factor productivity is often calculated as the

(a) difference between potential and actual economic growth.

(b) growth in output per unit of labor.

(c) growth in labor productivity not explained by the growth in capital per hour of labor (after

adjusting by the 1/3 rule).

(d) growth of capital utilization.

Answer: c.

11. Recent increases in real estate prices have resulted in higher property tax revenues for the IRS. The US

government is evaluating two possible uses for this money. Option 1 consists of investing in research

and development and is expected to increase total factor productivity by 2%. Option 2 is as costly as

option 1. Option 2 consists of providing subsidies to investments in physical capital. These subsidies

are expected to result in a 3% increase in the capital stock of the US economy. The “one-third” rule

applies for the US economy. If the government wants to generate the maximum increase in GDP from

its policy it should

(a) Follow option 1

(b) Follow option 2

Answer: a. Option 1 increases GDP by 2%. Option one increases GDP by 3%/3=1%.

12. The following table provides you with data about the behavior of productive inputs in Japan during

two different periods.

Period Growth in GDP per worker Growth in capital per worker Growth in hours worked

Page 399: Microeconomics Study Guide

1960-1973 7.2% 6% 0%

1991-2000 0.5% 0.6% 0%

Clearly, and relative to the 1960-73 period, the economy of Japan slowed-down substantially during

the last decade. If the one third-rule holds for Japan then most of the slowdown in GDP growth

can be attributed to a slowdown in

(a) the growth rate of capital per worker

(b) the growth rate of hours worked

(c) the growth rate of total factor productivity

Answer: c (explained during the review session)

13. One implication of the Information Technology Revolution is that computers, semiconductors, etc.

constitute now a large proportion of the total capital stock of the United States. Information Tech-

nologies have a very high rate of depreciation. Thus, the Information Technology revolution resulted

in a sudden increase in the depreciation rate of physical capital in the United States. According to the

Solow model, this increase in capital depreciation (on its own)

(a) should translate into a lower long-run capital stock, and thus lower long-run GDP

3(b) should translate into a higher long-run capital stock, and thus higher long-run GDP

Answer: a. Higher depreciation results in a lower Solow curve, and thus a lower long

run capital and GDP.

14. The U.S. economy experienced one of its strongest economic expansions during the 1990-2000 period.

Some people attribute this economic expansion to the massive introduction of Information Technologies.

Based on your answer to the previous question, and using a Solow diagram, it is possible to say

(a) that the increase in TFP associated to the use of information technologies more than compensated

for the negative impact of higher depreciation, which can help explain why the economy ended

up with a higher GDP

(b) that the increase in GDP attributed to the higher depreciation of capital was further expanded

by the increase in TFP associated to the use of information technologies

Answer: a. Higher TFP results in a higher Solow curve, and thus higher long run

capital and GDP. Given the answer to 11, option b does not make sense. Option a

Page 400: Microeconomics Study Guide

has to be true. for GDP to grow the positive impact of TFP must have more than

compensated the negative impact of higher depreciation.

15. According to the Solow growth model (normalizing population to 1 and assuming no population

growth), capital evolves through time following the law of motion

K t+1 = sAK

1

3

t + (1 − δ)K t .

Suppose the saving rate of Weird country is equal to 15%, that the level of total factor productivity (A)

is equal to 40, and that capital depreciates at a 6% annual rate. Let K 2007 = 1000 (Note: 1000

1

3 = 10).

According to the Solow model, the stock of capital for year 2008 will be

(a) 900

(b) 1540

(c) 460

(d) 1000

(e) none of the above

K 2008 = 0.15 ∗ 40 ∗ 10 + (1 − 0.06) ∗ 1000 = 1000

16. Based on your previous answer, the long-run capital stock of the economy is

(a) impossible to determine with the information provided

(b) less than 1000

(c) larger than 1000 but lower than 1540

(d) 1000

(e) more than 1540

Answer: d (explained during the review session)

17. If a firm has capital stock valued at $4,500 at the beginning of the year, invested (in physical capital)

$1,500 during the year, and ended up with a capital stock valued at $5,000 at the end of the year then

Page 401: Microeconomics Study Guide

we know that

(a) depreciation equaled $500.

(b) gross investment equaled $1,500 and depreciation equaled $1,000.

(c) net investment equaled $1,500.

4(d) depreciation equaled $3,500.

Answer: b, Capital at the end of the year = Investment + capital at beginning of

year - depreciation. Here, 5000=1500+4500-depreciation

18. A firm is considering a one-year project that is expected to deliver a 7% nominal profit rate. The

current real interest rate is 5% and inflation is expected to equal 3%. Then,

(a) The firm should invest in the project

(b) The firm should not invest in the project

Answer: b. The real profit rate is profit rate - inflation = 4% which is less than what

the firm can obtain in the bank (in a deposit that pays the real interest rate, 5%).

Figure 1.

19. In Figure 1 above, if the government increases the taxes on corporate profits then the real interest rate

____ and the equilibrium quantity of investment and saving _____.

(a) rises; increases

(b) rises; decreases

(c) falls; increases

(d) falls; decreases

Answer: d, higher taxes shift the investment demand left. As a result, the interest

rate falls, and investment and saving fall.

20. In Figure 1 above, if future personal income is suddenly expected to be much higher than current

personal income, the real interest rate ____ and the equilibrium quantity of investment ____.

(a) rises; increases

(b) rises; decreases

(c) falls; increases

(d) falls; decreases

Page 402: Microeconomics Study Guide

Answer: b, higher future income lowers saving, ss shifts left so that a new equilibrium

with higher interest and lower investment and saving appears.

5Figure 2

21. In the labor market described by Figure 2 above, if the real wage temporarily dropped to $10 dollars

per hour firms would like to hire _______ hours and workers would like to work ______ hours.

The latter situation implies there is an ____________.

(a) Approximately 100 billion hours per year; approximately 200 billion hours per year; excess supply

of workers that would eventually translate into higher wages

(b) Approximately 200 billion hours per year; approximately 100 billion hours per year; excess demand

for workers that would eventually translate into higher wages

Answer: b

22. Suppose that the government imposes a minimum wage of $20 dollars per hour to the labor market

described in the previous question. As a result,

(a) All workers are better off

(b) All workers are worse off

(c) Employment equals 100 bn hours and the wage rate equals $20 dollars per hour. Relative to

equilibrium the imposition of a minimum wage generates unemployment equivalent to 100 bn

hours

(d) Employment equals 100 bn hours and the wage rate equals $20 dollars per hour. Relative to

equilibrium the imposition of a minimum wage generates excess demand for workers and pressures

for wages to go up even further.

Answer: c.

23. Consider an economy where the labor market goes back to equilibrium immediately after any change

in the labor market. The FED considers that the GDP of the economy is lower than it should be and

lowers the interest rate. As a result

(a) Aggregate demand shifts to the right, GDP and the price level go up (i.e., there is inflation).

(b) Aggregate demand shifts to the right, GDP is unchanged and the price level goes up (i.e., there

is inflation).

Page 403: Microeconomics Study Guide

Answer: b.

624. Consider an economy where the labor market goes back to equilibrium slowly (it takes more than a

quarter) after any change in the labor market. The FED considers that the GDP of the economy is

lower than it should be and lowers the interest rate. As a result

(a) Aggregate demand shifts to the right, GDP and the price level goes up (i.e., there is inflation).

(b) Aggregate demand shifts to the right, GDP is unchanged and the price level goes up (i.e., there

is inflation).

Answer: a

25. An increase in the reserve requirement ratio

(a) lowers the amount of money commercial banks create through loans and deposits.

(b) increases the amount of money commercial banks create through loans and deposits.

(c) has nothing to do with the amount of money commercial banks create.

Answer: a.

26. To increase commercial bank lending the Fed can

(a) lower the discount rate, or sell government securities.

(b) raise the discount rate, or sell government securities.

(c) lower the discount rate, or buy govmt. securities.

(d) raise the discount rate, or buy government securities.

Answer: c

Use the following data to answer question 27

27. In the short-run equilibrium of this economy the labor market is

(a) at a full-employment equilibrium and wages will not change.

(b) above full-employment equilibrium and wages will rise.

(c) above full-employment equilibrium and wages will fall.

(d) below full-employment equilibrium and wages will fall.

Answer: b.

28. According to the theory of real business cycles a typical economic recession occurs when

(a) The consumer confidence index falls so that people spend less driving equilibrium GDP down.

Page 404: Microeconomics Study Guide

(b) The FED increases the interest rate so that people spend less driving equilibrium GDP down.

(c) The growth rate of total factor productivity slowsdown.

7(d) The economy of any of the major trading partners of the US slowsdown.

Answer: c.

29. According to Edward C. Prescott in his paper "Theory ahead business cycle measurement," economic

policy in relation to business cycles should focus on

(a) Leaving the economy alone.

(b) Changing the interest rate to smooth the business cycle (i.e. lower it during recessions, and

increasing it during expansions so that inflation is contained).

(c) Understanding the factors that determine the rate at which technology advances.

Answer: c.

Short answer problems

Answer four out of the five following questions as concisely as possible.

30. (10 points total) The following questions are based on our discussion of the paper "Theory ahead

business cycle measurement," by Edward C. Prescott

(a) (5 points) What is the business cycle phenomena?

(b) (5 points) What are, according the E.C. Prescott, the main forces accounting for business cycle

fluctuations?

31. (10 points total) The following questions are based on our discussion of the papers "On the mechanics of

economic development," by Robert E. Lucas Jr., and "Needed: A theory of total factor productivity,"

by Edward C. Prescott

(a) (3 points) How does Lucas define the problem of economic development?

(b) (3 points) Explain the orders of magnitude of the difference between poor and rich countries in

terms of their levels of per-capita GDP? (i.e. the GDP per capita of the richest countries is how

many times that of the poorest ones?)

(c) (4 points) According to Prescott, what are the main factors accounting for the observed cross-

country disparity in per capita GDP?

32. (10 points) The following table provides data for the output and capital per worker for Chile during

Page 405: Microeconomics Study Guide

the years 1980 and 2000.

Chile

year Output per worker Capital per worker

1980 19,451.79 $ 36,659.13 $

2000 32,064.61 $ 71,894.77 $

ln(32064)-ln(19451)= 0.50

0.3*( (ln(71894)-ln(36659)) )= 0.20

Using either one of the following two formulas below, answer each one of the following two questions

(a) How much of the increase in GDP can be accounted for by changes in capital per worker? How

much by changes in TFP?

CHANGE IN GDP ACCOUNTED FOR BY CHANGES IN CAPITAL: 20% (SEE

LOG FORMULAS BELOW THE TABLE)

CHANGE IN GDP ACCOUNTED FOR BY CHANGES IN TFP : 30%

(SINCE CHANGES IN CAPITAL AND CHANGES IN TFP MUST ADD UP TO

CHANGES IN GDP)

8(b) what has been the most important factor behind the observed growth in output per worker for

Chile?

CHANGES IN TFP ARE THE MOST IMPORTANT FACTOR (30%>20%)

33. (10 points) Deviations from trend in total factor productivity are positively correlated with deviations

from trend in GDP. This observation motivated the development of the Real Business Cycle Theory.

However it is also true that the magnitude of the deviations from trend in GDP are larger than the

deviations from trend in TFP. Discuss in detail the mechanisms through which a 1% deviation from

trend in TFP can generate a larger than 1% deviation from trend in GDP.

SEE SLIDES ON REAL BUSINESS CYCLE THEORY.

Cross-Price Elasticity of Demand

Page 406: Microeconomics Study Guide

The percentage by which the quantity demanded of the first good changes in response to a 1 percent change in the price of the second.

Income Elasticity of Demand

The percentage by which quantity demanded changes in response to a 1 percent change in income.

Price Elasticity of Supply

The percentage change in quantity supplied that occurs in response to a 1 percent change in price.

Perfectly Inelastic Supply

Supply is perfectly inelastic with respect to price if elasticity is zero.

Perfectly Elastic Supply

Supply is perfectly elastic with respect to price elasticity of supply is infinite

Marginal Utility

The additional utility gained from consuming an additional unit of a good.

Law of Diminishing Marginal Utility

The tendency for the additional utility gained from consuming an additional unit of a good to diminish as consumption increases beyond some point.

Optimal Combination of Goods

The affordable combination that yields the highest total utility.

The Rational Spending Rule

Spending should be allocated across goods so that the marginal utility per dollar is the same for each good.MU/P (1) = MU/P (2)

Real Price

The dollar price of a good relative to the average dollar price of all other goods.

Nominal Price

The absolute price of a good in dollar terms

Consumer Surplus

The difference between a buyer's reservation price for a product and the price actually paid.

Profit

Page 407: Microeconomics Study Guide

The total revenue a firm receives from the sale of its product minus all costs - explicit and implicit- incurred in producing it.

Profit-Maximizing Firm

A firm whose primary goal is to maximize the difference b/t its total revenues and total costs.

Perfectly Competitive Market

A market in which no individual suppliers has significant influence on the market price of the product.

Price Taker

A firm that has no influence over the price at which it sells its product.

Imperfectly Competitive Firm

A firm that has at least some control over the market price of its product

Factor of Production

An input used in the production of a good or service.

Short Run

A period of time sufficiently short that at least some of the firm's factors of production are fixed.

Long Run

A period of time of sufficient length that all the firm's factors of production are variable.

Law of Diminishing Returns

A property of the relationship between the amount of a good or service produced and the amount of a variable factor required to produce it; the law says that when some factors of production of the good eventually requires ever-larger increases in the variable factor.

Fixed Factor of Production

An input whose quantity cannot be altered in the short run.

Variable Factor of Production

An input whose quantity can be altered in the short run.

Fixed Cost

the sum of all payments made to the firm's fixed factors of production.

Variable cost

Page 408: Microeconomics Study Guide

the sum of all payments made to the firm's variable factors of production.

Total cost

the sum of all payments made to the firm's fixed and variable factors of production.

Marginal cost

as output changes from one level to another, the change in total cost divided by the corresponding change in output.

Average variable cost

variable cost divided by total output

average total cost

total cost divided by total output

Profitable firm

a firm whose total revenue exceeds its total cost

Producer Surplus

the amount by which price exceeds the seller's reservation price

Explicit costs

the actual payments a firm makes to its factors of production and other suppliers

Accounting profits

the difference b/t a firm's total revenue and its explicit costs

Implicit costs

the opportunity costs of the resources supplied by the firm's owners

Economic profit

the difference b/t a firm's total revenue and the sum of its explicit and implicit costs

Normal profit

the opportunity cost of the resources supplied by a firm's owners, equal to accounting profit minus economic profit

economic loss

Page 409: Microeconomics Study Guide

an economic profit that is less than zero

Rationing function of price

changes in prices distribute goods to those consumers who value them most highly

Allocative function of price

changes in prices direct resources away from overcrowded markets and toward markets that are undeserved

Invisible hand theory

Adam Smith's theory that the actions of independent, self-interested buyers and sellers will often result in the most efficient allocation of resources

Barrier to Entry

any force that prevents firms from entering a new market

Economic rent

that part of the payment for a factor of production that exceeds the owner's reservation price, the price below which the owner would not supply the factor

Price setter

a firm with at least some latitude to set its own price

Pure monopoly

the only supplier of a unique product with no close substitutes

Monopolistic competition

an industry structure in which a large number of firms produce slightly differentiated products that are reasonably close substitutes for one another

Oligopoly

an industry structure in which a small number of large firms produce products that are either close or perfect substitutes

market power

a firm's ability to raise the price of a good without losing all its sales

Constant returns to scale

a production process is said to have constant returns to scale if, when all inputs are changed by a given proportion, output changes by the same proportion

Page 410: Microeconomics Study Guide

Natural monopoly

a monopoly that results from economies of scale (increasing returns to scale)

Increasing returns to scale

a production process is said to have increasing returns to scale if, when all inputs are changed by a given proportion, output changes by more than that proportion

Marginal revenue

the change in a firm's total revenue that results from a one-unit change in output

Price discrimination

the practice of charging different buyers different prices for essentially the same good or service

Perfectly discriminating monopolist

a firm that charges each buyer exactly his or her reservation price

hurdle method of price discrimination

the practice by which a seller offers a discount to all buyers who overcome some obstacle

one who manages a household

The work economy comes from the Greek word for

scarcity

Economics deals primarily with the concept of

Page 411: Microeconomics Study Guide

everyone in that society cannot have all they want of the good

A good is considered scarce in a society when...?

how society manages its scarce resources

Economics is the study of...?

equity

Economists use the word _____ to describe a situation in which the benefits of society's resources are distributed fairly among society's members

opportunity cost

What you give up to obtain an item is called your...?

comparing costs and benefits

People make decisions at the margin by...?

marginal benefit is greater than the marginal cost

A rational decision maker takes an action only if the...?

Shawn has the comparative advantage.

If Shawn can produce donuts at a lower opportunity cost than Sue, then...?

differences in opportunity costs.

Comparative advantage is based on...?

should export that product.

The country that has a comparative advantage in a product...?

Imports

_____ are produced abroad and sold domestically.

Exports

_____ are produced domestically and sold abroad.

the price of wheat

One result of a drought in the midwest could be an increase in...?

demand, supply

Page 412: Microeconomics Study Guide

Buyers determine _____, sellers determine _____

price, quantity

For each good produced in a market economy, demand and supply determine both _____ and _____

competitive

A _____ market is one in which each seller has limited control over the price of his product.

Technology

_____ is not a determinant of demand.

an increase in demand for the good

If a good is "normal", then an increase in income will result in...?

reduces the quantity demanded of the other good

Two goods are substitutes if a decrease in the price of one good...?

increases the quantity demanded of the other good.

Two goods are complements if a decrease in the price of one good...?

elasticity

In general, _____ is a measure of how much buyers and sellers respond to changes in market conditions

buyers are to a change in price

The price elasticity of demand measures how responsive...?

inelastic

If a good is a necessity, demand for the good would tend to be...?

elastic

When quantity demanded responds substantially to changes in price, demand is said to be...?

the absolute value of the actual number.

Most economists report the elasticity of demand as...?

greater than 1

Demand is elastic if elasticity is...?

Page 413: Microeconomics Study Guide

less than 1

Demand is inelastic if elasticity is...?

equal to 1

Demand is unit elastic if elasticity is...?

horizontal

A perfectly elastic demand curve will be...?

vertical

A perfectly inelastic demand curve will be...?

Insulin

_____ would probably have the most inelastic demand.

ceiling.

A legal maximum price at which a good can be sold is a price _____

floor.

A legal minimum price at which a good can be sold is a price _____.

maximum amount that a buyer will pay for a good.

Willingness to pay measures the...?

have an adverse impact on the well-being of a bystander.

Negative externalities occur when one person's actions...?

if price increases, quantity supplied increases.

According to the law of supply...?

total revenue

The amount of money that a firm receives from the sale of its output is called _____

total cost or opportunity cost

The amount of money that a firm pays to buy inputs is called...?

Explicit

Page 414: Microeconomics Study Guide

_____ costs require monetary outlay.

entrepreneurial talents.

An example of an implicit cost of production would be...?

competitive market

When a firm has little ability to influence market prices it is said to be in what kind of market?

fixed costs

Firms that shut down in the short run still have to pay their...?

firms have little ability to influence market prices.

A competitive market is one in which...?

price of his product

In a competitive market each seller has limited control over the...?

hamburgers and fries

An example of complementary goods would be...?

ownership of a key resource.

The De Beers diamond monopoly is a classic example of a monopoly that arises from...?

firms and factory sizes

The three average total cost curves on the diagram correspond to three different...?

Q1-Q2

The firm experiences economies of scale if it changes its level of output

Q4-Q5

The firm experiences diseconomies of scale if it changes its level of output

oligopolies

Markets with only a few sellers, each offering a product similar or identical to the others, are typically referred to as...?

there are few sellers offering a similar or identical product

An oligopoly is a market in which...?

Page 415: Microeconomics Study Guide

cooperation would make each other better off

A prisoner's dilemma game demonstrates how cooperative action is often not rational even though...?

monopoly

In the short run, a firm in a monopolistically competitive market operates much like a(n)...?

inputs a firm uses

The factors of production are best defined as the...?

marginal rate of substitution

The rate at which a consumer is willing to exchange one good for another, and maintain a constant level of satisfaction, is called the...?

wages

Most of the total income earned in the US economy is ultimately paid to households in the form of...?

decreases

What happens to the labor supply in the pear-picking market when the wage paid to apple pickers increases?

labor supply in Mexico decreases, labor supply in US increases

What happens to the labor supply curves in both countries when Mexican workers leave Mexico and move to the US?

the factors of production

Economists refer to the inputs that firms use to produce goods and services as...?

Increase

If the price in airline tickets falls, what will happen to the demand curve for flight attendants?

Human capital theory

Which theory is supportive of the idea that increasing educational levels for all workers would raise all workers' productivity and thereby their wages?

education

The most important source of human capital is...?

the supply and demand for that person's labor

A person's earnings depend on...?

Page 416: Microeconomics Study Guide

(answer will be all of the above)

Poverty is found to be correlated with...?

quantity demanded increases

When the price of a pair of jeans rises, the...?

absolute advantage

the ability to produce a good using fewer inputs than another producer

average fixed cost (AFC)

Total fixed cost divided by the number of units of output; a per-unit measure of fixed costs. AFC = FC/Q

average total cost (ATC)

Total cost divided by the number of units of output ATC = TC/Q or ATC = AFC + AVC

average variable cost (AVC)

variable cost divided by the number of units of output AVC = VC/Q

budget constraint

the limits imposed on household choices by income, wealth, and product prices.

capital

goods used to produce other goods

cartel

a group of firms that gets together and makes joint price and output decisions to maximize joint profits

ceteris paribus

a devise used to analyze the relationship between two variable while the values of other variables are held unchanged.

clayton act

act outlawed specific monopolistic behaviors such as tying contracts

Page 417: Microeconomics Study Guide

command economy

An economy in which a central government either directly or indirectly sets output targets, incomes, and prices

comparative advantage

the ability to produce a good at a lower opportunity cost than another producer

complements

two goods for which an increase in the price of one leads to a decrease in the demand for the other and vice versa

consumer goods

goods produced for present consumption

consumer sovereignty

The idea that consumers ultimately dictate what will be produced (or not produced) by choosing what to purchase (and what not to purchase).

consumer surplus

The difference between the maximum amount a person is willing to pay for a good and its current market price.

cross price elasticity of demand

measures the responsiveness of the quantity demand of a good to a change in the price of another good.

diseconomies of scale

The property whereby long-run average total cost rises as the quantity of output increases (right-most upward sloping part of the long-run ATC)

demand curve

a graph that shows the amount of a product that would be bought at all possible prices in the market

depreciation

the decline in an asset's economic value over time

diminishing marginal utility

the point reached when an additional unit of a product consumed is less satisfying than the one before

economic theory

A statement or set of related statements about cause and effect, action and reaction

Page 418: Microeconomics Study Guide

economics

the study of how individuals and nations make choices about ways to use scarce resources to fulfill their needs and wants

efficiency

producing what people want at the least possible cost

elastic demand

the percentage change in quantity demanded is greater than the percentage change in price in absolute value

elasticity

a measure of responsiveness that tells us how a dependent variable such as quantity responds to a change in an independent variable such as price

equilibrium

the point at which quantity demanded and quantity supplied are equal

entrepreneur

a person who organizes, manages, and takes on the risks of a business

equity

(n.) the state of being just, fair, or impartial; fair and equal treatment; something that is fair; the money value of a property above and beyond any mortgage or other claim

shortage

quantity demanded exceeds quantity supplied

surplus

quantity supplied exceeds quantity demanded

externality

unintended side effect that either benefits or harms a third party not involved in the activity that caused it

fallacy of composition

The incorrect belief that what is true for the individual, or part, must necessarily be true for the group, or whole.

firm

Page 419: Microeconomics Study Guide

an organization that comes into being when a person or a group of people decides to produce a good or service to meet a perceived demand. transforms inputs into outputs. primary producing unit in a market economy

fixed cost

a cost that does not change, no matter how much of a good is produced. there are none in the long-run

free enterprise

the freedom of individuals to start and operate private businesses in search of profits

free rider problem

getting the benefits of something without having to contribute.. this problem also arises when individuals receive the benefit whether they contributed or not (Cleaner air).

Heckscher-Ohlin theorem

a country has a comparative advantage in the production of a product if that country is relatively well endowed with inputs used intensively in the production of that product.

homogenous products

undifferentiated products; products that are identical to, or indistinguishable from, one another

income

the money a person gets from salary or wages, profits, interest, investments, and other sources

imperfectly competitive industry

An industry in which single firms have some control over the price of their output

income elasticity of demand

measures the responsiveness of the quantity demanded of a good to the change in the income of the people demanding the good. Formula: (%change in quantity demanded) / (%change in income)

economies of scale

as a company produces larger numbers of a particular product, the cost of each of these products goes down aka increasing returns to scale

indifference curve

a curve showing the combinations of two goods that leave the consumer with the same level of utility.

inelastic demand

demand that responds a little bit to change in price. always has a numerical value between 0 and -1

Page 420: Microeconomics Study Guide

inferior goods

good that demand falls when income rises

inputs

anything provided by nature or previous generations that can be used directly or indirectly to satisfy human wants

microeconomics

the branch of economics that studies the economy of consumers or households or individual firms

interest

the price paid for the use of borrowed money

monopolistic competition

market structure of an industry in which there are many firms and freedom of entry and exit but in which each firm has a product somewhat differentiated from the others, giving it some control over its price

monopoly

one firm that produces a product which there are no close substitutes for and in which significant barriers exist to prevent new firms from entering the industry

laissez faire economy

government has zero to no control over the economy

law of demand

consumers buy more of a good when its price decreases and less when its price increases

law of diminishing marginal utility

As the quantity of a good consumed increases the extra satisfaction gained decreases

law of diminishing returns

a law affirming that to continue after a certain level of performance has been reached will result in a decline in effectiveness

law of supply

the claim that, other things equal, the quantity supplied of a good rises when the price of the good rises

lorenz curve

Page 421: Microeconomics Study Guide

A widely used graph of the distribution of income, with cumulative percentage of families plotted along the horizontal axis and cumulative percentage of income plotted along the vertical axis.

macroeconomics

the branch of economics that studies the overall working of a national economy

marginal cost (mc)

the increase in total cost that results form producing one more unit of output

marginal revenue (mr)

the additional revenue that a firm takes in when it increases output by one additional unit

marginal social cost (msc)

the total cost to society of producing an addiotnal unit of a good or service

marginal utility (mu)

additional satisfaction gained by the consumption or use of one more unit of a good or service

marginalism

the process of analyzing the additional or incremental costs or benefits arising from a choice or decision

market

A group of buyers and sellers of a good or service and the institution or arrangement by which they come together to trade

market failure

Situation in which an unregulated competitive market is inefficient because prices fail to provide proper signals to consumers and producers

market demand

sum of all the quantities of a good or service demanded per period by all the households buying in the market for that good or service

market power

a firm's ability to raise the price of a good without losing all its sales

moral hazard

The risk that the behavior of one party may change to the detriment of another after a contract has been agreed upon. Example: Those with insurance may be less likely to guard against loss than those without insurance.

Page 422: Microeconomics Study Guide

movement along the demand curve

the change in quantity demanded brought about by a change in price

movement along a supply curve

the change quantity supplied brought about by a change in price

natural monopoly

a monopoly that arises because a single firm can supply a good or service to an entire market at a smaller cost than could two or more firms

negative relationship

a relationship between two variables which a decrease in one is associated with an increase in another or vice versa

normal goods

goods which demand goes up when income is higher and goes down when income is lower

normative economics

the part of economics involving value judgments about what the economy should be like; focused on which economic goals and policies should be implemented; policy economics

north american free trade agreement

an agreement signed by the united states, canada, and mexico in which it was agreed that north america be a "free-trade" zone

ockham's razor

irrelevant detail should be cut away

oligopoly

a market in which control over the supply of a commodity is in the hands of a small number of producers and each one can influence prices and affect competitors

opportunity cost

The next best alternative given up when making a choice

outputs

goods and services of value to households

payoff

Page 423: Microeconomics Study Guide

an advantage or profit that you get as a result of doing something

perfect competition

a market structure that is characterized by a large number of small firms, a homogeneous product,freedom of entry and exit

perfect substitutes

identical products

perfectly elastic demand

demand in which quantity drops to zero at the slightest increase in price

perfectly inelastic demand

demand in which quantity demanded does not respond at all to a change in price

positive economics

An approach to economics that seeks to understand behavior and the operation of systems without making judgments. It describes what exists and how it works.

positive relationship

a relationship between two variables which a decrease in one is associated with a decrease in another. and a increase in one is associated with an increase in another

pos hoc, ergo propter hoc

the common misconception that if event a happens before event b that event b happened because of event a

producer surplus

the amount a seller is paid for a good minus the seller's cost of providing it.

product differentiation

a positioning strategy that many firms use to distinguish their products from those of competitors

price ceiling

a maximum price that sellers may charge for a good

price discrimination

charging different prices to different buyers

price elasticity of demand

Page 424: Microeconomics Study Guide

a measure of how much the quantity demanded of a good responds to a change in the price of that good, computed as the percentage change in quantity demanded divided by the percentage change in price

price floor

a minimum price below which exchange is not permitted

profit

the difference between revenues and cost

variable cost

a cost that depends on the level of production chosen

quantity demanded

how much people are willing to buy at a given price, only changes when price changes (move along curve)

quantity supplied

the amount of a particular product that a firm would be willing and able to offer for sale at a particular price during a given time period.

real income

Set of opportunities to purchase real goods and services available to a household as determined by prices and money income.

scarce

limited

shift of a demand curve

The change that takes place in a demand curve corresponding to a new relationship between quantities demanded of a good and price of that good. The shift is brought about by a change in the original conditions.

shift of a supply curve

the change that takes place in a supply curve corresponding to a new relationship between quantity supplied of a good and the price of that good. The shift is brought about by a change in the original conditions.

short run

a period of time sufficiently short that at least one of the firm's factors of production cannot be varied

slope

Page 425: Microeconomics Study Guide

a measurement that indicates weather the relationship between variables is positive or negative and how much of a response there is

social capital

capital that provides services to the public

substitues

goods that can serve as replacements for one another; wen the price of one increases demand for the other increases

supply curve

a graph illustrating how much of a product a firm will sell at different prices

theory of comparative advantage

Ricardo's theory that specialization and free trade will benefit all trading parties, even those that may be absolutely more efficient producers.

total cost (tc)

total fixed costs plus total variable costs TC = TFC + TVC

total fixed cost (tfc)

the total of all costs that do not change with output even if output is zero

total revenue (tr)

the amount received from the sale of the product; the price per unit X the quantity of output the firm decides to produce

total utility

total amount of satisfaction obtained form consumption of a good or service

total variable cost (tvc)

the total of all costs that vary with output in the short run

trade deficit

the situation when a country imports more than it exports

trade surplus

the situation when a country exports more than it imports

Page 426: Microeconomics Study Guide

unitary elasticity

A situation in which total revenue remains the same when prices change.

utility

the satisfaction a product yields

variable

a measure that can change from time to time or form observation to observation

isoquant

a graph that shows all the combinations of capital and labor that can be used to produce a given amount of output

Absolute advantage is found by comparing the productivity of one nation to that of another.

The Circular flow diagram is a visual model of how the economy is organized.

A leftward shift in supply is a decrease in supply.

Microeconomics is the study of how individual households & firms make decisions.

If there is a shortage of laborers, we would expect the wages of farm laborers to increase.

Another term for factors of production is goods

Imports are goods produced abroad & sold domestically.

Economics is the study of how society manages its scarce resources.

A movement along the supply curve might be caused by the price of the good or service.

The forces that make market economies work are demand and supply.

The primary determinant of a counties standard of living is its ability to produce goods and services.

What you give up to obtain an item is called opportunity cost

An economy's scarce resources are allocated by price for resources

Page 427: Microeconomics Study Guide

When goods that are produced in the United States are sold to China, the goods are

exported by the united states and imported by china.

A country has a comparative advantage in a product if the world price is

higher than its domestic price.

An example of a complementary good is lawnmowers and automobiles

Economies deals primarily with the concept of scarcity.

What are the 2 basic reasons why economist often appear to give conflicting advice to policymakers are differences in?

scientific judgement and values.

The country with the comparative advantage in a product None of them are correct.

Trade based on absolute advantage is NOT correct

When a country allows trade and becomes an exporter of a good, which of the following would NOT be true?

The losses of domestic consumer exceeds the gains of domestic producers.

If the demand for a product increases, we would expect equilibrium price

and equilibrium quantity to both increase.

Letters that are not on the Demand curve CANNOT produce.

Economist use model in order to learn how the economy works.

When economist are trying to help improve the world they are policy advisors.

Technology is NOT a determinant of demand

Suppose there is an increase in input prices. We would expect supply

to decrease.

Common Stock Equity

Preferred Stock No Equity

DJIA Top 30 industrial companies, price per share.

Law of Demand inverse relationship between price and quantity demanded.

Normative Economics What should be.

Restrictions on Trade Jobs, National Security, Infant Industries, Unfair competition, Protection as a barganship

Why do some countries like restrictions on trade? Some countries are socialist economies and dont encourage competition.

Change in Quantity Demanded Single Curve

Page 428: Microeconomics Study Guide

Change in Demand Two curves

If DEMAND is MORE than SUPPLY Shortage (-)

If supply is MORE than DEMAND Surplus (+)

Inferior Good Negative income-elasticity coefficient designates inferior good.

Production Possibilities Curve Showing different combinations of goods and services that can be produced in a fully employed economy assuming resources and technology are fixed.

Microeconomics Study of how individual households and firms make decisions.

Resources

1. Land (Rent)2. Labor (Wages and Salaries)3. Capital (not money - buildings and equipment)4. Entrepreneur (Profit)

4 Sectors

1. Household2. Business3. Government4. The rest of the WORLD (imports and exports) (OPEN)

Sunk Cost Once and for all cost that once incurred cannot be reserved.

Implicit Cost Cost of inputs used in production but is not paid for.

Explicit Cost Cost of inputs used and paid for.

Fixed Cost Produced of not still paid for.

Marginal Cost Cost of producing one more unit.

ATC short run U Law of Diminishing Returns

Economic Profit TR - Economic Cost

Accounting Cost TR - Explicit Cost

Causes of Economies to Scale 1. Labor Specialization2. Lower Raw Material3. Better use of by products

Total Revenue Price x Quanity

Page 429: Microeconomics Study Guide

Complementary Negative

Substitute Positive

Normal Positive

Inferior Negative

Elastic Price and TR = Opposite & More than 1

Inelastic Price and TR = Direct & Less than 1

Consumer Surplus (top) Demand - Market Price

Producers Surplus (bottom) What producer gets - minimum he'll except.

Short Run a. Fixedb. Variable

Long Run a. Variable = depends on production

Perfect Competition

1. Many buyer, many sellers2. Homo3. Easy4. No influence5. Regular Curve6. Less than 50

Monopoly / Monopsony

1. One seller, many buyers2. Homo3. Hard4. Seller Control5. Steep Curve with 2nd line6. 50-70

Oligopoly

1. Few sellers, many buyer2. Hetero3. Hard4. Has influence5. Kinked Curve6. 70-100

Price discrimination 1. Price Elasticity2. Separate Market (time, age, 3. No Resale

Monopolistic Competition 1. Many buyers & Sellers2. Hetero3. Easy than Oligopoly

Page 430: Microeconomics Study Guide

4. Firm try to influence

Coordination of Cullsion

1. Few buyers many sellers2. Hetero3. Hard4. Buys influence

Market Concentration Market dominated by few large

Concentration Ratio Largest 4

Causes of Market Concentration 1. Economies of Scale2. Barrier for Entry

If TR is less than TC its a loss

If price is above AT its Profit

If price is between AT and AV its Loss but will still produce

If price is below AT its Loss and will shut down

Assuming a firm experiences decreasing marginal products of labor with the addition of each worker regardless of the current output level. AT will be

Constant

Negative externalities occur when one persons actions adversely affects the well being of a bystander who is not party to the action.

The minimum wage was instituted in order to ensure workers a minimally adequate standard of living.

Producer Surplus is the amount a seller is paid less the cost of production.

Accounting Profit is total revenue minus the explicit cost of producing goods and services.

Profit is defined as total revenue minus total cost

Demand is said to be inelastic if the quantity demanded changes only slightly when the price of good changes.

For a firm, the production function represents the relationship between

quantity of inputs and quantity of outputs

If the quantity supplied is the same regardless of price, then the supply curve would be

perfectly inelastic

Consumer surplus measures the difference between the amount a consumer has to pay and the amount the consumer was willing to pay

The price elasticity of demand measures a buyers responsiveness to change in the price of a

Page 431: Microeconomics Study Guide

good.

A local pizza restaurant makes great bread sticks that the consumers don not respond much to a change in the price. If the owner is only interest in increasing revenue he should

raise the price of the breadsticks

Economist compute the price elasticity of demand as the percentage change in the quantity demanded divided by the percentage change in price.

If two goods are substitutes, their cross elasticity will be positive

Implicit cost do not require an outlay of money by the firm.

Economies of scale occur when long-run average total costs fall as outputs increase.

Explicit Cost ALL the ABOVE

Demand is inelastic if elasticity is less than 1

Demand for a good would tend to be more inelastic the fewer the available substitutes.

In the long run inputs that were fixed in short run become variable.

An example of an implicit cost of production would be the income an entrepreneur could have earned working for someone else

In any market total revenue is Price multiplied by quantity.

Uncongested roads are a good example public good

Since almost all forms of transportation produce some type of pollution

society has to weigh the cost and benefits and decide how much pollution to allow.

Fixed cost can be defined as costs that are incurred even if nothing is produced

A payroll tax is a tax on the wages and firms pay their workers

In an increase in income results in the quantity demanded of a good, then the good is

an inferior good.

Page 432: Microeconomics Study Guide

Chapter 23: Pure CompetitionType: G E: 419-420 MI: 175-176224.Refer to the above diagram. At the profit-maximizing output total revenue will be 0

GLD

.Answer: FalseType: G E: 420 MI: 176225.Refer to the above diagram. At output

C

production will result in an economic profit.Answer: TrueType: G E: 420 MI: 176226.Refer to the above diagram. At output

C

total variable cost is

FGKJ

.Answer: FalseType: G E: 420 MI: 176227.Refer to the above diagram. At output

C

average fixed cost is

GF

.Answer: FalseType: G E: 424 MI: 180228.Refer to the above diagram. At any price below

R

the firm will shut down in the short run.Answer: TrueType: G E: 424 MI: 180229.Refer to the above diagram. If demand fell to the level of

FNJ

, there would be no output at which the firmcould realize an economic profit.Answer: FalseType: G E: 424 MI: 180230.Refer to the above diagram. If the firm produced

D

units of output at price

G

, it would earn a normal profit.Answer: TrueType: G E: 417 MI: 173231.Refer to the above diagram. Total costs are minimized at output level

B

.Answer: FalseType: A E: 425 MI: 181232.Although individual purely competitive firms can influence the price of their product, these firms as agroup cannot influence market price.Answer: False

Page 433: Microeconomics Study Guide

Type: T Topic: 3 E: 417 MI: 173 135. The above data are for: A) the long run. C) both the short run and the long run. the short run. D) the intermediate market period only. Answer: B

Type: T Topic: 3 E: 419-420 MI: 175-176 136. Refer to the above data. At 5 units of output average fixed cost, average variable cost, and average total cost are:

A) $10, $60, and $70 respectively. C) $10, $70, and $80 respectively. $50, $40, and $90 respectively. D) $5, $25, and $30 respectively. Answer: A

Type: T Topic: 3 E: 419 MI: 175 137. Refer to the above data. The marginal cost of the fifth unit of output is: A) $80. $90. C) $50. D) $20.Answer: A

Type: T Topic: 3 E: 419-420 MI: 175-176 138. Refer to the above data. If product price is $75, the firm will produce: A) 3 units of output. 4 units of output. C) 5 units of output. D) 6 units of output. Answer: B

Type: T Topic: 3 E: 419-420 MI: 175-176 139. Refer to the above data. Given the $75 product price, at its optimal output the firm will: A) realize a $25 economic profit. C) incur a $25 loss. realize a $30 economic profit. D) realize a $30 loss. Answer: B

Type: A Topic: 3 E: 424 MI: 180 140. A purely competitive firm's short-run supply curve is:A) the upward sloping portion of its marginal cost curve.the upward sloping portion of its average variable cost curve.C) its marginal cost curve above average variable cost.D) its average total cost curve.Answer: C

Type: A Topic: 3 E: 424 MI: 180 141. In the short run, a purely competitive firm will earn a normal profit when:A) P = AVC. P > MC. C) that firm's MR = market equilibrium price. D) P = ATC.Answer: D

Use the following to answer questions 142-147:

Page 434: Microeconomics Study Guide

The following table applies to a purely competitive industry composed of 100 identical firms.

Type: T Topic: 3 E: 425 MI: 181 142. Refer to the above table. The equilibrium price in this purely competitive market is:A) $5. $4. C) $3. D) $2.Answer: C

Type: T Topic: 3 E: 425 MI: 181 143. Refer to the above table. At the equilibrium price, each of the 100 firms in this industry will produce:A) 600,000 units of output. C) 6,000 units of output60,000 units of output. D) 600 units of output.Answer: C

Type: T Topic: 3 E: 425-426 MI: 181-182 144. Refer to the above table. For each of the 100 firms in this industry, marginal revenue and total revenue will be: A) $4 and $400, respectively. C) $4 and $20,000, respectively.$3 and $30,000, respectively. D) $3 and $18,000, respectively.Answer: D

Type: T Topic: 3 E: 425-426 MI: 181-182 145. Refer to the above table. If each of the 100 firms in the industry is maximizing its profit, each must have a marginal cost of:A) $5. $4. C) $3. D) $2.Answer: C

Type: T Topic: 3 E: 425-426 MI: 181-182 146. Refer to the above table. If each of the 100 firms in the industry is maximizing its profit and earning only a normal profit, each must have a total cost of:A) $18,000. $20,000. C) $22,000. D) $24,000.Answer: A

Type: T Topic: 3 E: 425-426 MI: 181-182 147. Refer to the above table. If each of the 100 firms in the industry is maximizing its profit and earning only a normal profit, each must have an average total cost of:A) $2. $3. C) $4. D) $5.Answer: B

Profit maximizing in long run

Type: A Topic: 4 E: 427 MI: 183 148. Suppose a firm in a purely competitive market discovers that the price of its product is above its minimum AVC point but everywhere below ATC. Given this, the firm:A) minimizes losses by producing at the minimum point of its AVC curve.

Page 435: Microeconomics Study Guide

maximizes profits by producing where MR = ATC.C) should close down immediately.D) should continue producing in the short run, but leave the industry in the long run.Answer: D

Type: A Topic: 4 E: 427 MI: 183 149. Which of the following is true concerning purely competitive industries?A) There will be economic losses in the long run because of cut-throat competition.Economic profits will persist in the long run if consumer demand is strong and stable.C) In the short run, firms may incur economic losses or earn economic profits, but in the long run they earn normal profits.D) There are economic profits in the long run, but not in the short run.Answer: C

Type: A Topic: 4 E: 428 MI: 184 150. If a purely competitive firm is producing at the MR = MC output level and earning an economic profit, then:A) the selling price for this firm is above the market equilibrium price.new firms will enter this market.C) some existing firms in this market will leave.D) there must be price fixing by the industry's firms.Answer: B

Type: A Topic: 4 E: 427 MI: 183 151. Long-run competitive equilibrium: A) is realized only in constant-cost industries. C) is not economically efficient. will never change once it is realized. D) results in zero economic profits. Answer: D

Type: A Topic: 4 E: 428 MI: 184 152. We would expect an industry to expand if firms in that industry are: A) earning normal profits. C) incurring economic losses. earning economic profits. D) earning accounting profits. Answer: B

Type: A Topic: 4 E: 422-423 MI: 178-179 153. Which of the following statements is correct? A) Economic profits induce firms to enter an industry; losses encourage firms to leave. Economic profits induce firms to leave an industry; profits encourage firms to leave. C) Economic profits and losses have no significant impact on the growth or decline of an industry. D) Normal profits will cause an industry to expand. Answer: A

Type: A Topic: 4 E: 430 MI: 186 154. Suppose a purely competitive increasing-cost industry is in long-run equilibrium. Now assume that a decrease in consumer demand occurs. After all resulting adjustments have been completed, the new equilibrium price:

Page 436: Microeconomics Study Guide

A) and industry output will be less than the initial price and output. will be greater than the initial price, but the new industry output will be less than the original output. C) will be less than the initial price, but the new industry output will be greater than the original output. D) and industry output will be greater than the initial price and output. Answer: A

Type: A Topic: 4 E: 430 MI: 186 155. Which of the following statements is correct? A) The long-run supply curve for a purely competitive increasing-cost industry will be upsloping. The long-run supply curve for a purely competitive increasing-cost industry will be perfectly elastic. C) The long-run supply curve for a purely competitive industry will be less elastic than the industry's short-run supply curve. D) The long-run supply curve for a purely competitive decreasing-cost industry will be upsloping. Answer: A

Type: A Topic: 4 E: 429 MI: 185 156. A constant-cost industry is one in which: A) a higher price per unit will not result in an increased output. if 100 units can be produced for $100, then 150 can be produced for $150, 200 for $200, and so forth. C) the demand curve and therefore the unit price and quantity sold seldom change. D) the total cost of producing 200 or 300 units is no greater than the cost of producing 100 units. Answer: B

Type: A Topic: 4 E: 432 MI: 188 157. Which of the following will not hold true for a competitive firm in long-run equilibrium? A) P equals AFC P equals minimum ATC C) MC equals minimum ATC D) P equals MC Answer: A

Type: A Topic: 4 E: 430 MI: 186 158. Assume a purely competitive increasing-cost industry is initially in long-run equilibrium and that an increase in consumer demand occurs. After all economic adjustments have been completed product price will be: A) lower, but total output will be larger than originally. higher and total output will be larger than originally. C) lower and total output will be smaller than originally. D) higher, but total output will be smaller than originally. Answer: B

Type: A Topic: 4 E: 430 MI: 186 159. Assume a purely competitive, increasing-cost industry is in long-run equilibrium. If a decline in demand occurs, firms will: A) leave the industry, price will decrease, and quantity produced will increase. enter the industry and price and quantity will both increase. C) leave the industry and price and output will both increase. D) leave the industry and price and output will both decline. Answer: D

Page 437: Microeconomics Study Guide

Type: A Topic: 4 E: 427 MI: 183 160. When a purely competitive firm is in long-run equilibrium: A) marginal revenue exceeds marginal cost. price equals marginal cost. C) total revenue exceeds total cost. D) minimum average total cost is less than the product price. Answer: B

Type: A Topic: 4 E: 427 MI: 183 161. A purely competitive firm: A) must earn a normal profit in the short run. cannot earn economic profit in the long run. C) may realize either economic profit or losses in the long run. D) cannot earn economic profit in the short run. Answer: B

Type: A Topic: 4 E: 429 MI: 185 162. A constant-cost industry is one in which:A) resource prices fall as output is increased. resource prices rise as output is increased. C) resource prices remain unchanged as output is increased. D) small and large levels of output entail the same total costs. Answer: C

Type: A Topic: 4 E: 430 MI: 186 163. An increasing-cost industry is associated with: A) a perfectly elastic long-run supply curve. C) a perfectly inelastic long-run supply curve. an upsloping long-run supply curve. D) an upsloping long-run demand curve. Answer: B

Use the following to answer questions 164-166:

Type: G Topic: 4 E: 426 MI: 182 164. Refer to the above diagrams, which pertain to a purely competitive firm producing output q and the industry in which it operates. Which of the following is correct? A) The diagrams portray neither long-run nor short-run equilibrium. The diagrams portray both long-run and short-run equilibrium. C) The diagrams portray short-run equilibrium, but not long-run equilibrium. D) The diagrams portray long-run equilibrium, but not short-run equilibrium. Answer: C

Type: G Topic: 4 E: 427-428 MI: 183-184

Page 438: Microeconomics Study Guide

165. Refer to the above diagrams, which pertain to a purely competitive firm producing output q and the industry in which it operates. In the long run we should expect: A) firms to enter the industry, market supply to rise, and product price to fall. firms to leave the industry, market supply to rise, and product price to fall. C) firms to leave the industry, market supply to fall, and product price to rise. D) no change in the number of firms in this industry. Answer: C

Type: G Topic: 4 E: 432 MI: 188 166. Refer to the above diagrams, which pertain to a purely competitive firm producing output q and the industry in which it operates. The predicted long-run adjustments in this industry might be offset by: A) a decline in product demand. an increase in resource prices. C) a technological improvement in production methods. D) entry of new firms into the industry. Answer: C

Type: A Topic: 4 E: 427 MI: 183 167. Assume a purely competitive firm is maximizing profit at some output at which long-run average total cost is at a minimum. Then:A) the firm is earning an economic profit. there is no tendency for the firm's industry to expand or contract. C) allocative but not productive efficiency is being achieved. D) other firms will enter this industry.Answer: B

Type: A Topic: 4 E: 430 MI: 186 168. An increasing-cost industry is the result of: A) higher resource prices which occur as the industry expands. a change in the industry's minimum efficient scale. C) X-inefficiency. D) the law of diminishing returns. Answer: A

Type: A Topic: 4 E: 427 MI: 183 169. A purely competitive firm is precluded from making economic profit in the long run because: A) it is a "price taker." C) of unimpeded entry to the industry. its demand curve is perfectly elastic. D) it produces a differentiated product. Answer: C

Type: A Topic: 4 E: 428-429 MI: 184-185 170. If a purely competitive constant-cost industry is realizing economic profits, we can expect industry supply to: A) increase, output to increase, price to decrease, and profits to decrease. increase, output to increase, price to increase, and profits to decrease. C) decrease, output to decrease, price to increase, and profits to increase.

Page 439: Microeconomics Study Guide

D) increase, output to decrease, price to decrease, and profits to decrease. Answer: A

Type: A Topic: 4 E: 427 MI: 183 171. Assume that a decline in consumer demand occurs in a purely competitive industry which is initially in long-run equilibrium. We can: A) predict that the new price will be greater than the original price. predict that the new price will be less than the original price. C) predict that the new price will be the same as the original price. D) not compare the original and the new price without knowing about cost conditions in the industry. Answer: D

Type: D Topic: 4 E: 430-431 MI: 186-187 172. A decreasing-cost industry is one in which: A) contraction of the industry will decrease unit costs.input prices fall or technology improves as the industry expands. C) the long-run supply curve is perfectly elastic. D) the long-run supply curve is upsloping. Answer: B

Type: A Topic: 4 E: 430-431 MI: 186-187 173. When compact disc (CD) players first came on the market, they sold for over $1,000. Now they cost only $100. These facts imply that: A) the CD industry was once competitive, but is now monopolistic. fewer firms produce CD players than was the case five or ten years ago. C) the demand curve for CD players has shifted leftward. D) the CD player industry is a decreasing-cost industry. Answer: D

Type: A Topic: 4 E: 431 MI: 187 174. Suppose that an industry's long-run supply curve is downsloping. This suggests that: A) it is an increasing-cost industry. relevant inputs have become more expensive as the industry has expanded. C) technology has become less efficient as a result of the industry's expansion. D) it is a decreasing-cost industry. Answer: D

Type: A Topic: 4 E: 430-431 MI: 186-187 175. Suppose an increase in product demand occurs in a decreasing-cost industry. As a result: A) the new long-run equilibrium price will be lower than the original long-run equilibrium price. B) equilibrium quantity will decline. C) firms will eventually leave the industry. D) the new long-run equilibrium price will be higher than the original price. Answer: A

Page 440: Microeconomics Study Guide

Type: C Topic: 4 E: 429-430 MI: 185-186 176. Purely competitive industry X has constant costs and its product is an inferior good. The industry is currently in long-run equilibrium. The economy now goes into a recession and average incomes decline. The result will be:A) an increase in output and in the price of the product. B) an increase in output, but not in the price, of the product. C) a decrease in the output, but not in the price, of the product. D) a decrease in output and in the price of the product. Answer: B

Type: A Topic: 4 E: 430 MI: 186 177. Suppose losses cause industry X to contract and, as a result, the prices of relevant inputs decline. Industry X is: A) a constant-cost industry. C) an increasing-cost industry. B) a decreasing-cost industry. D) encountering X-inefficiency. Answer: C

Use the following to answer questions 178-183:

Type: G Topic: 4 E: 420 MI: 176 178. Refer to the above diagram showing the average total cost curve for a purely competitive firm. Suppose this firm is maximizing its total profit and the market price is $15. The firm's per unit profit is: A) $5. B) $200. C) a positive amount less than $5. D) a positive amount more than $200. Answer: C

Type: G Topic: 4 E: 420 MI: 176 179. Refer to the above diagram showing the average total cost curve for a purely competitive firm. Suppose that total variable cost is $300 at 40 units of output. At that level of output, average fixed cost: A) is $2.50. B) is $4. C) is $100. D) cannot be determined from the information provided. Answer: A

Type: G Topic: 4 E: 420 MI: 176 180. Refer to the above diagram showing the average total cost curve for a purely competitive firm. Suppose that average variable cost is $8 at 40 units of output. At that level of output, total fixed cost: A) is $2. B) is $40. C) is $80. D) cannot be determined from the information provided. Answer: C

Type: G Topic: 4 E: 415-416, 427 MI: 171-172, 183 181. Refer to the above diagram showing the average total cost curve for a purely competitive firm. At the long-run equilibrium level of output, this firm's total revenue: A) is $10. B) is $40. C) is $400. D) cannot be determined from the information provided. Answer: C

Type: G Topic: 4 E: 420, 427 MI: 176, 183 182. Refer to the above diagram showing the average total cost curve for a purely competitive firm. At the long-run

Page 441: Microeconomics Study Guide

equilibrium level of output, this firm's total cost: A) is $10. B) is $40. C) is $400. D) cannot be determined from the information provided. Answer: C

Type: G Topic: 4 E: 427 MI: 183 183. Refer to the above diagram showing the average total cost curve for a purely competitive firm. At the long-run equilibrium level of output, this firm's economic profit: A) is zero. B) is $400. C) is $200. D) cannot be determined from the information provided. Answer: A Type: A Topic: 4 E: 427 MI: 183 184. The MR = MC rule applies: A) in the short run, but not in the long run. C) in both the short run and the long run. B) in the long run, but not in the short run. D) only to a purely competitive firm. Answer: C

Type: A Topic: 4 E: 430 MI: 186 185. If the long-run supply curve of a purely competitive industry slopes upward, this implies that the prices of relevant resources: A) will fall as the industry expands. C) rise as the industry contracts. B) are constant as the industry expands. D) rise as the industry expands. Answer: D

Use the following to answer questions 186-187:

Type: G Topic: 4 E: 430 MI: 186 186. Refer to the above diagram. Line (1) reflects the long-run supply curve for: A) a constant-cost industry. C) an increasing-cost industry. B) a decreasing-cost industry. D) technologically progressive industry.Answer: C

Type: G Topic: 4 E: 429 MI: 185 187. Refer to the above diagram. Line (2) reflects the long-run supply curve for: A) a constant-cost industry. C) an increasing-cost industry. B) a decreasing-cost industry. D) technologically progressive industry.Answer: A

Pure competition and efficiency

Type: A Topic: 5 E: 432 MI: 188 188. Allocative efficiency is achieved when the production of a good occurs where: A) P = minimum ATC. B) P = MC. C) P = minimum AVC. D) total revenue is equal to TFC. Answer: B

Page 442: Microeconomics Study Guide

Type: A Topic: 5 E: 432 MI: 188 189. A firm is producing an output such that the benefit from one more unit is more than the cost of producing that additional unit. This means the firm is: A) producing more output than allocative efficiency requires. B) producing less output than allocative efficiency requires. C) achieving productive efficiency. D) producing an inefficient output, but we cannot say whether output should be increased or decreased. Answer: B

Type: A Topic: 5 E: 431 MI: 187 190. Resources are efficiently allocated when production occurs where:A) marginal cost equals average variable cost. C) price is equal to marginal cost. B) price is equal to average revenue. D) price is equal to average variable cost. Answer: C

Type: D Topic: 5 E: 432 MI: 188 191. The term productive efficiency refers to: A) any short-run equilibrium position of a competitive firm. B) the production of the product-mix most desired by consumers. C) the production of a good at the lowest average total cost. D) fulfilling the condition P = MC. Answer: C

Type: A Topic: 5 E: 432 MI: 188 192. If the price of product Y is $25 and its marginal cost is $18: A) Y is being produced with the least-cost combination of resources. B) society will realize a net gain if less of Y is produced. C) resources are being underallocated to Y. D) resources are being overallocated to Y. Answer: C

Type: D Topic: 5 E: 432 MI: 188 193. The term allocative efficiency refers to: A) the level of output that coincides with the intersection of the MC and AVC curves. B) minimization of the AFC in the production of any good. C) the production of the product-mix most desired by consumers. D) the production of a good at the lowest average total cost. Answer: C

Type: A Topic: 5 E: 433 MI: 189 194. Under pure competition in the long run: A) neither allocative efficiency nor productive efficiency are achieved. B) both allocative efficiency and productive efficiency are achieved. C) productive efficiency is achieved, but allocative efficiency is not. D) allocative efficiency is achieved, but productive efficiency is not. Answer: B

Page 443: Microeconomics Study Guide

Type: A Topic: 5 E: 432 MI: 188 195. If for a firm P = minimum ATC = MC, then:A) neither allocative efficiency nor productive efficiency is being achieved. B) productive efficiency is being achieved, but allocative efficiency is not. C) both allocative efficiency and productive efficiency are being achieved. D) allocative efficiency is being achieved, but productive efficiency is not. Answer: C

Use the following to answer questions 196-201:

Type: G Topic: 5 E: 431 MI: 187 196. The above diagram portrays: A) a competitive firm that should shut down in the short run. B) the equilibrium position of a competitive firm in the long run. C) a competitive firm that is realizing an economic profit. D) the loss-minimizing position of a competitive firm in the short run. Answer: B

Type: G Topic: 5 E: 431 MI: 187 Status: New 197. Refer to the above diagram. If this competitive firm produces output Q, it will:A) suffer an economic loss.B) earn a normal profit.C) earn an economic profit.D) achieve productive efficiency, but not allocative efficiency. Answer: B

Type: G Topic: 5 E: 433 MI: 189 198. Refer to the above diagram. By producing output level Q: A) neither productive nor allocative efficiency are achieved. B) both productive and allocative efficiency are achieved. C) allocative efficiency is achieved, but productive efficiency is not. D) productive efficiency is achieved, but allocative efficiency is not. Answer: B

Type: G Topic: 5 E: 432 MI: 188 199. Refer to the above diagram. At output level Q1: A) neither productive nor allocative efficiency are achieved. B) both productive and allocative efficiency are achieved. C) allocative efficiency is achieved, but productive efficiency is not. D) productive efficiency is achieved, but allocative efficiency is not. Answer: A

Page 444: Microeconomics Study Guide

Type: G Topic: 5 E: 432 MI: 188 200. Refer to the above diagram. At output level Q1: A) resources are overallocated to this product and productive efficiency is not realized. B) resources are underallocated to this product and productive efficiency is not realized. C) productive efficiency is achieved, but resources are underallocated to this product. D) productive efficiency is achieved, but resources are overallocated to this product. Answer: B

Type: G Topic: 5 E: 432-433 MI: 188-189 201. Refer to the above diagram. At output level Q2: A) resources are overallocated to this product and productive efficiency is not realized. B) resources are underallocated to this product and productive efficiency is not realized. C) productive efficiency is achieved, but resources are underallocated to this product. D) productive efficiency is achieved, but resources are overallocated to this product. Answer: A

Type: A Topic: 5 E: 432 MI: 188 202. Assume that society places a higher value on the last unit of X produced than the value of the resources used to produce that unit. With no spillovers, this information means that: A) total cost is greater than total revenue. C) marginal cost is greater than price. B) price is greater than marginal cost. D) resources are being overallocated to X. Answer: B

Type: A Topic: 5 E: 432-433 MI: 188-189 203. If production is occurring where marginal cost exceeds price, the purely competitive firm will: A) maximize profit, but resources will be underallocated to the product. B) maximize profit, but resources will be overallocated to the product. C) fail to maximize profit and resources will be overallocated to the product. D) fail to maximize profit and resources will be underallocated to the product. Answer: C

Type: A Topic: 5 E: 432 MI: 188 204. If a purely competitive firm is producing where price exceeds marginal cost, then: A) the firm will fail to maximize profit, but resources will be efficiently allocated. B) the firm will fail to maximize profit and resources will be overallocated to the product. C) the firm will fail to maximize profit and resources will be underallocated to the product. D) resources will be underallocated to the product, but the firm will maximize profit. Answer: C

Type: A Topic: 5 E: 432 MI: 188 Status: New 205. Which of the following conditions is true for a purely competitive firm in long-run equilibrium?A) P > MC = minimum ATC. C) P = MC = minimum ATC.B) P > MC > minimum ATC. D) P < MC < minimum ATC.Answer: C

Page 445: Microeconomics Study Guide

Consider This Questions

Type: A E: 423 MI: 179 Status: New 206. (Consider This) An unprofitable motel will stay open in the short-run if:A) price (average nightly room rate) exceeds average variable cost.B) marginal revenue exceeds marginal cost.C) price (average nightly room rate) exceeds average fixed cost.D) marginal revenue exceeds price.Answer: A

Type: A E: 423 MI: 179 Status: New 207. (Consider This) An otherwise unprofitable motel located on a largely abandoned roadway might be able to stay open for several years by:A) increasing its nightly room rates.B) reducing or eliminating its annual maintenance expenses.C) charging room rates that exceed marginal revenue.D) eliminating its fixed costs, including its opportunity costs.Answer: B

Last Word Questions

Use the following to answer questions 208-210:

Type: G E: 434 MI: 190 208. (Last Word) Refer to the above graph of the market for asparagus. At the market price of $2, area A + B represents:A) total consumer utility. C) consumer surplus.B) total revenue to sellers. D) returns to capital and to labor.Answer: A

Type: G E: 434 MI: 190 209. (Last Word) Refer to the above graph of the market for asparagus. At the market price of $2, area A represents:A) total consumer utility. B) total revenue to sellers. C) consumer surplus. D) economic profit.Answer: C

Type: G E: 434 MI: 190 210. (Last Word) In long-run equilibrium, purely competitive markets:A) minimize total cost.B) maximize consumer surplus.C) yield economic profits to most sellers.D) inevitably degenerate into monopoly in increasing cost industries.Answer: B

Page 446: Microeconomics Study Guide

True/False Questions

Type: A E: 416 MI: 172 211. In maximizing profit a firm will always produce that output where total revenues are at a maximum. Answer: False

Type: A E: 422 MI: 178 212. In the short run a competitive firm will always choose to shut down if product price is less than the lowest attainable average total cost. Answer: False

Type: A E: 427 MI: 183 213. After all long-run adjustments have been completed, a firm in a competitive industry will produce that level of output where average total cost is at a minimum. Answer: True

Type: A E: 430-431 MI: 186-187 214. The long-run supply curve for a decreasing-cost industry is downsloping. Answer: True

Type: A E: 420 MI: 176 215. A competitive firm will produce in the short run so long as its price exceeds its average fixed cost.Answer: False

Type: A E: 432 MI: 188 216. Marginal cost is a measure of the alternative goods which society forgoes in using resources to produce an additional unit of some specific product. Answer: True

Type: A E: 415-416 MI: 171-172 217. Price and marginal revenue are identical for an individual purely competitive seller. Answer: True

Type: A E: 432 MI: 188 218. Because the equilibrium position of a purely competitive seller entails an equality of price and marginal costs, competition produces up to an efficient allocation of economic resources. Answer: True

Type: A E: 423-424 MI: 179-180 219. The short-run supply curve slopes upward because producers must be compensated for rising marginal costs. Answer: True

Type: A E: 425 MI: 181 220. The demand curve for a purely competitive industry is perfectly elastic, but the demand curves faced by individual firms in such an industry are downsloping.

Page 447: Microeconomics Study Guide

Answer: False

Type: A E: 417-418 MI: 173-174 221. The total revenue curve of a competitive seller graphs as a straight, upsloping line. Answer: True

Type: D E: 416 MI: 172 222. Marginal revenue is the addition to total revenue resulting from the sale of one more unit of output. Answer: True

Use the following to answer questions 223-231:

Type: G E: 419-420 MI: 175-176 223. Refer to the above diagram. This firm will maximize profits by producing output D. Answer: False

Question 10 1.5 out of 1.5 points

The usual starting point for a master budget is:

Selected Answer: the sales forecast or sales budget.

Correct Answer: the sales forecast or sales budget.

Question 11 1.5 out of 1.5 points

Which of the following benefits could an organization reasonably expect from an effective budget program?

Selected Answer: All of these.

Correct Answer: All of these.

Question 12 1.5 out of 1.5 points

On January 1, Barnes Company has 8,000 units of Product A on hand. During the year, the company plans to sell 30,000 units of Product A, and plans to have 6,500 units on hand at year end. How many units of Product A must be produced during the year?

Selected Answer: 28,500

Page 448: Microeconomics Study Guide

Correct Answer: 28,500

Question 13 0 out of 1.5 points

Poorly trained workers could have an unfavorable effect on which of the following variances?

Selected Answer:

Correct Answer:

Question 14 1.5 out of 1.5 points

The variance that is most useful in assessing the performance of the purchasing department manager is:

Selected Answer: the materials price variance.

Correct Answer: the materials price variance.

Question 15 1.5 out of 1.5 points

The purpose of a flexible budget is to:

Selected Answer: compare actual and budgeted results at virtually any level of activity. Correct Answer: compare actual and budgeted results at virtually any level of activity.

Question 16 1.5 out of 1.5 points

If operating income is $60,000, average operating assets are $240,000, and the minimum required rate of return is 20%, what is the residual income?

Selected Answer: $12,000

Correct Answer: $12,000

Question 17 1.5 out of 1.5 points

Kinsi Corporation manufactures five different products. All five of these products must pass through a stamping machine in its fabrication department. This machine is Kinsi's constrained resource. Kinsi would make the most profit if it produces the product that:

Page 449: Microeconomics Study Guide

Selected Answer: generates the highest contribution margin per stamping machine hour. Correct Answer: generates the highest contribution margin per stamping machine hour.

Question 18 1.5 out of 1.5 points

The following information relates to next year's projected operating results of the Children's Division of Grunge Clothing Corporation:

If Children's Division is dropped, half of the fixed costs above can be eliminated. What will be the effect on Grunge's profit next year if Children's Division is dropped instead of being kept?

Selected Answer: $50,000 increase

Correct Answer: $50,000 increase

Question 19 1.5 out of 1.5 points

A company has unlimited funds to invest at its discount rate. The company should invest in all projects having:

Selected Answer: a net present value greater than zero.

Correct Answer: a net present value greater than zero.

Question 20 1.5 out of 1.5 points

An opportunity cost is:

Selected Answer: the benefit forgone when one alternative is selected rather than another. Correct Answer: the benefit forgone when one alternative is selected rather than another.

Question 21 1.5 out of 1.5 points

A favorable materials price variance of $380 and an unfavorable materials quantity variance of $120 were reported during the current year. Based on these variances, you can conclude that:

Selected Answer: the actual cost per unit of materials was less than the standard cost per unit. Correct Answer: the actual cost per unit of materials was less than the standard cost per unit.

Question 22 1.5 out of 1.5 points

The process of assigning indirect costs is called

Page 450: Microeconomics Study Guide

Selected Answer: cost allocation.

Correct Answer: cost allocation.

1. Luella Corporation prepares its statement of cash flows using the indirect method. Which of the following would be added to net income in the operating activities section of the statement?

A. B. C. D. 2. Which of the following should be classified as an investing activity on a statement of cash flows? A. cash received from the sale of office equipment that was sold at a loss.B. cash used to purchase a long-term investment in bonds of another corporation.C. cash received from the issuance of Iguato Corporation common stock.D. both A and B aboveE. All of these3. Three potential investment projects (A, B, and C) at Nit Corporation all require the same initial investment, have the same useful life (3 years), and have no expected salvage value. Expected net cash inflows from these three projects each year is as follows:

What can be determined from the information provided above? A. the net present value of project C will be the highest.B. the internal rate of return of projects A and C cannot be computed.C. the net present value and the internal rate of return will be the same for all three projects.D. both A and B above.4. Which of the following statements is false?I. For capital budgeting decisions, the net present value method is superior to the simple rate of return method. II. When using the payback method, any cash flows for a project that occur after the payback period are not considered in computing the payback period for that project.III. The present value of a given future cash flow will increase as the discount rate decreases.

A. Statement IB. Statement IIC. Statement III.D. Exactly two of the statements.E. None of the statements is false.

5. (Ignore income taxes in this problem.) How much would you have to invest today in the bank at an interest rate of 8% to have an annuity of $4,800 per year for 7 years, with nothing left in the bank at the end of the 7 years? Select the amount below that is closest to your answer. A. $33,600B. $2,798C. $24,989D. $31,1116. Which of the following statements is true?I. Sunk costs and future costs that do not differ between the alternatives are not relevant in a decision.

Page 451: Microeconomics Study Guide

II. A future cost that does not vary among alternatives under consideration is irrelevant.III. Opportunity costs represent economic benefits that are forgone as a result of pursuing some course of action.

A. Statement IB. Statement IIC. Statement III.D. Exactly two of the statements.E. All three statements are true.

7. Consider the following statements:

I. A division's net operating income, after deducting both traceable and allocated common corporate costs, is negative.II. The division's avoidable fixed costs exceed its contribution margin.III. The division's traceable fixed costs plus its allocated common corporate costs exceed its contribution margin.

Which of the above statements give an economic reason for eliminating the division? A. Only IB. Only IIC. Only IIID. Only I and IIE. None of the above.

8. Once the break-even point is reached: A. the total contribution margin changes from negative to positive. B. net income will increase by the unit contribution margin for each additional item sold.C. variable expenses will remain constant in total.D. the contribution margin ratio begins to decrease.E. none of the above.

9. The Jabba Company manufactures the "Snack Buster" which consists of a wooden snack chip bowl with an attached porcelain dip bowl. Which of the following would be relevant in Jabba's decision to make the dip bowls or buy them from an outside supplier?

A. B. C. D.

10. Which of the following statements is true?I. A responsibility center is a business segment whose manager has control over costs, revenues, or investments in operating assets. II. A segment is any part or activity of an organization about which a manager seeks cost, revenue, or profit data.III. The use of return on investment (ROI) as a performance measure may lead managers to reject a project that would be favorable for the company as a whole.

A. Statement IB. Statement II

Page 452: Microeconomics Study Guide

C. Statement III.D. Only two of the statements are true.E. All three statements are true.

11. Mike Corporation uses residual income to evaluate the performance of its divisions. The company's minimum required rate of return is 14%. In January, the Commercial Products Division had average operating assets of $970,000 and net operating income of $143,700. What was the Commercial Products Division's residual income in January? A. $7,900B. -$20,118C. $20,118D. -$7,900

12. Which of the following statements is false?I. In a flexible budget, when the activity declines, the variable cost per unit also declines. II. The higher the denominator activity level used to compute the predetermined overhead rate, the higher the predetermined overhead rate.III. If the denominator level of activity is more than the standard hours allowed for the output of the period, then the volume variance is unfavorable, indicating an under utilization of available facilities.

A. Statement IB. Statement IIC. Statement III.D. Exactly two of the statements.E. All of the statements are false.13. The variance that is most useful in assessing the performance of the purchasing department manager is: A. the materials quantity variance.B. the materials price variance.C. the labor rate variance.D. the labor efficiency variance.14. All of the following statements are correct when referring to process costing except: A. Process costing would be appropriate for a jeweler who makes custom jewelry to order.B. A process costing system has the same basic purposes as a job-order costing system.C. Units produced are indistinguishable from each other.D. Costs are accumulated by department.15. During a recent lengthy strike at Morell Manufacturing Company, management replaced striking assembly line workers with office workers. The assembly line workers were being paid $18 per hour while the office workers are only paid $10 per hour. What is the most likely effect on the labor variances in the first month of this strike?

A. B. C. D. 16. Which of the following statements is true?I. Ideal standards do not allow for machine breakdowns and other normal inefficiencies. II. The standard price per unit for direct materials should reflect the final, delivered cost of the materials, net of any discounts taken.III. The standard quantity or standard hours allowed refers to the amount of the input that should have been used to produce the actual output of the period.

Page 453: Microeconomics Study Guide

A. Statement IB. Statement IIC. Statement III.D. Only two of the statements are true.E. All three statements are true.

17. Which of the following statements is true?I. The direct materials to be purchased for a period can be obtained by subtracting the desired ending inventory of direct materials from the total direct materials needed for the period. II. The usual starting point in budgeting is to make a forecast of net income.III. In a production budget, if the number of units in finished goods inventory at the end of the period is less than the number of units in finished goods inventory at the beginning of the period, then the expected number of units sold is greater than the number of units to be produced during the period.

A. Statement IB. Statement IIC. Statement III.D. Only two of the statements are true.E. All three statements are true.18. The completion of goods is recorded as a decrease in the work in process inventory account when using:

A. B. C. D.

19. Which of the following statements is true?I. In a process costing system, the costs of one processing department become part of the costs of the next processing department. II. The units in beginning work in process inventory plus the units started into production must equal the units transferred out of the department plus the units in ending work in process inventory.III. Process costing is employed in industries that produce basically homogeneous products such as bricks, flour, or cement but would not be appropriate for assembly-type operations such as those that manufacture computers.

A. Statement IB. Statement IIC. Statement III.D. Only two of the statements are true.E. All three statements are true.20. If company A has a higher degree of operating leverage than company B, then: A. the company A has higher variable expenses. B. the company A's profits are more sensitive to percentage changes in sales.C. the company A is more profitable.D. the company A is less risky.E. none of the above.

21. Which of the following statements is false?I. Process costing is used where many different products are produced each period to customer specifications. II. When a company changes from a traditional costing system to an activity-based costing system, the unit

Page 454: Microeconomics Study Guide

product costs of high-volume products typically change more than the unit product costs of low-volume products.III. Activity-based costing uses a number of activity cost pools, each of which is allocated to products on the basis of direct labor-hours.

A. Statement IB. Statement IIC. Statement III.D. Exactly two of the statements.E. All of the statements are false.

22. Which of the following would probably be the most accurate measure of activity to use for allocating the costs associated with a factory's purchasing department? A. Machine-hoursB. Direct labor-hoursC. Number of orders processedD. Cost of materials purchased23. In activity-based costing, unit product costs computed for external financial reports do NOT include: A. direct materials.B. direct labor.C. manufacturing overhead.D. selling costs.

24. Which of the following statements is correct concerning job-order costing? A. Job-order costing would be appropriate for a textbook publisher.B. All the costs appearing on a job cost sheet are actual costs.C. Indirect materials are charged to a specific job.D. Job-order costing is mainly used in firms with homogeneous products such as oil refineries.

25. Which of the following types of firms typically would use process costing rather than job-order costing? A. A small appliance repair shop.B. A manufacturer of commercial passenger aircraft.C. A specialty equipment manufacturer.D. A breakfast cereal manufacturer.

26. In a job-order costing system, the application of manufacturing overhead would be recorded as a debit to: A. Manufacturing Overhead inventory.B. Finished Goods inventory.C. Work in Process inventory.D. Cost of Goods Sold.

27. How would the following costs be classified (product or period) under variable costing at a retail clothing store?

A. B. C. D.

28. In a job-order costing system, the cost of a completed but unsold job is: A. closed to Cost of Goods Sold.

Page 455: Microeconomics Study Guide

B. part of the Work in Process inventory balance.C. adjusted to exclude any applied overhead.D. part of the Finished Goods inventory balance.29. If overhead is underapplied, then: A. actual overhead cost is less than estimated overhead cost.B. the amount of overhead cost applied to Work in Process is less than the actual overhead cost incurred.C. the predetermined overhead rate is too high.D. the Manufacturing Overhead account will have a credit balance at the end of the year.

30. If both the fixed and variable expenses associated with a product decrease, what will be the effect on the contribution margin ratio and the break-even point, respectively?

A. B. C. D.

31. Which of the following is true regarding the contribution margin ratio of a single product company? A. As fixed expenses decrease, the contribution margin ratio increases.B. The contribution margin ratio multiplied by the selling price per unit equals the contribution margin per unit.C. The contribution margin ratio will decline as unit sales decline.D. The contribution margin ratio equals the selling price per unit less the variable expense ratio.

32. If a company is operating at the break-even point: A. its contribution margin will be equal to its variable expenses.B. its margin of safety will be equal to zero.C. its fixed expenses will be equal to its variable expenses.D. its selling price will be equal to its variable expense per unit.

33. Which of the following strategies could be used to reduce the break-even point?

A. B. C. D.

34. Which costs will change with an increase in activity within the relevant range? A. Unit fixed cost and total fixed costB. Unit variable cost and total variable costC. Unit fixed cost and total variable costD. Unit fixed cost and unit variable cost

35. Which of the following would usually be considered a discretionary fixed cost for a soft drink bottling company? A. the cost of advertising its productsB. the cost of fire insurance on its factory buildingC. depreciation on its manufacturing equipmentD. both a and b above

Like · · Share

Page 456: Microeconomics Study Guide

Recent Posts by OthersSee All

Muhiimbo Leonald Amootti

What is the price of motherboard and powersupply?

2 · September 16 at 5:38am

Martinsville Indiana Computer Repair - 46151

September 15Question 23 1.5 out of 1.5 points

Which of the following is not an advantage of decentralization for a company?

Selected Answer: subunit managers will act to benefit the organization as a whole Correct Answer: subunit managers will act to benefit the organization as a whole

Question 24 1.5 out of 1.5 points ...See More

Like · · Share

Martinsville Indiana Computer Repair - 46151

September 15Question 9 1.5 out of 1.5 points

Which of the following is not one of the questions you should ask when faced with an ethical dilemma?

Selected Answer: Do I think I will get caught? Correct Answer: Do I think I will get caught?

Question 10 1.5 out of 1.5 points

Which of the following costs is part of manufacturing overhead?

Selected Answer: Indirect labor

Correct Answer: Indirect labor

Page 457: Microeconomics Study Guide

Question 11 1.5 out of 1.5 points

Product costs

Selected Answer: All of the above answers are correct.

Correct Answer: All of the above answers are correct.

Question 12 1.5 out of 1.5 points

Which of the following is not a period cost?

Selected Answer: Direct materials

Correct Answer: Direct materials

Question 13 1.5 out of 1.5 points

Which of the following companies is most likely to use a process costing system?

Selected Answer: A company that produces petroleum products Correct Answer: A company that produces petroleum products

Question 14 1.5 out of 1.5 points

An allocation base is

Selected Answer: a common characteristic that jobs share, which is used to spread the overhead costs among the various jobs. Correct Answer: a common characteristic that jobs share, which is used to spread the overhead costs among the various jobs.

Question 15 0 out of 1.5 points

Which of the following statements about job-order costing is not true?

Selected Answer: Manufacturing overhead cannot be traced directly to jobs, so it is assigned using the overhead allocation rate. Correct Answer: Indirect labor is traced to jobs using time tickets.

Question 16 1.5 out of 1.5 points

Page 458: Microeconomics Study Guide

Which of the following is the most reasonable allocation base for a highly mechanized process?

Selected Answer: machine hours

Correct Answer: machine hours

Question 17 1.5 out of 1.5 points

A predetermined overhead rate is preferred over an actual overhead rate because a predetermined overhead rate

Selected Answer: Both A and C Correct Answer: Both A and C

Question 18 1.5 out of 1.5 points

The balance in the Finished Goods Inventory account on July 1, 2004, was $34,000 and the June 30, 2005, balance in the Finished Goods Inventory account was $41,000. If the cost of goods manufactured was $200,000, what was the cost of goods sold?

Selected Answer: $193,000

Correct Answer: $193,000

Question 19 0 out of 1.5 points

Black Company's Work in Process Inventory account has a beginning balance of $40,000 and an ending balance of $50,000. Current manufacturing costs total $125,000. What is the cost of goods manufactured?

Selected Answer: $125,000

Correct Answer: $115,000

Question 20 0 out of 1.5 points

Emerald Company estimates that the total overhead costs for 2004 will be $150,000 and that the production employees will work 30,000 direct labor hours and earn $600,000 during the year. If the company allocates overhead based on direct labor hours, what is the predetermined overhead rate?

Selected Answer: $20.00 per direct labor hour

Correct Answer: $5.00 per direct labor hour

Question 21

Page 459: Microeconomics Study Guide

1.5 out of 1.5 points

Companies that use process costing systems

Selected Answer: All of the above answers are correct.

Correct Answer: All of the above answers are correct.

Question 22 1.5 out of 1.5 points

The type of costing system commonly used by companies that produce a large number of homogeneous units in a continuous production process is called a

Selected Answer: process costing system.

Correct Answer: process costing system.

Question 23 0 out of 1.5 points

The equivalent units are calculated by:

Selected Answer: taking the units started plus the equivalent units in ending inventory. Correct Answer: taking the units completed plus the equivalent units in ending inventory.

Question 24 1.5 out of 1.5 points

In a process costing system, when items are sold, the cost of the items is moved from

Selected Answer: Finished Goods to Cost of Goods Sold. Correct Answer: Finished Goods to Cost of Goods Sold.

Question 25 1.5 out of 1.5 points

Which of the following sequences describes the typical flow of costs in a company that is using process costing?

Selected Answer: Work in Process, Finished Goods, Raw Materials Inventory, Cost of Goods Sold Correct Answer: Work in Process, Finished Goods, Raw Materials Inventory, Cost of Goods Sold

Question 26 1.5 out of 1.5 points

When partially completed units are converted to a comparable number of completed units, they are referred to as

Selected Answer: equivalent units.

Page 460: Microeconomics Study Guide

Correct Answer: equivalent units.

Question 27 1.5 out of 1.5 points

A production cost report

Selected Answer: provides a reconciliation of units and a reconciliation of costs as well as the details of the cost per equivalent unit calculation. Correct Answer: provides a reconciliation of units and a reconciliation of costs as well as the details of the cost per equivalent unit calculation.

Question 28 1.5 out of 1.5 points

If beginning inventory consisted of 3,500 units, ending inventory was 1,500 units, and 9,000 units were started during the period, how many units were completed and transferred out of the department?

Selected Answer: 11,000 units

Correct Answer: 11,000 units

Question 29 1.5 out of 1.5 points

If ending inventory consists of 2,600 units which are 100% complete with respect to materials and 30% complete with respect to conversion costs, the number of equivalent units for materials and for conversion costs are

Selected Answer: 2,600 and 780, respectively.

Correct Answer: 2,600 and 780, respectively.

Question 30 1.5 out of 1.5 points

Which of the following is most likely to be a variable cost?

Selected Answer: direct materials

Correct Answer: direct materials

Question 31 1.5 out of 1.5 points

When the level of activity increases, total variable costs

Page 461: Microeconomics Study Guide

Selected Answer: Increases in direct proportion to the increase in activity Correct Answer: Increases in direct proportion to the increase in activity

Question 32 1.5 out of 1.5 points

Which of the following statements is correct?

Selected Answer: Profit is equal to revenue minus total variable costs minus total fixed costs. Correct Answer: Profit is equal to revenue minus total variable costs minus total fixed costs.

Question 33 1.5 out of 1.5 points

The number of units that must be sold in order for a company to exactly cover its fixed and variable costs is the

Selected Answer: break-even point.

Correct Answer: break-even point.

Question 34 1.5 out of 1.5 points

Marro Manufacturing is operating at its break-even point of 10,000 units. Which of the following statements is not true?

Selected Answer: Marro's fixed costs equal its variable costs.

Correct Answer: Marro's fixed costs equal its variable costs.

Question 35 1.5 out of 1.5 points

Holding all other factors constant, the break-even point will be decreased by

Selected Answer: increasing the selling price.

Correct Answer: increasing the selling price.

Question 36 0 out of 1.5 points

Which of the following is not true for a firm with high operating leverage?

Selected Answer: It has a larger contribution margin ratio than similar firms. Correct Answer: If sales increase, its profits will increase slower than a company with lower operating leverage.

Question 37

Page 462: Microeconomics Study Guide

1.5 out of 1.5 points

When considering a process that involves a resource constraint, the optimal decision

Selected Answer: maximizes the contribution margin per unit of the constraint. Correct Answer: maximizes the contribution margin per unit of the constraint.

Question 38 1.5 out of 1.5 points

Fixed costs are $20,570. At the break-even point, variable costs are $1.63 per unit and the sales price is $3.50 per unit. What is the contribution margin per unit?

Selected Answer: $1.87

Correct Answer: $1.87

Question 39 1.5 out of 1.5 points

Copper Corporation sells a single product at a price of $275 per unit. Variable cost per unit is $135 and fixed costs total $356,860. If sales are expected to be $825,000, what is Cooper's margin of safety?

Selected Answer: $124,025

Correct Answer: $124,025

Question 40 1.5 out of 1.5 points

Anchor Company produces a single product that is sold for $85 per unit. If variable costs per unit are $26 and fixed costs total $47,500, how many units must Anchor sell in order to earn a profit of $100,000?

Selected Answer: 2,500

Correct Answer: 2,500

EXAM 2Question 1 1.5 out of 1.5 points

Which of the following is treated differently in full costing than in variable costing?

Selected Answer: fixed manufacturing overhead Correct Answer: fixed manufacturing overhead

Question 2 1.5 out of 1.5 points

Page 463: Microeconomics Study Guide

Which of the following is treated as a product cost in variable costing?

Selected Answer: direct labor

Correct Answer: direct labor

Question 3 1.5 out of 1.5 points

Full costing is:

Selected Answer: required for financial reporting under generally accepted accounting principles. Correct Answer: required for financial reporting under generally accepted accounting principles.

Question 4 1.5 out of 1.5 points

In variable costing, when does fixed manufacturing overhead become an expense?

Selected Answer: in the period when the expense is incurred

Correct Answer: in the period when the expense is incurred

Question 5 1.5 out of 1.5 points

In variable costing, which of the following would not be included in inventory?

Selected Answer: none of the above items would be in inventory in variable costing Correct Answer: none of the above items would be in inventory in variable costing

Question 6 1.5 out of 1.5 points

Which of the following items appears on a variable costing income statement but not on a full costing income statement?

Selected Answer: contribution margin

Correct Answer: contribution margin

Question 7 1.5 out of 1.5 points

When the number of units sold is equal to the number of units produced, net income using full costing will be

Selected Answer: equal to net income using variable costing.

Correct Answer: equal to net income using variable costing.

Page 464: Microeconomics Study Guide

Question 8 1.5 out of 1.5 points

The process of assigning indirect costs is called

Selected Answer: cost allocation.

Correct Answer: cost allocation.

Question 9 1.5 out of 1.5 points

A contract which specifies that the suppler will be paid for the cost of production as well as some fixed amount or percentage of cost is called a(n)

Selected Answer: cost-plus contract.

Correct Answer: cost-plus contract.

Question 10 1.5 out of 1.5 points

A grouping of individual costs whose total is allocated using one allocation base is called

Selected Answer: cost pool.

Correct Answer: cost pool.

Question 11 1.5 out of 1.5 points

Companies which use only one or two cost pools rather than several cost pools

Selected Answer: may have seriously distorted product costs. Correct Answer: may have seriously distorted product costs.

Question 12 1.5 out of 1.5 points

When activity based costing is implemented, the initial outcome is normally that:

Selected Answer: The cost of low volume products will be higher and the cost of high volume products will be lower Correct Answer: The cost of low volume products will be higher and the cost of high volume products will be lower

Question 13

Page 465: Microeconomics Study Guide

1.5 out of 1.5 points

Cool Company uses ABC costing. Which of the following is most likely to be the cost driver for the cost of ordering parts?

Selected Answer: number of orders placed

Correct Answer: number of orders placed

Question 14 1.5 out of 1.5 points

Which of the following is not generally true when a company compares ABC and traditional costing?

Selected Answer: ABC allocates cost based solely on production volume. Correct Answer: ABC allocates cost based solely on production volume.

Question 15 1.5 out of 1.5 points

The production departments at Cook Corporation occupy a total area of 600,000 square feet. Heating costs total $900,000 and are allocated based on the area that each department occupies. The finishing department occupies 30,000 square feet and the packaging department occupies 20,000 square feet. What amount of heating cost will be allocated to the finishing and packaging departments, respectively?

Selected Answer: $45,000 and $30,000 Correct Answer: $45,000 and $30,000

Question 16 1.5 out of 1.5 points

Maintenance cost is allocated to the three producing departments based on the machine hours used in each department. The maintenance cost for May was $100,000. The three departments had the following usage for May:Department Machine hours used Direct labor hours usedAssembly 500 2,000Fabrication 800 5,000Testing 700 3,000

The maintenance cost that should be allocated to the Fabrication Department for May should be:

Selected Answer: $40,000

Correct Answer: $40,000

Question 17 1.5 out of 1.5 points

Which of the following is likely to occur when more overhead cost pools are used?

Selected Answer: All of the above are true.

Page 466: Microeconomics Study Guide

Correct Answer: All of the above are true.

Question 18 1.5 out of 1.5 points

Which of the following is not a term used to describe the additional costs incurred as a result of selecting one decision alternative over another?

Selected Answer: sunk costs

Correct Answer: sunk costs

Question 19 1.5 out of 1.5 points

A company is trying to decide whether to sell partially completed goods in their current state or incur additional costs to finish the goods and sell them as complete units. Which of the following is not relevant to the decision?

Selected Answer: the costs incurred to process the units to this point Correct Answer: the costs incurred to process the units to this point

Question 20 1.5 out of 1.5 points

A company is trying to decide whether to keep or drop the sporting goods department in its department store. If the segment is dropped, the manager will be fired. The manager's salary, in relation to the decision to keep or drop the sporting goods department, is

Selected Answer: avoidable and therefore relevant

Correct Answer: avoidable and therefore relevant

Question 21 1.5 out of 1.5 points

The value of benefits foregone by selecting one decision alternative over another is a(n)

Selected Answer: opportunity cost. Correct Answer: opportunity cost.

Question 22 0 out of 1.5 points

A product line should be dropped when

Selected Answer: there will be a positive change in income if the product line is dropped. Correct Answer: all of the above.

Page 467: Microeconomics Study Guide

Question 23 1.5 out of 1.5 points

Which of the following is a direct cost of a specific department in a retail store?

Selected Answer: cost of the department's inventory Correct Answer: cost of the department's inventory

Question 24 1.5 out of 1.5 points

Two or more products that result from common inputs are called

Selected Answer: joint products. Correct Answer: joint products.

Question 25 1.5 out of 1.5 points

Oster Medical Supply Company is trying to decide whether or not to continue distributing hospital supplies. The following information is available for Oster's business segments. Assume that all direct fixed costs could be avoided if a segment is dropped and that the total common fixed costs would remain unchanged if a segment is dropped.Hospital Supplies Retail Stores Mail OrderSales $120,000 $440,000 $360,000Variable Costs $64,000 $200,000 $140,000Contribution Margin $56,000 $240,000 $220,000Direct Fixed Costs $50,000 $80,000 $90,000Allocated Common Fixed Costs $20,000 $70,000 $60,000Net Income ($14,000) $90,000 $70,000

Reference: Ref 7-4

If hospital supplies are dropped, overall net income would:

Selected Answer: Decrease by $6,000

Correct Answer: Decrease by $6,000

Question 26 1.5 out of 1.5 points

Ritz Company makes two products from a joint input that have the following information:

Units Produced Sales Value per unit at split off Total Additional processing costs Sales value per unit after additional processingProduct A 50,000 $10 $400,000 $15Product B 30,000 $8 $300,000 $20

The joint cost incurred to produce the two products to the split off point is $600,000.Reference: Ref 7-7

Page 468: Microeconomics Study Guide

Which products should be processed further?

Selected Answer: B

Correct Answer: B

Question 27 1.5 out of 1.5 points

Which of the following would not appear in a capital budget?

Selected Answer: purchase of inventory to be sold next month Correct Answer: purchase of inventory to be sold next month

Question 28 1.5 out of 1.5 points

Capital expenditure decisions

Selected Answer: All of the above are correct.

Correct Answer: All of the above are correct.

Question 29 1.5 out of 1.5 points

The basic concept involved in time value of money calculations is that

Selected Answer: it is better to receive a dollar today than to receive a dollar in the future. Correct Answer: it is better to receive a dollar today than to receive a dollar in the future.

Question 30 1.5 out of 1.5 points

Which of the following techniques uses time value of money concepts?

Selected Answer: internal rate of return

Correct Answer: internal rate of return

Question 31 0 out of 1.5 points

Which of the following would most likely be the present value of a 5 year annuity of $5,000 per year (assuming a positive discount rate)?

Selected Answer: $25,000

Correct Answer: $18,000

Page 469: Microeconomics Study Guide

Question 32 0 out of 1.5 points

Assuming a 10% rate of return, how does the present value of an amount to be received two years from today compare to the present value of the same amount to be received three years from today?

Selected Answer: It is impossible to tell unless the actual amount is known. Correct Answer: The present value of the amount to be received in two years is larger.

Question 33 1.5 out of 1.5 points

The sum of the present values of all cash flows (inflows and outflows) is called the

Selected Answer: net present value.

Correct Answer: net present value.

Question 34 1.5 out of 1.5 points

Projects with a negative net present value will always have a(n)

Selected Answer: internal rate of return that is less than the required rate of return. Correct Answer: internal rate of return that is less than the required rate of return.

Question 35 1.5 out of 1.5 points

An annuity is

Selected Answer: a series of equal payments.

Correct Answer: a series of equal payments.

Question 36 1.5 out of 1.5 points

The rate of return that equates the present value of future cash flows to the investment outlay is the

Selected Answer: internal rate of return.

Correct Answer: internal rate of return.

Question 37 0 out of 1.5 points

Page 470: Microeconomics Study Guide

Projects should be accepted when:

Selected Answer: The net present value is positive

Correct Answer: Both A and B

Question 38 1.5 out of 1.5 points

If the internal rate of return is greater than the required rate of return

Selected Answer: the project should be accepted. Correct Answer: the project should be accepted.

Question 39 1.5 out of 1.5 points

Which of the following statements about the payback period method is not true?

Selected Answer: The payback period method is more sophisticated and yields better decisions than the internal rate of return method. Correct Answer: The payback period method is more sophisticated and yields better decisions than the internal rate of return method.

Question 40 0 out of 1.5 points

Calculate the present value of $216,000 to be received in seven years if the discount rate is 16%.

Selected Answer: $73,159

Correct Answer: $76,421

EXAM 3Question 1 1.5 out of 1.5 points

A budget

Selected Answer: All of the above are true.

Correct Answer: All of the above are true.

Question 2 1.5 out of 1.5 points

A method of budget preparation that requires all budgeted amounts to be justified by the department, even if the amounts were supported in prior periods, is called

Page 471: Microeconomics Study Guide

Selected Answer: zero base budgeting.

Correct Answer: zero base budgeting.

Question 3 1.5 out of 1.5 points

The comprehensive planning document that incorporates a number of individual budgets is the

Selected Answer: master budget. Correct Answer: master budget.

Question 4 1.5 out of 1.5 points

Which of the following is a reasonable order in which to prepare budgets?

Selected Answer: sales budget, production budget, material purchases budget Correct Answer: sales budget, production budget, material purchases budget

Question 5 1.5 out of 1.5 points

If the number of units in beginning inventory is more than the number of units in ending inventory, the number of units sold is

Selected Answer: greater than the number of units produced. Correct Answer: greater than the number of units produced.

Question 6 1.5 out of 1.5 points

When constructing the production budget, the desired ending inventory for the first period is

Selected Answer: the beginning inventory for the second period. Correct Answer: the beginning inventory for the second period.

Question 7 1.5 out of 1.5 points

A significant difference between the direct material purchases budget and the direct labor budget is that the direct material purchases budget

Selected Answer: considers beginning and ending inventory amounts, which are not part of a direct labor budget. Correct Answer: considers beginning and ending inventory amounts, which are not part of a direct labor budget.

Question 8 1.5 out of 1.5 points

Page 472: Microeconomics Study Guide

Which of the following transactions will affect the cash receipts and disbursements budget for the month in which the transaction occurs?

Selected Answer: payment for direct labor

Correct Answer: payment for direct labor

Question 9 1.5 out of 1.5 points

The cash receipts and disbursements budget alerts management to all of the following except when

Selected Answer: stockouts will cause customer dissatisfaction.

Correct Answer: stockouts will cause customer dissatisfaction.

Question 10 1.5 out of 1.5 points

When budgets are used for evaluation, the differences between budgeted and actual amounts are called

Selected Answer: budget variances.

Correct Answer: budget variances.

Question 11 1.5 out of 1.5 points

The main difference between a static budget and a flexible budget is that the static budget is

Selected Answer: for a single level of activity while a flexible budget can be adjusted for different activity levels. Correct Answer: for a single level of activity while a flexible budget can be adjusted for different activity levels.

Question 12 1.5 out of 1.5 points

A flexible budget takes into account the fact that when production levels change

Selected Answer: total fixed costs remain the same.

Correct Answer: total fixed costs remain the same.

Question 13 1.5 out of 1.5 points

Knight Company has found that 30% of its sales are collected in the month of the sale and the remainder of the

Page 473: Microeconomics Study Guide

sales is collected in the next month. If sales are expected to be $100,000 in April, $120,000 in May, and $80,000 in June, what is the estimated amount of cash receipts for May?

Selected Answer: $106,000

Correct Answer: $106,000

Question 14 1.5 out of 1.5 points

The standard cost is

Selected Answer: the cost that should have been incurred to produce the item. Correct Answer: the cost that should have been incurred to produce the item.

Question 15 1.5 out of 1.5 points

The relationship between standard costs and budgeted costs is the

Selected Answer: standard cost times the expected production level equals the budgeted cost. Correct Answer: standard cost times the expected production level equals the budgeted cost.

Question 16 1.5 out of 1.5 points

Standards that are developed under the assumption that a variety of things may lead to less than perfect performance are

Selected Answer: attainable standards. Correct Answer: attainable standards.

Question 17 1.5 out of 1.5 points

Variances are usually analyzed for

Selected Answer: direct materials, direct labor, and manufacturing overhead. Correct Answer: direct materials, direct labor, and manufacturing overhead.

Question 18 1.5 out of 1.5 points

If the actual price per unit of material is greater than the standard price, the material price variance will be

Selected Answer: unfavorable.

Correct Answer: unfavorable.

Question 19

Page 474: Microeconomics Study Guide

1.5 out of 1.5 points

If the material quantity variance is favorable, the

Selected Answer: production manager has used materials efficiently. Correct Answer: production manager has used materials efficiently.

Question 20 1.5 out of 1.5 points

Which of the following would cause a variance to be unfavorable?

Selected Answer: the standard hours allowed are less than the actual hours worked Correct Answer: the standard hours allowed are less than the actual hours worked

Question 21 0 out of 1.5 points

A possible cause for an unfavorable labor rate variance is

Selected Answer: hiring new, inexperienced employees.

Correct Answer: using more experienced workers than planned.

Question 22 1.5 out of 1.5 points

Erie Company produces one product, the E4501. The standards for E4501 include the use of 25 yards of raw material at a standard price of $4.42 per yard. During a recent month, the company used 65,000 yards of raw material to produce 2,580 units of E4501. Erie had purchased this material at a cost of $4.37 per yard. Calculate the material quantity variance.

Selected Answer: $2,210 unfavorable

Correct Answer: $2,210 unfavorable

Question 9 1.5 out of 1.5 points

Which of the following is not one of the questions you should ask when faced with an ethical dilemma?

Selected Answer: Do I think I will get caught? Correct Answer: Do I think I will get caught?

Question 10 1.5 out of 1.5 points

Which of the following costs is part of manufacturing overhead?

Page 475: Microeconomics Study Guide

Selected Answer: Indirect labor

Correct Answer: Indirect labor

Question 11 1.5 out of 1.5 points

Product costs

Selected Answer: All of the above answers are correct.

Correct Answer: All of the above answers are correct.

Question 12 1.5 out of 1.5 points

Which of the following is not a period cost?

Selected Answer: Direct materials

Correct Answer: Direct materials

Question 13 1.5 out of 1.5 points

Which of the following companies is most likely to use a process costing system?

Selected Answer: A company that produces petroleum products Correct Answer: A company that produces petroleum products

Question 14 1.5 out of 1.5 points

An allocation base is

Selected Answer: a common characteristic that jobs share, which is used to spread the overhead costs among the various jobs. Correct Answer: a common characteristic that jobs share, which is used to spread the overhead costs among the various jobs.

Question 15 0 out of 1.5 points

Page 476: Microeconomics Study Guide

Which of the following statements about job-order costing is not true?

Selected Answer: Manufacturing overhead cannot be traced directly to jobs, so it is assigned using the overhead allocation rate. Correct Answer: Indirect labor is traced to jobs using time tickets.

Question 16 1.5 out of 1.5 points

Which of the following is the most reasonable allocation base for a highly mechanized process?

Selected Answer: machine hours

Correct Answer: machine hours

Question 17 1.5 out of 1.5 points

A predetermined overhead rate is preferred over an actual overhead rate because a predetermined overhead rate

Selected Answer: Both A and C Correct Answer: Both A and C

Question 18 1.5 out of 1.5 points

The balance in the Finished Goods Inventory account on July 1, 2004, was $34,000 and the June 30, 2005, balance in the Finished Goods Inventory account was $41,000. If the cost of goods manufactured was $200,000, what was the cost of goods sold?

Selected Answer: $193,000

Correct Answer: $193,000

Question 19 0 out of 1.5 points

Black Company's Work in Process Inventory account has a beginning balance of $40,000 and an ending balance of $50,000. Current manufacturing costs total $125,000. What is the cost of goods manufactured?

Selected Answer: $125,000

Correct Answer: $115,000

Page 477: Microeconomics Study Guide

Question 20 0 out of 1.5 points

Emerald Company estimates that the total overhead costs for 2004 will be $150,000 and that the production employees will work 30,000 direct labor hours and earn $600,000 during the year. If the company allocates overhead based on direct labor hours, what is the predetermined overhead rate?

Selected Answer: $20.00 per direct labor hour

Correct Answer: $5.00 per direct labor hour

Question 21 1.5 out of 1.5 points

Companies that use process costing systems

Selected Answer: All of the above answers are correct.

Correct Answer: All of the above answers are correct.

Question 22 1.5 out of 1.5 points

The type of costing system commonly used by companies that produce a large number of homogeneous units in a continuous production process is called a

Selected Answer: process costing system.

Correct Answer: process costing system.

Question 23 0 out of 1.5 points

The equivalent units are calculated by:

Selected Answer: taking the units started plus the equivalent units in ending inventory. Correct Answer: taking the units completed plus the equivalent units in ending inventory.

Question 24 1.5 out of 1.5 points

Page 478: Microeconomics Study Guide

In a process costing system, when items are sold, the cost of the items is moved from

Selected Answer: Finished Goods to Cost of Goods Sold. Correct Answer: Finished Goods to Cost of Goods Sold.

Question 25 1.5 out of 1.5 points

Which of the following sequences describes the typical flow of costs in a company that is using process costing?

Selected Answer: Work in Process, Finished Goods, Raw Materials Inventory, Cost of Goods Sold Correct Answer: Work in Process, Finished Goods, Raw Materials Inventory, Cost of Goods Sold

Question 26 1.5 out of 1.5 points

When partially completed units are converted to a comparable number of completed units, they are referred to as

Selected Answer: equivalent units.

Correct Answer: equivalent units.

Question 27 1.5 out of 1.5 points

A production cost report

Selected Answer: provides a reconciliation of units and a reconciliation of costs as well as the details of the cost per equivalent unit calculation. Correct Answer: provides a reconciliation of units and a reconciliation of costs as well as the details of the cost per equivalent unit calculation.

Question 28 1.5 out of 1.5 points

If beginning inventory consisted of 3,500 units, ending inventory was 1,500 units, and 9,000 units were started during the period, how many units were completed and transferred out of the department?

Selected Answer: 11,000 units

Correct Answer: 11,000 units

Question 29 1.5 out of 1.5 points

Page 479: Microeconomics Study Guide

If ending inventory consists of 2,600 units which are 100% complete with respect to materials and 30% complete with respect to conversion costs, the number of equivalent units for materials and for conversion costs are

Selected Answer: 2,600 and 780, respectively.

Correct Answer: 2,600 and 780, respectively.

Question 30 1.5 out of 1.5 points

Which of the following is most likely to be a variable cost?

Selected Answer: direct materials

Correct Answer: direct materials

Question 31 1.5 out of 1.5 points

When the level of activity increases, total variable costs

Selected Answer: Increases in direct proportion to the increase in activity Correct Answer: Increases in direct proportion to the increase in activity

Question 32 1.5 out of 1.5 points

Which of the following statements is correct?

Selected Answer: Profit is equal to revenue minus total variable costs minus total fixed costs. Correct Answer: Profit is equal to revenue minus total variable costs minus total fixed costs.

Question 33 1.5 out of 1.5 points

The number of units that must be sold in order for a company to exactly cover its fixed and variable costs is the

Selected Answer: break-even point.

Correct Answer: break-even point.

Question 34

Page 480: Microeconomics Study Guide

1.5 out of 1.5 points

Marro Manufacturing is operating at its break-even point of 10,000 units. Which of the following statements is not true?

Selected Answer: Marro's fixed costs equal its variable costs.

Correct Answer: Marro's fixed costs equal its variable costs.

Question 35 1.5 out of 1.5 points

Holding all other factors constant, the break-even point will be decreased by

Selected Answer: increasing the selling price.

Correct Answer: increasing the selling price.

Question 36 0 out of 1.5 points

Which of the following is not true for a firm with high operating leverage?

Selected Answer: It has a larger contribution margin ratio than similar firms. Correct Answer: If sales increase, its profits will increase slower than a company with lower operating leverage.

Question 37 1.5 out of 1.5 points

When considering a process that involves a resource constraint, the optimal decision

Selected Answer: maximizes the contribution margin per unit of the constraint. Correct Answer: maximizes the contribution margin per unit of the constraint.

Question 38 1.5 out of 1.5 points

Fixed costs are $20,570. At the break-even point, variable costs are $1.63 per unit and the sales price is $3.50 per unit. What is the contribution margin per unit?

Selected Answer: $1.87

Correct Answer: $1.87

Page 481: Microeconomics Study Guide

Question 39 1.5 out of 1.5 points

Copper Corporation sells a single product at a price of $275 per unit. Variable cost per unit is $135 and fixed costs total $356,860. If sales are expected to be $825,000, what is Cooper's margin of safety?

Selected Answer: $124,025

Correct Answer: $124,025

Question 40 1.5 out of 1.5 points

Anchor Company produces a single product that is sold for $85 per unit. If variable costs per unit are $26 and fixed costs total $47,500, how many units must Anchor sell in order to earn a profit of $100,000?

Selected Answer: 2,500

Correct Answer: 2,500

EXAM 2Question 1 1.5 out of 1.5 points

Which of the following is treated differently in full costing than in variable costing?

Selected Answer: fixed manufacturing overhead Correct Answer: fixed manufacturing overhead

Question 2 1.5 out of 1.5 points

Which of the following is treated as a product cost in variable costing?

Selected Answer: direct labor

Correct Answer: direct labor

Question 3 1.5 out of 1.5 points

Full costing is:

Page 482: Microeconomics Study Guide

Selected Answer: required for financial reporting under generally accepted accounting principles. Correct Answer: required for financial reporting under generally accepted accounting principles.

Question 4 1.5 out of 1.5 points

In variable costing, when does fixed manufacturing overhead become an expense?

Selected Answer: in the period when the expense is incurred

Correct Answer: in the period when the expense is incurred

Question 5 1.5 out of 1.5 points

In variable costing, which of the following would not be included in inventory?

Selected Answer: none of the above items would be in inventory in variable costing Correct Answer: none of the above items would be in inventory in variable costing

Question 6 1.5 out of 1.5 points

Which of the following items appears on a variable costing income statement but not on a full costing income statement?

Selected Answer: contribution margin

Correct Answer: contribution margin

Question 7 1.5 out of 1.5 points

When the number of units sold is equal to the number of units produced, net income using full costing will be

Selected Answer: equal to net income using variable costing.

Correct Answer: equal to net income using variable costing.

Question 8 1.5 out of 1.5 points

Page 483: Microeconomics Study Guide

The process of assigning indirect costs is called

Selected Answer: cost allocation.

Correct Answer: cost allocation.

Question 9 1.5 out of 1.5 points

A contract which specifies that the suppler will be paid for the cost of production as well as some fixed amount or percentage of cost is called a(n)

Selected Answer: cost-plus contract.

Correct Answer: cost-plus contract.

Question 10 1.5 out of 1.5 points

A grouping of individual costs whose total is allocated using one allocation base is called

Selected Answer: cost pool.

Correct Answer: cost pool.

Question 11 1.5 out of 1.5 points

Companies which use only one or two cost pools rather than several cost pools

Selected Answer: may have seriously distorted product costs. Correct Answer: may have seriously distorted product costs.

Question 12 1.5 out of 1.5 points

When activity based costing is implemented, the initial outcome is normally that:

Selected Answer: The cost of low volume products will be higher and the cost of high volume products will be lower Correct Answer: The cost of low volume products will be higher and the cost of high volume products will be lower

Question 13 1.5 out of 1.5 points

Page 484: Microeconomics Study Guide

Cool Company uses ABC costing. Which of the following is most likely to be the cost driver for the cost of ordering parts?

Selected Answer: number of orders placed

Correct Answer: number of orders placed

Question 14 1.5 out of 1.5 points

Which of the following is not generally true when a company compares ABC and traditional costing?

Selected Answer: ABC allocates cost based solely on production volume. Correct Answer: ABC allocates cost based solely on production volume.

Question 15 1.5 out of 1.5 points

The production departments at Cook Corporation occupy a total area of 600,000 square feet. Heating costs total $900,000 and are allocated based on the area that each department occupies. The finishing department occupies 30,000 square feet and the packaging department occupies 20,000 square feet. What amount of heating cost will be allocated to the finishing and packaging departments, respectively?

Selected Answer: $45,000 and $30,000 Correct Answer: $45,000 and $30,000

Question 16 1.5 out of 1.5 points

Maintenance cost is allocated to the three producing departments based on the machine hours used in each department. The maintenance cost for May was $100,000. The three departments had the following usage for May:Department Machine hours used Direct labor hours usedAssembly 500 2,000Fabrication 800 5,000Testing 700 3,000

The maintenance cost that should be allocated to the Fabrication Department for May should be:

Selected Answer: $40,000

Correct Answer: $40,000

Question 17 1.5 out of 1.5 points

Page 485: Microeconomics Study Guide

Which of the following is likely to occur when more overhead cost pools are used?

Selected Answer: All of the above are true.

Correct Answer: All of the above are true.

Question 18 1.5 out of 1.5 points

Which of the following is not a term used to describe the additional costs incurred as a result of selecting one decision alternative over another?

Selected Answer: sunk costs

Correct Answer: sunk costs

Question 19 1.5 out of 1.5 points

A company is trying to decide whether to sell partially completed goods in their current state or incur additional costs to finish the goods and sell them as complete units. Which of the following is not relevant to the decision?

Selected Answer: the costs incurred to process the units to this point Correct Answer: the costs incurred to process the units to this point

Question 20 1.5 out of 1.5 points

A company is trying to decide whether to keep or drop the sporting goods department in its department store. If the segment is dropped, the manager will be fired. The manager's salary, in relation to the decision to keep or drop the sporting goods department, is

Selected Answer: avoidable and therefore relevant

Correct Answer: avoidable and therefore relevant

Question 21 1.5 out of 1.5 points

The value of benefits foregone by selecting one decision alternative over another is a(n)

Selected Answer: opportunity cost.

Page 486: Microeconomics Study Guide

Correct Answer: opportunity cost.

Question 22 0 out of 1.5 points

A product line should be dropped when

Selected Answer: there will be a positive change in income if the product line is dropped. Correct Answer: all of the above.

Question 23 1.5 out of 1.5 points

Which of the following is a direct cost of a specific department in a retail store?

Selected Answer: cost of the department's inventory Correct Answer: cost of the department's inventory

Question 24 1.5 out of 1.5 points

Two or more products that result from common inputs are called

Selected Answer: joint products. Correct Answer: joint products.

Question 25 1.5 out of 1.5 points

Oster Medical Supply Company is trying to decide whether or not to continue distributing hospital supplies. The following information is available for Oster's business segments. Assume that all direct fixed costs could be avoided if a segment is dropped and that the total common fixed costs would remain unchanged if a segment is dropped.Hospital Supplies Retail Stores Mail OrderSales $120,000 $440,000 $360,000Variable Costs $64,000 $200,000 $140,000Contribution Margin $56,000 $240,000 $220,000Direct Fixed Costs $50,000 $80,000 $90,000Allocated Common Fixed Costs $20,000 $70,000 $60,000Net Income ($14,000) $90,000 $70,000

Reference: Ref 7-4

If hospital supplies are dropped, overall net income would:

Selected Answer: Decrease by $6,000

Page 487: Microeconomics Study Guide

Correct Answer: Decrease by $6,000

Question 26 1.5 out of 1.5 points

Ritz Company makes two products from a joint input that have the following information:

Units Produced Sales Value per unit at split off Total Additional processing costs Sales value per unit after additional processingProduct A 50,000 $10 $400,000 $15Product B 30,000 $8 $300,000 $20

The joint cost incurred to produce the two products to the split off point is $600,000.Reference: Ref 7-7

Which products should be processed further?

Selected Answer: B

Correct Answer: B

Question 27 1.5 out of 1.5 points

Which of the following would not appear in a capital budget?

Selected Answer: purchase of inventory to be sold next month Correct Answer: purchase of inventory to be sold next month

Question 28 1.5 out of 1.5 points

Capital expenditure decisions

Selected Answer: All of the above are correct.

Correct Answer: All of the above are correct.

Question 29 1.5 out of 1.5 points

The basic concept involved in time value of money calculations is that

Selected Answer: it is better to receive a dollar today than to receive a dollar in the future.

Page 488: Microeconomics Study Guide

Correct Answer: it is better to receive a dollar today than to receive a dollar in the future.

Question 30 1.5 out of 1.5 points

Which of the following techniques uses time value of money concepts?

Selected Answer: internal rate of return

Correct Answer: internal rate of return

Question 31 0 out of 1.5 points

Which of the following would most likely be the present value of a 5 year annuity of $5,000 per year (assuming a positive discount rate)?

Selected Answer: $25,000

Correct Answer: $18,000

Question 32 0 out of 1.5 points

Assuming a 10% rate of return, how does the present value of an amount to be received two years from today compare to the present value of the same amount to be received three years from today?

Selected Answer: It is impossible to tell unless the actual amount is known. Correct Answer: The present value of the amount to be received in two years is larger.

Question 33 1.5 out of 1.5 points

The sum of the present values of all cash flows (inflows and outflows) is called the

Selected Answer: net present value.

Correct Answer: net present value.

Question 34 1.5 out of 1.5 points

Projects with a negative net present value will always have a(n)

Page 489: Microeconomics Study Guide

Selected Answer: internal rate of return that is less than the required rate of return. Correct Answer: internal rate of return that is less than the required rate of return.

Question 35 1.5 out of 1.5 points

An annuity is

Selected Answer: a series of equal payments.

Correct Answer: a series of equal payments.

Question 36 1.5 out of 1.5 points

The rate of return that equates the present value of future cash flows to the investment outlay is the

Selected Answer: internal rate of return.

Correct Answer: internal rate of return.

Question 37 0 out of 1.5 points

Projects should be accepted when:

Selected Answer: The net present value is positive

Correct Answer: Both A and B

Question 38 1.5 out of 1.5 points

If the internal rate of return is greater than the required rate of return

Selected Answer: the project should be accepted. Correct Answer: the project should be accepted.

Question 39 1.5 out of 1.5 points

Which of the following statements about the payback period method is not true?

Page 490: Microeconomics Study Guide

Selected Answer: The payback period method is more sophisticated and yields better decisions than the internal rate of return method. Correct Answer: The payback period method is more sophisticated and yields better decisions than the internal rate of return method.

Question 40 0 out of 1.5 points

Calculate the present value of $216,000 to be received in seven years if the discount rate is 16%.

Selected Answer: $73,159

Correct Answer: $76,421

EXAM 3Question 1 1.5 out of 1.5 points

A budget

Selected Answer: All of the above are true.

Correct Answer: All of the above are true.

Question 2 1.5 out of 1.5 points

A method of budget preparation that requires all budgeted amounts to be justified by the department, even if the amounts were supported in prior periods, is called

Selected Answer: zero base budgeting.

Correct Answer: zero base budgeting.

Question 3 1.5 out of 1.5 points

The comprehensive planning document that incorporates a number of individual budgets is the

Selected Answer: master budget. Correct Answer: master budget.

Page 491: Microeconomics Study Guide

Question 4 1.5 out of 1.5 points

Which of the following is a reasonable order in which to prepare budgets?

Selected Answer: sales budget, production budget, material purchases budget Correct Answer: sales budget, production budget, material purchases budget

Question 5 1.5 out of 1.5 points

If the number of units in beginning inventory is more than the number of units in ending inventory, the number of units sold is

Selected Answer: greater than the number of units produced. Correct Answer: greater than the number of units produced.

Question 6 1.5 out of 1.5 points

When constructing the production budget, the desired ending inventory for the first period is

Selected Answer: the beginning inventory for the second period. Correct Answer: the beginning inventory for the second period.

Question 7 1.5 out of 1.5 points

A significant difference between the direct material purchases budget and the direct labor budget is that the direct material purchases budget

Selected Answer: considers beginning and ending inventory amounts, which are not part of a direct labor budget. Correct Answer: considers beginning and ending inventory amounts, which are not part of a direct labor budget.

Question 8 1.5 out of 1.5 points

Which of the following transactions will affect the cash receipts and disbursements budget for the month in which the transaction occurs?

Selected Answer: payment for direct labor

Correct Answer: payment for direct labor

Question 9

Page 492: Microeconomics Study Guide

1.5 out of 1.5 points

The cash receipts and disbursements budget alerts management to all of the following except when

Selected Answer: stockouts will cause customer dissatisfaction.

Correct Answer: stockouts will cause customer dissatisfaction.

Question 10 1.5 out of 1.5 points

When budgets are used for evaluation, the differences between budgeted and actual amounts are called

Selected Answer: budget variances.

Correct Answer: budget variances.

Question 11 1.5 out of 1.5 points

The main difference between a static budget and a flexible budget is that the static budget is

Selected Answer: for a single level of activity while a flexible budget can be adjusted for different activity levels. Correct Answer: for a single level of activity while a flexible budget can be adjusted for different activity levels.

Question 12 1.5 out of 1.5 points

A flexible budget takes into account the fact that when production levels change

Selected Answer: total fixed costs remain the same.

Correct Answer: total fixed costs remain the same.

Question 13 1.5 out of 1.5 points

Knight Company has found that 30% of its sales are collected in the month of the sale and the remainder of the sales is collected in the next month. If sales are expected to be $100,000 in April, $120,000 in May, and $80,000 in June, what is the estimated amount of cash receipts for May?

Selected Answer: $106,000

Page 493: Microeconomics Study Guide

Correct Answer: $106,000

Question 14 1.5 out of 1.5 points

The standard cost is

Selected Answer: the cost that should have been incurred to produce the item. Correct Answer: the cost that should have been incurred to produce the item.

Question 15 1.5 out of 1.5 points

The relationship between standard costs and budgeted costs is the

Selected Answer: standard cost times the expected production level equals the budgeted cost. Correct Answer: standard cost times the expected production level equals the budgeted cost.

Question 16 1.5 out of 1.5 points

Standards that are developed under the assumption that a variety of things may lead to less than perfect performance are

Selected Answer: attainable standards. Correct Answer: attainable standards.

Question 17 1.5 out of 1.5 points

Variances are usually analyzed for

Selected Answer: direct materials, direct labor, and manufacturing overhead. Correct Answer: direct materials, direct labor, and manufacturing overhead.

Question 18 1.5 out of 1.5 points

If the actual price per unit of material is greater than the standard price, the material price variance will be

Selected Answer: unfavorable.

Correct Answer: unfavorable.

Page 494: Microeconomics Study Guide

Question 19 1.5 out of 1.5 points

If the material quantity variance is favorable, the

Selected Answer: production manager has used materials efficiently. Correct Answer: production manager has used materials efficiently.

Question 20 1.5 out of 1.5 points

Which of the following would cause a variance to be unfavorable?

Selected Answer: the standard hours allowed are less than the actual hours worked Correct Answer: the standard hours allowed are less than the actual hours worked

Question 21 0 out of 1.5 points

A possible cause for an unfavorable labor rate variance is

Selected Answer: hiring new, inexperienced employees.

Correct Answer: using more experienced workers than planned.

Question 22 1.5 out of 1.5 points

Erie Company produces one product, the E4501. The standards for E4501 include the use of 25 yards of raw material at a standard price of $4.42 per yard. During a recent month, the company used 65,000 yards of raw material to produce 2,580 units of E4501. Erie had purchased this material at a cost of $4.37 per yard. Calculate the material quantity variance.

Selected Answer: $2,210 unfavorable

Correct Answer: $2,210 unfavorable

Chapter 5Question 1 1 out of 1 points

Full costing

Page 495: Microeconomics Study Guide

Selected Answer: All of the above choices are correct.

Correct Answer: All of the above choices are correct.

Question 2 1 out of 1 points

Which of the following is treated differently in full costing than in variable costing?

Selected Answer: fixed manufacturing overhead Correct Answer: fixed manufacturing overhead

Question 3 1 out of 1 points

Which of the following is treated as a product cost in variable costing?

Selected Answer: direct labor

Correct Answer: direct labor

Question 4 1 out of 1 points

Full costing is:

Selected Answer: required for financial reporting under generally accepted accounting principles. Correct Answer: required for financial reporting under generally accepted accounting principles.

Question 5 1 out of 1 points

In variable costing, when does fixed manufacturing overhead become an expense?

Selected Answer: in the period when the expense is incurred

Correct Answer: in the period when the expense is incurred

Question 6 1 out of 1 points

In full costing, when does fixed manufacturing overhead become an expense?

Page 496: Microeconomics Study Guide

Selected Answer: in the period when the product is sold Correct Answer: in the period when the product is sold

Question 7 1 out of 1 points

If the number of units sold is less than the number of units produced

Selected Answer: full costing will assign some fixed manufacturing costs to the units in ending inventory. Correct Answer: full costing will assign some fixed manufacturing costs to the units in ending inventory.

Question 8 1 out of 1 points

Last month, Active Company manufactured 20,000 units and sold 18,000 of these units at a price of $8.00 per unit. Manufacturing costs consisted of direct labor, $30,000; direct materials, $32,000; variable manufacturing overhead, $3,600; fixed manufacturing overhead, $21,600. Selling and administrative costs totaled $24,000.Reference: Ref 5-1

What is Active's net income using variable costing?

Selected Answer: $39,360

Correct Answer: $39,360

Question 9 1 out of 1 points

Last month, Active Company manufactured 20,000 units and sold 18,000 of these units at a price of $8.00 per unit. Manufacturing costs consisted of direct labor, $30,000; direct materials, $32,000; variable manufacturing overhead, $3,600; fixed manufacturing overhead, $21,600. Selling and administrative costs totaled $24,000.Reference: Ref 5-1

What is Active's net income using full costing?

Selected Answer: $41,520

Correct Answer: $41,520

Question 10 0 out of 1 points

Slinky Manufacturing makes a single product - the Slinky. Information for 2005 appears below:Sales in units: 200,000

Page 497: Microeconomics Study Guide

Production in units: 250,000Beginning inventory 0Variable production cost per unit: $1.00Variable selling cost per unit: $0.30Fixed production cost per year $100,000Fixed selling and administrative cost per year $50,000Selling price per unit $3.00

Reference: Ref 5-5

What is the cost per unit of inventory using variable costing?

Selected Answer: $1.30

Correct Answer: $1.00

Question 11 0 out of 1 points

Slinky Manufacturing makes a single product - the Slinky. Information for 2005 appears below:Sales in units: 200,000Production in units: 250,000Beginning inventory 0Variable production cost per unit: $1.00Variable selling cost per unit: $0.30Fixed production cost per year $100,000Fixed selling and administrative cost per year $50,000Selling price per unit $3.00

Reference: Ref 5-5

What is the full cost per unit of inventory?

Selected Answer: $1.70

Correct Answer: $1.40

Chapter 6Question 1 1 out of 1 points

The process of assigning indirect costs is called

Selected Answer: cost allocation.

Page 498: Microeconomics Study Guide

Question 2 1 out of 1 points

A contract which specifies that the suppler will be paid for the cost of production as well as some fixed amount or percentage of cost is called a(n)

Selected Answer: cost-plus contract.

Question 3 1 out of 1 points

The thing for which a cost is being calculated is a(n)

Selected Answer: cost object

Question 4 1 out of 1 points

An allocation base

Selected Answer: relates the cost pool to the cost objectives.

Question 5 1 out of 1 points

Companies which use only one or two cost pools rather than several cost pools

Selected Answer: may have seriously distorted product costs.

Question 6 1 out of 1 points

When activity based costing is implemented, the initial outcome is normally that:

Selected Answer: The cost of low volume products will be higher and the cost of high volume products will be lower

Question 7 1 out of 1 points

Cool Company uses ABC costing. Which of the following is most likely to be the cost driver for the cost of ordering parts?

Page 499: Microeconomics Study Guide

Selected Answer: number of orders placed

Question 8 1 out of 1 points

A grouping of individual costs whose total is allocated using one allocation base is called

Selected Answer: cost pool.

Question 9 1 out of 1 points

Pumper Company sells fine collectible statues and has implemented activity-based costing. Costs in the shipping department have been divided into three cost pools. The first cost pool contains costs that are related to packaging and shipping and Pumper has determined that the number of boxes shipped is an appropriate cost driver for these costs. The second cost pool is made up of costs related to the final inspection of each item before it is shipped and the cost driver for this pool is the number of individual items that are inspected and shipped. The final cost pool is used for general operations and supervision of the department and the cost driver is the number of shipments.

Information about the department is summarized below:Cost Pool Total Costs Cost Driver Annual ActivityPackaging and shipping $170,000 Number of boxes shipped 25,000 boxesFinal inspection $200,000 Number of individualitems shipped 100,000 itemsGeneral operations and supervision $85,000 Number of orders 10,000 orders

Reference: Ref 6-6

What amount is allocated to each order for the general operations and supervision of the department?

Selected Answer: $8.50

Question 10 1 out of 1 points

Pumper Company sells fine collectible statues and has implemented activity-based costing. Costs in the shipping department have been divided into three cost pools. The first cost pool contains costs that are related to packaging and shipping and Pumper has determined that the number of boxes shipped is an appropriate cost driver for these costs. The second cost pool is made up of costs related to the final inspection of each item before it is shipped and the cost driver for this pool is the number of individual items that are inspected and shipped. The final cost pool is used for general operations and supervision of the department and the cost driver is the number of shipments.

Information about the department is summarized below:Cost Pool Total Costs Cost Driver Annual Activity

Page 500: Microeconomics Study Guide

Packaging and shipping $170,000 Number of boxes shipped 25,000 boxesFinal inspection $200,000 Number of individualitems shipped 100,000 itemsGeneral operations and supervision $85,000 Number of orders 10,000 orders

Reference: Ref 6-6

An order is shipped to a retail customer who has ordered 10 individual items. This order will be shipped in 3 separate boxes. What is the shipping department cost that will be allocated to the order?

Selected Answer: $48.90

Question 11 1 out of 1 points

Which of the following is not an advantage of Activity Based Costing (ABC) over traditional volume based costing systems?

Selected Answer: ABC is less costly to implement than traditional systems

Chapter 7Question 1 1 out of 1 points

Which of the following is not a term used to describe the additional costs incurred as a result of selecting one decision alternative over another?

Selected Answer: sunk costs

Correct Answer: sunk costs

Question 2 1 out of 1 points

When deciding between two alternatives, the preferred alternative always has

Selected Answer: greater incremental profit than the other alternatives. Correct Answer: greater incremental profit than the other alternatives.

Question 3 1 out of 1 points

The value of benefits foregone by selecting one decision alternative over another is a(n)

Page 501: Microeconomics Study Guide

Selected Answer: opportunity cost. Correct Answer: opportunity cost.

Question 4 1 out of 1 points

Two or more products that result from common inputs are called

Selected Answer: joint products. Correct Answer: joint products.

Question 5 1 out of 1 points

When making a decision to sell a joint product at the split off point or process it further, which of the following is not relevant?

Selected Answer: Joint cost assigned

Correct Answer: Joint cost assigned

Question 6 1 out of 1 points

The Stale Store has 12,000 cans of tuna fish just a week past the expiration date. Each can cost $0.31. The cans could be sold as is for $0.20 each, or relabeled and sold as gourmet cat food. The cost of relabeling the cans would be $0.04 per can and the cans would then sell for $0.29 per can. What should be done with the cans and why?

Selected Answer: The cans should be relabeled into cat food since the sales price increases $0.09 per can and the cost is only $0.04 per can Correct Answer: The cans should be relabeled into cat food since the sales price increases $0.09 per can and the cost is only $0.04 per can

Question 7 1 out of 1 points

Smith Company manufactures widgets. Newman Company has approached Smith with a proposal to sell the company one of the components used to make widgets at a price of $100,000 for 50,000 units. Smith is currently making these components in its own factory. The following costs are associated with this part of the process when 50,000 units are produced:Direct material $44,000Direct labor 20,000Manufacturing overhead 60,000Total $124,000

Page 502: Microeconomics Study Guide

The manufacturing overhead consists of $32,000 of costs that will be eliminated if the components are no longer produced by Smith. The remaining manufacturing overhead will continue whether or not Smith makes the components. Answer the following questions from Smith's point of view.Reference: Ref 7-2

Should Smith make or buy the components for the widgets?

Selected Answer: Continue to make them because the incremental cost of buying from Newman is $4,000. Correct Answer: Continue to make them because the incremental cost of buying from Newman is $4,000.

Question 8 1 out of 1 points

Huxley Sports Company sells logo sports merchandise and does custom screen printing. They are trying to decide whether or not to continue screen printing. The following information is available for the segments. Assume that all direct fixed costs could be avoided if a segment is dropped and that the total common fixed costs would remain unchanged if the screen printing were dropped.Screen Printing Apparel SalesSales $120,000 $420,000Variable Costs $72,000 $220,000Contribution Margin $48,000 $200,000Direct Fixed Costs $32,000 $70,000Allocated Common Fixed costs $20,000 $70,000Net Income ($4,000) $60,000

Reference: Ref 7-3

If screen printing is dropped, apparel sales will be unaffected. All direct fixed costs associated with screen printing will be eliminated. What will the effect on overall profits be if the segment is eliminated?

Selected Answer: Overall profits will decrease by $16,000

Correct Answer: Overall profits will decrease by $16,000

Question 9 0 out of 1 points

The raw material, Grover, is processed into two joint products, Bert and Ernie. When processed, each pound of Grover yields six units of Bert and eight units of Ernie. Bert sells for $3 per unit and Ernie sells for $2.50 per unit. The total cost to process a 10-pound batch of Grover is $280.Reference: Ref 7-6

If the physical quantities method is used to allocate the $280 in joint costs, how much will be allocated to Bert?

Selected Answer: $180

Page 503: Microeconomics Study Guide

Correct Answer: $120

Question 10 1 out of 1 points

The raw material, Grover, is processed into two joint products, Bert and Ernie. When processed, each pound of Grover yields six units of Bert and eight units of Ernie. Bert sells for $3 per unit and Ernie sells for $2.50 per unit. The total cost to process a 10-pound batch of Grover is $280.Reference: Ref 7-6

If the relative sales value method is used to allocate the joint costs, how much will be allocated to Bert?

Selected Answer: $132.63

Correct Answer: $132.63

Chapter 9Question 1 1 out of 1 points

Capital expenditure decisions

Selected Answer: All of the above are correct.

Correct Answer: All of the above are correct.

Question 2 1 out of 1 points

Which of the following techniques uses time value of money concepts?

Selected Answer: internal rate of return

Correct Answer: internal rate of return

Question 3 1 out of 1 points

The sum of the present values of all cash flows (inflows and outflows) is called the

Selected Answer: net present value.

Page 504: Microeconomics Study Guide

Correct Answer: net present value.

Question 4 1 out of 1 points

An annuity is

Selected Answer: a series of equal payments.

Correct Answer: a series of equal payments.

Question 5 1 out of 1 points

The rate of return that equates the present value of future cash flows to the investment outlay is the

Selected Answer: internal rate of return.

Correct Answer: internal rate of return.

Question 6 1 out of 1 points

The length of time it takes to recover the initial cost of an investment is called the

Selected Answer: payback period.

Correct Answer: payback period.

Question 7 0 out of 1 points

You've just won a lottery and will receive $60,000 per year over the next 20 years. If you feel that 10% is an appropriate rate of return, what is the present value of your winnings?

Selected Answer: $1,200,000

Correct Answer: $510,816

Question 8 1 out of 1 points

Page 505: Microeconomics Study Guide

A proposed project will cost $600,000 and will provide returns of $150,000 in Year 1, $300,000 in Year 2, and $500,000 in Year 3. There will not be any cash flows associated with the project after Year 3. If taxes are ignored, and the company's hurdle rate is 15%, what is the net present value of the project?

Selected Answer: $86,020

Correct Answer: $86,020

Question 9 1 out of 1 points

An investment is expected to generate returns of $60,000 per year for each of the next six years. If the initial amount invested is $240,000 and taxes are ignored, which of the following is closest to the internal rate of return?

Selected Answer: 13%

Correct Answer: 13%

Question 10 1 out of 1 points

A project with an initial cost of $80,000 is expected to produce returns of $20,000 per year for each of the next nine years. What is the projected payback period?

Selected Answer: 4 years

Correct Answer: 4 years

Chapter 10Question 1 1 out of 1 points

A budget

Selected Answer: All of the above are true.

Correct Answer: All of the above are true.

Question 2 1 out of 1 points

Page 506: Microeconomics Study Guide

A method of budget preparation that requires all budgeted amounts to be justified by the department, even if the amounts were supported in prior periods, is called

Selected Answer: zero base budgeting.

Correct Answer: zero base budgeting.

Question 3 1 out of 1 points

Which of the following budgets is prepared first?

Selected Answer: sales budget

Correct Answer: sales budget

Question 4 1 out of 1 points

Let Prod = the number of units to be produced; EndInv = the desired ending inventory in units; BeginInv = the number of units in beginning inventory; and Sales = the number of units expected to be sold. Which of the following is true?

Selected Answer: Prod = Sales + EndInv – BeginInv Correct Answer: Prod = Sales + EndInv – BeginInv

Question 5 1 out of 1 points

When budgets are used for evaluation, the differences between budgeted and actual amounts are called

Selected Answer: budget variances.

Correct Answer: budget variances.

Question 6 1 out of 1 points

The main difference between a static budget and a flexible budget is that the static budget is

Selected Answer: for a single level of activity while a flexible budget can be adjusted for different activity levels. Correct Answer: for a single level of activity while a flexible budget can be adjusted for different activity levels.

Question 7

Page 507: Microeconomics Study Guide

1 out of 1 points

Volckmann Company's policy is to keep 25% of the next month's sales in ending inventory. If there are 1,200 units in inventory at the end of March, sales in April are expected to be 4,800 units, and sales in May are expected to be 9,000 units, how many units should be produced in April?

Selected Answer: 5,850

Correct Answer: 5,850

Question 8 1 out of 1 points

Jackie's Jump Ropes is planning to produce 2,000 boxes of jump ropes during June. Each box of jump ropes (1 unit) requires 75 feet of rope and 0.50 hours of direct labor. Rope costs $0.12 per foot and employees of the company are paid $11.00 per hour. Manufacturing overhead is applied at a rate of 150% of direct labor costs. Jackie's has 15,000 feet of rope in beginning inventory and wants to have 20,000 feet of rope in ending inventory.Reference: Ref 10-1

How many feet of rope should Jackie's Jump Ropes plan to buy during June?

Selected Answer: 155,000

Correct Answer: 155,000

Question 9 1 out of 1 points

Jackie's Jump Ropes is planning to produce 2,000 boxes of jump ropes during June. Each box of jump ropes (1 unit) requires 75 feet of rope and 0.50 hours of direct labor. Rope costs $0.12 per foot and employees of the company are paid $11.00 per hour. Manufacturing overhead is applied at a rate of 150% of direct labor costs. Jackie's has 15,000 feet of rope in beginning inventory and wants to have 20,000 feet of rope in ending inventory.Reference: Ref 10-1

What is the total amount that will be budgeted for direct labor for the month?

Selected Answer: $11,000

Correct Answer: $11,000

Question 10 1 out of 1 points

Page 508: Microeconomics Study Guide

In recent years, Queens Company has collected 20% of its sales in the month of the sale and the other 80% in the following month. During the first three months of 2005, Princess is anticipating sales of $350,000; $403,000; and $389,000, respectively. What is the amount of cash receipts budgeted for February?

Selected Answer: $360,600

Correct Answer: $360,600

Question 11 1 out of 1 points

GW Corporation's expected manufacturing costs are summarized below:Variable costsDirect material $16.30 per unitDirect labor $15.80 per unitVariable overhead $12.00 per unitFixed costs per month Factory depreciation $12,000Supervisory salaries 16,800Other fixed factory costs 2,500Reference: Ref 10-7

What is the flexible budget amount for a month when 6,000 units are produced?

Selected Answer: $295,900

Correct Answer: $295,900

Chapter 11Question 1 1 out of 1 points

The difference between standard and actual costs is a(n)

Selected Answer: standard cost variance. Correct Answer: standard cost variance.

Question 2 1 out of 1 points

The difference between standard costs and budgeted costs is that standard costs

Selected Answer: refer to a single unit while budgeted costs refer to the cost, at standard, for the total number of budgeted units.

Page 509: Microeconomics Study Guide

Correct Answer: refer to a single unit while budgeted costs refer to the cost, at standard, for the total number of budgeted units.

Question 3 1 out of 1 points

Standard costs are developed for all of the following except

Selected Answer: commissions per unit.

Correct Answer: commissions per unit.

Question 4 1 out of 1 points

In general, an unfavorable material variances arise from

Selected Answer: Both A and B are correct. Correct Answer: Both A and B are correct.

Question 5 1 out of 1 points

The material price variance uses the quantity of material _____, while the material quantity variance uses the quantity of material _____.

Selected Answer: purchased, used Correct Answer: purchased, used

Question 6 1 out of 1 points

The overhead volume variance indicates that

Selected Answer: the quantity of production differed from what was anticipated. Correct Answer: the quantity of production differed from what was anticipated.

Question 7 1 out of 1 points

Green Company manufactures a single product, the AZQR. The standards for materials for each unit have been set as 5 pounds of KR-27 at a standard cost of $35.00 per pound. During May, the company purchased and used 512 pounds of KR-27 to make 100 units of the AZQR. Green paid $33.00 per pound for the material. What is the material price variance?

Page 510: Microeconomics Study Guide

Selected Answer: $1,024 favorable

Correct Answer: $1,024 favorable

Question 8 1 out of 1 points

Huron Company produces one product, the H2001. Each unit of H2001 requires 6.5 pounds of raw material with a standard cost of $12.00 per pound. During July, Huron purchased 3,500 pounds of this raw material at a price of $12.25 per pound and used 3,280 pounds to produce 500 units of the H2001. Calculate the material quantity variance.

Selected Answer: $360 unfavorable

Correct Answer: $360 unfavorable

Question 9 1 out of 1 points

At Huron Company, the standard time required to assemble one transmitter is 2.30 hours. The standard wage is $9.60 per hour. During June, employees at Huron produced 860 transmitters and worked 1,935 labor hours at a total cost of $18,963. What is Huron's labor rate variance?

Selected Answer: $387.00 unfavorable

Correct Answer: $387.00 unfavorable

Question 10 1 out of 1 points

Northern Lights Company has a standard wage rate of $9.50 per hour. It has determined that the standard time to assemble one versascope is 2.75 hours. During August, Northern Lights employees assembled 600 versascopes. They were paid $15,974 for 1,630 hours of work. What is Northern Light's labor efficiency variance?

Selected Answer: $190.00 favorable

Correct Answer: $190.00 favorable

Chapter 12Question 1 1 out of 1 points

Firms that grant substantial decision making authority to the managers of subunits are referred to as

Page 511: Microeconomics Study Guide

Selected Answer: decentralized organizations.

Correct Answer: decentralized organizations.

Question 2 1 out of 1 points

Which of the following is not an advantage of decentralization for a company?

Selected Answer: subunit managers will act to benefit the organization as a whole Correct Answer: subunit managers will act to benefit the organization as a whole

Question 3 1 out of 1 points

Goal congruence refers to the match between

Selected Answer: goals of the individual managers and those of the company as a whole. Correct Answer: goals of the individual managers and those of the company as a whole.

Question 4 1 out of 1 points

The type of center that has responsibility for generating revenue as well as controlling costs is a(n)

Selected Answer: profit center.

Correct Answer: profit center.

Question 5 1 out of 1 points

Which of the following is a responsibility that distinguishes an investment center manager from a profit center manager?

Selected Answer: significantly influencing investment decisions

Correct Answer: significantly influencing investment decisions

Question 6 1 out of 1 points

Page 512: Microeconomics Study Guide

ROI is the ratio of

Selected Answer: investment center income to invested capital.

Correct Answer: investment center income to invested capital.

Question 7 1 out of 1 points

On a balanced scorecard, the measure of the number of new patents developed through research and development would be an example of a measure in the:

Selected Answer: learning and growth dimension

Correct Answer: learning and growth dimension

Question 8 1 out of 1 points

Fruit Company's Lime Division has invested capital of $900,000, sales of $750,000, and income (NOPAT) of $60,000. What is the division's turnover?

Selected Answer: 0.83

Correct Answer: 0.83

Question 9 1 out of 1 points

ROI can be improved by

Selected Answer: Both A and B are correct. Correct Answer: Both A and B are correct.

Question 10 1 out of 1 points

The magazine division of KRG Company has invested capital of $1,250,000. Sales of $1,000,000 generated a income (NOPAT) of $100,000. What is the division's ROI?

Selected Answer: 8%

Correct Answer: 8%

Page 513: Microeconomics Study Guide

Question 11 1 out of 1 points

Cat Food Company has two divisions, Turkey and Fish. Operating results for the two divisions are as follows:Turkey FishNOPAT $50,000 $60,000Investment $250,000 $400,000

The required rate of return for the Cat Food Company is 18%.Reference: Ref 12-4

What is the residual income for the Fish Division?

Selected Answer: ($12,000)

Correct Answer: ($12,000)

Chapter 14Question 1 1 out of 1 points

Horizontal analysis is

Selected Answer: consists of analyzing changes in financial statements across time. Correct Answer: consists of analyzing changes in financial statements across time.

Question 2 1 out of 1 points

Which type of analysis would highlight the percentage increase in sales from one year to the next?

Selected Answer: Horizontal analysis

Correct Answer: Horizontal analysis

Question 3 1 out of 1 points

Which ratio measures the rate earned on the total capital used by a firm?

Selected Answer: Return on total assets Correct Answer: Return on total assets

Page 514: Microeconomics Study Guide

Question 4 0 out of 1 points

The gross margin

Selected Answer: is the same as operating income.

Correct Answer: is sales less the cost of the inventory that generated those sales.

Question 5 1 out of 1 points

Asset turnover is a measure of

Selected Answer: the overall efficiency with which the company uses assets to generate revenues. Correct Answer: the overall efficiency with which the company uses assets to generate revenues.

Question 6 0 out of 1 points

The higher the accounts receivable turnover,

Selected Answer: neither A nor B

Correct Answer: both A and B

Question 7 1 out of 1 points

If a company has an inventory turnover of 12, that means that

Selected Answer: it has about one month's sales in inventory.

Correct Answer: it has about one month's sales in inventory.

Question 8 1 out of 1 points

The higher the debt to equity ratio, the higher the level of

Selected Answer: risk.

Correct Answer: risk.

Page 515: Microeconomics Study Guide

Question 9 1 out of 1 points

The ratio that measures how many multiples of the firm's earning investors are willing to pay for its stock is:

Selected Answer: the price earnings ratio

Correct Answer: the price earnings ratio

Question 10 1 out of 1 points

Earnings per share represents the dividends shareholders receive for each share they own.

Selected Answer: False

Correct Answer: False

EXAM 1Question 1 1.5 out of 1.5 points

Managerial accounting stresses accounting concepts and procedures that are relevant to preparing reports for

Selected Answer: internal users of accounting information.

Correct Answer: internal users of accounting information.

Question 2 1.5 out of 1.5 points

The principle that managers follow when they only investigate departures from the plan that appear to be significant is commonly known as

Selected Answer: management by exception.

Correct Answer: management by exception.

Question 3 1.5 out of 1.5 points

Page 516: Microeconomics Study Guide

Which of the following costs does not change when the level of business activity changes?

Selected Answer: total fixed costs Correct Answer: total fixed costs

Question 4 1.5 out of 1.5 points

Which of the following is not likely to be a fixed cost?

Selected Answer: direct materials

Correct Answer: direct materials

Question 5 1.5 out of 1.5 points

A sunk cost is a cost

Selected Answer: incurred in the past which is not relevant to present decisions. Correct Answer: incurred in the past which is not relevant to present decisions.

Question 6 1.5 out of 1.5 points

In a period when anticipated production is 10,000 units, budgeted variable costs are $85,000 and budgeted fixed costs are $45,000. If 12,000 units are actually produced, what is the expected total cost?

Selected Answer: $147,000

Correct Answer: $147,000

Question 7 1.5 out of 1.5 points

Variable cost per unit

Selected Answer: does not change when the number of units produced increases. Correct Answer: does not change when the number of units produced increases.

Question 8 1.5 out of 1.5 points

Page 517: Microeconomics Study Guide

You own a car and are trying to decide whether or not to trade it in and buy a new car. Which of the following costs is an opportunity cost in this situation?

Selected Answer: the trip to Europe that you will not be able to take if you buy the car Correct Answer: the trip to Europe that you will not be able to take if you buy the car

Question 23 1.5 out of 1.5 points

Which of the following is not an advantage of decentralization for a company?

Selected Answer: subunit managers will act to benefit the organization as a whole Correct Answer: subunit managers will act to benefit the organization as a whole

Question 24 1.5 out of 1.5 points

Goal congruence refers to the match between

Selected Answer: goals of the individual managers and those of the company as a whole. Correct Answer: goals of the individual managers and those of the company as a whole.

Question 25 1.5 out of 1.5 points

Responsibility accounting holds managers responsible for

Selected Answer: only the costs which they can control.

Correct Answer: only the costs which they can control.

Question 26 1.5 out of 1.5 points

A subunit that has responsibility for controlling costs, but does not have responsibility for generating revenue is a(n)

Selected Answer: cost center.

Correct Answer: cost center.

Question 27 1.5 out of 1.5 points

Page 518: Microeconomics Study Guide

Which of the following is not a characteristic of a profit center?

Selected Answer: The center's manager has no control over the product pricing decisions. Correct Answer: The center's manager has no control over the product pricing decisions.

Question 28 1.5 out of 1.5 points

Which of the following is a responsibility that distinguishes an investment center manager from a profit center manager?

Selected Answer: significantly influencing investment decisions

Correct Answer: significantly influencing investment decisions

Question 29 1.5 out of 1.5 points

ROI is used to evaluate

Selected Answer: investment centers.

Correct Answer: investment centers.

Question 30 1.5 out of 1.5 points

Profit margin is

Selected Answer: All of the above are correct.

Correct Answer: All of the above are correct.

Question 31 1.5 out of 1.5 points

If a manager is evaluated based on ROI, and is managing a division which has attained a high ROI, the manager

Selected Answer: may not want to invest in projects that have an ROI that is higher than the firm's cost of capital, but lower than the division's ROI. Correct Answer: may not want to invest in projects that have an ROI that is higher than the firm's cost of capital, but lower than the division's ROI.

Page 519: Microeconomics Study Guide

Question 32 1.5 out of 1.5 points

Which of the following is not one of the set of dimensions considered in the Balanced Scorecard?

Selected Answer: depreciation

Correct Answer: depreciation

Question 33 1.5 out of 1.5 points

Managers need to analyze financial reports to:

Selected Answer: all of the above.

Correct Answer: all of the above.

Question 34 1.5 out of 1.5 points

The percentage increase in sales from one year to the next would be most obvious by using:

Selected Answer: horizontal analysis

Correct Answer: horizontal analysis

Question 35 1.5 out of 1.5 points

The gross margin

Selected Answer: is sales less the cost of the inventory that generated those sales. Correct Answer: is sales less the cost of the inventory that generated those sales.

Question 36 1.5 out of 1.5 points

Asset turnover is a measure of

Selected Answer: the overall efficiency with which the company uses assets to generate revenues. Correct Answer: the overall efficiency with which the company uses assets to generate revenues.

Page 520: Microeconomics Study Guide

Question 37 1.5 out of 1.5 points

The higher the accounts receivable turnover,

Selected Answer: both A and B Correct Answer: both A and B

Question 38 1.5 out of 1.5 points

If a company has an inventory turnover of 12, that means that

Selected Answer: it has about one month's sales in inventory.

Correct Answer: it has about one month's sales in inventory.

Question 39 0 out of 1.5 points

Which of the following would be included in the current ratio but not the quick ratio?

Selected Answer: Marketable securities

Correct Answer: Inventory

Question 40 1.5 out of 1.5 points

Comparative financial statements for Alpha Company are shown belowAlpha Company Balance Sheet December 31, 2005 December 31, 2004Assets: Current assets: Cash $89,103 $68,203Accounts receivable $138,209 130,394Inventory 96,708 85,694Other 21,203 5,118Total current assets 345,223 289,409Property, plant and equipment, net 790,120 688,902Intangible assets 32,456 29242Total assets $1,167,799 $1,007,553Liabilities and stockholders' equity: Current liabilities: Accounts payable $85,443 $62,394Other current liabilities 34,321 28,901

Page 521: Microeconomics Study Guide

Total current liabilities 119,764 91,295Long term debt 302,430 290,324Total liabilities 422,194 381,619Stockholders' equity: Common stock 400,000 400,000Additional paid in capital 200,000 200,000Retained earnings 145,605 25,934Total stockholders' equity 745,605 625,934Total liabilities and stockholders' equity $1,167,799 $1,007,553

Alpha Company Statement of Earnings For the year ended December 31, 2005 For the year ended December 31, 2004Sales $15,005,852 $13,809,585Cost of Goods Sold 10,250,257 9,825,614Gross profit 4,755,595 3,983,971Operating expense 3,585,657 3,400,258Income before interest and tax 1,169,937 583,713Interest expense 28,500 27,300Earnings before tax 1,141,437 556,413Tax expense 342,431 166,924Net earnings $799,006 $389,489

Alpha Company uses ending balance sheet balances when calculating ratios.Reference: Ref 13-2

Using horizontal analysis, the percentage change in accounts receivable from 2004 to 2005 was:

Selected Answer: 6.0% increase

Correct Answer: 6.0% increase

EXAM 3Question 1 1.5 out of 1.5 points

A budget

Selected Answer: All of the above are true.

Correct Answer: All of the above are true.

Question 2 1.5 out of 1.5 points

A method of budget preparation that requires all budgeted amounts to be justified by the department, even if the

Page 522: Microeconomics Study Guide

amounts were supported in prior periods, is called

Selected Answer: zero base budgeting.

Correct Answer: zero base budgeting.

Question 3 1.5 out of 1.5 points

The comprehensive planning document that incorporates a number of individual budgets is the

Selected Answer: master budget. Correct Answer: master budget.

Question 4 1.5 out of 1.5 points

Which of the following is a reasonable order in which to prepare budgets?

Selected Answer: sales budget, production budget, material purchases budget Correct Answer: sales budget, production budget, material purchases budget

Question 5 1.5 out of 1.5 points

If the number of units in beginning inventory is more than the number of units in ending inventory, the number of units sold is

Selected Answer: greater than the number of units produced. Correct Answer: greater than the number of units produced.

Question 6 1.5 out of 1.5 points

When constructing the production budget, the desired ending inventory for the first period is

Selected Answer: the beginning inventory for the second period. Correct Answer: the beginning inventory for the second period.

Question 7 1.5 out of 1.5 points

A significant difference between the direct material purchases budget and the direct labor budget is that the direct material purchases budget

Page 523: Microeconomics Study Guide

Selected Answer: considers beginning and ending inventory amounts, which are not part of a direct labor budget. Correct Answer: considers beginning and ending inventory amounts, which are not part of a direct labor budget.

Question 8 1.5 out of 1.5 points

Which of the following transactions will affect the cash receipts and disbursements budget for the month in which the transaction occurs?

Selected Answer: payment for direct labor

Correct Answer: payment for direct labor

Question 9 1.5 out of 1.5 points

The cash receipts and disbursements budget alerts management to all of the following except when

Selected Answer: stockouts will cause customer dissatisfaction.

Correct Answer: stockouts will cause customer dissatisfaction.

Question 10 1.5 out of 1.5 points

When budgets are used for evaluation, the differences between budgeted and actual amounts are called

Selected Answer: budget variances.

Correct Answer: budget variances.

Question 11 1.5 out of 1.5 points

The main difference between a static budget and a flexible budget is that the static budget is

Selected Answer: for a single level of activity while a flexible budget can be adjusted for different activity levels. Correct Answer: for a single level of activity while a flexible budget can be adjusted for different activity levels.

Question 12 1.5 out of 1.5 points

A flexible budget takes into account the fact that when production levels change

Page 524: Microeconomics Study Guide

Selected Answer: total fixed costs remain the same.

Correct Answer: total fixed costs remain the same.

Question 13 1.5 out of 1.5 points

Knight Company has found that 30% of its sales are collected in the month of the sale and the remainder of the sales is collected in the next month. If sales are expected to be $100,000 in April, $120,000 in May, and $80,000 in June, what is the estimated amount of cash receipts for May?

Selected Answer: $106,000

Correct Answer: $106,000

Question 14 1.5 out of 1.5 points

The standard cost is

Selected Answer: the cost that should have been incurred to produce the item. Correct Answer: the cost that should have been incurred to produce the item.

Question 15 1.5 out of 1.5 points

The relationship between standard costs and budgeted costs is the

Selected Answer: standard cost times the expected production level equals the budgeted cost. Correct Answer: standard cost times the expected production level equals the budgeted cost.

Question 16 1.5 out of 1.5 points

Standards that are developed under the assumption that a variety of things may lead to less than perfect performance are

Selected Answer: attainable standards. Correct Answer: attainable standards.

Question 17 1.5 out of 1.5 points

Page 525: Microeconomics Study Guide

Variances are usually analyzed for

Selected Answer: direct materials, direct labor, and manufacturing overhead. Correct Answer: direct materials, direct labor, and manufacturing overhead.

Question 18 1.5 out of 1.5 points

If the actual price per unit of material is greater than the standard price, the material price variance will be

Selected Answer: unfavorable.

Correct Answer: unfavorable.

Question 19 1.5 out of 1.5 points

If the material quantity variance is favorable, the

Selected Answer: production manager has used materials efficiently. Correct Answer: production manager has used materials efficiently.

Question 20 1.5 out of 1.5 points

Which of the following would cause a variance to be unfavorable?

Selected Answer: the standard hours allowed are less than the actual hours worked Correct Answer: the standard hours allowed are less than the actual hours worked

Question 21 0 out of 1.5 points

A possible cause for an unfavorable labor rate variance is

Selected Answer: hiring new, inexperienced employees.

Correct Answer: using more experienced workers than planned.

Question 22 1.5 out of 1.5 points

Erie Company produces one product, the E4501. The standards for E4501 include the use of 25 yards of raw material at a standard price of $4.42 per yard. During a recent month, the company used 65,000 yards of raw material to produce 2,580 units of E4501. Erie had purchased this material at a cost of $4.37 per yard. Calculate the material quantity

Page 526: Microeconomics Study Guide

variance.

Selected Answer: $2,210 unfavorable

Correct Answer: $2,210 unfavorable

Question 23 1.5 out of 1.5 points

Which of the following is not an advantage of decentralization for a company?

Selected Answer: subunit managers will act to benefit the organization as a whole Correct Answer: subunit managers will act to benefit the organization as a whole

Question 24 1.5 out of 1.5 points

Goal congruence refers to the match between

Selected Answer: goals of the individual managers and those of the company as a whole. Correct Answer: goals of the individual managers and those of the company as a whole.

Question 25 1.5 out of 1.5 points

Responsibility accounting holds managers responsible for

Selected Answer: only the costs which they can control.

Correct Answer: only the costs which they can control.

Question 26 1.5 out of 1.5 points

A subunit that has responsibility for controlling costs, but does not have responsibility for generating revenue is a(n)

Selected Answer: cost center.

Correct Answer: cost center.

Question 27 1.5 out of 1.5 points

Page 527: Microeconomics Study Guide

Which of the following is not a characteristic of a profit center?

Selected Answer: The center's manager has no control over the product pricing decisions. Correct Answer: The center's manager has no control over the product pricing decisions.

Question 28 1.5 out of 1.5 points

Which of the following is a responsibility that distinguishes an investment center manager from a profit center manager?

Selected Answer: significantly influencing investment decisions

Correct Answer: significantly influencing investment decisions

Question 29 1.5 out of 1.5 points

ROI is used to evaluate

Selected Answer: investment centers.

Correct Answer: investment centers.

Question 30 1.5 out of 1.5 points

Profit margin is

Selected Answer: All of the above are correct.

Correct Answer: All of the above are correct.

Question 31 1.5 out of 1.5 points

If a manager is evaluated based on ROI, and is managing a division which has attained a high ROI, the manager

Selected Answer: may not want to invest in projects that have an ROI that is higher than the firm's cost of capital, but lower than the division's ROI. Correct Answer: may not want to invest in projects that have an ROI that is higher than the firm's cost of capital, but lower than the division's ROI.

Page 528: Microeconomics Study Guide

Question 32 1.5 out of 1.5 points

Which of the following is not one of the set of dimensions considered in the Balanced Scorecard?

Selected Answer: depreciation

Correct Answer: depreciation

Question 33 1.5 out of 1.5 points

Managers need to analyze financial reports to:

Selected Answer: all of the above.

Correct Answer: all of the above.

Question 34 1.5 out of 1.5 points

The percentage increase in sales from one year to the next would be most obvious by using:

Selected Answer: horizontal analysis

Correct Answer: horizontal analysis

Question 35 1.5 out of 1.5 points

The gross margin

Selected Answer: is sales less the cost of the inventory that generated those sales. Correct Answer: is sales less the cost of the inventory that generated those sales.

Question 36 1.5 out of 1.5 points

Asset turnover is a measure of

Selected Answer: the overall efficiency with which the company uses assets to generate revenues. Correct Answer: the overall efficiency with which the company uses assets to generate revenues.

Page 529: Microeconomics Study Guide

Question 37 1.5 out of 1.5 points

The higher the accounts receivable turnover,

Selected Answer: both A and B Correct Answer: both A and B

Question 38 1.5 out of 1.5 points

If a company has an inventory turnover of 12, that means that

Selected Answer: it has about one month's sales in inventory.

Correct Answer: it has about one month's sales in inventory.

Question 39 0 out of 1.5 points

Which of the following would be included in the current ratio but not the quick ratio?

Selected Answer: Marketable securities

Correct Answer: Inventory

Question 40 1.5 out of 1.5 points

Comparative financial statements for Alpha Company are shown belowAlpha Company Balance Sheet December 31, 2005 December 31, 2004Assets: Current assets: Cash $89,103 $68,203Accounts receivable $138,209 130,394Inventory 96,708 85,694Other 21,203 5,118Total current assets 345,223 289,409Property, plant and equipment, net 790,120 688,902Intangible assets 32,456 29242Total assets $1,167,799 $1,007,553Liabilities and stockholders' equity: Current liabilities: Accounts payable $85,443 $62,394Other current liabilities 34,321 28,901

Page 530: Microeconomics Study Guide

Total current liabilities 119,764 91,295Long term debt 302,430 290,324Total liabilities 422,194 381,619Stockholders' equity: Common stock 400,000 400,000Additional paid in capital 200,000 200,000Retained earnings 145,605 25,934Total stockholders' equity 745,605 625,934Total liabilities and stockholders' equity $1,167,799 $1,007,553

Alpha Company Statement of Earnings For the year ended December 31, 2005 For the year ended December 31, 2004Sales $15,005,852 $13,809,585Cost of Goods Sold 10,250,257 9,825,614Gross profit 4,755,595 3,983,971Operating expense 3,585,657 3,400,258Income before interest and tax 1,169,937 583,713Interest expense 28,500 27,300Earnings before tax 1,141,437 556,413Tax expense 342,431 166,924Net earnings $799,006 $389,489

Alpha Company uses ending balance sheet balances when calculating ratios.Reference: Ref 13-2

Using horizontal analysis, the percentage change in accounts receivable from 2004 to 2005 was:

Selected Answer: 6.0% increase

Correct Answer: 6.0% increase

Final ReviewQuestion 1 1.5 out of 1.5 points

The cost of the cushions that are used to manufacture sofas is best described as a:

Selected Answer: variable cost.

Correct Answer: variable cost.

Question 2 1.5 out of 1.5 points

Materials used in the operation of a factory, such as cleaning supplies, that are not an integral part of the final product

Page 531: Microeconomics Study Guide

should be classified as:

Selected Answer: manufacturing overhead.

Correct Answer: manufacturing overhead.

Question 3 1.5 out of 1.5 points

Which of the following is NOT a period cost?

Selected Answer: Monthly depreciation on production equipment used by factory workers. Correct Answer: Monthly depreciation on production equipment used by factory workers.

Question 4 1.5 out of 1.5 points

A sunk cost is:

Selected Answer: irrelevant for decision making.

Correct Answer: irrelevant for decision making.

Question 5 1.5 out of 1.5 points

During July, the cost of goods manufactured at Xxis Corporation was $70,000. The beginning finished goods inventory was $19,000 and the ending finished goods inventory was $15,000. What was the cost of goods sold for the month?

Selected Answer: $74,000

Correct Answer: $74,000

Question 6 1.5 out of 1.5 points

In a predetermined overhead rate in a job-order costing system that is based on machine-hours, which of the following would be used in the numerator and denominator?

Selected Answer:

Page 532: Microeconomics Study Guide

Correct Answer:

Question 7 1.5 out of 1.5 points

If overhead is underapplied, then:

Selected Answer: the amount of overhead cost applied to Work in Process is less than the actual overhead cost incurred.

Correct Answer: the amount of overhead cost applied to Work in Process is less than the actual overhead cost incurred.

Question 8 1.5 out of 1.5 points

A is a fixed cost; B is a variable cost. During the current year the level of activity has decreased but is still within the relevant range. We would expect that:

Selected Answer: The cost per unit of B has remained unchanged. Correct Answer: The cost per unit of B has remained unchanged.

Question 9 0 out of 1.5 points

Contribution margin is computed as sales revenue minus:

Selected Answer: cost of goods sold Correct Answer: variable expenses

Which of the following is not a difference between financial accounting and managerial accounting?

Selected Answer: Managerial accounting is primarily concerned with providing information for external users while financial accounting is concerned with internal users. Correct Answer: Managerial accounting is primarily concerned with providing information for external users while financial accounting is concerned with internal users.

Question 2 1 out of 1 points

Which of the following is most likely to make use of Spruce Company's managerial accounting information?

Selected Answer: the production manager of Spruce's plant in Minnesota Correct Answer: the production manager of Spruce's plant in Minnesota

Question 3 1 out of 1 points

Page 533: Microeconomics Study Guide

A difference between actual costs and planned costs

Selected Answer: should be investigated if the amount is exceptional. Correct Answer: should be investigated if the amount is exceptional.

Question 4 1 out of 1 points

Costs incurred in the past which are not relevant to present decisions are

Selected Answer: sunk costs.

Correct Answer: sunk costs.

Question 5 1 out of 1 points

The benefits that are given up when another alternative is selected is a(n)

Selected Answer: opportunity cost. Correct Answer: opportunity cost.

Question 6 1 out of 1 points

A cost which is directly traceable to a product, activity, or department is a

Selected Answer: direct cost. Correct Answer: direct cost.

Question 7 1 out of 1 points

A manager should be evaluated based on

Selected Answer: controllable costs.

Correct Answer: controllable costs.

Question 8 1 out of 1 points Calculating the difference in revenue and the difference in cost between decision alternatives is called

Page 534: Microeconomics Study Guide

Selected Answer: incremental analysis.

Correct Answer: incremental analysis.

Question 9 1 out of 1 points

When making ethical decisions, you should consider:

Selected Answer: what is right.

Correct Answer: what is right.

Question 10 0 out of 1 points

Which of the following statements regarding fixed costs is true?

Selected Answer: When production decreases, total fixed costs decrease. Correct Answer: When production increases, fixed cost per unit decreases.

Chapter 2Question 1 1 out of 1 points

Which of the following is not a manufacturing cost?

Selected Answer: Administrative expenses

Correct Answer: Administrative expenses

Question 2 0 out of 1 points

Cold Company manufactures refrigerators. Which of the following items is most likely to be an indirect material cost for Cold Company?

Selected Answer: Factory supervisor's salary

Correct Answer: Lubricant for refrigerator door hinges

Question 3

Page 535: Microeconomics Study Guide

1 out of 1 points

Product costs

Selected Answer: All of the above answers are correct.

Correct Answer: All of the above answers are correct.

Question 4 1 out of 1 points

Brandon Company allocates overhead based on machine hours used on a job. The predetermined overhead rate is $15 per machine hour. If 6,000 machine hours were used on jobs and $97,000 in overhead costs were incurred, what is the amount of underapplied or overapplied overhead?

Selected Answer: $7,000 underapplied

Correct Answer: $7,000 underapplied

Question 5 1 out of 1 points

Which of the following is a period cost?

Selected Answer: Commissions paid on each unit sold

Correct Answer: Commissions paid on each unit sold

Question 6 1 out of 1 points

A job-order costing system is likely to be used by a

Selected Answer: custom home builder.

Correct Answer: custom home builder.

Question 7 1 out of 1 points

Predetermined overhead rates use

Page 536: Microeconomics Study Guide

Selected Answer: estimated overhead costs and estimated levels of the allocation base. Correct Answer: estimated overhead costs and estimated levels of the allocation base.

Question 8 1 out of 1 points

The Sienna Company has a beginning balance in Finished Goods Inventory of $22,000 and an ending balance in Finished Goods Inventory of $20,000. If the cost of goods manufactured is $380,000, what is the cost of goods sold?

Selected Answer: $382,000

Correct Answer: $382,000

Question 9 1 out of 1 points

Yellow Company has a beginning balance in the Work in Process Inventory account of $230,000. Current manufacturing costs for the period are $580,000. If the cost of goods manufactured is $750,000, what is the ending balance in the Work in Process Inventory account?

Selected Answer: $60,000

Correct Answer: $60,000

Question 10 1 out of 1 points

Well Done Company applies overhead using machine hours as the allocation base, at a rate of $17 per machine hour. Job KD requires $500 worth of material, 12 hours of labor at $15 per hour and 11 machine hours. What is the cost of job KD?

Selected Answer: $867

Correct Answer: $867

Chapter 3Question 1 1 out of 1 points

Which of the following companies is most likely to use a process costing system?

Selected Answer: a food manufacturer Correct Answer: a food manufacturer

Page 537: Microeconomics Study Guide

Question 2 1 out of 1 points

Which of the following describes the differences between job-order and process costing?

Selected Answer: Job-order costing traces costs to jobs while process costing traces costs to departments and averages the costs among the units worked on during the period. Correct Answer: Job-order costing traces costs to jobs while process costing traces costs to departments and averages the costs among the units worked on during the period.

Question 3 1 out of 1 points

In the manufacturing operations of a company using process costing

Selected Answer: a product typically must pass through two or more production departments. Correct Answer: a product typically must pass through two or more production departments.

Question 4 1 out of 1 points

Labor and overhead are often grouped together and referred to as

Selected Answer: conversion costs.

Correct Answer: conversion costs.

Question 5 1 out of 1 points

A production cost report

Selected Answer: provides a reconciliation of units and a reconciliation of costs as well as the details of the cost per equivalent unit calculation. Correct Answer: provides a reconciliation of units and a reconciliation of costs as well as the details of the cost per equivalent unit calculation.

Question 6 1 out of 1 points

If beginning inventory consisted of 3,500 units, ending inventory was 1,500 units, and 9,000 units were started during the period, how many units were completed and transferred out of the department?

Selected Answer: 11,000 units

Page 538: Microeconomics Study Guide

Correct Answer: 11,000 units

Question 7 1 out of 1 points

In a process costing system, manufacturing overhead is added to Work in Process

Selected Answer: using a predetermined overhead rate or actual overhead. Correct Answer: using a predetermined overhead rate or actual overhead.

Question 8 1 out of 1 points

The ending Work in Process inventory in the mixing department contains 300 units that are 30% complete with respect to labor costs. How many equivalent units are in the ending inventory?

Selected Answer: 90

Correct Answer: 90

Question 9 1 out of 1 points

A company has 10,000 equivalent units in ending work in process, that are 100% complete with respect to materials and 80% complete with respect to conversion costs. The cost per unit for inventory is $2.00, which consists of $1.20 for materials and $0.80 for conversion costs. What is the dollar value assigned to the ending work in process inventory?

Selected Answer: $18,400

Correct Answer: $18,400

Question 10 1 out of 1 points

Equivalent units

Selected Answer: All of the above are true.

Correct Answer: All of the above are true.

Chapter 4

Page 539: Microeconomics Study Guide

Question 1 1 out of 1 points

Which of the following costs is least likely to be a variable cost?

Selected Answer: supervisory salaries

Correct Answer: supervisory salaries

Question 2 1 out of 1 points

When the level of activity increases, total fixed costs

Selected Answer: remain the same.

Correct Answer: remain the same.

Question 3 1 out of 1 points

The contribution margin per unit is the difference between

Selected Answer: selling price and variable cost per unit.

Correct Answer: selling price and variable cost per unit.

Question 4 1 out of 1 points

Holding all other factors constant, the break-even point will be decreased by

Selected Answer: increasing the selling price.

Correct Answer: increasing the selling price.

Question 5 1 out of 1 points

A firm with high operating leverage has high

Selected Answer: fixed costs.

Page 540: Microeconomics Study Guide

Correct Answer: fixed costs.

Question 6 1 out of 1 points

When considering a process that involves a resource constraint, the optimal decision

Selected Answer: maximizes the contribution margin per unit of the constraint. Correct Answer: maximizes the contribution margin per unit of the constraint.

Question 7 1 out of 1 points

Nick's Novelties has collected the following information over the last six months.Month Units produced Total costsMarch 10,000 $25,600April 12,000 26,200May 18,000 27,600June 13,000 26,450July 12,000 26,000August 15,000 26,500

Reference: Ref 4-1

Using the high-low method, what is the variable cost per unit?

Selected Answer: $0.25

Correct Answer: $0.25

Question 8 1 out of 1 points

Nick's Novelties has collected the following information over the last six months.Month Units produced Total costsMarch 10,000 $25,600April 12,000 26,200May 18,000 27,600June 13,000 26,450July 12,000 26,000August 15,000 26,500

Reference: Ref 4-1

Using the high-low method, what is the total fixed cost?

Page 541: Microeconomics Study Guide

Selected Answer: $23,100

Correct Answer: $23,100

Question 9 1 out of 1 points

Copper Corporation sells a single product at a price of $275 per unit. Variable cost per unit is $135 and fixed costs total $356,860. If sales are expected to be $825,000, what is Cooper's margin of safety?

Selected Answer: $124,025

Correct Answer: $124,025

Question 10 1 out of 1 points

Bryant Company makes a product that sells for $80 per unit and has $63 per unit in variable costs. Annual fixed costs are $17,000.Reference: Ref 4-2

How many units must be sold to earn a profit of $51,000?

Selected Answer: 4,000

Correct Answer: 4,000

What is an externality? a benefit or cost that affects someone not directly involved in the production orconsumption of a good or serviceDescribe negative externalities. 1. costs imposed on individuals not directlyinvolved in producing or consuming a good or service2.causes the social cost of productionof a good or service to exceed the private cost borne by the producer3. causes output to exceed theeconomically efficient amountDescribe positive externalities. 1. benefits received byindividuals not directly involved in producing or consuming a good or service2. causes the social benefit from consuming agood or service to exceed the private benefit3. causes output to be less than theeconomically efficient amountWhat is the result of negative and positive externalities? market failureWhat is the underlying cause of externalities and other forms of market failure? The absence of private property rights or the

Page 542: Microeconomics Study Guide

lack of sufficient enforcement of existing property rightsWhen are private solutions possible and efficient? if there are low transactions costsDescribe the command and control approach. the government sets specific quantitative limitsDescribe rivalry. occurs when consumption of one unit of a good precludes its consumption by someone elseDescribe excludability. means that anyone who does not pay for a good cannot consume itDescribe tragedy of the commons. Oneperson's use of a common resource can impose costs on othersWhat is a private cost? a cost borne by the producer of a good or serviceWhat is a social cost? the total cost ofproduction, including both the private cost and any external costWhat is a private benefit? the benefit receivedby the consumer of a good or serviceWhat is a social benefit? the total benefit from consuming a good,including both the private benefit and any external benefitWhat are property rights? the rights individuals or businesses have to the exclusive use of their property,including the right to buy or sell it.What is market failure? situations where the market fails to produce the efficient level of outputDescribe the Coase Theorem. 1. if transactions costs are low, private bargaining will result in an efficientsolution to the problem of externalities2. services. Successfulapplication of the Coase Theorem requires that the bargaining parties have full information regarding thecosts and benefits associated with the externalities and are willing to accept a reasonable agreementWhat are transaction costs? the costs in time and other resources thatparties incur in the process of agreeing to and carrying out an exchange of goods and servicesDescribe Pigovian tax and subsidy. 1. cause consumers and firms to internalize the externalitiesassociated with production and consumption2. The tax or subsidy would be equal to the dollar amount ofthe externalityWhat is a private good? a good that is both rival and excludableWhat is a common resource? a good that is rival but not excludableWhat is a public good? a good that is bothnonrivalrous and nonexcludableDescribe free riding? 1. benefiting from a good without paying for it2. Because of free riders, public goods are usually supplied by government rather than private firms.1. What is the cost that affects someone who is not directly involved in the production or consumptionof a good called?a. Private costb. Indirect costc. An externalityd. All of the above fit that definition c. An externality2. What is the term used to describe the total cost of producing a good?a. Private costb. Social costc. Externalityd. All of the above fit that definition. b. Social cost3. What is the difference between private benefit and social benefit?

Page 543: Microeconomics Study Guide

a. An external benefitb. Private costc. Social costd. A negative externality a. An external benefit4. What is the benefit received by the consumer of a good or service called?a. Private benefitb. Social benefitc. Private costd. A positive externality a. Private benefit10. Fill in the blanks. When a negative externality is present in producing a good or service,___________ of the good or service will be produced at market equilibrium.a. too muchb. too littlec. the optimal quantityd. none a. too much11. What is the situation called in which the market fails to produce the efficient level of output?a. An externalityb. Market failurec. External disequilibriumd. The Coase Theorem b. Market failure12. When we talk about property rights in the discussion of externalities, which rights do we refer to?a. The rights of individuals to polluteb. The rights of individuals to have exclusive use of their propertyc. The rights of individuals to buy but not sell their propertyd. All of the above b. The rights of individuals to have exclusive use of their property13. What are the sources of externalities and market failure?a. Incomplete property rightsb. The difficulty of enforcing property rights in certain situations.c. Both a. and b.d. Lack of understanding of the market system c. Both a. and b.14. What type of solution to externalities is the Coase Theorem?a. A private solution to externalitiesb. A public solution to externalitiesc. The only solution to externalitiesd. The least preferred solution a. A private solution to externalities15. Which of the following statements is correct according to Ronald Coase's argument for dealing withexternalities and market failure?a. In some situations, a private solution to the problem of externalities can be found.b. Only public solutions exist for solving externalities.c. Completely eliminating an externality is almost always the most efficient solution.d. The only cure to externalities is taxation. a. In some situations, a private solution to the problem of externalities can be found16. Which of the following is correct?a. Completely eliminating an externality is usually not economically efficient.b. As reductions in pollution increase, the additional benefits will decline.c. When levels of pollution are high, the marginal benefit of reducing pollution is also high.

Page 544: Microeconomics Study Guide

d. All of the above d. All of the above17. The net benefit to society from reducing pollution is equal toa. the sum of the benefits of reducing pollution and the costs.b. the difference between the benefits and the costs.c. the additional benefit plus the additional costs.d. the quantity of pollution, such as the tons of reduction in sulfur dioxide. b. the difference between the benefits and the costs.1. An externality isA) a cost paid for by the producer of a good or service.a benefit realized by the purchaser of a good or service.C) a benefit or cost experienced by someone who is not a producer or consumer of a good or service.D) anything that is external or not relevant to the production of a good or service. C) a benefit or cost experienced by someone who is not a producer or consumer of a good or service2. Which of the following is a source of market failure?A) a lack of government intervention in a marketunforeseen circumstances which leads to the bankruptcy of many firmsC) an inequitable income distributionD) incomplete property rights or inability to enforce property rights D) incomplete property rights or inability to enforce property rights3. What is a market failure?A) It refers to a breakdown in a market economy because of widespread corruption in government.It refers to a situation where an entire sector of the economy (for example, the airline industry) collapsesbecause of some unforeseen event.C) It refers the inability of the market to allocate resources efficiently up to the point where marginal socialbenefit equals marginal social cost.D) It refers to the inability of the market to allocate resources efficiently up to the point where marginal socialbenefit equals marginal private cost C) It refers the inability of the market to allocate resources efficiently up to the point where marginal socialbenefit equals marginal social cost.4. Which of the following activities create a negative externality?A) graduating from collegekeeping a junked car parked on your front lawnC) cleaning up the sidewalk on your blockD) repainting the house you live in to improve its appearance keeping a junked car parked on your front lawn5. A negative externality exists ifA) there are price controls in a market.the marginal social cost of producing a good or service exceeds the private cost.C) there are quantity controls in a market.D) the marginal private cost of producing a good or service exceeds the social cost the marginal social cost of producing a good or service exceeds the private cost.6. Private costsA) are borne by producers of a good while social costs are borne by those who cannot afford to purchase thegood.are borne by consumers of a good while social costs are borne by government.C) are borne by producers of a good while social costs are borne by government.D) are borne by producers of a good while social costs are borne by society at large. D) are borne by producers of a good

Page 545: Microeconomics Study Guide

while social costs are borne by society at large.. An external cost can be calculated as the difference betweenA) the social cost of production and the social benefit of production.the private cost of production and the social benefit of production.C) the social cost of production and the private cost of production.D) a producer's cost of production and the price at which the good is sold. C) the social cost of production and the private cost of production8. The social cost of cutting trees for firewood in a government forest isA) the increased likelihood of flooding as more trees are cut.the marginal costs of cutting the last tree.C) opportunity cost to the individual of cutting the wood.D) the increased likelihood of flooding as more trees are cut plus the private cost of cutting the trees. D) the increased likelihood of flooding as more trees are cut plus the private cost of cutting the trees9. Which of the following represents the economic benefit of production when firms produce goods that createpositive externalities?A) the difference between social benefits and private benefits created by the goods.the sum of private benefits from consumption.C) the external benefits created by the goods.D) the sum of private benefits and external benefits created by the goods. D) the sum of private benefits and external benefits created by the goods.10. A positive externality causesA) the marginal social benefit to be equal to the marginal private cost of the last unit produced.the marginal private benefit to exceed the marginal social cost of the last unit produced.C) the marginal social benefit to be less than the marginal private cost of the last unit produced.D) the marginal social benefit to exceed the marginal private cost of the last unit produced. D) the marginal social benefit to exceed the marginal private cost of the last unit produce14. When there is a negative externality in a free market,A) too little of the good is produced and consumed.an economically efficient level of the good is produced and consumed.C) a productively efficient level of the good is produced and consumed.D) too much of the good is produced and consumed. D) too much of the good is produced and consumed20. Which of the following conditions holds in an economically efficient competitive market equilibrium?A) There are no positive and no negative external effects from consumption and production.The marginal benefit of the last unit produced and consumed is maximized.C) Producer and consumer surplus are exactly equal in size.D) The deadweight loss is positive but at a minimum. A) There are no positive and no negative external effects from consumption and production.21. A market demand curve reflects theA) social benefits of consuming a product.the sum of private and social benefits of consuming a product.C) private benefits of consuming a product.D) external benefits of consuming a product C) private benefits of consuming a product.22. Which of the following statements is false?A) Social benefits are external benefits minus private benefits.Social costs are private costs and any external costs.C) Private benefits are received by the consumer of the good or service.

Page 546: Microeconomics Study Guide

D) Private costs are borne by the producer of the good or service. A) Social benefits are external benefits minus private benefits.27. In which of these situations is there no deadweight loss?A) free market production of a private good with a negative externalityfree market consumption of a private good with a positive externalityC) consumption of a common resource without government restrictionsD) free market production and consumption of a private good without any externalities D) free market production and consumption of a private good without any externalities28. Economists argue that the level of pollution should beA) ignored because it has always been present since the beginning of history.best determined by elected officials who can speak on behalf of the public.C) reduced completely to zero because by definition, it is a negative external effect.D) reduced to the point where the marginal benefit of pollution reduction is equal to the marginal cost ofpollution reduction to society. D) reduced to the point where the marginal benefit of pollution reduction is equal to the marginal cost ofpollution reduction to society29. The Coase theorem states thatA) government intervention is always needed if externalities are present.a free market equilibrium is the best solution to address externalities.C) if transactions costs are low, private bargaining will result in an efficient solution to the problem ofexternalities.D) assigning property rights is the only thing the government should do in a market economy C) if transactions costs are low, private bargaining will result in an efficient solution to the problem ofexternalities.30. If the paint on your house was eaten away by the fumes from a factory nearby and you hired a lawyer to sue thepolluting firm, your legal fees would be consideredA) external costs.social costs of the pollution.C) marginal benefits.D) transaction costs D) transaction costs.31. Consider the following characteristics: a. low transaction costsb. small levels of pollutionc. high levels of pollutiond. clear assignment of property rights. Which of the above are assumptions behind the Coase Theorem?A) a and da, b, and dC) a, c, and dD) a only A) a and d32. In the city of Alvarez, with the exception of guide dogs for blind people, all dogs are banned from its threepublic parks, regardless of whether the animals are leashed. Many residents are pushing for a change in policy.Canine lover Sara Northridge observed, "There are 800 or more homes here. There are three parks within 10minutes, and almost everyone has a dog, but we can't take our dogs there." Others fear that allowing dogswould detract from their enjoyment of the parks. Tim Cortis retorted, "We're not preventing dog lovers fromenjoying the park, just come without your dog." Which of the following is a way of dealing with the problem by

Page 547: Microeconomics Study Guide

assigning property rights to a particular group?A) impose a fee only for dog-owners to use the public parks; non dog owners do not pay a feeimpose a two-tier entry fee system - a lower fee for non dog owners and a higher fee for dog ownersC) dedicate some parks, or at least one park, exclusively for the use of visitors bringing dogs to the park.D) allow dog owners to bring their dogs to the park but insist that they keep watch over their dogs C) dedicate some parks, or at least one park, exclusively for the use of visitors bringing dogs to the park33. In the United States, many beekeepers travel from state to state, renting out their bee colonies to farmers forpollination services. This is an example ofA) a tradable exchange contract.a Pigouvian solution to a positive externality problem.C) command and control policy.D) a Coasian solution to a positive externality problem. D) a Coasian solution to a positive externality problem34. What does the phrase "internalizing an external cost" mean?A) forcing producers to factor into their production costs, the cost of the externalities created in theproduction of their outputprohibiting economic activities that create externalitiesC) limiting the extent to which domestic firms can outsource productionD) finding a way to address cross-border pollution A) forcing producers to factor into their production costs, the cost of the externalities created in theproduction of their output35. An advantage of imposing a tax on the producer that generates pollution is thatA) it will eliminate pollution.the government can keep tabs on exactly what is produced in an industry.C) a producer can pass the cost of the pollution to consumers.D) it forces the polluting producer to internalize the external cost of the pollution. D) it forces the polluting producer to internalize the external cost of the pollution36. Suppose a tax equal to the value of the marginal external cost at the optimal output is imposed on a pollutiongenerating good. All of the following will result from the tax exceptA) a decrease in market supply of the good.an increase in the equilibrium market price.C) a decrease in the equilibrium quantity produced and consumed.D) an increase in the demand for the good D) an increase in the demand for the good.37. Governments can increase the consumption of a product that creates positive externalities byA) assigning property rights to the producers of the product.taxing the production and consumption of the product.C) convincing everyone to consume the good.D) subsidizing the production of the product so that the supply is increased and market price is reduced D) subsidizing the production of the product so that the supply is increased and market price is reduced.39. Government imposed quantitative limits on the amount of pollution firms are allowed is an example ofA) a tradable emission allowance system of pollution control.the Pigovian method of pollution control.C) Coasian solution to pollution reduction.D) command and control approach to pollution reduction D) command and control approach to pollution reductionIssuing marketable emission allowance permits to polluting firmsA) gives the industry the right to a specific level of pollution.generates income for the government.

Page 548: Microeconomics Study Guide

C) has the same impact as a subsidy on production.D) encourages more pollution. A) gives the industry the right to a specific level of pollution41. Compared to the command and control approach to pollution control, the tradable emissions allowance systemisA) less efficient, because the tradable emissions allowance system gives firms the right to pollute a certainamount.less efficient, because it does not raise funds for the government whereas under the command and controlapproach, firms are penalized for non-compliance.C) more efficient, because firms with low costs of pollution control would pollute the least and sell their extrapermits to other firms with higher costs of pollution control.D) more efficient, because there are implementation and monitoring costs with the tradable emissionsallowance system. C) more efficient, because firms with low costs of pollution control would pollute the least and sell their extrapermits to other firms with higher costs of pollution control.Anyone can purchase sulfur dioxide emissions allowances on the Chicago Mercantile Exchange. Severalenvironmental groups have raised money to buy allowances (which it subsequently destroys). As part of theirfund-raising, these groups have urged contributors to buy the allowances as gifts. As one newspaper story putit, "For the environmentalist in your life, here's a gift that is sold by the ton, fits in an envelope and will lastforever." Source for quote: Randall Edwards, "Dear Santa: Please Bring Me Sulfur Dioxide for Christmas," Columbus Dispatch, December19, 1999.What would be the impact on the price of the emission allowances in the market?A) The price rises.The price falls but not to zero.C) The price remains unchanged because the allowances purchased by the environmental groups aredestroyed.D) The price falls to zero. A) The price rises.43. Under the European Union's tradable carbon emissions rights scheme, utilities are given a set level of permits toemit carbon dioxide. If a utility wants to pollute more than its allowance it could buy more permits in themarket. Under what situation will a utility that wants to pollute more than its allowance not purchaseadditional permits?A) if there is an excess demand for permits in the marketif it is cheaper to switch from heavy polluting energies to cleaner fuels than to purchase pollution permitsC) if it is cheaper to burn heavy polluting energies than to switch to cleaner fuelsD) if the market price of a permit is so low that it has virtually no resale value if it is cheaper to switch from heavy polluting energies to cleaner fuels than to purchase pollution permitsA product is considered to be nonexcludable ifA) you cannot keep those who did not pay for the item from enjoying its benefits.your consumption of the product reduces the quantity available for others to consume.C) you can keep those who did not pay for the item from enjoying its benefits.D) it is jointly owned by all members of a community A) you cannot keep those who did not pay for the item from enjoying its benefits.5. Which of the following displays these two characteristics: rivalry and nonexcludability?A) a private good.a common resource.

Page 549: Microeconomics Study Guide

C) a quasi-public good.D) a public good. a common resource. Which of the following displays nonrivalry and nonexcludability in consumption?A) quasi-public goodspublic goodsC) common resourcesD) private goods public goods7. Which of the following is an example of a product that is nonexcludable and rivalrous?A) free concert (with limited seating) in a parkflu vaccinationsC) automobilesD) national defense A) free concert (with limited seating) in a park8. Common resources differ from public goods in thatA) common resources are collectively owned by a group of people while public goods are governmentowned.unlike public goods, common resources are rivalrous in consumption.C) common resources are resources that cannot be renewed but the production of public goods can beincreased any time.D) common resources are non-excludable while public goods are excludable to those who do not pay for thegood. B) unlike public goods, common resources are rivalrous in consumption49. In economics, the term "free-rider" refers toA) one who waits for other s to produce a good and then enjoys its benefits without paying for it.B) a supervisor who delegates menial time-consuming activities to othersC) a person who evades taxes.D) one who volunteers her services. A) one who waits for other s to produce a good and then enjoys its benefits without paying for it50. How does the construction of a market demand curve for a private good differ from that for a public good?A) The market demand curve for a private good is determined by adding up the quantities demanded byeach consumer at each price but the market demand curve for a public good is determined by adding upthe price each consumer is willing to pay for each quantity of the good.B) There is no difference; in both cases the demand curve is determined by adding up the quantitiesdemanded by each consumer at each priceC) There is no difference; in both cases the demand curve is determined by adding up the price eachconsumer is willing to pay for each quantity of the good.D) The market demand curve for a private good is determined by adding up the price each consumer iswilling to pay for each quantity of the good but the market demand curve for a public good is determinedby adding up but the quantities demanded by each consumer at each price. A) The market demand curve for a private good is determined by adding up the quantities demanded byeach consumer at each price but the market demand curve for a public good is determined by adding upthe price each consumer is willing to pay for each quantity of the good.51. The efficient output level of a public good occurs where theA) greatest number of free riders occurs.B) marginal cost of producing the last unit is equal to the marginal benefit realized by consumers.C) marginal cost of production is at its lowest.D) total cost of production is affordable. B) marginal cost of producing the last unit is equal to the marginal benefit realized by consumers.

Page 550: Microeconomics Study Guide

52. For-profit producers will produce only private goods becauseA) buyers will be willing to pay for the goods since the benefits are excludable.B) the cost of production can be easily determined easily found.C) markets exist for private goods but not for public goods.D) all external benefits can be internalized using market prices. A) buyers will be willing to pay for the goods since the benefits are excludable.53. According to an article in the Wall Street Journal, economist Paul Romer of Stanford University has argued:"The market mechanism and property rights are excellent at conserving scarce resources and putting them tothe most profitable use. They aren't so good at encouraging the production and distribution of new ideas,which are critical to progress."Source: David Wessel, "Precepts from Professor Summers," Wall Street Journal, October 17, 2002.What characteristics of the production and distribution of new ideas might make it difficult for the market toproduce the optimal amount?A) rivalry and non-excludabilityB) rivalry and excludabilityC) rivalry and excludabilityD) non-rivalry and non-excludability D) non-rivalry and non-excludability54. The "tragedy of the commons" refers to the phenomenon whereA) individuals are free riders.B) there is rivalry in consumption.C) people do not internalize an externality.D) people overuse a common resource D) people overuse a common resource.55. Which of the following exemplifies the tragedy of the commons?A) The Pleasant Hill community is growing so fast that the city's only post office is not able to keep pace withthe population growth in the community.B) Residents in the northern coast of California receive only one public broadcasting signal which may beeliminated altogether if government funding is cut.C) Canadian citizens receive free universal health care administered by provincial governments.D) The Malaysian tapir, distinguished for its unusual coloration, is a target for poachers who hunt it for itstough and leathery hide. D) The Malaysian tapir, distinguished for its unusual coloration, is a target for poachers who hunt it for itstough and leathery hide.56. Consider the stock of ocean tuna which is massively overfished. It is rational for an individual to exploit theresource rather than to conserve the stock becauseA) the social cost of harvesting the fish is lower than the private cost.B) the private cost of harvesting the fish is lower than the social cost.C) the private benefit of harvesting tuna is lower than the social benefit of harvesting it.D) the private benefit of harvesting tuna is higher than the social benefit of harvesting it. B) the private cost of harvesting the fish is lower than the social cost60. The basic cause of deadweight losses from the existence of common resources and externalities isA) a lack of clearly defined and enforceable property rights.B) the absence of government intervention.C) the use of a market system to deal with scarcity.D) the self interested rationality of human beings. A) a lack of clearly defined and enforceable property rights.61. Global warming refers to the effect of global pollutants such as carbon dioxide on climates on the earth.Climate-induced changes in temperatures affect, among other things, agriculture. Which of the following is a

Page 551: Microeconomics Study Guide

reason why policymakers are still debating if this problem should be addressed and how it should beaddressed?A) Scientists agree about the damage caused by carbon emissions but disagree about the methods of reducingemissions.B) The marginal cost of reducing carbon emissions is known with certainly but the marginal benefit fromreduction is not known with certainty.C) There is much disagreement about all aspects of the problem: how much carbon emissions contribute todamage from climate change which in turn informs the benefits from reduction, and what methods to usewhich in turn determines the cost of reductionD) The marginal cost of reducing carbon emissions is not known with certainly but the marginal benefit fromreduction is known with certainty C) There is much disagreement about all aspects of the problem: how much carbon emissions contribute todamage from climate change which in turn informs the benefits from reduction, and what methods to usewhich in turn determines the cost of reduction62. Private producers have no incentive to provide public goods becauseA) once produced, it will not be possible to exclude to those who do not pay for the goodB) they cannot avoid the tragedy of the commons.C) the government subsidy granted is usually insufficient to enable private producer to make a profit.D) production of huge quantities of public goods entails huge fixed costs. A) once produced, it will not be possible to exclude to those who do not pay for the goodSet ChampionsThere are no high scores or champions for this set yet. You can sign up or log in to be the first!

Practice Exam One - Micro Fall 2011Student: ___________________________________________________________________________

1. When economists say that people act rationally in their self interest, they mean that individuals: A. look for and pursue opportunities to increase their utility.B. generally disregard the interests of others.C. are mainly creatures of habit.D. are usually impulsive and unpredictable.

2. A person should consume more of something when its marginal: A. benefit exceeds its marginal cost.B. cost exceeds its marginal benefit.C. cost equals its marginal benefit.D. benefit is still positive.

3. Marginal costs exist because: A. the decision to engage in one activity means forgoing some other activity.B. wants are scarce relative to resources.C. households and businesses make rational decisions.D. most decisions do not involve sacrifices or tradeoffs.

4. Even though local newspapers are very inexpensive, people rarely buy more than one of them each day. This fact: A. is an example of irrational behavior.

Page 552: Microeconomics Study Guide

B. implies that reading should be taught through phonics rather than the whole language method.C. contradicts the economic perspective.D. implies that, for most people, the marginal benefit of reading a second newspaper is less than the marginal cost.

5. In deciding whether to study for an economics quiz or go to a movie, one is confronted by the idea(s) of: A. scarcity and opportunity costs.B. money and real capital.C. complementary economic goals.D. full production.

6. Which of the following most closely relates to the idea of opportunity costs? A. tradeoffs.B. economic growth.C. technological change.D. capitalism.

7. Economic theories: A. are useless because they are not based on laboratory experimentation.B. that are true for individual economic units are never true for the economy as a whole.C. are generalizations based on a careful observation of facts.D. are abstractions and therefore of no application to real situations.

8. Macroeconomics approaches the study of economics from the viewpoint of: A. the entire economy.B. governmental units.C. the operation of specific product and resource markets.D. individual firms.

9. Any combination of goods lying outside of the budget line: A. implies that the consumer is not spending all his income.B. yields less utility than any point on the budget line.C. yields less utility than any point inside the budget line.D. is unattainable, given the consumer's income.

10. Refer to the above graphs. Assume that pizza is measured in slices and beer in pints. In which of the graphs is the opportunity cost of a pint of beer equal to one slice of pizza? A. graph AB. graph BC. graph CD. graph D

11. A production possibilities curve shows: A. that resources are unlimited.B. that people prefer one of the goods more than the other.

Page 553: Microeconomics Study Guide

C. the maximum amounts of two goods that can be produced, assuming the full use of available resources.D. combinations of capital and labor necessary to produce specific levels of output.

12. The typical production possibilities curve is: A. an upsloping line that is bowed out from the origin.B. a downsloping line that is bowed in toward the origin.C. a downsloping line that is bowed out from the origin.D. a straight upsloping line.

13. The construction of a production possibilities curve assumes: A. the quantities of all resources are unlimited.B. technology is fixed.C. some resources are unemployed.D. there is no inflation in the economy.

Answer the question on the basis of the following production possibilities tables for two countries, North Cantina and South Cantina:

14. Refer to the above tables. If North Cantina is producing at production alternative B, the opportunity cost of the eleventh unit of consumer goods will be: A. 10 units of capital goods.B. 1/4 of a unit of capital goods.C. 8 units of capital goods.D. 1/8 of a unit of capital goods.

15. Which of the following is a distinguishing feature of a market system? A. public ownership of all capital.B. central planning.C. wide-spread private ownership of capital.D. a circular flow of goods, resources, and money.

16. Economic systems differ according to which two main characteristics? A. Who owns the factors of production, and the methods used to coordinate economic activity.B. The technology used in production, and the quantity and quality of natural resources.C. How goods are produced, and who gets them.D. The political system in place, and the degree of scarcity facing the economy.

17. The regulatory mechanism of the market system is: A. self-interest.B. private property.C. competition.D. specialization.

18. Specialization-the division of labor-enhances productivity and efficiency by: A. allowing workers to take advantage of existing differences in their abilities and skills.

Page 554: Microeconomics Study Guide

B. avoiding the time loss involved in shifting from one production task to another.C. allowing workers to develop skills by working on one, or a limited number, of tasks.D. all of the means identified in the other answers.

19. If consumer desire for product X increases, all of the following will occur except: A. an increase in the profits of industry X.B. an increase in the quantity of resources employed by industry X.C. an increase in the output of industry X.D. a decrease in the quantity of resources employed in industry X.

20. The market system's answer to the fundamental question "Who will get the goods and services?" is essentially: A. "Those willing and able to pay for them."B. "Those who physically produced them."C. "Those who most need them."D. "Those who get utility from them."

21. Which of the following best describes the invisible-hand concept? A. The desires of resource suppliers and producers to further their own self-interest will automatically further the public interest.B. The non-substitutability of resources creates a conflict between private and public interests and calls for government intervention.C. The market system is the best system for overcoming the scarce resources-unlimited wants problem.D. Central direction by the government will improve resource allocation in a capitalistic economy.

22. In the simple circular flow model: A. households are buyers of resources.B. businesses are sellers of final products.C. households are sellers of final products.D. there are real flows of goods, services, and resources, but not money flows.

23. A market: A. reflects upsloping demand and downsloping supply curves.B. entails the exchange of goods, but not services.C. is an institution that brings together buyers and sellers.D. always requires face-to-face contact between buyer and seller.

24. Economists use the term "demand" to refer to: A. a particular price-quantity combination on a stable demand curve.B. the total amount spent on a particular commodity over a fixed time period.C. an upsloping line on a graph that relates consumer purchases and product price.D. a schedule of various combinations of market prices and amounts demanded.

25. When the price of Nike soccer balls fell, Ronaldo purchased more Nike soccer balls and fewer Adidas soccer balls. Which of the following best explains Ronaldo's decision to buy more Nike soccer balls? A. the substitution effectB. the income effect

Page 555: Microeconomics Study Guide

C. an increase in the demand for Nike soccer ballsD. the price effect

26. Which of the following will not cause the demand for product K to change? A. a change in the price of close-substitute product JB. an increase in incomes of buyers of product KC. a change in the price of KD. a change in consumer tastes for K

27. If Z is an inferior good, an increase in money income will shift the: A. supply curve for Z to the left.B. supply curve for Z to the right.C. demand curve for Z to the left.D. demand curve for Z to the right.

28. A decrease in the price of digital cameras will: A. cause the demand curve for memory cards to become vertical.B. shift the demand curve for memory cards to the right.C. shift the demand curve for memory cards to the left.D. not affect the demand for memory cards.

29. A decrease in the demand for recreational fishing boats might be caused by an increase in the: A. income of sports fishers.B. price of outboard motors.C. size and number of fish available.D. price of sailing boats.

30. Refer to the above diagram. A decrease in supply is depicted by a: A. move from point x to point y.B. shift from S1 to S2.C. shift from S2 to S1.D. move from point y to point x.

31. Suppose product X is an input in the production of product Y. Product Y in turn is a substitute for product Z. An increase in the price of X can be expected to: A. decrease the demand for Z.B. increase the demand for Z.C. have no effect on the demand for Z.D. decrease the supply of Z.

32. An increase in the excise tax on cigarettes raises the price of cigarettes by shifting the: A. demand curve for cigarettes rightward.B. demand curve for cigarettes leftward.C. supply curve for cigarettes rightward.

Page 556: Microeconomics Study Guide

D. supply curve for cigarettes leftward.

33. If the supply and demand curves for a product both decrease, then equilibrium: A. quantity must fall and equilibrium price must rise.B. price must fall, but equilibrium quantity may rise, fall, or remain unchanged.C. quantity must decline, but equilibrium price may rise, fall, or remain unchanged.D. quantity and equilibrium price must both decline.

34. In the above market, economists would call a government-set minimum price of $50 a: A. price ceiling.B. price floor.C. equilibrium price.D. fair price.

35. The basic formula for the price elasticity of demand coefficient is: A. absolute decline in quantity demanded/absolute increase in price.B. percentage change in quantity demanded/percentage change in price.C. absolute decline in price/absolute increase in quantity demanded.D. percentage change in price/percentage change in quantity demanded.

36. Which of the following is not characteristic of the demand for a commodity that is elastic? A. The relative change in quantity demanded is greater than the relative change in price.B. Buyers are relatively sensitive to price changes.C. Total revenue declines if price is increased.D. The elasticity coefficient is less than one.

37. The price elasticity of demand for widgets is 0.80. Assuming no change in the demand curve for widgets, a 16 percent increase in sales implies a: A. 1 percent reduction in price.B. 12 percent reduction in price.C. 40 percent reduction in price.D. 20 percent reduction in price.

38. The price elasticity of demand is generally: A. negative, but the minus sign is ignored.B. positive, but the plus sign is ignored.C. positive for normal goods and negative for inferior goods.D. positive because price and quantity demanded are inversely related.

39. The concept of price elasticity of demand measures: A. the slope of the demand curve.B. the number of buyers in a market.C. the extent to which the demand curve shifts as the result of a price decline.D. the sensitivity of consumer purchases to price changes.

Page 557: Microeconomics Study Guide

40. A firm can sell as much as it wants at a constant price. Demand is thus: A. perfectly inelastic.B. perfectly elastic.C. relatively inelastic.D. relatively elastic.

41. Which of the following statements is not correct? A. If the relative change in price is greater than the relative change in the quantity demanded associated with it, demand is inelastic.B. In the range of prices in which demand is elastic, total revenue will diminish as price decreases.C. Total revenue will not change if price varies within a range where the elasticity coefficient is unity.D. Demand tends to be elastic at high prices and inelastic at low prices.

42. If a firm finds that it can sell $13,000 worth of a product when its price is $5 per unit and $11,000 worth of it when its price is $6, then: A. the demand for the product is elastic in the $6-$5 price range.B. the demand for the product must have increased.C. elasticity of demand is 0.74.D. the demand for the product is inelastic in the $6-$5 price range.

43. In which of the following cases will total revenue increase? A. price falls and demand is inelasticB. price falls and supply is elasticC. price rises and demand is inelasticD. price rises and demand is elastic

44. Suppose that the price of peanuts falls from $3 to $2 per bushel and that, as a result, the total revenue received by peanut farmers changes from $16 to $14 billion. Thus: A. the demand for peanuts is elastic.B. the demand for peanuts is inelastic.C. the demand curve for peanuts has shifted to the right.D. no inference can be made as to the elasticity of demand for peanuts.

45. The demand for autos is likely to be: A. less price elastic than the demand for Honda Accords.B. more price elastic than the demand for Honda Accords.C. of the same price elasticity as the demand for Honda Accords.D. perfectly inelastic.

46. We would expect the cross elasticity of demand between dress shirts and ties to be: A. positive, indicating normal goods.B. positive, indicating complementary goods.C. negative, indicating substitute goods.D. negative, indicating complementary goods.

Page 558: Microeconomics Study Guide

47. Suppose that a 10 percent increase in the price of normal good Y causes a 20 percent increase in the quantity demanded of normal good X. The coefficient of cross elasticity of demand is: A. negative and therefore these goods are substitutes.B. negative and therefore these goods are complements.C. positive and therefore these goods are substitutes.D. positive and therefore these goods are complements.

48. Which of the following is an example of market failure? A. negative externalitiesB. positive externalitiesC. public goodsD. all of these

49. If the demand curve reflects consumers' full willingness to pay, and the supply curve reflects all costs of production, then which of the following is true? A. The benefit surpluses shared between consumers and producers will be maximized.B. The benefit surpluses received by consumers and producers will be equal.C. There will be no consumer or producer surplus.D. Consumer surplus will be maximized, and producer surplus will be minimized.

50. Amanda buys a ruby for $330 for which she was willing to pay $340. The minimum acceptable price to the seller, Tony, was $140. Amanda experiences: A. a consumer surplus of $10 and Tony experiences a producer surplus of $190.B. a producer surplus of $200 and Tony experiences a consumer surplus of $10.C. a consumer surplus of $670 and Tony experiences a producer surplus of $200.D. a producer surplus of $10 and Tony experiences a consumer surplus of $190.

51. An efficiency loss (or deadweight loss): A. is measured as the combined loss of consumer surplus and producer surplus.B. results from producing a unit of output for which the maximum willingness to pay exceeds the minimum acceptable price.C. can result from underproduction, but not from overproduction.D. can result from overproduction, but not from underproduction.

52. (Last Word) In a cap-and-trade program: A. government fixes the price of pollution rights and firms choose how many permits to purchase.B. government fixes the maximum amount of a pollutant that firms can discharge and issues permits that firms can buy from and sell to each other.C. each firm is provided a fixed number of permits for a particular pollutant and no individual firm is allowed to acquire additional permits.D. firms can emit whatever type of pollutant they want, so long as the total tonnage does not exceed a government established quantity.

53. Nonexcludability describes a condition where: A. one person's consumption of a good does not prevent consumption of the good by others.B. there is no effective way to keep people from using a good once it comes into being.

Page 559: Microeconomics Study Guide

C. sellers can withhold the benefits of a good from those unwilling to pay for it.D. there is no potential for free-riding behavior.

Practice Exam One - Micro Fall 2011 Key

1. When economists say that people act rationally in their self interest, they mean that individuals: A. look for and pursue opportunities to increase their utility.B. generally disregard the interests of others.C. are mainly creatures of habit.D. are usually impulsive and unpredictable.

2. A person should consume more of something when its marginal: A. benefit exceeds its marginal cost.B. cost exceeds its marginal benefit.C. cost equals its marginal benefit.D. benefit is still positive.

3. Marginal costs exist because: A. the decision to engage in one activity means forgoing some other activity.B. wants are scarce relative to resources.C. households and businesses make rational decisions.D. most decisions do not involve sacrifices or tradeoffs.

4. Even though local newspapers are very inexpensive, people rarely buy more than one of them each day. This fact: A. is an example of irrational behavior.B. implies that reading should be taught through phonics rather than the whole language method.C. contradicts the economic perspective.D. implies that, for most people, the marginal benefit of reading a second newspaper is less than the marginal cost.

5. In deciding whether to study for an economics quiz or go to a movie, one is confronted by the idea(s) of: A. scarcity and opportunity costs.B. money and real capital.C. complementary economic goals.D. full production.6. Which of the following most closely relates to the idea of opportunity costs? A. tradeoffs.B. economic growth.C. technological change.D. capitalism.7. Economic theories: A. are useless because they are not based on laboratory experimentation.B. that are true for individual economic units are never true for the economy as a whole.C. are generalizations based on a careful observation of facts.D. are abstractions and therefore of no application to real situations.

Page 560: Microeconomics Study Guide

8. Macroeconomics approaches the study of economics from the viewpoint of: A. the entire economy.B. governmental units.C. the operation of specific product and resource markets.D. individual firms.

9. Any combination of goods lying outside of the budget line: A. implies that the consumer is not spending all his income.B. yields less utility than any point on the budget line.C. yields less utility than any point inside the budget line.D. is unattainable, given the consumer's income.

10. Refer to the above graphs. Assume that pizza is measured in slices and beer in pints. In which of the graphs is the opportunity cost of a pint of beer equal to one slice of pizza? A. graph AB. graph BC. graph CD. graph D

11. A production possibilities curve shows: A. that resources are unlimited.B. that people prefer one of the goods more than the other.C. the maximum amounts of two goods that can be produced, assuming the full use of available resources.D. combinations of capital and labor necessary to produce specific levels of output.

12. The typical production possibilities curve is: A. an upsloping line that is bowed out from the origin.B. a downsloping line that is bowed in toward the origin.C. a downsloping line that is bowed out from the origin.D. a straight upsloping line.

13. The construction of a production possibilities curve assumes: A. the quantities of all resources are unlimited.B. technology is fixed.C. some resources are unemployed.D. there is no inflation in the economy.

Answer the question on the basis of the following production possibilities tables for two countries, North Cantina and South Cantina:

Page 561: Microeconomics Study Guide

14. Refer to the above tables. If North Cantina is producing at production alternative B, the opportunity cost of the eleventh unit of consumer goods will be: A. 10 units of capital goods.B. 1/4 of a unit of capital goods.C. 8 units of capital goods.D. 1/8 of a unit of capital goods.

15. Which of the following is a distinguishing feature of a market system? A. public ownership of all capital.B. central planning.C. wide-spread private ownership of capital.D. a circular flow of goods, resources, and money.

16. Economic systems differ according to which two main characteristics? A. Who owns the factors of production, and the methods used to coordinate economic activity.B. The technology used in production, and the quantity and quality of natural resources.C. How goods are produced, and who gets them.D. The political system in place, and the degree of scarcity facing the economy.

17. The regulatory mechanism of the market system is: A. self-interest.B. private property.C. competition.D. specialization.

18. Specialization-the division of labor-enhances productivity and efficiency by: A. allowing workers to take advantage of existing differences in their abilities and skills.B. avoiding the time loss involved in shifting from one production task to another.C. allowing workers to develop skills by working on one, or a limited number, of tasks.D. all of the means identified in the other answers.

19. If consumer desire for product X increases, all of the following will occur except: A. an increase in the profits of industry X.B. an increase in the quantity of resources employed by industry X.C. an increase in the output of industry X.D. a decrease in the quantity of resources employed in industry X.

20. The market system's answer to the fundamental question "Who will get the goods and services?" is essentially: A. "Those willing and able to pay for them."B. "Those who physically produced them."C. "Those who most need them."D. "Those who get utility from them."

21. Which of the following best describes the invisible-hand concept? A. The desires of resource suppliers and producers to further their own self-interest will automatically further the public

Page 562: Microeconomics Study Guide

interest.B. The non-substitutability of resources creates a conflict between private and public interests and calls for government intervention.C. The market system is the best system for overcoming the scarce resources-unlimited wants problem.D. Central direction by the government will improve resource allocation in a capitalistic economy.

22. In the simple circular flow model: A. households are buyers of resources.B. businesses are sellers of final products.C. households are sellers of final products.D. there are real flows of goods, services, and resources, but not money flows.

23. A market: A. reflects upsloping demand and downsloping supply curves.B. entails the exchange of goods, but not services.C. is an institution that brings together buyers and sellers.D. always requires face-to-face contact between buyer and seller.

24. Economists use the term "demand" to refer to: A. a particular price-quantity combination on a stable demand curve.B. the total amount spent on a particular commodity over a fixed time period.C. an upsloping line on a graph that relates consumer purchases and product price.D. a schedule of various combinations of market prices and amounts demanded.

25. When the price of Nike soccer balls fell, Ronaldo purchased more Nike soccer balls and fewer Adidas soccer balls. Which of the following best explains Ronaldo's decision to buy more Nike soccer balls? A. the substitution effectB. the income effectC. an increase in the demand for Nike soccer ballsD. the price effect

26. Which of the following will not cause the demand for product K to change? A. a change in the price of close-substitute product JB. an increase in incomes of buyers of product KC. a change in the price of KD. a change in consumer tastes for K

27. If Z is an inferior good, an increase in money income will shift the: A. supply curve for Z to the left.B. supply curve for Z to the right.C. demand curve for Z to the left.D. demand curve for Z to the right.

28. A decrease in the price of digital cameras will:

Page 563: Microeconomics Study Guide

A. cause the demand curve for memory cards to become vertical.B. shift the demand curve for memory cards to the right.C. shift the demand curve for memory cards to the left.D. not affect the demand for memory cards.

29. A decrease in the demand for recreational fishing boats might be caused by an increase in the: A. income of sports fishers.B. price of outboard motors.C. size and number of fish available.D. price of sailing boats.

30. Refer to the above diagram. A decrease in supply is depicted by a: A. move from point x to point y.B. shift from S1 to S2.C. shift from S2 to S1.D. move from point y to point x.

31. Suppose product X is an input in the production of product Y. Product Y in turn is a substitute for product Z. An increase in the price of X can be expected to: A. decrease the demand for Z.B. increase the demand for Z.C. have no effect on the demand for Z.D. decrease the supply of Z.

32. An increase in the excise tax on cigarettes raises the price of cigarettes by shifting the: A. demand curve for cigarettes rightward.B. demand curve for cigarettes leftward.C. supply curve for cigarettes rightward.D. supply curve for cigarettes leftward.

33. If the supply and demand curves for a product both decrease, then equilibrium: A. quantity must fall and equilibrium price must rise.B. price must fall, but equilibrium quantity may rise, fall, or remain unchanged.C. quantity must decline, but equilibrium price may rise, fall, or remain unchanged.D. quantity and equilibrium price must both decline.

34. In the above market, economists would call a government-set minimum price of $50 a: A. price ceiling.B. price floor.C. equilibrium price.

Page 564: Microeconomics Study Guide

D. fair price.

35. The basic formula for the price elasticity of demand coefficient is: A. absolute decline in quantity demanded/absolute increase in price.B. percentage change in quantity demanded/percentage change in price.C. absolute decline in price/absolute increase in quantity demanded.D. percentage change in price/percentage change in quantity demanded.

36. Which of the following is not characteristic of the demand for a commodity that is elastic? A. The relative change in quantity demanded is greater than the relative change in price.B. Buyers are relatively sensitive to price changes.C. Total revenue declines if price is increased.D. The elasticity coefficient is less than one.

37. The price elasticity of demand for widgets is 0.80. Assuming no change in the demand curve for widgets, a 16 percent increase in sales implies a: A. 1 percent reduction in price.B. 12 percent reduction in price.C. 40 percent reduction in price.D. 20 percent reduction in price.

38. The price elasticity of demand is generally: A. negative, but the minus sign is ignored.B. positive, but the plus sign is ignored.C. positive for normal goods and negative for inferior goods.D. positive because price and quantity demanded are inversely related.

39. The concept of price elasticity of demand measures: A. the slope of the demand curve.B. the number of buyers in a market.C. the extent to which the demand curve shifts as the result of a price decline.D. the sensitivity of consumer purchases to price changes.

40. A firm can sell as much as it wants at a constant price. Demand is thus: A. perfectly inelastic.B. perfectly elastic.C. relatively inelastic.D. relatively elastic.

41. Which of the following statements is not correct? A. If the relative change in price is greater than the relative change in the quantity demanded associated with it, demand is inelastic.B. In the range of prices in which demand is elastic, total revenue will diminish as price decreases.C. Total revenue will not change if price varies within a range where the elasticity coefficient is unity.D. Demand tends to be elastic at high prices and inelastic at low prices.

Page 565: Microeconomics Study Guide

42. If a firm finds that it can sell $13,000 worth of a product when its price is $5 per unit and $11,000 worth of it when its price is $6, then: A. the demand for the product is elastic in the $6-$5 price range.B. the demand for the product must have increased.C. elasticity of demand is 0.74.D. the demand for the product is inelastic in the $6-$5 price range.

43. In which of the following cases will total revenue increase? A. price falls and demand is inelasticB. price falls and supply is elasticC. price rises and demand is inelasticD. price rises and demand is elastic

44. Suppose that the price of peanuts falls from $3 to $2 per bushel and that, as a result, the total revenue received by peanut farmers changes from $16 to $14 billion. Thus: A. the demand for peanuts is elastic.B. the demand for peanuts is inelastic.C. the demand curve for peanuts has shifted to the right.D. no inference can be made as to the elasticity of demand for peanuts.

45. The demand for autos is likely to be: A. less price elastic than the demand for Honda Accords.B. more price elastic than the demand for Honda Accords.C. of the same price elasticity as the demand for Honda Accords.D. perfectly inelastic.

46. We would expect the cross elasticity of demand between dress shirts and ties to be: A. positive, indicating normal goods.B. positive, indicating complementary goods.C. negative, indicating substitute goods.D. negative, indicating complementary goods.

47. Suppose that a 10 percent increase in the price of normal good Y causes a 20 percent increase in the quantity demanded of normal good X. The coefficient of cross elasticity of demand is: A. negative and therefore these goods are substitutes.B. negative and therefore these goods are complements.C. positive and therefore these goods are substitutes.D. positive and therefore these goods are complements.

48. Which of the following is an example of market failure? A. negative externalitiesB. positive externalitiesC. public goodsD. all of these

Page 566: Microeconomics Study Guide

49. If the demand curve reflects consumers' full willingness to pay, and the supply curve reflects all costs of production, then which of the following is true? A. The benefit surpluses shared between consumers and producers will be maximized.B. The benefit surpluses received by consumers and producers will be equal.C. There will be no consumer or producer surplus.D. Consumer surplus will be maximized, and producer surplus will be minimized.

50. Amanda buys a ruby for $330 for which she was willing to pay $340. The minimum acceptable price to the seller, Tony, was $140. Amanda experiences: A. a consumer surplus of $10 and Tony experiences a producer surplus of $190.B. a producer surplus of $200 and Tony experiences a consumer surplus of $10.C. a consumer surplus of $670 and Tony experiences a producer surplus of $200.D. a producer surplus of $10 and Tony experiences a consumer surplus of $190.

51. An efficiency loss (or deadweight loss): A. is measured as the combined loss of consumer surplus and producer surplus.B. results from producing a unit of output for which the maximum willingness to pay exceeds the minimum acceptable price.C. can result from underproduction, but not from overproduction.D. can result from overproduction, but not from underproduction.

52. (Last Word) In a cap-and-trade program: A. government fixes the price of pollution rights and firms choose how many permits to purchase.B. government fixes the maximum amount of a pollutant that firms can discharge and issues permits that firms can buy from and sell to each other.C. each firm is provided a fixed number of permits for a particular pollutant and no individual firm is allowed to acquire additional permits.D. firms can emit whatever type of pollutant they want, so long as the total tonnage does not exceed a government established quantity.

53. Nonexcludability describes a condition where: A. one person's consumption of a good does not prevent consumption of the good by others.B. there is no effective way to keep people from using a good once it comes into being.C. sellers can withhold the benefits of a good from those unwilling to pay for it.D. there is no potential for free-riding behavior.

Test 2 practice MCQ (Answers are at the end)

1. If the demand for product X is inelastic, a 4 percent increase in the price of X will:

A. decrease the quantity of X demanded by more than 4 percent.

B. decrease the quantity of X demanded by less than 4 percent.

Page 567: Microeconomics Study Guide

C. increase the quantity of X demanded by more than 4 percent.

D. increase the quantity of X demanded by less than 4 percent.

2. Suppose Aiyanna's Pizzeria currently faces a linear demand curve and is charging a very high price per pizza and doing very little business. Aiyanna now decides to lower pizza prices by 5 percent per week for an indefinite period of time. We can expect that each successive week:

A. demand will become more price elastic.

B. price elasticity of demand will not change as price is lowered.

C. demand will become less price elastic.

D. the elasticity of supply will increase.

3. Refer to the above diagram and assume a single good. If the price of the good increased from $5.70 to $6.30 along D1, the price elasticity of demand along this portion of the demand curve would be:

A. 0.8.

B. 1.0.

C. 1.2.

D. 2.0.

4. The elasticity of supply of product X is unitary if the price of X rises by:

A. 5 percent and quantity supplied rises by 7 percent.

B. 8 percent and quantity supplied rises by 8 percent.

C. 10 percent and quantity supplied stays the same.

D. 7 percent and quantity supplied rises by 5 percent.

5. It takes a considerable amount of time to increase the production of pork. This implies that:

A. a change in the demand for pork will not affect its price in the short run.

Page 568: Microeconomics Study Guide

B. the short-run supply curve for pork is less elastic than the long-run supply curve for pork.

C. an increase in the demand for pork will elicit a larger supply response in the short run than in the long run.

D. the long-run supply curve for pork is less elastic than the short-run supply curve for pork.

6. The supply curve of a one-of-a-kind original painting is:

A. relatively elastic.

B. relatively inelastic.

C. perfectly inelastic.

D. perfectly elastic.

7. We would expect the cross elasticity of demand between dress shirts and ties to be:

A. positive, indicating normal goods.

B. positive, indicating complementary goods.

C. negative, indicating substitute goods.

D. negative, indicating complementary goods.

8. Suppose that a 10 percent increase in the price of normal good Y causes a 20 percent increase in the quantity demanded of normal good X. The coefficient of cross elasticity of demand is:

A. negative and therefore these goods are substitutes.

B. negative and therefore these goods are complements.

C. positive and therefore these goods are substitutes.

D. positive and therefore these goods are complements.

9. Which of the following goods will least likely suffer a decline in demand during a recession?

A. Dinner at a nice restaurant

Page 569: Microeconomics Study Guide

B. iPods

C. Toothpaste

D. Plasma screen and LCD TVs

10. Generally speaking, the demand for luxury goods is more price elastic than is the demand for necessities.

True False

11. When the percentage change in price is greater than the resulting percentage change in quantity demanded:

A. a decrease in price will increase total revenue.

B. demand may be either elastic or inelastic.

C. an increase in price will increase total revenue.

D. demand is elastic.

12. If a firm finds that it can sell $13,000 worth of a product when its price is $5 per unit and $11,000 worth of it when its price is $6, then:

A. the demand for the product is elastic in the $6-$5 price range.

B. the demand for the product must have increased.

C. elasticity of demand is 0.74.

D. the demand for the product is inelastic in the $6-$5 price range.

13. Which of the following generalizations is not correct?

A. The larger an item is in one's budget, the greater the price elasticity of demand.

B. The price elasticity of demand is greater for necessities than it is for luxuries.

C. The larger the number of close substitutes available, the greater will be the price elasticity of demand for a particular product.

D. The price elasticity of demand is greater the longer the time period under consideration.

Page 570: Microeconomics Study Guide

14. Marginal utility can be:

A. positive, but not negative.

B. positive or negative, but not zero.

C. positive, negative, or zero.

D. decreasing, but not negative.

15. Suppose that MUx/Px exceeds MUy/Py. To maximize utility the consumer who is spending all her money income should buy:

A. less of X only if its price rises.

B. more of Y only if its price rises.

C. more of Y and less of X.

D. more of X and less of Y.

16. When a consumer shifts purchases from product X to product Y the marginal utility of:

A. X falls and the marginal utility of Y rises.

B. X rises and the marginal utility of Y falls.

C. both X and Y rises.

D. both X and Y falls.

17. Diminishing marginal utility explains why:

A. the income effect exceeds the substitution effect.

B. the substitution effect exceeds the income effect.

C. supply curves are upsloping.

D. demand curves are downsloping.

18. According to prospect theory, firms are more likely to shrink packages than raise prices because:

Page 571: Microeconomics Study Guide

A. consumers feel the loss of a price increase more than they feel the loss of buying a smaller package for their money.

B. they don't understand that consumers recognize price increases easily, regardless of what form they take.

C. consumers associate smaller packages with higher quality luxury goods.

D. consumers are generally trying to downsize their purchases and lead simpler lives.

19. Prospect theory and the work of behavioral economists confirm that consumers are economically rational.

True False

20. If the price of A is $12 and the price of B is $3, the budget line tells us that a consumer in effect can trade:

A. 12 units of A for 3 of B.

B. 1 unit of A for 4 of B.

C. 1 unit of A for 3 of B.

D. 1 unit of B for 4 of A.

21. AA is Al's indifference curve and BB is Betty's. Al and Betty have the same budget line, LL. This information implies that:

A. Al's demand for X is greater than Betty's.

B. Al's demand for Y is greater than Betty's.

C. Al and Betty have the same demand for both products.

D. Al will buy some of X, but Betty will not.

22. In the above diagram:

A. the consumer is indifferent between points A and B, but neither point maximizes his utility.

Page 572: Microeconomics Study Guide

B. the consumer is indifferent between points A and B and either point will maximize his utility.

C. any combination of X and Y entailing more of Y and less of X than shown at B would be preferred.

D. any combination of X and Y entailing more of X and less of Y than shown at A would be preferred.

23. "Essential" water is cheaper than "nonessential" diamonds because:

A. new industrial uses for diamonds have been discovered.

B. the supply of water is great relative to demand and the supply of diamonds is small relative to demand.

C. although the total utility of diamonds is greater, their marginal utility is small.

D. the supply of diamonds is great relative to demand and the supply of water is small relative to demand.

24. Why do people tend to eat more at all-you-can-eat buffet restaurants than at restaurants where each item is purchased separately?

A. Once the all-you-can-eat meal is purchased, consumers view additional trips back to the buffet as having a price of zero.

B. MU/P is greater at all-you-can-eat restaurants.

C. People who eat at all-you-can-eat restaurants do not experience diminishing marginal utility.

D. Food at all-you-can-eat restaurants tends to have fewer calories, so consumers feel the need to consume a greater volume of food.

25. Refer to the above diagram. The budget line shift that moves the consumer's equilibrium from point A to point B suggests:

A. an increase in the demand for product X.

B. a decrease in the demand for product X.

C. no change in the demand for product X.

Page 573: Microeconomics Study Guide

D. that X is an inferior good.

test 2 practice Key

1. B

2. C

3. C

4. B

5. B

6. C

7. D

8. C

9. C

10. TRUE

11. C

12. A

13. B

14. C

15. D

16. B

17. D

18. A

19. FALSE

20. B

Page 574: Microeconomics Study Guide

21. B

22. A

23. B

24. A

25. B

Share this set• Share on Facebook • Share on Twitter• • About this setCreated by:Midlightstar on March 21, 2012 Subjects:EconomyDescription:Consumer BehaviorLog in to favorite or report as inappropriate.Pop outDiscussNo MessagesYou must log in to discuss this set.Chapter 6 (MC)Both Sides is the satisfaction or pleasure one gets from consuming itThe utility of a good or service:

1/76Preview our new flashcards mode! Study: SpellerLearnTest

Games: ScatterSpace Race

Tools: PrintExportCopy

Page 575: Microeconomics Study Guide

CombineEmbed

Order by 76 termsTerms Definitionsis the satisfaction or pleasure one gets from consuming it The utility of a good or service:positive, negative, or zero Marginal utility can be:negative Mary says, "You would have to pay me $50 to attend that pro wrestling event." For Mary, the marginal utility of that event is:utility The ability of a good or service to satisfy wants is called:satisfies consumer wants A product has utility if it:beyond some point additional units of a product will yield less and less extra satisifacion to a consumer The law of diminishing marginal utility states that:increase the marginal utility of the last unit consumed of this good If the price of product X rises, then the resulting decline in the amount purchased will:change in total utility obtained by consuming one more unit of a good Marginal utility is the:satisfaction that a consumer derives from a good or service Utility refers to the:summing the marginal utilities of each unit consumed Total utility may be detrmined by:is positive, but may be either increasing or decreasing If total utility is increasing, marginal utility:zero Where total utility is at a maximum, marginal utility is:demand curves slope downward The law of diminishing marginal utility explains why:Alex may have to lower the price to convince Kara to buy a second slice While eating at Alex's "Pizza by the slice" restaurant., Kara experiences diminishing marginal utility. She gained 10 units of satisfaction from her first slice pizza consumed, and would only recieve 5 units of satisfaction from consuing a second slice. Based on this information we can conclude that:consumers behave rationally, attempting to maximize their satisfaction The theory of consumer behavior assumes that:marginal utility obtained from the last dollar spent on each product is the same To maximize utility a consumer should allocate money income so that the:more of X and less of Y Suppose that MUx/Px exceeds MUy/Py. To maximize utility the consumer who is spending all her money income should buy:more of B and less of A Ben is exhausting his money income consuming products A and B in such quantities that MUa/Pa = 5 and MUb/Pb = 8. Ben should purchase:their marginal utilities are the same Suppose you have a limited money income and you are purchasing products products A and B whose prices happen to be the same. To maximize your utility you should purchase A and B in such amounts that:MUa/Pa = MUb/Pb = MUc/Pc ... = MUn/Pn A consumer is maximizing her utility with a particular money income when:should buy less B and more A Suppose that Ms. Thomson is currently exhausting her money income by purchasing 10 units of A and 8 units of B at prices of $2 and $4 respectively. The marginal utility of hte last units of A and B are 16 and 24 respectively. These data suggest that Ms. Thomson:the marginal utility of the last dollar spent on each good purchased will be the same If a rational consumer is in equilibrium, which of the following conditions will hold true?ratios of the marginal utility of each product purchased divided by its price are equal A consumer who has a limited budget will maximize utility or satisfaction when the:decrease the marginal utility per dollar spend on A An increase in the price of product A will:total utility The theory of consumer behavior assumes that consumers attempt to maximize:

Page 576: Microeconomics Study Guide

X rises and the marginal utility of Y falls When a consumer shifts purchases from product X to product Y the marginal utility of:has a higher "marginal utility to price ratio" for the hockey game than for the play Prashanth decides to buy a $75 ticket to a particular New York professional hockey game rather than $50 ticket for a particular Broadway play. We can conclude that Prashanth:demand curves are downsloping Diminishing marginal utility explains why:they all help explain the downsloping demand curve What do the income effect, the substitution effect, and diminishing marginal utility have in common?marginal utility diminishes as more of a product is consumed A consumer's demand curve for a product is downsloping becauseone should consume less of time-intensive goods In introducing the opportunity cost of time into the theory of consumer we find that, all else equal:Apple introduced new features to entice previous buyers to purchase new models How did Apple overcome consumers' diminishing marginal utility for iPods?essential goods may be cheap while nonessential goods may be expensive The diamond-water paradox arises because:the price of a product is related to its marginal utility, not its total utility The diamond-water paradox occurs because:the supply of water is great relative to demand and the supply of diamonds is small relative to demand "Essential" water is cheaper than "nonessential" diamonds because:increases the amount of health care consumed by reducing the price of additional units of care The fact that most medical purchases are financed through insurance:once the all-you-can-eat meal is purchased, consumers view additional trips back to the buffet as having a price of zero who do people tend to eat more at all-you-can-eat buffet restaurants than at restaurants where each item is purchased separately?

36. If a product is in surplus supply, we can conclude that its price:A. is below the equilibrium level.B. is above the equilibrium level.C. will rise in the near future.D. is in equilibrium. B37. If the supply of a product decreases and the demand for that product simultaneously increases, we canconclude that:A. equilibrium price must rise, but equilibrium quantity may either rise, fall, or remain unchanged.B. equilibrium price must rise and equilibrium quantity must fall.C. equilibrium price and equilibrium quantity must both increase.D. equilibrium price and equilibrium quantity must both decline. A38. The rationing function of prices refers to the:A. tendency of supply and demand to shift in opposite directions.B. fact that ration coupons are needed to alleviate wartime shortages of goods.C. capacity of a competitive market to equate the quantity demanded and the quantity supplied.D. ability of the market system to generate an equitable distribution of income. C39. If there is a shortage of product X, we can predict that:A. fewer resources will be allocated to the production of this good.

Page 577: Microeconomics Study Guide

B. the price of the product will rise.C. the price of the product will decline.D. the supply curve will shift to the left and the demand curve to the right, thereby eliminating the shortage. B40. In the following question, you are asked to determine, other things being equal, the effects of a given change in a determinant of demand or supply for product X upon: (i) the demand (D) for, or supply (S) of, X,(ii) the equilibrium price (P) of X and (iii) the equilibrium quantity (Q) of X. An increase in income, if X is a normal good, will:A. increase D, increase P, and increase Q.B. increase D, increase P, and decrease Q.C. increase S, increase P, and increase Q.D. decrease D, increase P, and increase Q. A41. In the following question you are asked to determine, other things being equal, the effects of a given change in a determinant of demand or supply for product X upon: (i) the demand (D) for, or supply (S) of, X,(ii) the equilibrium price (P) of X and (iii) the equilibrium quantity (Q) of X.An increase in the tastes and preferences for X will:A. increase S, decrease P, and increase Q.B. decrease S, decrease P, and decrease Q.C. increase D, increase P, and increase Q.D. decrease D, decrease P, and decrease Q. C42. Given a down-sloping demand curve and an up-sloping supply curve for a product, a decrease in resourceprices will:A. increase equilibrium price and quantity.B. decrease equilibrium price and quantity.C. decrease equilibrium price and increase equilibrium quantity.D. increase equilibrium price and decrease equilibrium quantity. C43. If an Australian importer can purchase GBP10 000 for AUD20 000, then the rate of exchange:A. in Australia will be AUD1 equals GBP2.B. in Australia will be AUD2 equals GBP1.C. in Great Britain will be AUD1 equals GBP2.D. in Great Britain will be AUD0.5 equals GBP1. B44. The Australian supply of British pounds is:A. down-sloping, because a lower dollar price of pounds means that Australian goods are cheaper to the British.B. up-sloping, because a higher dollar price of pounds means that Australian goods are cheaper to the British.C. up-sloping, because a lower dollar price of pounds means that Australian goods are cheaper to the British.D. down-sloping, because a higher dollar price of pounds means that Australian goods are cheaper to the British. B45. A depreciation of the Australian dollar will tend to:A. decrease the prices of both Australian imports and exports.B. increase the prices of both Australian imports and exports.C.decrease the prices of the goods that Australians import, but increase the prices to foreigners of the goods that Australians export.D.increase the prices of the goods that Australians import, but decrease the prices to foreigners of the goods that Australians export. D46. Assume that France and Britain have floating exchange rates. Other things being unchanged, if economic growth is more rapid in France than in Britain:A. gold bullion will flow out of France.B. the franc will depreciate.

Page 578: Microeconomics Study Guide

C. the pound will depreciate.D. the franc will appreciate. B47. Assume that France and Britain have floating exchange rates. Other things being unchanged, if the price level is stable in Britain but France experiences rapid inflation:A. gold bullion will flow into France.B. the franc will depreciate.C. the pound will depreciate.D. the franc will appreciate. B48. Surpluses drive competitive prices up; shortages drive them down.A. TrueB. False B49. If demand increases and supply simultaneously decreases, equilibrium price will rise.A. TrueB. False B50. The rationing function of prices refers to the fact that the government must distribute any surplus goods thatmay be left in a competitive market.A. TrueB. False B51. Consumers buy more of normal goods as incomes rise.A. TrueB. False B52. Toothpaste and toothbrushes are substitute goods.A. TrueB. False BSet ChampionsThere are no high scores or champions for this set yet. You can sign up or log in to be the first!

Demand, Supply, and Market Equilibrium 42ANSWERS TO END-OF-CHAPTER QUESTIONS3-1 Explain the law of demand. Why does a demand curve slope downward? How is a market demand curve derived from individual demand curves? As prices change because of a change in supply for a commodity, buyers will change the quantity they demand of that item. If the price drops, a larger quantity will be demanded. If the price rises, a lesser quantity will be demanded. The demand curve slopes downward because of diminishing marginal utility, and the substitution and income effects. Because successive units of a good provide less additional utility than the previous units, buyers will only pay for these smaller amounts of utility if the price is lowered. When the price of a commodity decreases relative tothat of substitutes, a buyer will substitute the now cheaper commodity for those whose prices

Page 579: Microeconomics Study Guide

have not changed. At the same time, the decreased price of the commodity under discussion will make the buyer wealthier in real terms. More can be bought of this commodity (as well as of others whose prices have not changed). Thus, the substitution and income effects reinforce each other: More will be bought of a normal (or superior) commodity as its price decreases. On a graph with price on the vertical axis and quantity on the horizontal, this is shown as a demand curve sloping downward from left to right. The market demand curve is derived by horizontally summing the individual demand curves. 3-2 What are the determinants of demand? What happens to the demand curve when each of these determinants changes? Distinguish between a change in demand and a change in the quantity demanded, noting the cause(s) of each. The fundamental determinant of demand is the price of the commodity under consideration: a change in price causes movement along the commodity’s demand curve. This movement is called a change in quantity demanded. Decreased price leads to movement down the demand curve: There is an increase in quantity demanded. Increased price leads to movement up the demand curve: There is a decrease in quantity demanded. In addition, there are determinants of demand, which are factors that may shift the demand curve, i.e., cause a “change in demand.” These are the number of buyers, the tastes (or desire) of the buyers for the commodity, the income of the buyers, the changes in price of related commodities (substitutes and complements), and expectations of the buyers regarding the future price of the commodity under discussion. The following will lead to increased demand: more buyers, greater desire for the commodity, higher incomes (assuming a normal good), lower incomes (assuming an inferior good), an increased price of substitutes, a decreased price of complements, and an expectation of higher future prices or incomes. This increased demand will show as a shift of the entire demand curve to the right. The reverse of all the above will lead to decreas

Page 580: Microeconomics Study Guide

ed demand and will show as a shift of the entire demand curve to the left. 3-3 (Key Question) What effect will each of the followinghave on the demand for small automobiles such as the Mini Cooper and Smart car? a. Small automobiles become more fashionable. b. The price of large automobiles rises (with the price of small autos remaining the same). c. Income declines and small autos are an inferior good. d. Consumers anticipate the price of small autos will greatly come down in the near future. e. The price of gasoline substantially drops. Demand increases in (a), (b), and (c); decreases in (d). The lastone (e) is ambiguous. As autos and gas are complements, one could argue that the decrease in gas prices would stimulate demand Demand, Supply, and Market Equilibrium 43for all cars, including small ones. However, one could also argue that small cars are attractive to consumers because of fuel efficiency, and that a decrease in gas prices effectively reduces the price of the “gas guzzling” substitutes. That would encourage consumers to switch from smaller to larger cars (SUVs), and demand for small automobiles would fall. [This presents a good illustration of the complexity of many of these changes.] 3-4 Explain the law of supply. Why does the supply curve slope upward? How is the market supply curve derived from the supply curves of individual producers? As prices rise because of increased demand for a commodity, producers find it more and more profitable to increase the quantity they offer for sale; that is, the supply curve will slope upward from left to right. Clearly, firms would rather sell at a higher price than at a lower price. Moreover, it is necessary for firms to demand a higher price as they increase production. This comes about because as they produce more and more, they start to run up against capacity constraints and costs rise. At any given time, a plant has a given size. As production increases, the firm will need to add an extra shift and then a third shift, both perhaps at higher wages. It

Page 581: Microeconomics Study Guide

may run out of warehouse space and have to rent at higher cost from another firm. It may have to pay extra to get increasingly urgent raw material, and so on. The market supply curve is derived by horizontally adding the individual supply curves. 3-5 What are the determinants ofsupply? What happens to the supply curve when each of these determinants changes? Distinguish between a change in supply and a change in the quantity supplied, noting the cause(s) of each. The fundamental determinant of supply is the price of the commodity. As price increases, the quantity supplied increases. An increase in price causes a movement up a given supply curve. A decrease in price causes a movement down a given supply curve. The non-price determinants of supply are: resource (input) prices, technology, taxes and subsidies, prices of other related goods, expectations, and the number of sellers. If one or more of these change, there will be a change in supply and the whole supply curve will shift to the right or the left. The following will cause an increase in supply: a decrease in resource (input) prices; improved (lower cost) technology; a decrease in business taxes, an increase in subsidies to business; a decrease in the price of another commodity that this firm was making, provided that commodity is a substitute in production (the firm can switch from the now lower priced one to our commodity); an expectation of lower prices in the future; and an increase in the number of sellers. The increase in supply caused by the noted changein one or more of the above will cause the entire supply curve to shift to the right. More will now be supplied at any given price. Alternatively expressed, any given amount will now be supplied at a lower price. The reverse of any or all the above changes inthe determinants of demand will cause a decrease in demand and will be shown as a shift of the supply curve to the left. Less will now be supplied at any given price. Alternatively expressed, any given amount will now be supplied at a higher price.

Page 582: Microeconomics Study Guide

3-6 (Key Question) What effect will each of the followinghave on the supply of automobile tires? a. A technological advance in the methods of producing tires. b. A decline in the number of firms in the tire industry. c. An increase in the price of rubber used in the production of tires. d. The expectation that the equilibrium price of auto tires will be lower in the future than it is currently. Demand, Supply, and Market Equilibrium 44e. A decline in the price of large tires used for semi-trucks and earth hauling rigs (with no change in the price of auto tires). f. The levying of a per-unit tax in each auto tire sold. g. The granting of a 50-cent-per-unit subsidy for each auto tire produced. Supply increases in (a), (d), (e), and (g); decreases in (b), (c), and (f). 3-7 “In the corn market, demand often exceeds supply and supply sometimes exceeds demand.” “The price of corn rises and falls in response to changes in supply and demand.” In which of these two statements are the terms “supply” and “demand” used correctly? Explain. In the first statement “supply” and “demand” are used incorrectly. Supply and demand are both schedules or curves that intersect where quantity supplied and quantity demanded are equal. One cannot talk of curves that intersect asexceeding or not exceeding each other. Supply and/or demand can change (the entire curves can shift). Each time this happens, it will create a new intersection of the two curves that will lead to changes in the equilibrium quantity and price of corn. Thus, the terms “supply” and “demand” are used correctly in the second statement. 3-8 (Key Question) Suppose the total demand for wheat and the total supply of wheat per month in the Kansas City grain market are as follows: Thousands of bushels

Page 583: Microeconomics Study Guide

demanded Price per bushel Thousand of bushels supplied Surplus (+) or shortage (-) 85 80 75 70 65 60 $3.40 3.70 4.00 4.30 4.60 4.90 72 73 75 77 79 81 _____ _____ _____ _____ _____ _____ a. What is the equilibrium price? What is the equilibrium quantity? Fill in the surplus-shortage column and use it to explain why your answers are correct. b. Graph the demand for wheat and the supply of wheat. Be sure to label the axes of your graph correctly. Label equilibrium price Pand the equilibrium quantity Q. c. Why will $3.40 not be the equilibrium price inthis market? Why not $4.90? “Surpluses drive

Page 584: Microeconomics Study Guide

prices up; shortages drive them down.” Do you agree? Data from top to bottom: -13; -7; 0; +7; +14; and +21. Demand, Supply, and Market Equilibrium 47At a price of $4.60, buyers only want to purchase 65,000 bushels, but sellers want to sell 79,000 bushels, resulting in a surplus of 14,000 bushels. The floor prevents the price from falling to eliminate the surplus. See the graph below. 3-15 What do economists mean when they say that“price floors and ceilings stifle the rationing function of prices and distort resource allocation”? When unrestrained, prices rise and fall to correct imbalances between the quantity supplied and quantity demanded in a market. If sellers find themselves at a given price with more output than consumers are willing to purchase, the price will fall. Likewise, if the market is not offering enough of a good to satisfy consumer demand, the price will rise. Price floors and ceilings prevent price movements to correct these imbalances.When a price is set above equilibrium (i.e. a price floor), sellers will produce more than the market can support, diverting resources away from more highly valued uses. Price ceilings result in an underallocation of resources toward a particular good, where the excess demand (shortage) reveals that consumers value the good (and therefore the resources used to produce it) morethan what the market currently offers. Demand, Supply, and Market Equilibrium 483-16 Advanced analysis: Assume that the demand for a commodity is represented by the equation P = 10 - .2Qdand supply by the equation P = 2 + .2Qs, where Qdand Qsare quantity demanded and

Page 585: Microeconomics Study Guide

quantity supplied, respectively, and Pis price. Using the equilibrium condition Qs= Qd, solve the equations to determine equilibrium price. Now determine equilibrium quantity. Graph the two equations to substantiate your answers. Demand is P= 10 – 2QdTherefore 5P= 50 – Qd= 50 – 5PSupply is P= 2 + 2QsTherefore 5P= 10 + Qsand Qs= –10 + 5PSubstitute Qdand Qsinto

Page 586: Microeconomics Study Guide

Qs= Qdequilibrium condition 50 – 5P= –10 + 5P 60 = 10Pand 6 = PNow substitute P= 6 in either Qdor Qsto determine equilibrium quantity Qd= 50 – 5P= 50 – 5(6) = 20 or Qs= –10 + 5P= –10 + 5(6) = 20 3-17 (Last Word) What is the current overall number of candidates waiting for an organ transplant? (For the answer, visit the United Network for Organ Sharing website, www.unos.org.) For what transplant organ is the waiting list the longest? (Select “Data” and “At a glance.”) Do you favor the establishment of a legal market for transplant organs? Why or why not? As of July 2006, there were nearly 99,000 on the waiting list, with kidneys having the longest list

Page 587: Microeconomics Study Guide

(about 71,000). Answers to the other questions will vary. 024681012140204060PriceQuantityQuestion 3-15SupplyDemandDemand, Supply, and Market Equilibrium 49

Private markets do not allocate resources in the most economically desirable way Market failure is said to occur whenever:the demand and supply curves don't reflect consumers' full willingness to pay for a good or service Demand-side market failures occur when:demand-side market failure People enjoy outdoor holiday lighting displays, and would be willing to pay to see these displays, but can't be made to pay. Because those who put up lights are unable to charge others to view them, they don't put up as many lights as people would like. This is an example of a:the demand and supply curves don't reflect the full cost of producing a good or service Supply-side market failures occur when:costs more to produce than it provides in benefits From soceity's perspective, in the presence of a supply-side market failure, the last unit of a good produced typically:supply-side market failure the trains of the transcontinental railway company, when shipping goods, sometimes emit sparks that start fies along the tracks and damage the property of others. If transcontinental does not pay for the damage it causes, what has occurred?Supply curves must reflect all costs of production, and demand curves must reflect consumers' full willingness to pay What two conditions must hold for a competitive market to produce efficient outcomes?the benefit surpluses shared between consumers and producers will be maximized If the demand curve reflects consumers' full willingess to pay, and the supply curve reflects all costs of production, then which of the following is true?is the difference between the maximum prices consumers are willing to pay for a product and the lower equilibrium price Consumer surplus:is the difference between he minimum prices producers are willng to accept for a product and the higher equilibrium price Producer surplus:under the demand curve and above the actual price Graphically, if the supply and demand curves are linear, consumer surplus is measured as the triangle:above the supply curve and below the actual price Graphically, producer surplus is measured as the area:equals the marginal cost of producing that particular unit A producer's minimum acceptable price for a particular unit fo a good:

Page 588: Microeconomics Study Guide

the combined amounts of consumer surplus and produceer surplus are maximized allocative efficiency occurs only at that output where:the maximum willingnes to pay for the last unit of output equals the minimum acceptable price of that unit of output At the output level defining allocative efficiency:the maximum willingness to pay for the last unit of output equals the minimum acceptable price of that unit of output At the output where the combined amounts of consumer and producer surplus are largest:is measured as the combined loss of consumer surplus and producer surplus An efficiency loss (or deadweight loss):maximum willingness to pay exceeds minimum acceptable price An efficiency loss (or deadweight loss) declines in size when a unit of output is produced for which:nonrivalry and nonexcludability The two main characteristics of a public good are:public goods Nonrivalry and nonexcludability are the main characteristics of:has benefits available to all, including nonpayers Unlike a private good, a public good:is available to all and cannot be denied to anyone A public good:private firms cannot stop consumers who are unwilling to pay for such goods from benefiting from them The market system does not produce public because:is nonrivalry and nonexcludability Public goods are those for which there:nonrival in consumption If one person's consumption of a good does not preclude another's consumption, the good is said to be:there is no effective way to keep people from using a good once it comes into being Nonexcludability describes a condition where:the market demand for a public good is nonexistent or understated Because of the free-rider problem:marginal benefit equals marginal cost At the optimal quantity of a public good:summing vertically the individual demand curves for the public good A demand curve for a public good is determined by:the 3rd unit should be produced Suppose that Mick and Cher are the only two members of society and are willing to pay $10 and $8, respectively, for the 3rd unit of a public good. Also, assume that the marginal cost of the 3rd unit is $17. We can conclude that:compare the benefits and costs associated with any economic project or activity Cost-benefit analysis attempts to:the optimal project size is the one for which MB = MC accoding to the marginal-cost-marginal-benefit rule:whenever they over-or underallocate resources to a project Economists consider governments to be "wasteful:"the benefits associated with a product exceed those accruing to people who consume it A positive externality or spillover benefit occurs when:the total cost of producing a good exceeds the cost borne by the producer A negaitve externality or spilloveer cost occurs when:price and output would increase If a good that generates positive externalities were produced and priced to take into account these spillover benefits, then its:resources and currently underallocated to the provision of holiday lighting in Anytown Suppose that the anytown city government asks private citizens to donate money to support the town's annual holiday lighting display. Assuming that the citizens of Anytown enjoy the lighting display, the request for donations suggests that:benefit of abatemnt equals its marginal cost of abatement The socially optimal amount of pollution abatement occurs where society's marginal:diminishing marginal utility The marginal benefit to society of reducing pollution declines with increases in pollution abatement because of the law of:diminishing returns The marginal cost to society of reducing pollution rises with increases in pollution abatement because of the law of:diminishing returns The MC curves in the above diagram slope upward because of the law of:those enjoying the art to "free ride" since they cannot be made to bear any of the cost Suppose that Susie creates a

Page 589: Microeconomics Study Guide

work of art and displays it in a public place. Economists would expect:people will understate their enjoyment of the art in order to "free ride" Brinley puts on an art show in a public space, asking for donations based on how much people enjoy his work. Economists would expect that:private individuals can often negotiate their own resolution of externality problems According to the Coase Theorem:Darcy and Rachel to negotiate a mutually agreeable level of volume and/or selection of music Darcy and Rachel live down the hal from each other in the same dorm. Darcy like to play her music loudly down the hall, and Rachel finds the music annoying. A coase theorem solution for this problem would be for:the coase theorem Suppose that a large tree on Betty's property is blocking Chuck's view of the lake below. Betty accepts Chuck's offer to pay Betty $100 for the right to cut down the tree. This situation describes:government fixes the maximum amount of a pollutant that firms can discharge and issues permits that firms can buy from and sell to each other In a cap-and-trade program:assigns a property right to polluting the atmosphere A cap-and-trade program:prefer a carbon tax to cap-and-trade for reducing carbon dioxide emissions Because there are so many sources of carbon dioxide, making monitoring difficult and costly, many economists:

A nation's production possibilities curve is bowed out from the origin because: resources are not equally efficient in producing every good.92. Refer to the above table. If the economy is producing at production alternative C, the opportunity cost of the tenth unit of consumer goods will be: A. 4 units of capital goods.B. 2 units of capital goods.C. 3 units of capital goods.D. 1/3 of a unit of capital goods. D. 1/3 of a unit of capital goods.93. Refer to the above table. As compared to production alternative D, the choice of alternative C would: A. tend to generate a more rapid growth rate.B. be unattainable.C. entail unemployment.D. tend to generate a slower growth rate. A. tend to generate a more rapid growth rate.94. Refer to the above table. A total output of 3 units of capital goods and 4 units of consumer goods: A. is irrelevant because the economy is capable of producing a larger total output.B. will result in the maximum rate of growth available to this economy.C. would involve an inefficient use of the economy's scarce resources.D. is unobtainable in this economy. C. would involve an inefficient use of the economy's scarce resources.95. Refer to the above table. For this economy to produce a total output of 3 units of capital goods and 13 units of consumer goods it must: A. achieve economic growth.B. use its resources more efficiently than the data in the table now indicate.C. allocate its available resources most efficiently among alternative uses.D. achieve the full employment of available resources. A. achieve economic growth.96. Refer to the above table. For these data the law of increasing opportunity costs is reflected in the fact that: A. the amount of consumer goods that must be sacrificed to get more capital goods diminishes beyond a point.B. larger and larger amounts of capital goods must be sacrificed to get additional units of consumer goods.C. the production possibilities data would graph as a straight downsloping line.D. the economy's resources are presumed to be scarce. B. larger and larger amounts of capital goods must be sacrificed

Page 590: Microeconomics Study Guide

to get additional units of consumer goods.When an economy is operating under conditions of full employment, the production of more of commodity A will mean the production of less of commodity B because: resources are limited.The production possibilities curve is a frontier between what? a frontier between all combinations of two goods that can be produced and those combinations that cannot be produced.Assume an economy is operating at some point on its production possibilities curve, which shows civilian and military goods. If the output of military goods is increased, the output of civilian goods: must be decreased.Any point inside the production possibilities curve indicates: that more output could be produced with available resources.101. Refer to the above diagram. Other things equal, this economy will achieve the most rapid rate of growth if: A. the ratio of capital to consumer goods is minimized.B. it chooses point C.C. it chooses point B.D. it chooses point A. D. it chooses point A.102. Refer to the above diagram. This economy will experience unemployment if it produces at point: A. A.B. B.C. C.D. D. D. D.In drawing the production possibilities curve we assume that: technology is fixed.Which of the following is assumed in constructing a typical production possibilities curve? production technology is fixed.If the production possibilities curve were a straight down-sloping line, this would suggest that: resources are perfectly shiftable between the production of these two goods.Assume an economy is incurring unemployment. The effect of resolving this problem will be to: move the level of actual output to the economy's production possibilities curve.107. Which of the following is not correct? A typical production possibilities curve: A. indicates how much of two products a society can produce.B. reveals how much each additional unit of one product will cost in terms of the other product.C. specifies how much of each product society should produce.D. indicates that to produce more of one product society must forgo larger and larger amounts of the other product. C. specifies how much of each product society should produce.A point inside a production possibilities curve best illustrates: unemployment.109. Refer to the above diagram. This production possibilities curve is constructed so that: A. resources are presumed to be perfectly shiftable between bread and tractors.B. the opportunity cost of bread diminishes as more bread is produced.C. the opportunity cost of tractors increases as more bread is produced.D. the opportunity cost of both bread and tractors increases as more of each is produced. D. the opportunity cost of both bread and tractors increases as more of each is produced.110. Refer to the above diagram. Starting at point A, the opportunity cost of producing each successive unit of tractors is: A. a constant 2 units of bread.B. 2, 4, 6, and 8 units of bread.C. 8, 6, 4, and 2 units of bread.D. the reciprocal of the output of tractors. B. 2, 4, 6, and 8 units of bread.111. Refer to the above diagram. Starting at point E, the production of successive units of bread will cost:

Page 591: Microeconomics Study Guide

A. a constant 8 units of tractors.B. a constant 6 units of tractors.C. 1/8, 1/6, 1/4, and 1/2 units of tractors.D. 1/2, 1/4, 1/6, and 1/8 units of tractors. C. 1/8, 1/6, 1/4, and 1/2 units of tractors.In drawing a production possibilities curve we hold constant: both technology and resource supplies.The construction of a production possibilities curve assumes: technology is fixed.If an economy is operating inside its production possibilities curve for consumer goods and capital goods, it: can produce more of both consumer goods and capital goods by using resources that are currently idle.115. Refer to the above diagram. Points A, B, C, D, and E show: A. that the opportunity cost of bicycles increases, while that of computers is constant.B. combinations of bicycles and computers that society can produce by using its resources efficiently.C. that the opportunity cost of computers increases, while that of bicycles is constant.D. that society's demand for computers is greater than its demand for bicycles. B. combinations of bicycles and computers that society can produce by using its resources efficiently.116. Refer to the above diagram. This production possibilities curve is: A. convex to the origin because opportunity costs are constant.B. linear because opportunity costs are constant.C. concave to the origin because of increasing opportunity costs.D. convex to the origin because of increasing opportunity costs. C. concave to the origin because of increasing opportunity costs.117. Refer to the above diagram. If society is currently producing 9 units of bicycles and 4 units of computers and it now decides to increase computer output to 6, the cost: A. will be 4 units of bicycles.B. will be 2 units of bicycles.C. will be zero because unemployed resources are available.D. of doing so cannot be determined from the information given. A. will be 4 units of bicycles.118. Refer to the above diagram. The combination of computers and bicycles shown by point G is: A. attainable, but too costly.B. unattainable, given currently available resources and technology.C. attainable, but involves unemployment.D. irrelevant because it is inconsistent with consumer preferences. B. unattainable, given currently available resources and technology.119. Refer to the above diagram. If society is currently producing the combination of bicycles and computers shown by point D, the production of 2 more units of bicycles: A. cannot be achieved because resources are fully employed.B. will cost 1 unit of computers.C. will cost 2 units of computers.D. will cause some resources to become unemployed. B. will cost 1 unit of computers.120. Refer to the above diagram. The combination of computers and bicycles shown by point F: A. is unattainable, given currently available resources and technology.B. is attainable, but implies that the economy is not using all its resources.C. is irrelevant because it is inconsistent with consumer preferences.D. suggests that opportunity costs are constant. B. is attainable, but implies that the economy is not using all its resources.121. Refer to the above diagram. The movement down the production possibilities curve from point A to point E suggests that the production of:

Page 592: Microeconomics Study Guide

A. computers, but not bicycles, is subject to increasing opportunity costs.B. bicycles, but not computers, is subject to increasing opportunity costs.C. both bicycles and computers are subject to constant opportunity costs.D. both bicycles and computers are subject to increasing opportunity costs. D. both bicycles and computers are subject to increasing opportunity costs.The law of increasing opportunity costs states that: if society wants to produce more of a particular good, it must sacrifice larger and larger amounts of other goods to do so.What does the concept of opportunity cost suggest? suggests that the use of resources in any particular line of production means that alternative outputs must be forgone.The law of increasing opportunity costs exists because: resources are not equally efficient in producing various goods.The law of increasing opportunity costs is reflected in a production possibilities curve that is: concave to the origin.Opportunity cost is best defined as: the amount of one product that must be given up to produce one more unit of another product.If the output of product X is such that marginal benefit equals marginal cost: the correct amount of resources is being allocated to X's production.128. Refer to the above diagram for athletic shoes. The optimal output of shoes is: A. Q1.B. Q2.C. Q3.D. greater than Q3. B. Q2.129. Refer to the above diagram for athletic shoes. If the current output of shoes is Q1, then: A. society would consider additional units of shoes to be more valuable than alternative uses of those resources.B. society would consider additional units of shoes to be less valuable than alternative uses of those resources.C. society would experience a net loss by producing more shoes.D. resources are being allocated efficiently to the production of shoes. A. society would consider additional units of shoes to be more valuable than alternative uses of those resources.130. Refer to the above diagram for athletic shoes. If the current output of shoes is Q3, then: A. resources are being allocated efficiently to the production of shoes.B. society would consider additional units of shoes to be more valuable than alternative products.C. society would consider additional units of shoes to be less valuable than alternative products.D. society would experience a net gain by producing more shoes. C. society would consider additional units of shoes to be less valuable than alternative products.131. Refer to the above diagram for athletic shoes. If the current output of shoes is Q3, then: A. society should produce fewer shoes to achieve the optimal allocation of resources.B. society should produce more shoes to achieve the optimal allocation of resources.C. resources are being allocated efficiently to the production of shoes.D. shoes are more valuable to society than alternative products A. society should produce fewer shoes to achieve the optimal allocation of resources.Suppose that an economy is producing on its production possibilities curve, but is not producing quantities of each good where the marginal benefit equals the marginal cost for each good. How can this economy improve its allocation? by producing more of one good and less of the other.The optimal allocation of resources is found: where MB = MC.The basic difference between consumer goods and capital goods is that consumer goods satisfy wants directly while capital goods satisfy wants indirectly.Which of the following would be most likely to shift the production possibilities curve to the right? A. a sudden and substantial expansion of consumer wants

Page 593: Microeconomics Study Guide

B. an improvement in the literacy level and general level of educationC. a decline in the size of the population and labor forceD. shifting resources from the production of capital goods to the production of consumer goods B. an improvement in the literacy level and general level of educationWhich of the following will shift the production possibilities curve to the right? A. an increase in the unemployment rate from 6 to 8 percentB. a decline in the efficiency with which the present labor force is allocatedC. a decrease in the unemployment rate from 8 to 6 percentD. a technological advance that allows farmers to produce more output from given inputs D. a technological advance that allows farmers to produce more output from given inputsOther things equal, which of the following would shift an economy's production possibilities curve to the left? A. the discovery of a low-cost means of generating and storing solar energyB. the entrance of more women into the labor forceC. a law requiring mandatory retirement from the labor force at age 55D. an increase in the proportion of total output that consists of capital or investment goods C. a law requiring mandatory retirement from the labor force at age 55138. Refer to the above diagram. The concept of opportunity cost is best represented by the: A. shift of the production possibilities curve from PP1 to PP2.B. move from B on PP1 to E on PP2.C. move from B on PP1 to C on PP1.D. move from D inside PP1 to B on PP1. C. move from B on PP1 to C on PP1.139. Refer to the above diagram. Which of the following positions relative to PP1 would be the most likely to result in a future production possibilities curve of PP3, rather than PP2? A. A.B. B.C. C.D. D. A. A.140. Refer to the above diagram. An improvement in technology will: A. shift the production possibilities curve from PP1 to PP2.B. shift the production possibilities curve from PP2 to PP1.C. move the economy from A to C along PP1.D. move the economy from A, B, or C on PP1 to D. A. shift the production possibilities curve from PP1 to PP2.Which of the following statements, if any, is correct for a nation that is producing only consumer and capital goods? Other things equal, the more capital goods a nation produces, the greater will be its future growth rate.(Consider This) A direct cost of going to college is: A. tuition, while an indirect cost (opportunity cost) is books and other supplies.B. forgone income while in college, while an indirect cost (opportunity cost) is tuition.C. tuition, while an indirect cost (opportunity cost) is forgone income while in college.D. books and supplies, while an indirect cost (opportunity cost) is food and housing. C. tuition, while an indirect cost (opportunity cost) is forgone income while in college.(Consider This) An exception to the advice "go to college, stay in college, and earn a degree" occurs when: A. tuition expenses are high and rising.B. the opportunity cost of attending college is extraordinarily high.C. the price of textbooks is high and rising.D. the economy is growing rapidly and jobs are plentiful. B. the opportunity cost of attending college is extraordinarily high.

Page 594: Microeconomics Study Guide

If we say that two variables are directly related, this means that: an increase in one variable is associated with an increase in the other variable.If we say that two variables are inversely related, this means that: an increase in one variable is associated with a decrease in the other.Which of the following statements is correct? A. The value of the independent variable is determined by the value of the dependent variable.B. The value of the dependent variable is determined by the value of the independent variable.C. The dependent variable designates the "cause" and the independent variable the "effect."D. Dependent variables graph as upsloping lines; independent variables graph as downsloping lines. B. The value of the dependent variable is determined by the value of the independent variable.If two variables are inversely related, then as the value of one variable: increases, the value of the other decreases.If a positive relationship exists between x and y: the relationship will graph as an upsloping line.The slope of a straight line can be determined by: comparing the absolute vertical change to the absolute horizontal change between two points on the line.150. Refer to the above diagram. The variables X and Y are: A. inversely related.B. directly related.C. unrelated.D. negatively related. B. directly related.151. Refer to the above diagram. The vertical intercept: A. is 40.B. is 50.C. is 60.D. cannot be determined from the information given. B. is 50.152. Refer to the above diagram. The slope of the line: A. is - 1/4.B. is + 1/4.C. is .40.D. cannot be determined from the information given. B. is + 1/4.153. Refer to the above diagram. The equation that shows the relationship between Y and X is: A. Y = 50 + 1/4 X.B. X = 1/4Y.C. Y = .4X.D. Y = 1/4 X - 50. A. Y = 50 + 1/4 X.Assume a household would consume $100 worth of goods and services per week if its weekly income were zero and would spend an additional $80 per week for each $100 of additional income. Letting C represent consumption and Y represent income, the equation that summarizes this relationship is: C = 100 + .8Y.If the equation y = 5 + 6x was graphed, the slope would be what? slope would be +6.If the equation y = 15 - 4x was plotted, the slope would be what? slope would be -4.If the equation y = -10 + 2.5x was plotted: A. the vertical intercept would be -10.B. the slope would be -7.5.C. it would graph as a downsloping line.D. the slope would be -10. A. the vertical intercept would be -10.158. The movement from line A to line A' represents a change in: A. the slope only.

Page 595: Microeconomics Study Guide

B. the intercept only.C. both the slope and the intercept.D. neither the slope nor the intercept. B. the intercept only.159. In the above diagram variables x and y are: A. both dependent variables.B. directly related.C. inversely related.D. unrelated. C. inversely related.160. In the above diagram the vertical intercept and slope are: A. 4 and -1 1/3 respectively.B. 3 and -1 1/3 respectively.C. 3 and + 3/4 respectively.D. 4 and + 3/4 respectively. A. 4 and -1 1/3 respectively.161. Refer to the above diagram. The slope of curve ZZ at point A is approximately: A. +2.B. +2 1/2.C. -2 1/2.D. +4. B. +2 1/2.162. Refer to the above diagram. The slope of curve ZZ at point B is: A. infinity.B. zero.C. +1.D. -1. B. zero.163. Refer to the above diagram. The slope of curve ZZ at point C is approximately: A. -4.B. -2.C. -2 2/5.D. +3. C. -2 2/5.164. The slope of a line parallel to the vertical axis is: A. zero.B. one.C. infinite.D. one-half. C. infinite.165. The slope of a line parallel to the horizontal axis is: A. zero.B. one.C. infinite.D. one-half. A. zero.

Share this set• Share on Facebook • Share on Twitter• • About this setCreated by:

Page 596: Microeconomics Study Guide

suqiyuan12 on September 2, 2012 Log in to favorite or report as inappropriate.Pop outDiscussNo MessagesYou must log in to discuss this set.econ910 chapter 3Both Sides 1. A market:A. reflects up-sloping demand and down-sloping supply curves.B. entails the exchange of goods, but not services.C. is an institution that brings together buyers and sellers.D. always entails face-to-face contact between buyer and seller.C

1/52Preview our new flashcards mode! Study: SpellerLearnTest

Games: ScatterSpace Race

Tools: PrintExportCopyCombineEmbed

Order by 52 termsTerms Definitions1. A market:A. reflects up-sloping demand and down-sloping supply curves.B. entails the exchange of goods, but not services.C. is an institution that brings together buyers and sellers.D. always entails face-to-face contact between buyer and seller. C2. The law of demand states that:A. price and quantity demanded are inversely related.B. the larger the number of buyers in a market, the lower the product price.C. price and quantity demanded are directly related.D. consumers will buy more of a given product at high prices than they will at low prices. A3. Represented graphically, the market demand curve is:A. steeper than any individual demand curve that comprises it.B. greater than the sum of the individual demand curves.

Page 597: Microeconomics Study Guide

C. the horizontal sum of individual demand curves.D. the vertical sum of individual demand curves. C4. Economists use the term 'demand' to refer to:A. a particular price-quantity combination on a stable demand curve.B. the total amount spent on a particular commodity over a stipulated time period.C. an up-sloping line on a graph that relates consumer purchases and product price.D. a schedule of various combinations of market prices and quantities demanded. D5. When the price of a product increases, a consumer is able to buy less of it with a given money income. Thisdescribes:A. the cost effect.B. the inflationary effect.C. the income effect.D. the substitution effect. C6. An increase in the price of a product will reduce the amount purchased because:A. supply curves are up-sloping.B. the higher price means that real incomes have risen.C. consumers will substitute other products for the one whose price has risen.D. consumers substitute relatively high-priced products for relatively low-priced products. C7. The construction of demand and supply curves assumes that the primary variable that influences decisionsto produce and purchase goods is:A. price.B. expectations.C. preferences.D. incomes. A8. Alyssa rents 5 movies per month when the price is $3.00 each and 7 movies per month when the price is$2.50. Alyssa has demonstrated the:A. law of price.B. law of supply.C. actions of an irrational consumer.D. law of demand. D9. A demand curve is:A. the downward-sloping line relating the price of the good to the quantity demanded.B. the upward-sloping line relating price to quantity supplied.C. the curve that relates income to quantity demanded.D. showing the same relationship between two goods as a production possibilities frontier. A10. If the number of buyers in the market decreases, the:A. demand in the market will increase.B. demand in the market will decrease.C. supply in the market will increase.D. supply in the market will decrease. B11. A market demand is:A. a vertical summation of individual demand curves.B. a horizontal summation of individual demand curves.C. not responsive to change in tastes and preferences.D. determined solely by the number of buyers and sellers in the market. B12. A higher price for batteries would tend to:

Page 598: Microeconomics Study Guide

A. increase the demand for flashlights.B. increase the demand for electricity.C. decrease the demand for electricity.D. D: increase the demand for batteries. B13. Which of the following will not cause the demand for product K to change?A. A change in the price of close-substitute product J.B. An increase in consumer incomes.C. A change in the price of K.D. A change in consumer tastes. C14. Which of the following would not shift the demand curve for beef?A. A widely publicised study that indicates beef increases one's cholesterol.B. A reduction in the price of cattle feed.C. An effective advertising campaign by pork producers.D. A change in the incomes of beef consumers. B15. If the price of product L increases, the demand curve for close-substitute product J will:A. shift downward toward the horizontal axis.B. shift to the left.C. shift to the right.D. remain unchanged. C16. Two goods are complements if a decrease in the price of one good:A. increases the quantity demanded of the other good.B. reduces the demand for the other good.C. reduces the quantity demanded of the other good.D. raises the demand for the other good. D17. If goods A and B are complements, an increase in the price of A will result in:A. more of good A sold.B. more of good B sold.C. less of good B sold.D. no difference in the quantity sold of either good. C18. If the price of K declines, the demand curve for the complementary product J will:A. shift to the left.B. decrease.C. shift to the right.D. remain unchanged. C19. Which of the following is most likely to be an inferior good?A. Fur coats.B. Used clothing.C. Steak.D. Butter. B20. If X is a normal good, a rise in money income will shift the:A. supply curve for X to the left.B. supply curve for X to the right.C. demand curve for X to the left.D. demand curve for X to the right. D21. If consumer incomes increase, the demand for product X:A. will necessarily remain unchanged.

Page 599: Microeconomics Study Guide

B. may shift either to the right or left.C. will necessarily shift to the right.D. will necessarily shift to the left. B22. You love peanut butter. You hear on the news that 50% of the peanut crop in the South has been wiped out,which will cause the price to double by the end of the year. As a result:A. your demand for peanut butter will increase by the end of the year.B. your demand for peanut butter increases today.C. your demand for peanut butter falls as you look for a substitute good.D. you decide to give up peanut butter completely. B23. The term 'quantity demanded' refers to:A. the entire series of prices and quantities that comprise the demand schedule.B. a situation where the income and substitution effects do not apply.C. the amount of a product that will be purchased at some specific price.D. none of the above. C24. The law of supply indicates that:A. producers will offer more of a product at high prices than they will at low prices.B. the product supply curve is down-sloping.C. consumers will purchase less of a good at high prices than they will at low prices.D. producers will offer more of a product at low prices than they will at high prices. A25. The supply of a good is negatively related to the:A. price of inputs used to make the good.B. demand for the good by consumers.C. price of the good itself.D. amount of profit a firm can expect to receive from sale of the good. A26. A leftward shift of a product supply curve might be caused by:A. an improvement in the relevant technique of production.B. a decline in the prices of needed inputs.C. an increase in consumer incomes.D. some firms leaving an industry. D27. If the number of sellers in a market increases, the:A; demand in that market will increase.B. supply in that market will increase.C. supply in that market will decrease.D. demand in that market will decrease. Answer: B28. Lead is an important input in the production of crystal. If the price of lead decreases, all else equal, wewould expect the supply of:A. crystal to be unaffected.B. crystal to decrease.C. crystal to increase.D. lead to increase. C29. An improvement in production technology will:A. tend to increase equilibrium price.B. shift the supply curve to the left.C. shift the supply curve to the right.D. shift the demand curve to the left. C30. 'Because of unseasonably cold weather, the supply of oranges has substantially decreased.' This statement

Page 600: Microeconomics Study Guide

indicates that:A. consumers will be willing and able to buy fewer oranges at each possible price.B. the equilibrium quantity of oranges will rise.C. the amount of oranges that will be available at various prices has declined.D. the price of oranges will fall. C31. An increase in the price of oranges would lead to:A. an increased supply of oranges.B. a reduction in the prices of inputs used in orange production.C. an increased demand for oranges.D. a movement up the supply curve for oranges. D32. At the equilibrium price:A. buyers have an incentive to buy more.B. it is possible for there to be a shortage.C. firms have an incentive to increase production.D. the plans of buyers and sellers in the market are mutually consistent D33. Which of the following statements is incorrect?A. If demand increases and supply decreases, equilibrium price will rise.B. If supply increases and demand decreases, equilibrium price will fall.C. If demand decreases and supply increases, equilibrium price will rise.D. If supply declines and demand remains constant, equilibrium price will rise. C34. In which of the following instances will the effect upon equilibrium price be indeterminate; that is,dependent on the magnitude of the given shifts in supply and demand?A. Demand rises and supply rises.B. Supply falls and demand remains constant.C. Demand rises and supply falls.D. Supply rises and demand falls. A35. The data below shows supply and demand data for wheat:P($)QdQs5 45 774 50 733 56 682 61 611 67 57Refer to the above information. Equilibrium price will be:A. $4.B. $3.C. $2.D. $1. C

Economics may best be defined as:A. the interaction between macro and micro considerations.B. the study of the behaviour of people and institutions in the production, distribution and consumption of scarce goods.C. the empirical testing of value judgments through the use of induction and deduction.D. the use of policy to refute facts and hypotheses. B2. The study of economics is primarily concerned with:

Page 601: Microeconomics Study Guide

A. keeping private businesses from losing money.B. demonstrating that capitalistic economies are superior to socialistic economies.C. choices that are made in seeking to use scarce resources efficiently.D. determining the most equitable distribution of society's output. C3. Economic theories:A. are useless because they are not based upon laboratory experimentation.B. that are true for individual economic units are never true for the economy as a whole.C. are generalisations based upon a careful observation of facts.D. are abstractions and therefore have no application to real situations. C4. Which of the following is a correct statement?A. Economic concepts or laws that are valid during depression are necessarily valid during prosperity.B.Though not quantitatively exact, economic laws are useful because they allow us to make predictions that are meaningful and useful.C.Economics is as scientific as physics and chemistry because economic laws are as quantitatively precise as the laws of physics or chemistry.D. Since economics is concerned with questions of 'ought', it is a branch of applied ethics and is not scientific. B5. Generally speaking, it may be said that the inductive method:A. begins with hypotheses that are tested against real-world facts.B. confuses correlation with cause and effect.C. moves from facts to generalisations or theory.D. cannot be applied to economic analysis. C6. The deductive method:A. begins with hypotheses that are tested against real-world facts.B. confuses correlation with cause and effect.C. begins with facts and moves to generalisations or theory.D. applies to the physical sciences but not to social sciences. A7. In constructing models, economists:A. make simplified assumptions.B. include all available information.C. must use mathematical equations.D. attempt to duplicate the real world. A8. Economic models:A. are of limited use because they cannot be tested empirically.B. are limited to variables that are directly related to one another.C. emphasise basic economic relationships by abstracting from the complexities of the real world.D. are unrealistic and therefore are of no practical consequence. C9. The term ceteris paribus means:A. if event A precedes event B, A has caused B.B. economics deals with facts, not values.C. other things being equal.D. prosperity inevitably follows recession. C10. The basic purpose of the ceteris paribus assumption is to:A. allow one to reason about the relationship between two variables, without the intrusion of other variables.B. allow one to focus upon micro variables by ignoring macro variables.C. allow one to focus upon macro variables by ignoring micro variables.D. determine whether X causes Y or vice versa. A

Page 602: Microeconomics Study Guide

11. A 'hypothesis' is:A. a fundamental truth which all economists accept.B. a tentative, untested principle.C. the same as a normative statement.D. always the result of induction. B12. The term ceteris paribus means that:A. the associated statement is normative.B. many variables affect the variable under consideration.C. a number of relevant variables are assumed to be constant.D. when variable X increases, so does the related variable. C13. Microeconomics is concerned with:A. the aggregate or total levels of income, employment and output.B. a detailed examination of specific economic units which comprise the economic system.C. the concealment of detailed information about specific segments of the economy.D. the establishment of an overall view of the operation of the economic system. B14. Macroeconomics approaches the study of economics from the viewpoint of:A. the entire economy.B. governmental units.C. the operation of specific product and resource markets.D. individual firms. A15. Which of the following is a microeconomic statement?A. The real domestic output increased by 2.5% last year.B. Unemployment was 6.8% of the labour force last year.C. The price of wheat declined last year.D. The general price level increased by 4% last year. C16. Which of the following is a macroeconomic statement?A. The gross profit of all Australian businesses was $182 billion last year.B. The price of beef declined by 3% last year.C. General Motors' profit increased in 1988.D. The productivity of steelworkers increased by 1% in 1992. A17. A normative statement is one that:A. is based on the law of averages.B. pertains only to microeconomics.C. pertains only to macroeconomics.D. is based upon value judgments. D18. A positive statement is:A. derived by induction.B. derived by deduction.C. subjective and based upon a value judgment.D. objective and based upon facts. D19. 'Economics is concerned with using scarce productive resources efficiently in attempting to satisfysociety's material wants.' This statement is:A. positive, but incorrect.B. positive and correct.C. normative, but incorrect.D. normative and correct. B

Page 603: Microeconomics Study Guide

20. Dean says that, 'The imposition of a tax on beer will raise its price'. Kylie argues that 'Taxes should be imposed on beer because university students drink too much'. We can conclude that:A. Dean's statement is normative, but Kylie's is positive.B. Kylie's statement is normative, but Dean's is positive.C. both statements are normative.D. both statements are positive. B21. Achieving full employment is most likely to conflict with the goal of:A. price level stability.B. economic growth.C. an equitable distribution of income.D. economic security. A22. Assume the relationship between unemployment and inflation is such that a low rate of unemploymentresults in a high rate of inflation, and vice versa. On the basis of this relationship, we can say that:A. the goals of full employment and price level stability are compatible.B. the goals of full employment and price level stability are conflicting.C. society should seek to eliminate inflation, even if it means a high levelD. of unemployment.E. society should seek to eliminate unemployment, even if it means a high rateF. of inflation. B23. The 'fallacy of composition' states that:A. because economic systems are comprised of so many diverse economic units, economic laws arenecessarily inexact.B. the anticipation of a particular event can affect the nature or composition of the event, when it occurs.C. what is true for the individual must necessarily be true for the group.D. because event A precedes event B, A is necessarily the cause of B. C24. The 'post-hoc' fallacy states that:A. because event A precedes event B, A is necessarily the cause of B.B. the very attempt to accomplish a certain objective may create conditions that prohibit the achievement ofthat goal.C. events may drastically alter plans; therefore, intentions and actual accomplishments may differ considerably.D. generalisations that are accurate at the level of microeconomics may be inaccurate at the level of macroeconomics. A25. The safest way for an individual to leave a burning theatre is to run for the nearest exit. It is, therefore, alsothe best means of escape for a large audience. This illustrates:A. the 'post-hoc' fallacy.B. Wagner's law.C. the fallacy of composition.D. the fallacy of limited decisions. C26. Which of the following pertains to the notion that generalisations which apply to individuals are not alwaysvalid for a group?A. The law of large numbers.B. The law of averages.C. The fallacy of composition.D. The post-hoc fallacy. C27. The 'post-hoc' fallacy states that:A. positive statements are always followed by normative judgments.B. normative statements can never be proven true or false.

Page 604: Microeconomics Study Guide

C. if one acts on one's expectations, those expectations will always be fulfilled.D. cause and effect can be determined merely by observing the sequence of events. D28. Which of the following best illustrates the post-hoc fallacy?A. Because it was 30 degrees today, I worked up a sweat playing tennis.B. I took the day off work to go to the beach and that's why it rained.C. Because it rained at the football game, my new sweater got wet.D. Because I have studied diligently this semester, my grade average has improved. B29. Economics deals primarily with the concept of:A. scarcity.B. poverty.C. change.D. power. A30. Scarcity exists when:A. there is less than an infinite amount of a resource or good.B. society can meet the wants of every individual.C. there is less of a good or resource available than people wish to have.D. the government fails to produce goods. C31. A rational decision maker takes an action only if:A. marginal benefit is less than the marginal cost.B. marginal benefit is greater than the marginal cost.C. average benefit is greater than the average cost.D. marginal benefit is greater than the average cost but less than the marginal cost. B32. Making decisions 'at the margin' means that people:A. make those decisions that do not impose a marginal cost.B. evaluate how easily a decision can be reversed if problems arise.C. compare the marginal costs and marginal benefits of each decision.D. can ignore the cost of forgoing an alternative. C33. One difficulty economists face that some other scientists do not, is that:A. unlike other sciences, economic studies must include the largest economic player, the government.B. economists receive less government funding than other scientists.C. corporations are reluctant to disclose information necessary for economic research.D. experiments are often difficult in economics. D34. A macroeconomist would study each of the following, except the:A. impact of minimum wage laws on employment in the fast food industry.B. effect of changes in savings rates on GDP.C. impact of monetary policy on the rate of inflation.D. effect of tax policy on the rate of economic growth. A35. The fact that economic generalisations are abstract renders them impractical and useless.A. TrueB. False B36. If economic theories are solidly based upon relevant facts, then there can be no question as to the characterof appropriate economic policy.A. TrueB. FalseTrue False B37. Positive statements are expressions of value judgments.

Page 605: Microeconomics Study Guide

A. TrueB. False B38. The primary objective of economic science is to help businesspeople operate their firms profitably.A. TrueB. False B39. The basic goals of Australian capitalism are always complementary, in that the achievement of any onegoal simultaneously furthers the attainment of other basic goals.A. TrueB. False B40. A rational decision maker takes an action if and only if the marginal cost exceeds the marginal benefit.A. TrueB. False B41. Economics is the study of how fairly goods and services are distributed within society.A. TrueB. False B42. Scarcity means that there is less of a good or resource available than people wish to have.A. TrueB. False B

For economists, the word "utility" means pleasure and satisfactionIn economics, the pleasure, happiness, or satisfaction received from a product is called what? utilityWhen economists say that people act rationally in their self interest, they mean that individuals do what? look for and pursue opportunities to increase their utilityAccording to Emerson: "Want is a growing giant whom the coat of Have was never large enough to cover." According to economists, "Want" exceeds "Have" because what? productive resources are limitedAccording to economists, economic self-interest is what? a reality that underlies economic behaviorWhen entering a building, Sam diverts his path to go through an open door rather than make the physical effort to open the closed door that is directly in his path. This is an example of what? marginal benefit-marginal cost analysisJoe sold gold coins for $1000 that he bought a year ago for $1000. He says, "At least I didn't lose any money on my financial investment." His economist friend points out that in effect he did lose money, because he could have received a 3 percent return on the $1000 if he had bought a bank certificate of deposit instead of the coins. The economist's analysis in this case incorporates the idea of what? opportunity costsA person should consume more of something when its marginal means what? benefit exceeds its marginal costEconomics may best be defined as what? the social science concerned with how individuals, institutions, and society make optimal choices under conditions of scarcityThe study of economics is primarily concerned with what? choices that are made in seeking the best use of resourcesThe economic perspective refers to what? the making of purposeful decisions in a context of marginal costs and marginal benefits.what does the economic perspective entail? a comparison of marginal benefits and marginal costs in decision makingWhat does purposeful behavior suggest? individuals will make different choices because of different desired outcomesPurposeful behavior means that: people weigh costs and benefits to make decisions.Economics involves marginal analysis because: most decisions involve changes from the present situationWhy should you decide to go to a movie? if the marginal benefit of the movie exceeds its marginal cost.Marginal costs exist because: the decision to engage in one activity means forgoing some other activity.The assertion that "There is no free lunch" means what? all production involves the use of scarce resources and thus the sacrifice of alternative goods.

Page 606: Microeconomics Study Guide

Consumers spend their incomes to get the maximum benefit or satisfaction from the goods and services they purchase. What is this a reflection of? purposeful behaviorIf someone produced too much of a good, this would suggest that: the good was produced to the point where its marginal cost exceeded its marginal benefit.Even though local newspapers are very inexpensive, people rarely buy more than one of them each day. What does this fact imply? implies that, for most people, the marginal benefit of reading a second newspaper is less than the marginal cost.In deciding whether to study for an economics quiz or go to a movie, one is confronted by the idea(s) of what? scarcity and opportunity costsWhich one of the following expressions best states the idea of opportunity cost? "There is no such thing as a free lunch."What is an economic explanation for why most college-aged movie stars do not attend college? the opportunity cost in terms of reduced income is too greatSuppose that a university decides to spend $1 million to upgrade personal computers and scientific equipment for faculty rather than spend $1 million to expand parking for students. This example illustrates what? Opportunity costsWhich of the following most closely relates to the idea of opportunity costs? tradeoffsEconomists contend that most economic decisions are what? purposefulAlex sees that his neighbors' lawns all need mowing. He offers to provide the service in exchange for a wage of $20 per hour. Some neighbors accept Alex's offer and others refuse. Economists would describe Alex's behavior as: rational self-interest, because he attempting to increase his own income by identifying and satisfying someone else's wants.Kara was out jogging and despite being tired, decided to run one more mile. Based on her actions, economists would conclude that Kara: decided that the marginal benefit of running one more mile would outweigh the cost of the additional mile.What are Economic theories? are generalizations based on a careful observation of facts.In constructing models, what do economists do? make simplifying assumptionsWhat do Economic models do? emphasize basic economic relationships by purposefully simplifying the complexities of the real world.What does the term "ceteris paribus" mean? other things equalWhat is the basic purpose of the other-things-equal assumption? allow one to reason about the relationship between variables X and Y without the intrusion of variable Z.Suppose an economist says that "Other things equal, the lower the price of bananas, the greater the amount of bananas purchased." This statement indicates that: all factors other than the price of bananas (for example, consumer tastes and incomes) are assumed to be constant.The term "other things equal" means that: a number of relevant variables are assumed to be constantMacroeconomics approaches the study of economics from the viewpoint of: the entire economyWhat is associated with macroeconomics? an empirical investigation of the general price level and unemployment rates since 1990The problems of aggregate inflation and unemployment are: major topics of macroeconomics.Which of the following statements pertains to macroeconomics? A. Because the minimum wage was raised, Mrs. Olsen decided to enter the labor force.B. A decline in the price of soybeans caused farmer Wanek to plant more land in wheat.C. National income grew by 2.7 percent last year.D. The Pumpkin Center State Bank increased its interest rate on consumer loans by 1 percentage point. C. National income grew by 2.7 percent last year.What can Macroeconomics best be described as? study of the large aggregates of the economy or the economy as a whole.What is Microeconomics is concerned with? a detailed examination of specific economic units that make up the

Page 607: Microeconomics Study Guide

economic system.Microeconomics: is concerned with individual economic units and specific markets.Which of the following is a macroeconomic statement?A. The gross profits of all U.S. businesses were $182 billion last year.B. The price of beef declined by 3 percent last year.C. General Motors' profits increased last year.D. The productivity of steelworkers increased by 1 percent last year. A. The gross profits of all U.S. businesses were $182 billion last year.Which of the following is a microeconomic statement? A. The real domestic output increased by 2.5 percent last year.B. Unemployment was 6.8 percent of the labor force last year.C. The price of personal computers declined last year.D. The general price level increased by 4 percent last year. C. The price of personal computers declined last year.Which of the following statements is true? A. Microeconomics focuses on specific decision-making units of the economy; macroeconomics examines the economy as a whole.B. Macroeconomics focuses on specific decision-making units of the economy; microeconomics examines the economy as a whole.C. Every topic in economics is either a microeconomic or a macroeconomic issue; a topic cannot be both.D. Topics in microeconomics have public policy implications; topics in macroeconomics do not. A. Microeconomics focuses on specific decision-making units of the economy; macroeconomics examines the economy as a whole.A normative statement is one that: is based on value judgments.Which of the following is a normative statement? A. The temperature is high today.B. The humidity is high today.C. It is too hot to play tennis today.D. It will cool off later this evening. C. It is too hot to play tennis today.A positive statement is one that is: objective and is based on facts.Which of the following is a positive statement? A. The humidity is too high today.B. It is too hot to jog today.C. The temperature is 92 degrees today.D. Summer evenings are nice when it cools off. C. The temperature is 92 degrees today.What are Normative statements concerned with? what ought to be.What are Positive statements concerned with? what is.Most of the disagreement among economists involves: Normative statementsBen says that "An increase in the tax on beer will raise its price." Holly argues that "Taxes should be increased on beer because college students drink too much." We can conclude that: Holly's statement is normative, but Ben's is positive.What is the economizing problem? the need to make choices because economic wants exceed economic means.The economizing problem is one of deciding how to make the best use of: limited resources to satisfy virtually unlimited wants.When the economist says that economic wants are insatiable, this means that: these wants are virtually unlimited and therefore incapable of complete satisfaction.Why does the scarcity problem persist? persists because economic wants exceed available productive resources.The alternative combination of two goods which a consumer can purchase with a given money income is shown by what? a budget line.

Page 608: Microeconomics Study Guide

What does a budget line show? all possible combinations of two goods that can be purchased, given money income and the prices of the goods.61. Refer to the budget line shown in the diagram above. If the consumer's money income is $20, the: A. prices of C and D cannot be determined.B. price of C is $2 and the price of D is $4.C. consumer can obtain a combination of 5 units of both C and D.D. price of C is $4 and the price of D is $2. D. price of C is $4 and the price of D is $2.62. Refer to the budget line shown in the diagram above. If the consumer's money income is $20, which of the following combination of goods is unattainable? A. 4 units of C, and 6 units of D.B. 5 units of C, and no units of D.C. 1 unit of C, and 8 units of D.D. 2 units of C, and 6 units of D. A. 4 units of C, and 6 units of D.63. Refer to the budget line shown in the diagram above. The absolute value of the slope of the budget line is: A. MUC/MUD.B. one-half.C. PD/PC.D. PC/PD. D. PC/PD.In moving along a given budget line: the prices of both products and money income are assumed to be constant.An increase in money income does what? shifts the consumer's budget line to the right.66. The shift of the budget line from cd to ab in the above figure is consistent with: A. decreases in the prices of both M and N.B. an increase in the price of M and a decrease in the price of N.C. a decrease in money income.D. an increase in money income. a decrease in money income.Any combination of goods lying outside of the budget line is what? is unattainable, given the consumer's income.68. Suppose you have a money income of $10, all of which you spend on Coke and popcorn. In the above diagram, the prices of Coke and popcorn respectively are: A. $.50 and $1.00.B. $1.00 and $.50.C. $1.00 and $2.00.D. $.40 and $.50. A. $.50 and $1.00.A budget line shows the: alternative combinations of two goods that a consumer can purchase with a given money income.Other things equal, an increase in a consumer's money income does what? shifts her budget line rightward because she can now purchase more of both products.What does the slope of a budget line reflect? price ratio of the two products72. Suppose Elroy's budget line is as shown on the above diagram. If his tastes change in favor of Coke and against popcorn, the budget line will: be unaffected.Assume the price of product Y (the quantity of which is on the vertical axis) is $15 and the price of product X (the quantity of which is on the horizontal axis) is $3. Also assume that money income is $60. The absolute value of the slope of the resulting budget line: is 1/5.Suppose that Julia receives a $20 gift card for the local coffee shop, where she only buys lattes and muffins. If the price of a latte is $4 and the price of a muffin is $2, then we can conclude that Julia: can buy 5 lattes or 10 muffins if she chooses to buy only one of the two goods.Which of the following is a land resource?

Page 609: Microeconomics Study Guide

A. a computer programmerB. a computerC. silicon (sand) used to make computer chipsD. a piece of software used by a firm C. silicon (sand) used to make computer chipsWhich of the following is a labor resource? A. a computer programmerB. a computerC. silicon (sand) used to make computer chipsD. a piece of software used by a firm A. a computer programmerWhich of the following is a capital resource? A. a computer programmerB. a corporate bond issued by a computer manufacturerC. silicon (sand) used to make computer chipsD. a piece of software used by a firm D. a piece of software used by a firmThe four factors of production are: land, labor, capital, and entrepreneurial abilityWhich of the following lists includes only capital resources (and therefore no labor or land resources)? A. an ice arena; a professional hockey player; hockey uniforms.B. the owner of a new startup firm; a chemistry lab; a researcher.C. a hydroelectric dam; water behind the dam; power lines.D. autos owned by a car rental firm; computers at the car rental agency; the vans that shuffle rental customers to and from the airport. D. autos owned by a car rental firm; computers at the car rental agency; the vans that shuffle rental customers to and from the airport.Why is money not an economic resource? money, as such, is not productive.Economic resources are also called: factors of production.Which of the following is real capital? A. a pair of stockingsB. a construction craneC. a savings accountD. a share of IBM stock B. a construction craneThe main function of the entrepreneur is to: innovateWhich of the following is not a main function of the entrepreneur? A. make routine pricing decisions.B. innovate.C. assume the risk of economic losses.D. makes strategic business decisions. A. make routine pricing decisions.The production possibilities curve illustrates the basic principle that: if all the resources of an economy are in use, more of one good can be produced only if less of another good is produced.Which of the following will not produce an outward shift of the production possibilities curve? A. an upgrading of the quality of a nation's human resourcesB. the reduction of unemploymentC. an increase in the quantity of a society's labor forceD. the improvement of a society's technological knowledge B. the reduction of unemploymentUnemployment is illustrated by what? by a point inside the production possibilities curve.If the production possibilities curve is a straight line: economic resources are perfectly substitutable between the production of the two products.A production possibilities curve illustrates: Scarcity

Page 610: Microeconomics Study Guide

A production possibilities curve shows: the maximum amounts of two goods that can be produced assuming the full use of available resources.

34. Which of the following is not an economic cost? A. wages.B. rents.C. economic profits.D. payments made to the entrepreneur for organizing production.

AACSB: Analytical SkillsBloom's: UnderstandingLearning Objective: 2-3Topic: Five fundamental questions

35. If competitive industry Z is making substantial economic profit, output will: A. fall in industry Z, and firms will likely leave the market.B. fall in all industries except industry Z.C. expand in industry Z, as more resources will move to that industry.D. expand in industry Z, but no new firms will enter the market.

AACSB: Reflective Thinking SkillsBloom's: ApplicationLearning Objective: 2-3Learning Objective: 2-4Topic: Five fundamental questions

36. From society's point of view the economic function of profits and losses is to: A. promote the equal distribution of real assets and wealth.B. achieve full employment and price level stability.C. contribute to a more equal distribution of income.D. reallocate resources from less desired to more desired uses.

AACSB: Reflective Thinking SkillsBloom's: UnderstandingLearning Objective: 2-3Learning Objective: 2-4Topic: Five fundamental questions

37. In a market economy a significant change in consumers' desire for product X will: A. alter the profits or losses received by certain firms.B. cause a reallocation of scarce resources.

Page 611: Microeconomics Study Guide

C. cause some industries to expand and others to contract.D. do all of these.

AACSB: Reflective Thinking SkillsBloom's: ApplicationLearning Objective: 2-3Learning Objective: 2-4Topic: Five fundamental questions

38. Economic profits in an industry suggest the industry: A. can earn more profits by increasing product price.B. should be larger to better satisfy consumers' desire for the product.C. has excess production capacity.D. is the size that consumers want it to be.

AACSB: Reflective Thinking SkillsBloom's: AnalysisLearning Objective: 2-3Topic: Five fundamental questions

39. Economic profits and losses: A. are both considered by economists to be a part of production costs.B. are essential to the reallocation of resources from less desired goods to more desired goods.C. have no influence on the composition of the domestic output.D. equalize the distribution of income in the long run.

AACSB: Analytical SkillsBloom's: UnderstandingLearning Objective: 2-3Learning Objective: 2-4Topic: Five fundamental questions

40. If consumer desire for product X increases, all of the following will occur except: A. an increase in the profits of industry X.B. an increase in the quantity of resources employed by industry X.C. an increase in the output of industry X.D. a decrease in the quantity of resources employed in industry X.

AACSB: Reflective Thinking SkillsBloom's: ApplicationLearning Objective: 2-3

Page 612: Microeconomics Study Guide

Learning Objective: 2-4Topic: Five fundamental questions

41. An increase in consumer desire for strawberries is most likely to: A. increase the number of strawberry pickers needed by farmers.B. reduce the supply of strawberries.C. reduce the number of people willing to pick strawberries.D. reduce the need for strawberry pickers.

AACSB: Reflective Thinking SkillsBloom's: ApplicationLearning Objective: 2-3Learning Objective: 2-4Topic: Five fundamental questions

42. If competitive industry Y is incurring substantial losses, output will: A. expand as resources move toward industry Y.B. contract as resources move toward industry Y.C. contract as resources move away from industry Y.D. expand as resources move away from industry Y.

AACSB: Reflective Thinking SkillsBloom's: UnderstandingLearning Objective: 2-3Learning Objective: 2-4Topic: Five fundamental questions

43. The economic function of profits and losses is to: A. bring about a more equal distribution of income.B. signal that resources should be reallocated.C. eliminate small firms and reduce competition.D. tell government which industries need to be subsidized.

AACSB: Analytical SkillsBloom's: UnderstandingLearning Objective: 2-3Learning Objective: 2-4Topic: Five fundamental questions

44. In a competitive economy, prices: A. influence consumers in their purchases of goods and services.B. influence businesses in their purchases of economic resources.

Page 613: Microeconomics Study Guide

C. influence workers in making occupational choices.D. do all of these.

AACSB: Analytical SkillsBloom's: UnderstandingLearning Objective: 2-3Topic: Five fundamental questions

45. If a competitive industry is neither expanding nor contracting, we would expect: A. total revenue to be zero.B. economic profits to be zero.C. total opportunity cost to be zero.D. more resources to flow to that industry.

AACSB: Reflective Thinking SkillsBloom's: UnderstandingLearning Objective: 2-3Topic: Five fundamental questions

46. Suppose industry A is realizing substantial economic profit. Which of the following best describes what will happen in this competitive market? A. Firms will leave the industry and output will fall.B. Firms will enter the industry and output will fall.C. Firms will leave the industry and output will rise.D. Firms will enter the industry and output will rise.

AACSB: Reflective Thinking SkillsBloom's: ApplicationLearning Objective: 2-4Topic: Five fundamental questions

47. The competitive market system: A. encourages innovation because government provides tax breaks and subsidies to those who develop new products or new productive techniques.B. discourages innovation because it is difficult to acquire additional capital in the form of new machinery and equipment.C. discourages innovation because firms want to get all the profits possible from existing machinery and equipment.D. encourages innovation because successful innovators are rewarded with economic profits.

AACSB: Reflective Thinking SkillsBloom's: Understanding

Page 614: Microeconomics Study Guide

Learning Objective: 2-4Topic: Five fundamental questions

48. In a market economy the distribution of output will be determined primarily by: A. consumer needs and preferences.B. the quantities and prices of the resources that households supply.C. government regulations that provide a minimum income for all.D. a social consensus as to what distribution of income is most equitable.

AACSB: Analytical SkillsBloom's: UnderstandingLearning Objective: 2-3Topic: Five fundamental questions

49. The most efficient combination of resources in producing any output is the combination that: A. comes closest to using the same quantities of land, labor, capital, and entrepreneurial ability.B. can be obtained for the smallest money outlay.C. uses the smallest total quantity of all resources.D. conserves most on the use of labor.

AACSB: Analytical SkillsBloom's: KnowledgeLearning Objective: 2-3Topic: Five fundamental questions

Answer the next question(s) using the following data which show all available techniques for producing 20 units of a particular commodity:

50. Refer to the above data. In view of the indicated resource prices, the economically most efficient production technique(s) is (are) technique(s): A. #1.B. #2 and #4.C. #3.D. #1 and #3.

AACSB: Analytical SkillsBloom's: AnalysisLearning Objective: 2-3Topic: Five fundamental questions

Page 615: Microeconomics Study Guide

51. Refer to the above data. Assuming that the firm is motivated by self-interest and that the 20 units which can be produced with each technique can be sold for $2 per unit, the firm will: A. realize an economic profit of $10.B. realize an economic profit of $4.C. not earn any economic profit.D. close down rather than incur a loss by producing.

AACSB: Analytical SkillsBloom's: ApplicationLearning Objective: 2-3Topic: Five fundamental questions

52. Refer to the above data. If a new production technique is developed that enables a firm to produce 20 units of output with 3 units of land, 3 of labor, 1 of capital, and 2 of entrepreneurial ability, this technique would: A. not be adopted because, although it reduces production costs, it does not increase profit.B. be adopted because it would lower production costs and increase economic profit.C. not be adopted because it entails higher production costs than other available techniques.D. be adopted, even though economic profits would be reduced slightly.

AACSB: Analytical SkillsBloom's: AnalysisLearning Objective: 2-3Topic: Five fundamental questions

Answer the next question(s) on the basis of the following information: Suppose 30 units of product A can be produced by employing just labor and capital in the four ways shown below. Assume the prices of labor and capital are $2 and $3 respectively.

53. Refer to the above information. Which technique is economically most efficient in producing A? A. IB. IIC. IIID. IV

AACSB: Analytical SkillsBloom's: AnalysisLearning Objective: 2-3Topic: Five fundamental questions

Page 616: Microeconomics Study Guide

54. Refer to the above information. If the price of product A is $0.50, the firm will realize: A. an economic profit of $4.B. an economic profit of $2.C. an economic profit of $6.D. a loss of $3.

AACSB: Analytical SkillsBloom's: ApplicationLearning Objective: 2-3Topic: Five fundamental questions

55. In a competitive market economy firms will select the least-cost production technique because: A. such choices will result in the full employment of available resources.B. to do so will maximize the firms' profits.C. this will prevent new firms from entering the industry.D. "dollar voting" by consumers mandates such a choice.

AACSB: Reflective Thinking SkillsBloom's: UnderstandingLearning Objective: 2-3Topic: Five fundamental questions

56. The market system's answer to the fundamental question "What will be produced?" is essentially: A. "Goods and services that are profitable."B. "Low cost goods and services."C. "Goods and service that can be produced using large amounts of capital."D. "Goods and services that possess lasting value."

AACSB: Reflective Thinking SkillsBloom's: UnderstandingLearning Objective: 2-3Topic: Five fundamental questions

57. The market system's answer to the fundamental question "How will the goods and services be produced?" is essentially: A. "With as much machinery as possible."B. "Using the latest technology."C. "By exploiting labor."D. "Using the least-cost production techniques."

AACSB: Reflective Thinking Skills

Page 617: Microeconomics Study Guide

Bloom's: UnderstandingLearning Objective: 2-3Topic: Five fundamental questions

58. The market system's answer to the fundamental question "Who will get the goods and services?" is essentially: A. "Those willing and able to pay for them."B. "Those who physically produced them."C. "Those who most need them."D. "Those who get utility from them."

AACSB: Reflective Thinking SkillsBloom's: UnderstandingLearning Objective: 2-3Topic: Five fundamental questions

59. The market system's answer to the fundamental question "How will the system accommodate change?" is essentially: A. "Through government leadership and direction."B. "Through the guiding function of prices and the incentive function of profits."C. "Through training and retraining programs."D. "Through random trial and error."

AACSB: Reflective Thinking SkillsBloom's: UnderstandingLearning Objective: 2-4Topic: Five fundamental questions

60. The market system's answer to the fundamental question "How will the system promote progress?" is essentially: A. "Through government funded research programs."B. "Through the redistribution of income to promote greater equality."C. "Through training and retraining programs."D. "Through the profit potential that encourages development of new technology."

AACSB: Reflective Thinking SkillsBloom's: UnderstandingLearning Objective: 2-4Topic: Five fundamental questions

61. The advent of DVDs has virtually demolished the market for videocassettes. This is an example of: A. creative destruction.B. derived demand.C. capital accumulation.

Page 618: Microeconomics Study Guide

D. the difference between normal and economic profits.

AACSB: Reflective Thinking SkillsBloom's: ApplicationLearning Objective: 2-4Topic: Five fundamental questions

62. Consumer sovereignty refers to the: A. fact that resource prices are higher than product prices in capitalistic economies.B. idea that the pursuit of self-interest is in the public interest.C. idea that the decisions of producers must ultimately conform to consumer demands..D. fact that a Federal agency exists to protect consumers from harmful and defective products.

AACSB: Analytical SkillsBloom's: UnderstandingLearning Objective: 2-3Topic: Five fundamental questions

63. The dollar votes of consumers ultimately determine the composition of output and the allocation of resources in a market economy. This statement best describes the concept of: A. derived demand.B. consumer sovereignty.C. the invisible hand.D. market failure.

AACSB: Analytical SkillsBloom's: KnowledgeLearning Objective: 2-3Topic: Five fundamental questions

64. Which of the following is not one of the five fundamental questions? A. What prices will be charged for goods and services?B. Who will get the goods and services?C. What goods and services will be produced?D. How will the system promote progress?

AACSB: Analytical SkillsBloom's: KnowledgeLearning Objective: 2-3Status: NewTopic: Five fundamental questions

Page 619: Microeconomics Study Guide

65. "Consumer sovereignty" means that: A. buyers can dictate the prices at which goods and services will be purchased.B. advertising is ineffective because consumers already know what they want.C. buyers control the quality of goods and services through regulatory agencies.D. buyers determine what will be produced based on their "dollar votes" for the goods and services offered by sellers.

AACSB: Analytical SkillsBloom's: UnderstandingLearning Objective: 2-3Status: NewTopic: Five fundamental questions

66. Which of the following best describes the invisible-hand concept? A. The desires of resource suppliers and producers to further their own self-interest will automatically further the public interest.B. The nonsubstitutability of resources creates a conflict between private and public interests and calls for government intervention.C. The market system is the best system for overcoming the scarce resources-unlimited wants problem.D. Central direction by the government will improve resource allocation in a capitalistic economy.

AACSB: Analytical SkillsBloom's: KnowledgeLearning Objective: 2-3Topic: Invisible hand

67. The invisible hand refers to the: A. fact that the U.S. tax system redistributes income from rich to poor.B. notion that, under competition, decisions motivated by self-interest promote the social interest.C. tendency of monopolistic sellers to raise prices above competitive levels.D. fact that government controls the functioning of the market system.

AACSB: Analytical SkillsBloom's: UnderstandingLearning Objective: 2-3Topic: Invisible hand

68. The invisible-hand concept suggests that: A. market failures imply the need for a national economic plan.B. big businesses are inherently more efficient than small businesses.C. the competitiveness of a capitalistic market economy invariably diminishes over time.

Page 620: Microeconomics Study Guide

D. assuming competition, private and public interests will coincide.

AACSB: Reflective Thinking SkillsBloom's: UnderstandingLearning Objective: 2-3Topic: Invisible hand

69. The invisible-hand concept suggests that: A. changes in product demands are only randomly reflected in changes in the demands for resources.B. profit maximization is inconsistent with an efficient allocation of resources.C. government action is necessary to correct for market failures.D. when firms maximize their profits, society's output will also be maximized.

AACSB: Reflective Thinking SkillsBloom's: UnderstandingLearning Objective: 2-3Topic: Invisible hand

70. Two major virtues of the market system are that it: A. allocates resources efficiently and allows economic freedom.B. results in an equitable personal distribution of income and always maintains full employment.C. results in price level stability and a fair personal distribution of income.D. eliminates discrimination and minimizes environmental pollution.

AACSB: Reflective Thinking SkillsBloom's: UnderstandingLearning Objective: 2-3Topic: Invisible hand

71. The market system: A. produces considerable inefficiency in the use of scarce resources.B. effectively harnesses the incentives of workers and entrepreneurs.C. is inconsistent with freedom of choice in the long run.D. has slowly lost ground to emerging command systems.

AACSB: Reflective Thinking SkillsBloom's: UnderstandingLearning Objective: 2-3Topic: Invisible hand

72. According to the concept of the "invisible hand," if Susie opens and operates a profitable childcare center, then:

Page 621: Microeconomics Study Guide

A. government should regulate the business to ensure quality.B. the profit Susie earns indicates that she is overcharging for her services.C. she has served society's interests by providing a desired good or service.D. this demonstrates that consumer sovereignty is not present in this market.

AACSB: Reflective Thinking SkillsBloom's: ApplicationLearning Objective: 2-3Status: NewTopic: Invisible hand

73. The invisible hand promotes society's interests because: A. individuals pursuing their self-interest will try to produce goods and services that people in society want and are willing to purchase.B. individuals will produce goods for others out of concern for their fellow human beings.C. it makes sure that everyone wins from competition in the market.D. government regulation pushes business into producing the right mix of goods and services.

AACSB: Reflective Thinking SkillsBloom's: UnderstandingLearning Objective: 2-3Status: NewTopic: Invisible hand

74. The coordination problem in the centrally planned economies refers to the idea that: A. planners had to direct required inputs to each enterprise.B. the price level and the level of employment were inversely related.C. the immediate effect of more investment was less consumption.D. exports had to be equal to imports for a central plan to work.

AACSB: Analytical SkillsBloom's: UnderstandingLearning Objective: 2-1Topic: Demise of the command systems

75. "Under central planning, some group has to decide how to get the necessary inputs produced in the right amounts and delivered to the right places at the right time. This is a nearly impossible task without markets and profits." This quotation best identifies the: A. incentive problem under central planning.B. coordination problem under central planning.C. self-sufficiency dilemma under communism.D. resource overcommitment problem under communism.

Page 622: Microeconomics Study Guide

AACSB: Reflective Thinking SkillsBloom's: ApplicationLearning Objective: 2-1Topic: Demise of the command systems

76. "Because the outputs of many industries are the inputs to other industries, the failure of any single industry to fulfill the output quantities specified in the central plan caused a chain-reaction of adverse repercussions on production." This quotation best identifies the: A. incentive problem under central planning.B. self-sufficiency dilemma under communism.C. resource overcommitment problem under communism.D. coordination problem under central planning.

AACSB: Reflective Thinking SkillsBloom's: ApplicationLearning Objective: 2-1Topic: Demise of the command systems

77. The incentive problem under communist central planning refers to the idea that: A. planners had to direct required inputs to each enterprise.B. workers, managers, and entrepreneurs could not personally gain by responding to shortages or surpluses or by introducing new and improved products.C. the immediate effect of more investment was less consumption.D. exports had to be equal to imports for a central plan to work.

AACSB: Reflective Thinking SkillsBloom's: UnderstandingLearning Objective: 2-1Topic: Demise of the command systems

78. Suppose that an individual sees a tremendous opportunity to produce and sell a new product, but dismisses the idea because there is no way to exploit this opportunity for personal gain. This situation best identifies the: A. coordination problem under communist central planning.B. self-sufficiency dilemma under communism.C. asymmetric information problem under communism.D. incentive problem under communist central planning.

AACSB: Reflective Thinking SkillsBloom's: ApplicationLearning Objective: 2-1

Page 623: Microeconomics Study Guide

Topic: Demise of the command systems

79. Shortages and unmet demand provide opportunities for individuals and firms to profit under capitalism, but they present no such opportunities under central planning. This reality represents central planning's: A. incentive problem.B. coordination problem.C. paradox of value.D. X-efficiency problem.

AACSB: Reflective Thinking SkillsBloom's: UnderstandingLearning Objective: 2-1Topic: Demise of the command systems

80. Innovation lagged in the centrally planned economies because:

Type: T Topic: 3 E: 417 MI: 173 135. The above data are for: A) the long run. C) both the short run and the long run. the short run. D) the intermediate market period only. Answer: B

Type: T Topic: 3 E: 419-420 MI: 175-176 136. Refer to the above data. At 5 units of output average fixed cost, average variable cost, and average total cost are: A) $10, $60, and $70 respectively. C) $10, $70, and $80 respectively. $50, $40, and $90 respectively. D) $5, $25, and $30 respectively. Answer: A

Type: T Topic: 3 E: 419 MI: 175 137. Refer to the above data. The marginal cost of the fifth unit of output is: A) $80. $90. C) $50. D) $20.Answer: A

Type: T Topic: 3 E: 419-420 MI: 175-176 138. Refer to the above data. If product price is $75, the firm will produce: A) 3 units of output. 4 units of output. C) 5 units of output. D) 6 units of output. Answer: B

Type: T Topic: 3 E: 419-420 MI: 175-176 139. Refer to the above data. Given the $75 product price, at its optimal output the firm will: A) realize a $25 economic profit. C) incur a $25 loss.

Page 624: Microeconomics Study Guide

realize a $30 economic profit. D) realize a $30 loss. Answer: B

Type: A Topic: 3 E: 424 MI: 180 140. A purely competitive firm's short-run supply curve is:A) the upward sloping portion of its marginal cost curve.the upward sloping portion of its average variable cost curve.C) its marginal cost curve above average variable cost.D) its average total cost curve.Answer: C

Type: A Topic: 3 E: 424 MI: 180 141. In the short run, a purely competitive firm will earn a normal profit when:A) P = AVC. P > MC. C) that firm's MR = market equilibrium price. D) P = ATC.Answer: D

Use the following to answer questions 142-147:

The following table applies to a purely competitive industry composed of 100 identical firms.

Type: T Topic: 3 E: 425 MI: 181 142. Refer to the above table. The equilibrium price in this purely competitive market is:A) $5. $4. C) $3. D) $2.Answer: C

Type: T Topic: 3 E: 425 MI: 181 143. Refer to the above table. At the equilibrium price, each of the 100 firms in this industry will produce:A) 600,000 units of output. C) 6,000 units of output60,000 units of output. D) 600 units of output.Answer: C

Type: T Topic: 3 E: 425-426 MI: 181-182 144. Refer to the above table. For each of the 100 firms in this industry, marginal revenue and total revenue will be: A) $4 and $400, respectively. C) $4 and $20,000, respectively.$3 and $30,000, respectively. D) $3 and $18,000, respectively.Answer: D

Type: T Topic: 3 E: 425-426 MI: 181-182 145. Refer to the above table. If each of the 100 firms in the industry is maximizing its profit, each must have a marginal cost of:A) $5. $4. C) $3. D) $2.Answer: C

Page 625: Microeconomics Study Guide

Type: T Topic: 3 E: 425-426 MI: 181-182 146. Refer to the above table. If each of the 100 firms in the industry is maximizing its profit and earning only a normal profit, each must have a total cost of:A) $18,000. $20,000. C) $22,000. D) $24,000.Answer: A

Type: T Topic: 3 E: 425-426 MI: 181-182 147. Refer to the above table. If each of the 100 firms in the industry is maximizing its profit and earning only a normal profit, each must have an average total cost of:A) $2. $3. C) $4. D) $5.Answer: B

Profit maximizing in long run

Type: A Topic: 4 E: 427 MI: 183 148. Suppose a firm in a purely competitive market discovers that the price of its product is above its minimum AVC point but everywhere below ATC. Given this, the firm:A) minimizes losses by producing at the minimum point of its AVC curve.maximizes profits by producing where MR = ATC.C) should close down immediately.D) should continue producing in the short run, but leave the industry in the long run.Answer: D

Type: A Topic: 4 E: 427 MI: 183 149. Which of the following is true concerning purely competitive industries?A) There will be economic losses in the long run because of cut-throat competition.Economic profits will persist in the long run if consumer demand is strong and stable.C) In the short run, firms may incur economic losses or earn economic profits, but in the long run they earn normal profits.D) There are economic profits in the long run, but not in the short run.Answer: C

Type: A Topic: 4 E: 428 MI: 184 150. If a purely competitive firm is producing at the MR = MC output level and earning an economic profit, then:A) the selling price for this firm is above the market equilibrium price.new firms will enter this market.C) some existing firms in this market will leave.D) there must be price fixing by the industry's firms.Answer: B

Type: A Topic: 4 E: 427 MI: 183 151. Long-run competitive equilibrium: A) is realized only in constant-cost industries. C) is not economically efficient. will never change once it is realized. D) results in zero economic profits. Answer: D

Page 626: Microeconomics Study Guide

Type: A Topic: 4 E: 428 MI: 184 152. We would expect an industry to expand if firms in that industry are: A) earning normal profits. C) incurring economic losses. earning economic profits. D) earning accounting profits. Answer: B

Type: A Topic: 4 E: 422-423 MI: 178-179 153. Which of the following statements is correct? A) Economic profits induce firms to enter an industry; losses encourage firms to leave. Economic profits induce firms to leave an industry; profits encourage firms to leave. C) Economic profits and losses have no significant impact on the growth or decline of an industry. D) Normal profits will cause an industry to expand. Answer: A

Type: A Topic: 4 E: 430 MI: 186 154. Suppose a purely competitive increasing-cost industry is in long-run equilibrium. Now assume that a decrease in consumer demand occurs. After all resulting adjustments have been completed, the new equilibrium price: A) and industry output will be less than the initial price and output. will be greater than the initial price, but the new industry output will be less than the original output. C) will be less than the initial price, but the new industry output will be greater than the original output. D) and industry output will be greater than the initial price and output. Answer: A

Type: A Topic: 4 E: 430 MI: 186 155. Which of the following statements is correct? A) The long-run supply curve for a purely competitive increasing-cost industry will be upsloping. The long-run supply curve for a purely competitive increasing-cost industry will be perfectly elastic. C) The long-run supply curve for a purely competitive industry will be less elastic than the industry's short-run supply curve. D) The long-run supply curve for a purely competitive decreasing-cost industry will be upsloping. Answer: A

Type: A Topic: 4 E: 429 MI: 185 156. A constant-cost industry is one in which: A) a higher price per unit will not result in an increased output. if 100 units can be produced for $100, then 150 can be produced for $150, 200 for $200, and so forth. C) the demand curve and therefore the unit price and quantity sold seldom change. D) the total cost of producing 200 or 300 units is no greater than the cost of producing 100 units. Answer: B

Type: A Topic: 4 E: 432 MI: 188 157. Which of the following will not hold true for a competitive firm in long-run equilibrium? A) P equals AFC P equals minimum ATC C) MC equals minimum ATC D) P equals MC Answer: A

Page 627: Microeconomics Study Guide

Type: A Topic: 4 E: 430 MI: 186 158. Assume a purely competitive increasing-cost industry is initially in long-run equilibrium and that an increase in consumer demand occurs. After all economic adjustments have been completed product price will be: A) lower, but total output will be larger than originally. higher and total output will be larger than originally. C) lower and total output will be smaller than originally. D) higher, but total output will be smaller than originally. Answer: B

Type: A Topic: 4 E: 430 MI: 186 159. Assume a purely competitive, increasing-cost industry is in long-run equilibrium. If a decline in demand occurs, firms will: A) leave the industry, price will decrease, and quantity produced will increase. enter the industry and price and quantity will both increase. C) leave the industry and price and output will both increase. D) leave the industry and price and output will both decline. Answer: D

Type: A Topic: 4 E: 427 MI: 183 160. When a purely competitive firm is in long-run equilibrium: A) marginal revenue exceeds marginal cost. price equals marginal cost. C) total revenue exceeds total cost. D) minimum average total cost is less than the product price. Answer: B

Type: A Topic: 4 E: 427 MI: 183 161. A purely competitive firm: A) must earn a normal profit in the short run. cannot earn economic profit in the long run. C) may realize either economic profit or losses in the long run. D) cannot earn economic profit in the short run. Answer: B

Type: A Topic: 4 E: 429 MI: 185 162. A constant-cost industry is one in which:A) resource prices fall as output is increased. resource prices rise as output is increased. C) resource prices remain unchanged as output is increased. D) small and large levels of output entail the same total costs. Answer: C

Type: A Topic: 4 E: 430 MI: 186 163. An increasing-cost industry is associated with: A) a perfectly elastic long-run supply curve. C) a perfectly inelastic long-run supply curve.

Page 628: Microeconomics Study Guide

an upsloping long-run supply curve. D) an upsloping long-run demand curve. Answer: B

Use the following to answer questions 164-166:

Type: G Topic: 4 E: 426 MI: 182 164. Refer to the above diagrams, which pertain to a purely competitive firm producing output q and the industry in which it operates. Which of the following is correct? A) The diagrams portray neither long-run nor short-run equilibrium. The diagrams portray both long-run and short-run equilibrium. C) The diagrams portray short-run equilibrium, but not long-run equilibrium. D) The diagrams portray long-run equilibrium, but not short-run equilibrium. Answer: C

Type: G Topic: 4 E: 427-428 MI: 183-184 165. Refer to the above diagrams, which pertain to a purely competitive firm producing output q and the industry in which it operates. In the long run we should expect: A) firms to enter the industry, market supply to rise, and product price to fall. firms to leave the industry, market supply to rise, and product price to fall. C) firms to leave the industry, market supply to fall, and product price to rise. D) no change in the number of firms in this industry. Answer: C

Type: G Topic: 4 E: 432 MI: 188 166. Refer to the above diagrams, which pertain to a purely competitive firm producing output q and the industry in which it operates. The predicted long-run adjustments in this industry might be offset by: A) a decline in product demand. an increase in resource prices. C) a technological improvement in production methods. D) entry of new firms into the industry. Answer: C

Type: A Topic: 4 E: 427 MI: 183 167. Assume a purely competitive firm is maximizing profit at some output at which long-run average total cost is at a minimum. Then:A) the firm is earning an economic profit. there is no tendency for the firm's industry to expand or contract. C) allocative but not productive efficiency is being achieved. D) other firms will enter this industry.Answer: B

Type: A Topic: 4 E: 430 MI: 186

Page 629: Microeconomics Study Guide

168. An increasing-cost industry is the result of: A) higher resource prices which occur as the industry expands. a change in the industry's minimum efficient scale. C) X-inefficiency. D) the law of diminishing returns. Answer: A

Type: A Topic: 4 E: 427 MI: 183 169. A purely competitive firm is precluded from making economic profit in the long run because: A) it is a "price taker." C) of unimpeded entry to the industry. its demand curve is perfectly elastic. D) it produces a differentiated product. Answer: C

Type: A Topic: 4 E: 428-429 MI: 184-185 170. If a purely competitive constant-cost industry is realizing economic profits, we can expect industry supply to: A) increase, output to increase, price to decrease, and profits to decrease. increase, output to increase, price to increase, and profits to decrease. C) decrease, output to decrease, price to increase, and profits to increase. D) increase, output to decrease, price to decrease, and profits to decrease. Answer: A

Type: A Topic: 4 E: 427 MI: 183 171. Assume that a decline in consumer demand occurs in a purely competitive industry which is initially in long-run equilibrium. We can: A) predict that the new price will be greater than the original price. predict that the new price will be less than the original price. C) predict that the new price will be the same as the original price. D) not compare the original and the new price without knowing about cost conditions in the industry. Answer: D

Type: D Topic: 4 E: 430-431 MI: 186-187 172. A decreasing-cost industry is one in which: A) contraction of the industry will decrease unit costs.input prices fall or technology improves as the industry expands. C) the long-run supply curve is perfectly elastic. D) the long-run supply curve is upsloping. Answer: B

Type: A Topic: 4 E: 430-431 MI: 186-187 173. When compact disc (CD) players first came on the market, they sold for over $1,000. Now they cost only $100. These facts imply that: A) the CD industry was once competitive, but is now monopolistic. fewer firms produce CD players than was the case five or ten years ago. C) the demand curve for CD players has shifted leftward. D) the CD player industry is a decreasing-cost industry. Answer: D

Page 630: Microeconomics Study Guide

Type: A Topic: 4 E: 431 MI: 187 174. Suppose that an industry's long-run supply curve is downsloping. This suggests that: A) it is an increasing-cost industry. relevant inputs have become more expensive as the industry has expanded. C) technology has become less efficient as a result of the industry's expansion. D) it is a decreasing-cost industry. Answer: D

Type: A Topic: 4 E: 430-431 MI: 186-187 175. Suppose an increase in product demand occurs in a decreasing-cost industry. As a result: A) the new long-run equilibrium price will be lower than the original long-run equilibrium price. B) equilibrium quantity will decline. C) firms will eventually leave the industry. D) the new long-run equilibrium price will be higher than the original price. Answer: A

Type: C Topic: 4 E: 429-430 MI: 185-186 176. Purely competitive industry X has constant costs and its product is an inferior good. The industry is currently in long-run equilibrium. The economy now goes into a recession and average incomes decline. The result will be:A) an increase in output and in the price of the product. B) an increase in output, but not in the price, of the product. C) a decrease in the output, but not in the price, of the product. D) a decrease in output and in the price of the product. Answer: B

Type: A Topic: 4 E: 430 MI: 186 177. Suppose losses cause industry X to contract and, as a result, the prices of relevant inputs decline. Industry X is: A) a constant-cost industry. C) an increasing-cost industry. B) a decreasing-cost industry. D) encountering X-inefficiency. Answer: C

Use the following to answer questions 178-183:

Type: G Topic: 4 E: 420 MI: 176 178. Refer to the above diagram showing the average total cost curve for a purely competitive firm. Suppose this firm is maximizing its total profit and the market price is $15. The firm's per unit profit is: A) $5. B) $200. C) a positive amount less than $5. D) a positive amount more than $200. Answer: C

Type: G Topic: 4 E: 420 MI: 176 179. Refer to the above diagram showing the average total cost curve for a purely competitive firm. Suppose that total

Page 631: Microeconomics Study Guide

variable cost is $300 at 40 units of output. At that level of output, average fixed cost: A) is $2.50. B) is $4. C) is $100. D) cannot be determined from the information provided. Answer: A

Type: G Topic: 4 E: 420 MI: 176 180. Refer to the above diagram showing the average total cost curve for a purely competitive firm. Suppose that average variable cost is $8 at 40 units of output. At that level of output, total fixed cost: A) is $2. B) is $40. C) is $80. D) cannot be determined from the information provided. Answer: C

Type: G Topic: 4 E: 415-416, 427 MI: 171-172, 183 181. Refer to the above diagram showing the average total cost curve for a purely competitive firm. At the long-run equilibrium level of output, this firm's total revenue: A) is $10. B) is $40. C) is $400. D) cannot be determined from the information provided. Answer: C

Type: G Topic: 4 E: 420, 427 MI: 176, 183 182. Refer to the above diagram showing the average total cost curve for a purely competitive firm. At the long-run equilibrium level of output, this firm's total cost: A) is $10. B) is $40. C) is $400. D) cannot be determined from the information provided. Answer: C

Type: G Topic: 4 E: 427 MI: 183 183. Refer to the above diagram showing the average total cost curve for a purely competitive firm. At the long-run equilibrium level of output, this firm's economic profit: A) is zero. B) is $400. C) is $200. D) cannot be determined from the information provided. Answer: A Type: A Topic: 4 E: 427 MI: 183 184. The MR = MC rule applies: A) in the short run, but not in the long run. C) in both the short run and the long run. B) in the long run, but not in the short run. D) only to a purely competitive firm. Answer: C

Type: A Topic: 4 E: 430 MI: 186 185. If the long-run supply curve of a purely competitive industry slopes upward, this implies that the prices of relevant resources: A) will fall as the industry expands. C) rise as the industry contracts. B) are constant as the industry expands. D) rise as the industry expands. Answer: D

Use the following to answer questions 186-187:

Type: G Topic: 4 E: 430 MI: 186 186. Refer to the above diagram. Line (1) reflects the long-run supply curve for:

Page 632: Microeconomics Study Guide

A) a constant-cost industry. C) an increasing-cost industry. B) a decreasing-cost industry. D) technologically progressive industry.Answer: C

Type: G Topic: 4 E: 429 MI: 185 187. Refer to the above diagram. Line (2) reflects the long-run supply curve for: A) a constant-cost industry. C) an increasing-cost industry. B) a decreasing-cost industry. D) technologically progressive industry.Answer: A

Pure competition and efficiency

Type: A Topic: 5 E: 432 MI: 188 188. Allocative efficiency is achieved when the production of a good occurs where: A) P = minimum ATC. B) P = MC. C) P = minimum AVC. D) total revenue is equal to TFC. Answer: B

Type: A Topic: 5 E: 432 MI: 188 189. A firm is producing an output such that the benefit from one more unit is more than the cost of producing that additional unit. This means the firm is: A) producing more output than allocative efficiency requires. B) producing less output than allocative efficiency requires. C) achieving productive efficiency. D) producing an inefficient output, but we cannot say whether output should be increased or decreased. Answer: B

Type: A Topic: 5 E: 431 MI: 187 190. Resources are efficiently allocated when production occurs where:A) marginal cost equals average variable cost. C) price is equal to marginal cost. B) price is equal to average revenue. D) price is equal to average variable cost. Answer: C

Type: D Topic: 5 E: 432 MI: 188 191. The term productive efficiency refers to: A) any short-run equilibrium position of a competitive firm. B) the production of the product-mix most desired by consumers. C) the production of a good at the lowest average total cost. D) fulfilling the condition P = MC. Answer: C

Type: A Topic: 5 E: 432 MI: 188 192. If the price of product Y is $25 and its marginal cost is $18: A) Y is being produced with the least-cost combination of resources. B) society will realize a net gain if less of Y is produced. C) resources are being underallocated to Y.

Page 633: Microeconomics Study Guide

D) resources are being overallocated to Y. Answer: C

Type: D Topic: 5 E: 432 MI: 188 193. The term allocative efficiency refers to: A) the level of output that coincides with the intersection of the MC and AVC curves. B) minimization of the AFC in the production of any good. C) the production of the product-mix most desired by consumers. D) the production of a good at the lowest average total cost. Answer: C

Type: A Topic: 5 E: 433 MI: 189 194. Under pure competition in the long run: A) neither allocative efficiency nor productive efficiency are achieved. B) both allocative efficiency and productive efficiency are achieved. C) productive efficiency is achieved, but allocative efficiency is not. D) allocative efficiency is achieved, but productive efficiency is not. Answer: B

Type: A Topic: 5 E: 432 MI: 188 195. If for a firm P = minimum ATC = MC, then:A) neither allocative efficiency nor productive efficiency is being achieved. B) productive efficiency is being achieved, but allocative efficiency is not. C) both allocative efficiency and productive efficiency are being achieved. D) allocative efficiency is being achieved, but productive efficiency is not. Answer: C

Use the following to answer questions 196-201:

Type: G Topic: 5 E: 431 MI: 187 196. The above diagram portrays: A) a competitive firm that should shut down in the short run. B) the equilibrium position of a competitive firm in the long run. C) a competitive firm that is realizing an economic profit. D) the loss-minimizing position of a competitive firm in the short run. Answer: B

Type: G Topic: 5 E: 431 MI: 187 Status: New 197. Refer to the above diagram. If this competitive firm produces output Q, it will:A) suffer an economic loss.B) earn a normal profit.C) earn an economic profit.D) achieve productive efficiency, but not allocative efficiency.

Page 634: Microeconomics Study Guide

Answer: B

Type: G Topic: 5 E: 433 MI: 189 198. Refer to the above diagram. By producing output level Q: A) neither productive nor allocative efficiency are achieved. B) both productive and allocative efficiency are achieved. C) allocative efficiency is achieved, but productive efficiency is not. D) productive efficiency is achieved, but allocative efficiency is not. Answer: B

Type: G Topic: 5 E: 432 MI: 188 199. Refer to the above diagram. At output level Q1: A) neither productive nor allocative efficiency are achieved. B) both productive and allocative efficiency are achieved. C) allocative efficiency is achieved, but productive efficiency is not. D) productive efficiency is achieved, but allocative efficiency is not. Answer: A

Type: G Topic: 5 E: 432 MI: 188 200. Refer to the above diagram. At output level Q1: A) resources are overallocated to this product and productive efficiency is not realized. B) resources are underallocated to this product and productive efficiency is not realized. C) productive efficiency is achieved, but resources are underallocated to this product. D) productive efficiency is achieved, but resources are overallocated to this product. Answer: B

Type: G Topic: 5 E: 432-433 MI: 188-189 201. Refer to the above diagram. At output level Q2: A) resources are overallocated to this product and productive efficiency is not realized. B) resources are underallocated to this product and productive efficiency is not realized. C) productive efficiency is achieved, but resources are underallocated to this product. D) productive efficiency is achieved, but resources are overallocated to this product. Answer: A

Type: A Topic: 5 E: 432 MI: 188 202. Assume that society places a higher value on the last unit of X produced than the value of the resources used to produce that unit. With no spillovers, this information means that: A) total cost is greater than total revenue. C) marginal cost is greater than price. B) price is greater than marginal cost. D) resources are being overallocated to X. Answer: B

Type: A Topic: 5 E: 432-433 MI: 188-189 203. If production is occurring where marginal cost exceeds price, the purely competitive firm will: A) maximize profit, but resources will be underallocated to the product. B) maximize profit, but resources will be overallocated to the product.

Page 635: Microeconomics Study Guide

C) fail to maximize profit and resources will be overallocated to the product. D) fail to maximize profit and resources will be underallocated to the product. Answer: C

Type: A Topic: 5 E: 432 MI: 188 204. If a purely competitive firm is producing where price exceeds marginal cost, then: A) the firm will fail to maximize profit, but resources will be efficiently allocated. B) the firm will fail to maximize profit and resources will be overallocated to the product. C) the firm will fail to maximize profit and resources will be underallocated to the product. D) resources will be underallocated to the product, but the firm will maximize profit. Answer: C

Type: A Topic: 5 E: 432 MI: 188 Status: New 205. Which of the following conditions is true for a purely competitive firm in long-run equilibrium?A) P > MC = minimum ATC. C) P = MC = minimum ATC.B) P > MC > minimum ATC. D) P < MC < minimum ATC.Answer: C

Consider This Questions

Type: A E: 423 MI: 179 Status: New 206. (Consider This) An unprofitable motel will stay open in the short-run if:A) price (average nightly room rate) exceeds average variable cost.B) marginal revenue exceeds marginal cost.C) price (average nightly room rate) exceeds average fixed cost.D) marginal revenue exceeds price.Answer: A

Type: A E: 423 MI: 179 Status: New 207. (Consider This) An otherwise unprofitable motel located on a largely abandoned roadway might be able to stay open for several years by:A) increasing its nightly room rates.B) reducing or eliminating its annual maintenance expenses.C) charging room rates that exceed marginal revenue.D) eliminating its fixed costs, including its opportunity costs.Answer: B

Last Word Questions

Use the following to answer questions 208-210:

Type: G E: 434 MI: 190 208. (Last Word) Refer to the above graph of the market for asparagus. At the market price of $2, area A + B represents:

Page 636: Microeconomics Study Guide

A) total consumer utility. C) consumer surplus.B) total revenue to sellers. D) returns to capital and to labor.Answer: A

Type: G E: 434 MI: 190 209. (Last Word) Refer to the above graph of the market for asparagus. At the market price of $2, area A represents:A) total consumer utility. B) total revenue to sellers. C) consumer surplus. D) economic profit.Answer: C

Type: G E: 434 MI: 190 210. (Last Word) In long-run equilibrium, purely competitive markets:A) minimize total cost.B) maximize consumer surplus.C) yield economic profits to most sellers.D) inevitably degenerate into monopoly in increasing cost industries.Answer: B

True/False Questions

Type: A E: 416 MI: 172 211. In maximizing profit a firm will always produce that output where total revenues are at a maximum. Answer: False

Type: A E: 422 MI: 178 212. In the short run a competitive firm will always choose to shut down if product price is less than the lowest attainable average total cost. Answer: False

Type: A E: 427 MI: 183 213. After all long-run adjustments have been completed, a firm in a competitive industry will produce that level of output where average total cost is at a minimum. Answer: True

Type: A E: 430-431 MI: 186-187 214. The long-run supply curve for a decreasing-cost industry is downsloping. Answer: True

Type: A E: 420 MI: 176 215. A competitive firm will produce in the short run so long as its price exceeds its average fixed cost.Answer: False

Type: A E: 432 MI: 188 216. Marginal cost is a measure of the alternative goods which society forgoes in using resources to produce an additional unit of some specific product. Answer: True

Page 637: Microeconomics Study Guide

Type: A E: 415-416 MI: 171-172 217. Price and marginal revenue are identical for an individual purely competitive seller. Answer: True

Type: A E: 432 MI: 188 218. Because the equilibrium position of a purely competitive seller entails an equality of price and marginal costs, competition produces up to an efficient allocation of economic resources. Answer: True

Type: A E: 423-424 MI: 179-180 219. The short-run supply curve slopes upward because producers must be compensated for rising marginal costs. Answer: True

Type: A E: 425 MI: 181 220. The demand curve for a purely competitive industry is perfectly elastic, but the demand curves faced by individual firms in such an industry are downsloping. Answer: False

Type: A E: 417-418 MI: 173-174 221. The total revenue curve of a competitive seller graphs as a straight, upsloping line. Answer: True

Type: D E: 416 MI: 172 222. Marginal revenue is the addition to total revenue resulting from the sale of one more unit of output. Answer: True

Use the following to answer questions 223-231:

Type: G E: 419-420 MI: 175-176 223. Refer to the above diagram. This firm will maximize profits by producing output D. Answer: False

Multiple Choice Questions

Four market models

Type: A Topic: 1 E: 414 MI: 170 1. Economists would describe the U.S. automobile industry as: A) purely competitive. an oligopoly. C) monopolistically competitive. D) a pure monopoly. Answer: B

Type: A Topic: 1 E: 414 MI: 170

Page 638: Microeconomics Study Guide

2. In which of the following market structures is there clear-cut mutual interdependence with respect to price-output policies? A) pure monopoly oligopoly C) monopolistic competition D) pure competition Answer: B

Type: A Topic: 1 E: 414 MI: 170 3. Which of the following industries most closely approximates pure competition? A) agriculture farm implements C) clothing D) steel Answer: A

Type: D Topic: 1 E: 414 MI: 170 4. Economists use the term imperfect competition to describe: A) all industries which produce standardized products. any industry in which there is no nonprice competition. C) a pure monopoly only. D) those markets which are not purely competitive. Answer: D

Type: A Topic: 1 E: 414 MI: 170 5. In which of the following industry structures is the entry of new firms the most difficult? A) pure monopoly oligopoly C) monopolistic competition D) pure competition Answer: A

Type: A Topic: 1 E: 414 MI: 170 6. An industry comprised of 40 firms, none of which has more than 3 percent of the total market for a differentiated product is an example of:A) monopolistic competition oligopoly C) pure monopoly D) pure competition Answer: A

Type: A Topic: 1 E: 413-414 MI: 169-170 7. A one-firm industry is known as: A) monopolistic competition oligopoly C) pure monopoly D) pure competition Answer: C

Type: A Topic: 1 E: 414 MI: 170 8. An industry comprised of four firms, each with about 25 percent of the total market for a product is an example of:A) monopolistic competition oligopoly C) pure monopoly D) pure competition Answer: B

Type: A Topic: 1 E: 413-414 MI: 169-170 9. An industry comprised of a very large number of sellers producing a standardized product is known as:A) monopolistic competition oligopoly C) pure monopoly D) pure competition Answer: D

Type: A Topic: 1 E: 414 MI: 170 10. An industry comprised of a small number of firms, each of which considers the potential reactions of its rivals in

Page 639: Microeconomics Study Guide

making price-output decisions is called:A) monopolistic competition oligopoly C) pure monopoly D) pure competition Answer: B

Pure competition defined; demand curve

Type: A Topic: 2 E: 414 MI: 170 11. Which of the following statements applies to a purely competitive producer? A) It will not advertise its product. In long-run equilibrium it will earn an economic profit. C) Its product will have a brand name. D) Its product is slightly different from those of its competitors. Answer: A

Type: A Topic: 2 E: 414 MI: 170 12. A purely competitive seller is: A) both a "price maker" and a "price taker." C) a "price taker." neither a "price maker" nor a "price taker." D) a "price maker." Answer: C

Type: A Topic: 2 E: 414 MI: 170 13. Which of the following is not characteristic of pure competition? A) price strategies by firms C) no barriers to entry a standardized product D) a larger number of sellers Answer: A

Type: A Topic: 2 E: 414 MI: 170 14. Which of the following is not a basic characteristic of pure competition? A) considerable nonprice competition C) a standardized or homogeneous product no barriers to the entry or exodus of firms D) a large number of buyers and sellers Answer: A

Type: A Topic: 2 E: 415 MI: 171 15. The demand schedule or curve confronted by the individual purely competitive firm is: A) relatively elastic, that is, the elasticity coefficient is greater than unity. perfectly elastic. C) relatively inelastic, that is, the elasticity coefficient is less than unity. D) perfectly inelastic. Answer: B

Type: A Topic: 2 E: 415-416 MI: 171-172 16. Which of the following is characteristic of a purely competitive seller's demand curve? A) Price and marginal revenue are equal at all levels of output. Average revenue is less than price. C) Its elasticity coefficient is 1 at all levels of output.

Page 640: Microeconomics Study Guide

D) It is the same as the market demand curve. Answer: A

Use the following to answer questions 17-19:

In answering the next question(s), assume a graph in which dollars are measured on the vertical axis and output on the horizontal axis.

Type: G Topic: 2 E: 415-416 MI: 171-172 17. Refer to the above information. For a purely competitive firm total revenue: A) graphs as a straight, upsloping line. C) is a straight line, parallel to the horizontal axis. is a straight line, parallel to the vertical axis. D) graphs as a straight, downsloping line. Answer: A

Type: G Topic: 2 E: 416 MI: 172 18. Refer to the above information. For a purely competitive firm marginal revenue: A) graphs as a straight, upsloping line. C) is a straight line, parallel to the horizontal axis. is a straight line, parallel to the vertical axis. D) graphs as a straight, downsloping line. Answer: C

Type: G Topic: 2 E: 416 MI: 172 19. Refer to the above information. For a purely competitive firm: A) marginal revenue will graph as an upsloping line. the demand curve will lie above the marginal revenue curve. C) the marginal revenue curve will lie above the demand curve. D) the demand and marginal revenue curves will coincide. Answer: D

Type: A Topic: 2 E: 415-416 MI: 171-172 20. If a firm in a purely competitive industry is confronted with an equilibrium price of $5, its marginal revenue: A) may be either greater or less than $5. C) will be less than $5. will also be $5. D) will be greater than $5. Answer: B

Type: A Topic: 2 E: 414 MI: 170 21. Price is constant or given to the individual firm selling in a purely competitive market because: A) the firm's demand curve is downsloping. of product differentiation reinforced by extensive advertising. C) each seller supplies a negligible fraction of total supply. D) there are no good substitutes for its product. Answer: C

Type: A Topic: 2 E: 415-416 MI: 171-172 22. For a purely competitive seller, price equals: A) average revenue. marginal revenue. C) total revenue divided by output. D) all of the above.

Page 641: Microeconomics Study Guide

Answer: D

Type: A Topic: 2 E: 415 MI: 171 23. For a purely competitive firm total revenue: A) is price times quantity sold. increases by a constant absolute amount as output expands. C) graphs as a straight upsloping line from the origin. D) has all of the above characteristics. Answer: D

Type: A Topic: 2 E: 416 MI: 172 24. The marginal revenue curve of a purely competitive firm: A) lies below the firm's demand curve. increases at an increasing rate as output expands. C) is horizontal at the market price. D) is downsloping because price must be reduced to sell more output. Answer: C

Type: A Topic: 2 E: 415 MI: 171 25. The demand curve in a purely competitive industry is ______, while the demand curve to a single firm in that industry is ______.A) perfectly inelastic, perfectly elastic C) downsloping, perfectly inelastic downsloping, perfectly elastic D) perfectly elastic, downsloping Answer: B

Type: A Topic: 2 E: 415 MI: 171 26. A perfectly elastic demand curve implies that the firm: A) must lower price to sell more output. can sell as much output as it chooses at the existing price. C) realizes an increase in total revenue which is less than product price when it sells an extra unit. D) is selling a differentiated (heterogeneous) product. Answer: B

Type: A Topic: 2 E: 415-416 MI: 171-172 27. The vertical distance between the horizontal axis and any point on a pure competitor's demand curve measures: A) total revenue. total cost. C) product price, marginal revenue, and average revenue. D) the quantity demanded. Answer: C

Type: A Topic: 2 E: 416 MI: 172 28. The fact that a purely competitive firm's total revenue curve is linear and upsloping to the right implies that: A) product price increases as output increases. product price decreases as output increases. C) product price is constant at all levels of output.

Page 642: Microeconomics Study Guide

D) marginal revenue declines as more output is produced. Answer: C

Type: A Topic: 2 E: 415 MI: 171 29. Which of the following statements is correct? A) The demand curve for a purely competitive firm is perfectly elastic, but the demand curve for a purely competitive industry is downsloping. The demand curve for a purely competitive firm is downsloping, but the demand curve for a purely competitive industry is perfectly elastic. C) The demand curves are downsloping for both a purely competitive firm and a purely competitive industry. D) The demand curves are perfectly elastic for both a purely competitive firm and a purely competitive industry. Answer: A

Use the following to answer questions 30-31:

Type: G Topic: 2 E: 416 MI: 172 30. Refer to the above diagram, which pertains to a purely competitive firm. Curve A represents: A) total revenue and marginal revenue. C) total revenue and average revenue. marginal revenue only. D) total revenue only. Answer: D

Type: G Topic: 2 E: 415-416 MI: 171-172 31. Refer to the above diagram, which pertains to a purely competitive firm. Curve C represents: A) total revenue and marginal revenue. C) total revenue and average revenue. marginal revenue only. D) average revenue and marginal revenue. Answer: D

Type: A Topic: 2 E: 416 MI: 172 32. A purely competitive seller's average revenue curve coincides with: A) its marginal revenue curve only. its demand curve only. C) both its demand and marginal revenue curves. D) neither its demand nor its marginal revenue curve. Answer: C

Type: D Topic: 2 E: 416 MI: 172 33. Marginal revenue is the: A) change in product price associated with the sale of one more unit of output. change in average revenue associated with the sale of one more unit of output. C) difference between product price and average total cost. D) change in total revenue associated with the sale of one more unit of output. Answer: D

Page 643: Microeconomics Study Guide

Type: A Topic: 2 E: 416 MI: 172 34. Marginal revenue for a purely competitive firm: A) is greater than price. C) is equal to price. is less than price. D) may be either greater or less than price. Answer: C

Profit maximizing in short run

Type: A Topic: 3 E: 416 MI: 172 35. Firms seek to maximize: A) per unit profit. total revenue. C) total profit. D) market share. Answer: C

Type: A Topic: 3 E: 418 MI: 174 36. A competitive firm in the short run can determine the profit-maximizing (or loss-minimizing) output by equating: A) price and average total cost. C) marginal revenue and marginal cost. price and average fixed cost. D) price and marginal revenue. Answer: C

Type: A Topic: 3 E: 417 MI: 173 37. In the short run a purely competitive firm that seeks to maximize profit will produce: A) where the demand and the ATC curves intersect. where total revenue exceeds total cost by the maximum amount. C) that output where economic profits are zero. D) at any point where the total revenue and total cost curves intersect. Answer: B

Use the following to answer questions 38-41:

Type: G Topic: 3 E: 417-418 MI: 173-174 38. Refer to the above short-run data. Total fixed cost for this firm is: A) about $67. $300. C) $200. D) $100. Answer: C

Type: G Topic: 3 E: 417 MI: 173 39. Refer to the above short-run data. The shape of the total cost curve reflects: A) diminishing opportunity costs. C) increasing and diminishing returns. the law of rising fixed costs. D) economies and diseconomies of scale. Answer: C

Type: G Topic: 3 E: 418 MI: 174 40. Refer to the above short-run data. The profit-maximizing output for this firm is:

Page 644: Microeconomics Study Guide

A) above 440 units. 440 units. C) 320 units. D) 100 units. Answer: C

Type: G Topic: 3 E: 418 MI: 174 41. Refer to the above short-run data. Which of the following is correct? A) This firm will maximize its profit at 440 untis of output.B) Any level of output between 100 and 440 units will yield an economic profit. C) This firm's marginal revenue rises with output.D) Any level of output less than 100 units or greater than 440 units is profitable. Answer: B

Type: A Topic: 3 E: 417 MI: 173 42. A competitive firm will maximize profits at that output at which: A) total revenue exceeds total cost by the greatest amount. B) total revenue and total cost are equal. C) price exceeds average total cost by the largest amount. D) the difference between marginal revenue and price is at a maximum. Answer: A

Use the following to answer questions 43-48:

Type: G Topic: 3 E: 418 MI: 174 43. Curve (1) in the above diagram is a purely competitive firm's:A) total cost curve. C) marginal revenue curveB) total revenue curve. D) total economic profit curve.Answer: D

Type: G Topic: 3 E: 416 MI: 172 44. Curve (2) in the above diagram is a purely competitive firm'sA) total cost curve. C) marginal revenue curveB) total revenue curve. D) total economic profit curve.Answer: C

Type: G Topic: 3 E: 418 MI: 174 45. Curve (3) in the above diagram is a purely competitive firm's A) total cost curve. C) marginal revenue curve.B) total revenue curve. D) total economic profit curve.Answer: B

Type: G Topic: 3 E: 418 MI: 174 46. Curve (4) in the above diagram is a purely competitive firm's:A) total cost curve. B) total revenue curve. C) marginal revenue curve. D) total profit curve.Answer: A

Page 645: Microeconomics Study Guide

Type: G Topic: 3 E: 418 MI: 174 47. Refer to the above diagram. Other things equal, an increase of product price would be shown as:A) an increase in the steepness of curve (3), an upward shift in curve (2), and upward shift in curve (1).B) a decrease in the steepness of curve (3), a downward shift in curve (2), and an upward shift in curve (1).C) an downward shift in curve (4) and an upward shift in curve (1), with no changes in lines (2) and (3).D) an upward shift in line (2) only.Answer: A

Type: G Topic: 3 E: 418 MI: 174 48. The firm represented by the above diagram would maximize its profit where:A) curves (2) and (1) intersect.B) curve (1) touches the horizontal axis for the second time.C) the vertical distance between curves (3) and (4) is the greatest.D) curves (3) and (4) intersect.Answer: C

Type: D Topic: 3 E: 417-418 MI: 173-174 49. A firm reaches a break-even point (normal profit position) where: A) marginal revenue cuts the horizontal axis. B) marginal cost intersects the average variable cost curve. C) total revenue equals total variable cost. D) total revenue and total cost are equal. Answer: D

Type: A Topic: 3 E: 419 MI: 175 50. The MR = MC rule applies: A) to firms in all types of industries. C) only to monopolies. B) only when the firm is a "price taker." D) only to purely competitive firms. Answer: A

Type: A Topic: 3 E: 418 MI: 174 51. When a firm is maximizing profit it will necessarily be: A) maximizing profit per unit of output. B) maximizing the difference between total revenue and total cost. C) minimizing total cost. D) maximizing total revenue. Answer: B

Type: A Topic: 3 E: 419 MI: 175 52. The MR = MC rule can be restated for a purely competitive seller as P = MC because: A) each additional unit of output adds exactly its price to total revenue. B) the firm's average revenue curve is downsloping. C) the market demand curve is downsloping. D) the firm's marginal revenue and total revenue curves will coincide. Answer: A

Page 646: Microeconomics Study Guide

Type: A Topic: 3 E: 424 MI: 180 53. In the short run the individual competitive firm's supply curve is that segment of the: A) average variable cost curve lying below the marginal cost curve. B) marginal cost curve lying above the average variable cost curve. C) marginal revenue curve lying below the demand curve. D) marginal cost curve lying between the average total cost and average variable cost curves. Answer: B

Type: A Topic: 3 E: 424 MI: 180 54. Which of the following is not a valid generalization concerning the relationship between price and costs for a purely competitive seller in the short run? A) Price must be at least equal to average total cost. B) Price times quantity produced must be equal to or greater than total variable cost for some level of output or the firm will close down in the short run. C) Price may be equal to, greater than, or less than average total cost. D) Price must be equal to or greater than minimum average variable cost for the firm to continue producing. Answer: A

Type: A Topic: 3 E: 418 MI: 174 55. Assume the XYZ Corporation is producing 20 units of output. It is selling this output in a purely competitive market at $10 per unit. Its total fixed costs are $100 and its average variable cost is $3 at 20 units of output. This corporation:A) should close down in the short run. C) is realizing a loss of $60. B) is maximizing its profits. D) is realizing an economic profit of $40. Answer: D

56. A purely competitive firm's short-run supply curve is: A) perfectly elastic at the minimum average total cost. upsloping and equal to the portion of the marginal cost curve that lies above the average variable cost curve. C) upsloping and equal to the portion of the marginal cost curve that lies above the average total cost curve. D) upsloping only when the industry has constant costs. Answer: B

Type: A Topic: 3 E: 424 MI: 180 57. Suppose you find that the price of your product is less than minimum AVC. You should: A) minimize your losses by producing where P = MC. maximize your profits by producing where P = MC. C) close down because, by producing, your losses will exceed your total fixed costs. D) close down because total revenue exceeds total variable cost. Answer: C

Type: A Topic: 3 E: 422 MI: 178 58. If a purely competitive firm shuts down in the short run:

Page 647: Microeconomics Study Guide

A) its loss will be zero. it will realize a loss equal to its total variable costs. C) it will realize a loss equal to its total fixed costs. D) it will realize a loss equal to its total costs. Answer: C

Type: A Topic: 3 E: 422 MI: 178 59. A purely competitive firm should produce in the short run if its total revenue is sufficient to cover its: A) total variable costs. total costs. C) total fixed costs. D) marginal costs. Answer: A

Use the following to answer questions 60-64:

Answer the next question(s) on the basis of the following data confronting a firm:

Type: T Topic: 3 E: 416 MI: 172 60. Refer to the above data. This firm is selling its output in a(n): A) imperfectly competitive market. C) purely competitive market. monopolistic market. D) oligopolistic market. Answer: C

Type: T Topic: 3 E: 424 MI: 180 61. Refer to the above data. If the firm's minimum average variable cost is $10, the firm's profit-maximizing level of output would be: A) 2. 3. C) 4. D) 5. Answer: B

Type: T Topic: 3 E: 417 MI: 173 62. Refer to the above data. At the profit-maximizing output the firm's total revenue is:A) $48. $32. C) $80. D) $64. Answer: A

Type: T Topic: 3 E: 417 MI: 173 63. Refer to the above data. At the profit-maximizing output the firm's total cost is:A) $48. $32. C) $80. D) $64. Answer: B

Type: T Topic: 3 E: 417 MI: 173 64. Refer to the above data. The firm's: A) economic profit is $12. economic profit is $16. C) loss is $14. D) economic profit is $3. Answer: B

Type: A Topic: 3 E: 424 MI: 180 65. In the short run a purely competitive firm will always make an economic profit if:

Page 648: Microeconomics Study Guide

A) P = ATC. P > AVC. C) P = MC. D) P > ATC. Answer: D

Type: A Topic: 3 E: 424 MI: 180 66. Suppose that at 500 units of output marginal revenue is equal to marginal cost. The firm is selling its output at $5 per unit and average total cost at 500 units of output is $6. On the basis of this information we: A) can say that the firm should close down in the short run. can say that the firm can produce and realize an economic profit in the short run. C) cannot determine whether the firm should produce or shut down in the short run. D) can assume the firm is not using the most efficient technology. Answer: C

Type: A Topic: 3 E: 424 MI: 180 67. If a firm is confronted with economic losses in the short run, it will decide whether or not to produce by comparing: A) marginal revenue and marginal cost. C) total revenue and total cost. price and minimum average variable cost. D) total revenue and total fixed cost. Answer: B

Type: A Topic: 3 E: 424 MI: 180 68. A firm finds that at its MR = MC output, its TC = $1000, TVC = $800, TFC = $200, and total revenue is $900. This firm should: A) shut down in the short run. produce because the resulting loss is less than its TFC. C) produce because it will realize an economic profit. D) liquidate its assets and go out of business. Answer: B

Type: A Topic: 3 E: 424 MI: 180 69. The lowest point on a purely competitive firm's short-run supply curve corresponds to: A) the minimum point on its ATC curve. C) the minimum point on its AFC curve. the minimum point on its AVC curve. D) the minimum point on its MC curve. Answer: B

Use the following to answer questions 70-73:

Type: G Topic: 3 E: 423-424 MI: 179-180 70. Refer to the above diagram for a purely competitive producer. The lowest price at which the firm should produce (as opposed to shutting down) is: A) P1. P2. C) P3. D) P4. Answer: B

Type: G Topic: 3 E: 423-424 MI: 179-180 71. Refer to the above diagram for a purely competitive producer. The firm will produce at a loss at all prices:

Page 649: Microeconomics Study Guide

A) above P1. above P3. C) above P4. D) between P2 and P 3. Answer: D

Type: G Topic: 3 E: 424 MI: 180 72. Refer to the above diagram for a purely competitive producer. If product price is P3: A) the firm will maximize profit at point d. C) economic profits will be zero. the firm will earn an economic profit. D) new firms will enter this industry. Answer: C

Type: G Topic: 3 E: 424 MI: 180 73. Refer to the above diagram for a purely competitive producer. The firm's short-run supply curve is: A) the abcd segment of the MC curve. C) the cd segment of the MC curve. the bcd segment of the MC curve. D) not shown. Answer: B

Type: A Topic: 3 E: 424 MI: 180 74. The short-run supply curve of a purely competitive producer is based on its: A) AVC curve. ATC curve. C) AFC curve. D) MC curve. Answer: D

Type: A Topic: 3 E: 424 MI: 180 75. On a per unit basis economic profit can be determined as the difference between: A) marginal revenue and product price. C) marginal revenue and marginal cost. product price and average total cost. D) average fixed cost and product price. Answer: B

Type: A Topic: 3 E: 423-424 MI: 179-180 76. In the short run a purely competitive seller will shut down if: A) it cannot produce at an economic profit. price is less than average variable cost at all outputs. C) price is less than average fixed cost at all outputs. D) there is no point at which marginal revenue and marginal cost are equal. Answer: B

Use the following to answer questions 77-81:

Type: G Topic: 3 E: 424 MI: 180 77. Refer to the above diagram. To maximize profit or minimize losses this firm will produce: A) K units at price C. D units at price J. C) E units at price A. D) E units at price B. Answer: C

Type: G Topic: 3 E: 420 MI: 176 78. Refer to the above diagram. At the profit-maximizing output, total revenue will be:

Page 650: Microeconomics Study Guide

A) 0AHE. 0BGE. C) 0CFE. D) ABGE. Answer: A

Type: G Topic: 3 E: 420 MI: 176 79. Refer to the above diagram. At the profit-maximizing output, total fixed cost is equal to: A) 0AHE. 0BGE. C) 0CFE. D) BCFG. Answer: D

Type: G Topic: 3 E: 420 MI: 176 80. Refer to the above diagram. At the profit-maximizing output, total variable cost is equal to: A) 0AHE. 0CFE. C) 0BGE. D) ABGH. Answer: B

Type: G Topic: 3 E: 420 MI: 176 81. Refer to the above diagram. At the profit-maximizing output, the firm will realize: A) a loss equal to BCFG. C) an economic profit of ACFH. a loss equal to ACFH. D) an economic profit of ABGH. Answer: D

Type: A Topic: 3 E: 420 MI: 176 82. If a purely competitive firm is producing at some level less than the profit-maximizing output, then:A) price is necessarily greater than average total cost. fixed costs are large relative to variable costs. C) price exceeds marginal revenue. D) marginal revenue exceeds marginal cost. Answer: D

Use the following to answer questions 83-87:

Answer the next question(s) on the basis of the following cost data for a firm that is selling in a purely competitive market:

Type: T Topic: 3 E: 421 MI: 177 83. Refer to the above data. If the market price for the firm's product is $12, the competitive firm will produce: A) 4 units at a loss of $109. C) 8 units at a loss of $48.80. 4 units at an economic profit of $31.75. D) zero units at a loss of $100. Answer: D

Type: T Topic: 3 E: 420 MI: 176 84. Refer to the above data. If the market price for the firm's product is $32, the competitive firm will produce: A) 8 units at an economic profit of $16. 5 units at a loss of $10. C) 8 units at a loss equal to the firm's total fixed cost.

Page 651: Microeconomics Study Guide

D) 7 units at an economic profit of $41.50. Answer: A

Type: T Topic: 3 E: 421 MI: 177 85. Refer to the above data. If the market price for the firm's product is $28, the competitive firm will: A) produce 4 units at a loss of $17.40. C) close down in the short run. produce 7 units at a loss of $14.00. D) produce 6 units at a loss of $23.80. Answer: B

Type: T Topic: 3 E: 423 MI: 179 86. Refer to the above data. Which of the following is the firm's short-run supply schedule?

Answer: C

Type: T Topic: 3 E: 425 MI: 181 87. Refer to the above data. If there were 1,000 identical firms in this industry and total or market demand is as shown below, equilibrium price will be:

A) $32. $42. C) $36. D) $13. Answer: C

Type: A Topic: 3 E: 423-424 MI: 179-180 88. If at the MC = MR output, AVC exceeds price: A) new firms will enter this industry. the firm should produce the MC = MR output and realize an economic profit. C) the firm should shut down in the short run. D) the firm should expand its plant. Answer: C

Use the following to answer questions 89-94:

Type: G Topic: 3 E: 424 MI: 180 89. Refer to the above diagram. The profit-maximizing output: A) is n. is k. C) is h. D) cannot be determined from the information given. Answer: A

Type: G Topic: 3 E: 424 MI: 180 90. Refer to the above diagram. At the profit-maximizing output, average variable cost is: A) ef. fg. C) na. D) ac.

Page 652: Microeconomics Study Guide

Answer: C

Type: G Topic: 3 E: 424 MI: 180 91. Refer to the above diagram. At the profit-maximizing output, total profit is: A) efbc. fgab. C) egac. D) 0fbn. Answer: A

Type: G Topic: 3 E: 424 MI: 180 92. Refer to the above diagram. For any level of output, total fixed cost: A) is fgab. is 0gan. C) is ba. D) is efbc. Answer: A

Type: G Topic: 3 E: 424 MI: 180 93. Refer to the above diagram. The short-run supply curve for this firm is the: A) entire MC curve. segment of the AVC curve lying to the right of the MC curve. C) segment of the MC curve lying above the ATC curve. D) segment of the MC curve lying above the AVC curve. Answer: D

Type: G Topic: 3 E: 416 MI: 172 94. Refer to the above diagram. This firm is selling its product in a(n): A) purely competitive market. C) monopsonistic market. imperfectly competitive market. D) monopolistic market. Answer: A

Type: A Topic: 3 E: 423-424 MI: 179-180 95. In the short run a purely competitive seller will shut down if product price: A) equals average revenue. is greater than MC. C) is less than AVC. D) is less than ATC. Answer: C

Type: A Topic: 3 E: 423-424 MI: 179-180 96. The short-run shut-down point for a purely competitive firm occurs: A) at any point where price is less than the minimum AVC. between the two break-even points. C) at any point where total revenue is less than total cost. D) at any point where the firm is not making an economic profit. Answer: A

Type: A Topic: 3 E: 427 MI: 183 97. In a purely competitive industry: A) there will be no economic profits in either the short run or the long run. B) economic profits may persist in the long run if consumer demand is strong and stable. C) there may be economic profits in the short run, but not in the long run. D) there may be economic profits in the long run, but not in the short run.

Page 653: Microeconomics Study Guide

Answer: C

Type: A Topic: 3 E: 424 MI: 180 98. The short-run supply curve for a purely competitive industry can be found by: A) multiplying the AVC curve of the representative firm by the number of firms in the industry. B) adding horizontally the AVC curves of all firms. C) summing horizontally the segments of the MC curves lying above the AVC curve for all firms. D) adding horizontally the immediate market period supply curves of each firm. Answer: C

Type: A Topic: 3 E: 419 MI: 175 99. DASH Airlines is considering the addition of a flight from Red Cloud to David City. The total cost of the flight would be $1100 of which fixed costs are $800. Expected revenues from the flight are $600. DASH should: A) not add this flight because only flights which cover their full costs are profitable. B) not add this flight because it is not profitable at the margin. C) add this flight because marginal revenue exceeds marginal costs. D) not add this flight because total costs exceed total revenue. Answer: C

Type: A Topic: 3 E: 421 MI: 177 100. In contrast to American firms, Japanese firms frequently make lifetime employment commitments to their workers and agree not to lay them off when product demand is weak. Other things being equal, we would expect Japanese firms to: A) face more elastic product demand curves than American firms. B) have relatively greater variable costs than American firms. C) discontinue production at higher product prices than would American firms. D) continue to produce in the short run at lower prices than would American firms. Answer: D

Type: A Topic: 3 E: 421 MI: 177 101. Assume for a competitive firm that MC = AVC at $12, MC = ATC at $20, and MC = MR at $16. This firm will: A) realize a profit of $4 per unit of output. B) maximize its profit by producing in the short run. C) minimize its losses by producing in the short run. D) shut down in the short run. Answer: C

Type: A Topic: 3 E: 418 MI: 174 102. The principle that a firm should produce up to the point where the marginal revenue from the sale of an extra unit of output is equal to the marginal cost of producing it is known as the: A) output-maximizing rule. B) profit-maximizing rule. C) shut-down rule. D) break-even rule. Answer: B

Type: A Topic: 3 E: 419 MI: 175 103. If a purely competitive firm is producing at the P = MC output and realizing an economic profit, at that output:

Page 654: Microeconomics Study Guide

A) marginal revenue is less than price. C) ATC is being minimized. B) marginal revenue exceeds ATC. D) total revenue equals total cost. Answer: B

Type: A Topic: 3 E: 418 MI: 174 104. If a profit-seeking competitive firm is producing its profit-maximizing output and its total fixed costs fall by 25 percent, the firm should: A) use more labor and less capital to produce a larger output. B) not change its output. C) reduce its output. D) increase its output. Answer: B

Use the following to answer questions 105-108:

Type: G Topic: 3 E: 424 MI: 180 105. Refer to the above diagram. At P2, this firm will: A) produce 44 units and realize an economic profit. B) produce 44 units and earn only a normal profit. C) produce 66 units and earn only a normal profit. D) shut down in the short run. Answer: B

Type: G Topic: 3 E: 424 MI: 180 106. Refer to the above diagram. At P1, this firm will produce: A) 47 units and break even. C) 66 units and earn only a normal profit. B) 47 units and realize an economic profit. D) 24 units and earn only a normal profit. Answer: B

Type: G Topic: 3 E: 423-424 MI: 179-180 107. Refer to the above diagram. At P4, this firm will: A) shut down in the short run. C) produce 30 units and earn only a normal profit. B) produce 30 units and incur a loss. D) produce 10 units and earn only a normal profit. Answer: A

Type: G Topic: 3 E: 423-424 MI: 179-180 108. Refer to the above diagram. At P3, this firm will: A) produce 14 units and realize an economic profit. B) produce 62 units and earn only a normal profit. C) produce 40 units and incur a loss. D) shut down in the short run. Answer: C

Page 655: Microeconomics Study Guide

Type: A Topic: 3 E: 423-424 MI: 179-180 109. The loss of a purely competitive firm which shuts down in the short run: A) is equal to its total variable costs. C) is equal to its total fixed costs. B) is zero. D) cannot be determined. Answer: C Type: A Topic: 3 E: 423-424 MI: 179-180 110. The Ajax Manufacturing Company is selling in a purely competitive market. Its output is 100 units which sell at $4 each. At this level of output total cost is $600, total fixed cost is $100, and marginal cost is $4. The firm should: A) reduce output to about 80 units. C) continue to produce 100 units. B) expand its production. D) produce zero units of output. Answer: D

Type: A Topic: 3 E: 419 MI: 175 111. The MR = MC rule can be restated for a purely competitive seller as P = MC because: A) each additional unit of output adds exactly its constant price to total revenue. B) the firm's average revenue curve is downsloping. C) the market demand curve is downsloping. D) the firm's marginal revenue and total revenue curves will coincide. Answer: A

Type: A Topic: 3 E: 421 MI: 177 112. If a purely competitive firm is maximizing economic profit: A) it is necessarily maximizing per-unit profit. B) it may or may not be maximizing per unit profit. C) then per-unit profit will be minimized. D) it is necessarily overallocating resources to its product. Answer: B

Use the following to answer questions 113-115:

Answer the next question(s) on the basis of the following cost data for a purely competitive seller:

Type: T Topic: 3 E: 417 MI: 173 113. Refer to the above data. If product price is $60, the firm will: A) shut down. B) produce 4 units and realize a $120 economic profit. C) produce 6 units and realize a $100 economic profit. D) produce 3 units and incur a $40 loss. Answer: C

Type: T Topic: 3 E: 417 MI: 173 114. Refer to the above data. If product price is $45, the firm will: A) shut down.

Page 656: Microeconomics Study Guide

B) produce 4 units and realize a $120 economic profit. C) produce 5 units and realize a $15 economic profit. D) produce 6 units and realize a $100 economic profit. Answer: C

Type: T Topic: 3 E: 422 MI: 178 115. Refer to the above data. If product price is $25, the firm will: A) shut down and incur a $90 loss. B) shut down and incur a $50 loss. C) produce 3 units and incur a $65 loss. D) produce 4 units and realize a $10 economic profit. Answer: B

Type: A Topic: 3 E: 422 MI: 178 116. If total revenue is less than total variable costs at the MR = MC output, a purely competitive firm should: A) shut down. C) produce and may or may not realize a profit. B) produce, but will necessarily realize a loss. D) increase its output. Answer: A

Type: A Topic: 3 E: 417 MI: 173 117. Assume a purely competitive firm is selling 200 units of output at $3 each. At this output its total fixed cost is $100 and its total variable cost is $350. This firm:A) is maximizing its profit. B) is making a profit, but not necessarily the maximum profit. C) is incurring losses. D) should shut down in the short run. Answer: B

Use the following to answer questions 118-122:

Type: G Topic: 3 E: 424 MI: 180 118. Refer to the above diagram. This firm will earn only a normal profit if product price is: A) P1. B) P2. C) P3. D) P4. Answer: C

Type: G Topic: 3 E: 424 MI: 180 119. Refer to the above diagram. The firm will realize an economic profit if price is: A) P1. B) P2. C) P3. D) P4. Answer: D

Type: G Topic: 3 E: 423-424 MI: 179-180 120. Refer to the above diagram. The firm will produce at a loss if price is: A) P1. B) P2. C) P3. D) P4.

Page 657: Microeconomics Study Guide

Answer: B

Type: G Topic: 3 E: 423-424 MI: 179-180 121. Refer to the above diagram. The firm will shut down at any price less than: A) P1. B) P2. C) P3. D) P4. Answer: A

Type: G Topic: 3 E: 424 MI: 180 Status: New 122. Refer to the above diagram. The firm's supply curve is the segment of the:A) MC curve above its intersection with the AVC curve.B) MC curve above its intersection with the ATC curve.C) AVC curve above its intersection with the MC curve.D) ATC curve above its intersection with the MC curve.Answer: A

Use the following to answer questions 123-132:

Answer the next question(s) on the basis of the following cost data for a firm that is selling in a purely competitive market.

Type: G Topic: 3 E: 402 MI: 158 Status: New 123. Refer to the above data. The marginal cost column reflects:A) the law of diminishing returns. C) diseconomies of scale.B) the law of diminishing marginal utility. D) economies of scale. Answer: A

Type: G Topic: 3 E: 398-399 MI: 154-155 Status: New 124. Refer to the above data. At 6 units of output, total fixed cost is ____ and total cost is ____:A) $25; $50. B) $50; $300. C) $100, $200. D) $150; $300.Answer: D

Type: G Topic: 3 E: 398-399 MI: 154-155 Status: New 125. Refer to the above data. At 3 units of output, total variable cost is ____ and total cost is ____:A) $20; $70. B) $60; $210. C) $20, $210. D) $60; $350.Answer: B

Type: G Topic: 3 E: 398-399 MI: 154-155 Status: New 126. Refer to the above data. We can infer that, at zero output, this firm's total fixed, total variable, and total costs are:A) zero, zero, and zero, respectively. C) $150, $25, and $175, respectively.B) zero, $25, and $175, respectively. D) $150, zero, and $150, respectively.Answer: D

Type: G Topic: 3 E: 419-420 MI: 175-176 Status: New

Page 658: Microeconomics Study Guide

127. Refer to the above data. If the market price for this firm's product is $86.95, it will produce:A) 9 units at an economic profit of zero. C) 9 units at an economic profit of $281.52.B) 6 units at a loss of $90. D) 8 units at an economic profit of $130.48.Answer: C

Type: G Topic: 3 E: 419-420 MI: 175-176 Status: New 128. Refer to the above data. If the market price for this firm's product is $68.07, it will produce:A) 8 units at an economic profit of zero. C) 9 units at an economic profit of $281.52.B) 6 units at a loss of $90. D) 8 units at an economic profit of $130.48.Answer: D

Type: G Topic: 3 E: 421-422 MI: 177-178 Status: New 129. Refer to the above data. If the market price for this firm's product is $35, it will produce:A) 6 units at a loss of $150. C) 9 units at an economic profit of $281.52.B) 6 units at a loss of $90. D) 8 units at an economic profit of $130.48.Answer: B

Type: G Topic: 3 E: 421-422 MI: 177-178 Status: New 130. Refer to the above data. If the market price for this firm's product is $24, it will produce:A) 4 units at a loss of $150. C) 3 units at an economic profit of zero.B) 6 units at a loss of $90. D) 4 units at a loss of $138.Answer: D

Type: G Topic: 3 E: 422-423 MI: 178-179 Status: New 131. Refer to the above data. If the market price for this firm's product is $14, it will produce:A) 0 units at a loss of $150. C) 3 units at an economic profit of zero.B) 3 units at a loss of $168. D) 4 units at a loss of $138.Answer: A

Type: G Topic: 3 E: 424 MI: 180 Status: New 132. Refer to the above data. The firm's supply schedule is reflected in the:A) marginal cost data for the fourth through tenth units of output.B) marginal cost data for the first through tenth units of output.C) average total cost data for the seventh through tenth units of output.D) average variable cost data for fifth through tenth units of output.Answer: A

Type: A Topic: 3 E: 417 MI: 173 133. A purely competitive seller should produce (rather than shut down) in the short run: A) only if total revenue exceeds total cost. B) only if total cost exceeds total revenue. C) if total revenue exceeds total cost or if total cost exceeds total revenue by some amount less than total fixed cost. D) if total cost exceeds total revenue by some amount greater than total fixed cost. Answer: C

Page 659: Microeconomics Study Guide

Type: A Topic: 3 E: 417-418 MI: 173-174 134. In the short run a purely competitive firm will maximize profit by producing that output at which: A) total revenue exceeds total cost by a maximum amount.B) total revenue exceeds total cost by a minimum amount.C) total revenue and total cost are equal. D) total fixed cost equals total variable cost. Answer: A

Type: A Topic: 6 E: 368 MI: 124 155. Suppose that when your income increases from $28,000 to $30,000 per year, your purchases of X increase from 4 to 5 units because of that income increase. Thus:A) X is an inferior good. X is a substitute good. C) the income effect exceeds the substitution effect. D) the demand for X is elastic with respect to income. Answer: D

Use the following to answer questions 156-163:

Type: G Topic: 6 E: 367 MI: 123 156. Refer to the above diagrams. The case of substitute goods is represented by figure: A) A. B. C) C. D) D. Answer: D

Type: G Topic: 6 E: 368 MI: 124 157. Refer to the above diagrams. The case of a normal good is represented by figure(s): A) A. B. C) C. D) D.Answer: A

Type: G Topic: 6 E: 368 MI: 124 158. Refer to the above diagrams. The case of an inferior good is represented by figure(s): A) A only. B only. C) C. D) D. Answer: B

Type: G Topic: 6 E: 367 MI: 123 159. Refer to the above diagrams. The case of complementary goods is represented by figure: A) A. B. C) C. D) D. Answer: C

Type: G Topic: 6 E: 368 MI: 124 160. Refer to the above diagrams. In which case would the coefficient of income elasticity be positive? A) A. B. C) C. D) D.

Page 660: Microeconomics Study Guide

Answer: A

Type: G Topic: 6 E: 368 MI: 124 161. Refer to the above diagrams. In which case would the coefficient of income elasticity be negative? A) A. B. C) C. D) D. Answer: B

Type: G Topic: 6 E: 368 MI: 124 162. Refer to the above diagrams. In which case would the coefficient of cross elasticity of demand be positive? A) A. B. C) C. D) D. Answer: D

Type: G Topic: 6 E: 368 MI: 124 163. Refer to the above diagrams. In which case would the coefficient of cross elasticity of demand be negative? A) A B C) C D) D Answer: C

Type: A Topic: 6 E: 367 MI: 123 164. Suppose that a 10 percent increase in the price of normal good Y causes a 20 percent increase in the quantity demanded of normal good X. The coefficient of cross elasticity of demand is: A) negative and therefore these goods are substitutes. negative and therefore these goods are complements. C) positive and therefore these goods are substitutes. D) positive and therefore these goods are complements. Answer: C

Type: A Topic: 6 E: 367 MI: 123 165. Suppose that a 20 percent increase in the price of normal good Y causes a 10 percent decline in the quantity demanded of normal good X. The coefficient of cross elasticity of demand is: A) negative and therefore these goods are substitutes. negative and therefore these goods are complements. C) positive and therefore these goods are substitutes. D) positive and therefore these goods are complements. Answer: B

Type: A Topic: 6 E: 368 MI: 124 166. Assume that a 4 percent increase in income in the economy produces an 8 percent increase in the quantity demanded of good X. The coefficient of income elasticity of demand is: A) negative and therefore X is an inferior good. C) positive and therefore X is an inferior good. negative and therefore X is a normal good. D) positive and therefore X is a normal good. Answer: D

Type: A Topic: 6 E: 368 MI: 124 167. Assume that a 3 percent increase in income in the economy produces a 1 percent decline in the quantity demanded of good X. The coefficient of income elasticity of demand for good X is:

Page 661: Microeconomics Study Guide

A) negative and therefore X is an inferior good. C) positive and therefore X is an inferior good. negative and therefore X is a normal good. D) positive and therefore X is a normal good. Answer: A

Consider This Questions

Type: F E: 358 MI: 114 Status: New 168. (Consider This) Elastic demand is analogous to a __________ and inelastic demand to a _________.A) normal wrench; socket wrench C) Ace bandage; firm rubber tie-downtight rubber band; loose rubber band D) one-foot ruler; tape measureAnswer: C

Type: F E: 358 MI: 114 Status: New 169. (Consider This) Elasticity can be thought of as degree of relative: A) video brightness. price bounce. C) audio volume. D) quantity stretch.Answer: D

Last Word Questions

Type: A E: 369 MI: 125 Status: New 170. (Last Word) Suppose that a firm has "pricing power" and can segregate its market into two distinct groups based on differences in elasticities of demand. The firm might charge:A) a lower price to the group that has the less elastic demand. a higher price to the group that has the less elastic demand.C) the same price to both groups but include a "free" related product for the group that has an inelastic demand.D) the same price to both groups but make it difficult for the group with the more elastic demand to gain access to the product.Answer: B

Type: A E: 369 MI: 125 Status: New 171. (Last Word) Microsoft charges a substantially lower price for a software upgrade than for the initial purchase of the software. This implies that Microsoft views the demand curve for the software upgrade to be:A) more elastic than the demand for the original software.upsloping rather than downsloping.C) less elastic than the demand for the original software.D) of less value than the original software. Answer: A

Type: A E: 369 MI: 125 Status: New 172. (Last Word) Which of the following is not an example of pricing based on group differences in elasticity of demand?A) Senior-citizen discounts at restaurants and motels. Cash rebates for purchases of automobiles.C) Children discounts for admissions to theme parks.D) Discounted student prices for visits to museums.Answer: B

Page 662: Microeconomics Study Guide

True/False Questions

Type: A E: 359 MI: 115 Status: New 173. A linear demand curve has a constant elasticity over the full range of the curve.Answer: False

Type: A E: 365 MI: 121 Status: New 174. The greater the ease of shifting resources from product X to Y in the production process, the greater is the elasticity of supply of product Y.Answer: True

Type: E E: 365 MI: 121 Status: New 175. If the elasticity coefficient of supply is 0.7, supply is elastic.Answer: False

Type: F E: 366 MI: 122 Status: New 176. Antiques tend to have highly inelastic supply curves.Answer: True

Type: A E: 362 MI: 118 177. The smaller the number of good substitutes for a product, the greater will be the price elasticity of demand for it. Answer: False

Type: A E: 363 MI: 119 178. If the demand for wheat is highly price inelastic, an extraordinarly large crop may reduce farm incomes. Answer: True

Type: A E: 362 MI: 118 179. Generally speaking, the demand for luxury goods is more price elastic than is the demand for necessities. Answer: True

Type: A E: 362 MI: 118 180. Generally speaking, the smaller the percentage of one's total budget devoted to a particular product, the more price elastic will be the demand for that product. Answer: False

Type: A E: 361 MI: 117 181. If price and total revenue are directly related, demand is inelastic. Answer: True

Type: A E: 361-362 MI: 117-118 182. If price changes and total revenue changes in the opposite direction, demand is relatively elastic. Answer: True

Use the following to answer questions 183-185:

Page 663: Microeconomics Study Guide

Answer the next question(s) on the basis of the following demand and supply data:

Type: A E: 48-49 MI: 48-49 183. Refer to the above data. The equilibrium price of this product is somewhere between $6 and $7. Answer: True

Type: T E: 359 MI: 115 184. Refer to the above data. The demand for this product is elastic in the $8-$7 price range. Answer: True

Type: T E: 365 MI: 121 185. Refer to the above data. The supply of this product is inelastic in the $6-$5 price range. Answer: False

Type: T E: 367 MI: 123 186. Cross elasticity of demand measures the effect of a change in the price of one product on the quantity demanded of another product. Answer: True

Type: D E: 368 MI: 124 187. Income elasticity measures the effect of a change in income on the purchases of some good or service. Answer: True

Type: D E: 368 MI: 124 188. If the coefficient of income elasticity of demand is positive, the product is an inferior good. Answer: False

Type: A E: 367 MI: 123 189. If the coefficient of cross elasticity of demand is positive, the two products are complementary goods. Answer: False

Type: A E: 368 MI: 124 190. An income elasticity coefficient of -1.8 means the product is a normal good. Answer: False

Type: A E: 367 MI: 123 191. A cross elasticity of demand coefficient of +2.5 indicates that the two products are substitutes. Answer: True

Type: A E: 367 MI: 123 192. We would expect the coefficient of cross elasticity of demand for DVD players and DVDs to be positive. Answer: False

Page 664: Microeconomics Study Guide

Summer 2003MBA 501-01Economics SurveyTong

Mock Exam (no credit)

Part I. Multiple Choice (One point each)Identify the letter of choice that best completes the statement or answers the question.

Use the following to answer question 1:

Answer the next question(s) on the basis of the following data. All figures are in billions of dollars.

1. The gross domestic product for the above economy is: A) $100. $95. C) $110. D) $107.

2. The total amount of income earned by U.S. resource suppliers in a year is measured by: A) gross domestic product. national income. C) personal income. D) disposable income.

3. The term "real GDP" refers to: A) the value of the domestic output after adjustments have been made for environmental pollution and changes in the distribution of income. GDP data that embody changes in the price level, but not changes in physical output. C) GDP data that reflect changes in both physical output and the price level. D) GDP data that have been adjusted for changes in the price level.

4. GDP differs from NDP in that: A) GDP is based on gross exports, while NDP is based on net exports. GDP includes, but NDP excludes, indirect business taxes. C) net investment is used in calculating GDP and gross investment is used in calculating NDP. D) gross investment is used in calculating GDP and net investment is used in calculating NDP.

5. A nation's stock of capital goods will decline when: A) gross investment exceeds net investment. net investment is positive, but less than gross investment. C) depreciation exceeds gross investment.

Page 665: Microeconomics Study Guide

D) gross investment exceeds depreciation.

6. Consumption of fixed capital (depreciation) can be determined by: A) adding indirect business taxes to NDP. subtracting NDP from GDP.C) subtracting net investment from GDP. D) adding net investment to gross investment.

7. If personal income exceedes national income in a particular year, we can conclude that: A) transfer payments exceeded the sum of social security contributions, corporate income taxes, and indirect business taxes. the sum of social security contributions, corporate income taxes, and undistributed corporate profits exceeded transfer payments. C) consumption of fixed capital and indirect business taxes exceeded personal taxes. D) transfer payments exceeded the sum of social security contributions, corporate income taxes, and undistributed corporate profits.

8. The consumption schedule is such that: A) both the APC and the MPC increase as income rises. the APC is constant and the MPC declines as income rises. C) the MPC is constant and the APC declines as income rises. D) the MPC and APC must be equal at all levels of income.

9. The 45-degree line on a chart relating consumption and income shows: A) all points where the MPC is constant. all points at which saving and income are equal. C) all the points at which consumption and income are equal. D) the amounts households will plan to save at each possible level of income.

Use the following to answer question 10:

Answer the next question(s) on the basis of the following data for a hypothetical economy.

10. Refer to the above data. The marginal propensity to consume is: A) .80. .75. C) .20. D) .25.

11. A specific investment will be undertaken if the expected rate of return, r, exceeds the interest rate, i. A) True b) False

12. The vertical distance between the horizontal axis and any point on a nondiscriminating monopolist's demand curve measures:

Page 666: Microeconomics Study Guide

A) the quantity demanded. product price and average revenue. C) total revenue. D) product price and marginal revenue.

Use the following to answer question 13:

Answer the next question(s) on the basis of the following demand and cost data for a pure monopolist:

13. Refer to the above data. Equilibrium price for the monopolist will be: A) $5.00 $2.90. C) $3.35. D) $4.50.

14. Natural monopoly may result where products produce substantial network effects and can be simultaneously consumed by a large number of consumers.A) True False

Use the following to answer question 15:

15. Refer to the above diagrams. If drawn, Firm A's average revenue curve would lie below its demand curve. A) True False

Use the following to answer question 16:

Answer the next question(s) on the basis of the following cost data for a purely competitive seller:

16. Refer to the above data. If product price is $60, the firm will: A) shut down. produce 4 units and realize a $120 economic profit. C) produce 6 units and realize a $100 economic profit. D) produce 3 units and incur a $40 loss.

17. Which of the following industries most closely approximates pure competition? A) agriculture farm implements C) clothing

Page 667: Microeconomics Study Guide

D) steel

18. A one-firm industry is known as: A) monopolistic competition oligopoly C) pure monopoly D) pure competition

19. A firm finds that at its MR = MC output, its TC = $1000, TVC = $800, TFC = $200, and total revenue is $900. This firm should: A) shut down in the short run. produce because the resulting loss is less than its TFC. C) produce because it will realize an economic profit. D) liquidate its assets and go out of business.

Use the following to answer question 20:

Answer the next question(s) on the basis of the following cost data for a firm that is selling in a purely competitive market:

20. Refer to the above data. If the market price for the firm's product is $12, the competitive firm will produce: A) 4 units at a loss of $109. 4 units at an economic profit of $31.75. C) 8 units at a loss of $48.80. D) zero units at a loss of $100.

Use the following to answer question 21:

Use the following cost information for the Creamy Crisp Donut Company to answer the next question(s):

Entrepreneur's potential earnings as a salaried worker = $50,000Annual lease on building = $22,000Annual revenue from operations = $380,000Payments to workers = $120,000Utilities (electricity, water, disposal) costs = $8,000Entrepreneur's potential economic profit from the next best entrepreneurial activity = $80,000Entrepreneur's forgone interest on personal funds used to finance the business = $6,000

21. Refer to the above data. Creamy Crisp's explicit costs are:A) $286,000.$150,000.C) $94,000.D) $156,000.

Page 668: Microeconomics Study Guide

Use the following to answer question 22:

22. The above diagram suggests that: A) when marginal product is zero, total product is at a minimum. when marginal product lies above average product, average product is rising. C) when marginal product lies below average product, average product is rising. D) when total product is at a maximum, so is marginal product and average product.

23. A demand curve which is parallel to the horizontal axis is:A) perfectly inelastic perfectly elastic C) relatively inelastic D) relatively elastic

Use the following to answer question 24:

In the following question(s) you are asked to determine, other things equal, the effects of a given change in a determinant of demand or supply for product X upon (1) the demand (D) for, or supply (S) of, X, (2) the equilibrium price (P) of X and (3) the equilibrium quantity (Q) of X.

24. Refer to the above. An improvement in the technology used to produce X will: A) decrease S, increase P, and decrease Q. B) decrease S, increase P, and increase Q. C) increase S, decrease P, and increase Q. D) decrease D, decrease P, and decrease Q.

Use the following to answer question 25:

25. Refer to the above diagram. Which of the following is a positive statement? A) A point inside the production possibilities curve is superior to a point on the curve because the former requires less work effort. B) Because any society should stress economic growth as its major goal, point D is superior to point C. C) Point B is preferable to point C because the ultimate goal of economic activity is to maximize consumption. D) Given its resources and technology, this society is incapable of simultaneously producing 3 units of tractors and 15 units of bread.

26. Allocative efficiency involves determining: A) which output-mix will result in the most rapid rate of economic growth. B) which production possibilities curve reflects the lowest opportunity costs. C) the point on the production possibilities curve that will maximize society's satisfaction. D) the optimal rate of technological progress.

Page 669: Microeconomics Study Guide

27. For most producing firms: A) marginal cost rises as output is carried to a certain level, and then begins to decline. B) total costs rise as output is carried to a certain level, and then begin to decline. C) average total costs decline as output is carried to a certain level, and then begin to rise. D) average total costs rise as output is carried to a certain level, and then begin to decline.

28. The lowest point on a purely competitive firm's short-run supply curve corresponds to: A) the minimum point on its ATC curve. B) the minimum point on its AVC curve. C) the minimum point on its AFC curve. D) the minimum point on its MC curve.

Use the following to answer question 29:

29. Refer to the above diagram. The quantity difference between areas A and C for the indicated price reduction measures: A) marginal cost. B) marginal revenue. C) monopoly price. D) a welfare or efficiency loss.

30.

The consumption schedule in the above diagram indicates that: A) consumers will maximize their satisfaction where the consumption schedule and 45° line intersect. B) up to a point consumption exceeds income, but then falls below income. C) the MPC falls as income increases. D) households consume as much as they earn.

Part II. Short Questions

1.

The following table shows nominal GDP and an appropriate price index for a group of selected years.

Nominal GDP, Price IndexYear Billions (1996 = 100)

1960 $527.40 22.191968 911.5 26.29

Page 670: Microeconomics Study Guide

1978 2295.9 48.221988 4742.5 80.221998 8790.2 103.22

a) Compute real GDP for the years 1978 and 1998. b) What is the growth rate of real GDP between 1988 and 1998? c) What is the inflation rate between 1968 and 1978?

2.

Do you agree or disagree with the statement that: “A monopolist always charges the highest possible price.” Explain.

3. Complete the accompanying table.

Level of outputand income(GDP = DI) Consumption Saving APC APS MPC MPS$480 $___ $–8 ____ ____ ___ ___520 ___ 0 ____ ____ ___ ___560 ___ 8 ____ ____ ___ ___600 ___ 16 ____ ____ ___ ___640 ___ 24 ____ ____ ___ ___680 ___ 32 ____ ____ ___ ___720 ___ 40 ____ ____ ___ ___760 ___ 48 ____ ____ ___ ___800 ___ 56 ____ ____ ___ ___

(a) Using the below graphs, show the consumption and saving schedules graphically.

(b) Locate the break-even level of income. How is it possible for households to dissave at very low income levels?

***** The End *****

Answer Key -- Mock Summer 2003

1. C 2. B 3. D 4. D 5. C 6. B 7. D 8. C

Page 671: Microeconomics Study Guide

9. C 10. A 11. True 12. B 13. D 14. True 15. False 16. C 17. A 18. C 19. B 20. D 21. B 22. B 23. B 24. C 25. D 26. C 27. C 28. B 29. B 30. B

Price elasticity of demand

Type: D Topic: 1 E: 356 MI: 112 1. The price elasticity of demand coefficient measures: A) buyer responsiveness to price changes. the extent to which a demand curve shifts as incomes change. C) the slope of the demand curve. D) how far business executives can stretch their fixed costs. Answer: A

Type: E Topic: 1 E: 356 MI: 112 2. The basic formula for the price elasticity of demand coefficient is: A) absolute decline in quantity demanded/absolute increase in price. percentage change in quantity demanded/percentage change in price. C) absolute decline in price/absolute increase in quantity demanded. D) percentage change in price/percentage change in quantity demanded. Answer: B

Type: D Topic: 1 E: 356 MI: 112 3. The demand for a product is inelastic with respect to price if: A) consumers are largely unresponsive to a per unit price change. the elasticity coefficient is greater than 1.

Page 672: Microeconomics Study Guide

C) a drop in price is accompanied by a decrease in the quantity demanded. D) a drop in price is accompanied by an increase in the quantity demanded. Answer: A

Type: A Topic: 1 E: 357 MI: 113 4. If the price elasticity of demand for a product is 2.5, then a price cut from $2.00 to $1.80 will: A) increase the quantity demanded by about 2.5 percent. decrease the quantity demanded by about 2.5 percent. C) increase the quantity demanded by about 25 percent. D) increase the quantity demanded by about 250 percent. Answer: C

Type: A Topic: 1 E: 357 MI: 113 5. Suppose that as the price of Y falls from $2.00 to $1.90 the quantity of Y demanded increases from 110 to 118. Then the price elasticity of demand is: A) 4.00. 2.09. C) 1.37. D) 3.94. Answer: C

Type: D Topic: 1 E: 357 MI: 113 6. Which of the following is not characteristic of the demand for a commodity that is elastic? A) The relative change in quantity demanded is greater than the relative change in price. Buyers are relatively sensitive to price changes. C) Total revenue declines if price is increased. D) The elasticity coefficient is less than one. Answer: D

Type: A Topic: 1 E: 357 MI: 113 7. If the demand for product X is inelastic, a 4 percent increase in the price of X will: A) decrease the quantity of X demanded by more than 4 percent. decrease the quantity of X demanded by less than 4 percent. C) increase the quantity of X demanded by more than 4 percent. D) increase the quantity of X demanded by less than 4 percent. Answer: B

Type: A Topic: 1 E: 357 MI: 113 8. If a firm can sell 3,000 units of product A at $10 per unit and 5,000 at $8, then: A) the price elasticity of demand is 0.44. C) the price elasticity of demand is 2.25. A is a complementary good. D) A is an inferior good. Answer: C

Type: A Topic: 1 E: 357-358 MI: 113-114 9. A perfectly inelastic demand schedule: A) rises upward and to the right, but has a constant slope. can be represented by a line parallel to the vertical axis. C) cannot be shown on a two-dimensional graph. D) can be represented by a line parallel to the horizontal axis.

Page 673: Microeconomics Study Guide

Answer: B

Type: C Topic: 1 E: 357 MI: 113 10. The larger the coefficient of price elasticity of demand for a product, the: A) larger the resulting price change for an increase in supply. more rapid the rate at which the marginal utility of that product diminishes. C) less competitive will be the industry supplying that product. D) smaller the resulting price change for an increase in supply. Answer: D

Type: A Topic: 1 E: 359 MI: 115 11. Most demand curves are relatively elastic in the upper-left portion because the original price: A) and quantity from which the percentage changes in price and quantity are calculated are both large. and quantity from which the percentage changes in price and quantity are calculated are both small. C) from which the percentage price change is calculated is small and the original quantity from which the percentage change in quantity is calculated is large. D) from which the percentage price change is calculated is large and the original quantity from which the percentage change in quantity is calculated is small. Answer: D

Type: A Topic: 1 E: 357 MI: 113 12. The price elasticity of demand for widgets is 0.80. Assuming no change in the demand curve for widgets, a 16 percent increase in sales implies a: A) 1 percent reduction in price. C) 40 percent reduction in price. 12 percent reduction in price. D) 20 percent reduction in price. Answer: D

Type: A Topic: 1 E: 359 MI: 115 13. Suppose Aiyanna's pizzeria currently faces a linear demand curve and is charging a very high price per pizza and doing very little business. Aiyanna now decides to lower pizza prices by 5 percent per week for an indefinite period of time. We can expect that each successive week: A) demand will become more price elastic. price elasticity of demand will not change as price is lowered. C) demand will become less price elastic. D) the elasticity of supply will increase. Answer: C

Type: A Topic: 1 E: 358 MI: 114 14. The price elasticity of demand of a straight-line demand curve is: A) elastic in high-price ranges and inelastic on low-price ranges. elastic, but does not change at various points on the curve. C) inelastic, but does not change at various points on the curve. D) 1 at all points on the curve. Answer: A

Page 674: Microeconomics Study Guide

Type: C Topic: 1 E: 359 MI: 115 15. A leftward shift in the supply curve of product X will increase equilibrium price to a greater extent the: A) more elastic the supply curve. C) more elastic the demand for the product. larger the elasticity of demand coefficient. D) more inelastic the demand for the product. Answer: D

Type: A Topic: 1 E: 359 MI: 115 16. If the demand for bacon is relatively elastic, a 10 percent decline in the price of bacon will: A) decrease the amount demanded by more than 10 percent. increase the amount demanded by more than 10 percent. C) decrease the amount demanded by less than 10 percent. D) increase the amount demanded by less than 10 percent. Answer: B

Type: A Topic: 1 E: 357 MI: 113 17. The price elasticity of demand is: A) negative, but the minus sign is ignored. positive, but the plus sign is ignored. C) positive for normal goods and negative for inferior goods. D) positive because price and quantity demanded are inversely related. Answer: A

Type: A Topic: 1 E: 359 MI: 115 18. For a linear demand curve: A) elasticity is constant along the curve. C) demand is elastic at low prices. elasticity is unity at every point on the curve. D) demand is elastic at high prices. Answer: D

Type: A Topic: 1 E: 359 MI: 115 19. The price of product X is reduced from $100 to $90 and, as a result, the quantity demanded increases from 50 to 60 units. Therefore demand for X in this price range: A) has declined. is of unit elasticity. C) is inelastic. D) is elastic. Answer: D

Use the following to answer question 20:

Type: G Topic: 1 E: 359 MI: 115 20. The above diagram shows two product demand curves. On the basis of this diagram we can say that: A) over range P1P2 price elasticity of demand is greater for D1 than for D2. over range P1P2 price elasticity of demand is greater for D2 than for D1. C) over range P1P2 price elasticity is the same for the two demand curves. D) not enough information is given to compare price elasticities. Answer: A

Page 675: Microeconomics Study Guide

Type: A Topic: 1 E: 359 MI: 115 21. Suppose we find that the price elasticity of demand for a product is 3.5 when its price is increased by 2 percent. We can conclude that quantity demanded: A) increased by 7 percent. C) decreased by 9 percent. decreased by 7 percent. D) decreased by 12 percent. Answer: B

Type: A Topic: 1 E: 359 MI: 115 22. The price elasticity of demand for beef is about 0.60. Other things equal, this means that a 20 percent increase in the price of beef will cause the quantity of beef demanded to: A) increase by approximately 12 percent. C) decrease by approximately 32 percent. decrease by approximately 12 percent. D) decrease by approximately 26 percent. Answer: B

Type: A Topic: 1 E: 359 MI: 115 23. The elasticity of demand: A) is infinitely large for a perfectly inelastic demand curve. tends to be inelastic in high-price ranges and elastic in low-price ranges. C) tends to be elastic in high-price ranges and inelastic in low-price ranges. D) is the same at each price-quantity combination on a stable demand curve. Answer: C

Type: D Topic: 1 E: 357 MI: 113 24. If a demand for a product is elastic, the value of the price elasticity coefficient is: A) zero. greater than one. C) equal to one. D) less than one. Answer: B

Type: D Topic: 1 E: 356 MI: 112 25. The concept of price elasticity of demand measures: A) the slope of the demand curve. the number of buyers in a market. C) the extent to which the demand curve shifts as the result of a price decline. D) the sensitivity of consumer purchases to price changes. Answer: D

Use the following to answer questions 26-28:

Type: G Topic: 1 E: 359 MI: 115 26. Refer to the above diagram. Between prices of $5.70 and $6.30:A) D1 is more elastic than D2. C) D1 and D2 have identical elasticities.D2 is an inferior good and D1 is a normal good. D) D2 is more elastic than D1Answer: A

Page 676: Microeconomics Study Guide

Type: G Topic: 1 E: 359 MI: 115 27. Refer to the above diagram and assume a single good. If the price of the good decreases from $6.30 to $5.70, consumer spending would:A) decrease if demand were D1 only. C) decrease if demand were either D1 or D2.decrease if demand were D2 only. D) increase if demand were either D1 or D2.Answer: B

Type: G Topic: 1 E: 359 MI: 115 28. Refer to the above diagram and assume a single good. If the price of the good increased from $5.70 to $6.30 along D1, the price elasticity of demand along this portion of the demand curve would be:A) 0.8. 1.0. C) 1.2. D) 2.0.Answer: C

Type: A Topic: 1 E: 359 MI: 115 29. Suppose the price of local cable TV service increased from $16.20 to $19.80 and as a result the number of cable subscribers decreased from 224,000 to 176,000. Along this portion of the demand curve, price elasticity of demand is:A) 0.8. 1.2. C) 1.6. D) 8.0Answer: B

Type: A Topic: 1 E: 359 MI: 115 30. If the price of hand calculators falls from $10 to $9 and, as a result, the quantity demanded increases from 100 to 125, then: A) demand is elastic. demand is inelastic. C) demand is of unit elasticity. D) not enough information is given to make a statement about elasticity. Answer: A

Type: A Topic: 1 E: 358 MI: 114 31. A perfectly inelastic demand curve: A) has a price elasticity coefficient greater than unity. has a price elasticity coefficient of unity throughout. C) graphs as a line parallel to the vertical axis. D) graphs as a line parallel to the horizontal axis. Answer: C

Type: A Topic: 1 E: 359 MI: 115 32. Moving upward on a downward-sloping straight-line demand curve, we find that price elasticity: A) is constant. C) decreases continuously. increases continuously. D) may either increase or decrease. Answer: B

Use the following to answer questions 33-37:

Answer the next question(s) on the basis of the following demand schedule:

Page 677: Microeconomics Study Guide

Type: T Topic: 1 E: 359 MI: 115 33. Refer to the above data. If this demand schedule were graphed, we would find that: A) its slope diminishes as we move southeast down the curve. its slope diminishes as we move northwest up the curve. C) its slope is constant throughout. D) the data is inconsistent with the law of demand. Answer: C

Type: T Topic: 1 E: 359 MI: 115 34. Refer to the above data. The price elasticity of demand is relatively elastic: A) in the $6-$4 price range. C) in the $3-$1 price range. over the entire $6-$1 price range. D) in the $6-$5 price range only. Answer: A

Type: T Topic: 1 E: 359 MI: 115 35. Refer to the above data. The price elasticity of demand is relatively inelastic: A) in the $6-$4 price range. C) in the $3-$1 price range. over the entire $6-$1 price range. D) in the $6-$5 price range only. Answer: C

Type: T Topic: 1 E: 359 MI: 115 36. Refer to the above data. The price elasticity of demand is unity: A) throughout the entire price range because the slope of the demand curve is constant. in the $4-$3 price range only. C) over the entire $3-$1 price range. D) over the entire $6-$4 price range. Answer: B

Type: T Topic: 1 E: 359 MI: 115 37. Refer to the above data. Which of the following is correct? A) Although the slope of the demand curve is constant, price elasticity declines as we move from high to low price ranges. Although the slope of the demand curve is constant, price elasticity increases as we move from high to low price ranges. C) Although the demand curve is concave to the origin, price elasticity of demand is constant throughout. D) A steep slope means demand is inelastic; a flat slope means demand is elastic. Answer: A

Type: A Topic: 1 E: 359 MI: 115 38. If the price elasticity of demand for gasoline is 0.20:A) the demand for gasoline is linear. a rise in the price of gasoline will reduce total revenue.C) a 10 percent rise in the price of gasoline will decrease the amount purchased by 2 percent.

Page 678: Microeconomics Study Guide

D) a 10 percent fall in the price of gasoline will increase the amount purchased by 20 percent. Answer: C

Type: T Topic: 1 E: 359 MI: 115 39. In which price range of the accompanying demand schedule is demand elastic?

A) $4-$3 $3-$2 C) $2-$1 D) below $1 Answer: A

Total revenue test

Type: A Topic: 2 E: 361 MI: 117 40. When the percentage change in price is greater than the resulting percentage change in quantity demanded: A) a decrease in price will increase total revenue. C) an increase in price will increase total revenue. demand may be either elastic or inelastic. D) demand is elastic. Answer: C

Type: A Topic: 2 E: 360-361 MI: 116-117 41. Suppose the price elasticity coefficients of demand are 1.43, 0.67, 1.11, and 0.29 for products W, X, Y, and Z respectively. A 1 percent decrease in price will increase total revenue in the case(s) of: A) W and Y. B) Y and Z. C) X and Z. D) Z and W. Answer: A

Use the following to answer questions 42-44:

Type: G Topic: 2 E: 361 MI: 117 42. Suppose that the above total revenue curve is derived from a particular linear demand curve. That demand curve must be:A) inelastic for price declines that increase quantity demanded from 6 units to 7 units.B) elastic for price declines that increase quantity demanded from 6 units to 7 units.C) inelastic for price increases that reduce quantity demanded from 4 units to 3 units.D) elastic for price increases that reduce quantity demanded from 8 units to 7 units.Answer: A

Type: G Topic: 2 E: 360 MI: 116 43. Suppose that the above total revenue curve is derived from a particular linear demand curve. That demand curve must be:A) inelastic for price declines that increase quantity demanded from 2 units to 3 units.B) elastic for price declines that increase quantity demanded from 5 units to 6 units.C) inelastic for price increases that reduce quantity demanded from 4 units to 3 units.D) elastic for price increases that reduce quantity demanded from 4 units to 3 units.Answer: D

Page 679: Microeconomics Study Guide

Type: G Topic: 2 E: 361 MI: 117 44. Suppose that the above total revenue curve is derived from a particular linear demand curve. That demand curve must be:A) inelastic for price declines that increase quantity demanded from 2 units to 3 units.B) elastic for price declines that increase quantity demanded from 5 units to 6 units.C) unit elastic for price increases that reduce quantity demanded from 5 units to 4 units..D) inelastic for price increases that reduce quantity demanded from 4 units to 3 units.Answer: C

Type: A Topic: 2 E: 360 MI: 116 45. Which of the following statements is not correct? A) If the relative change in price is greater than the relative change in the quantity demanded associated with it, demand is inelastic. B) In the range of prices in which demand is elastic, total revenue will diminish as price decreases. C) Total revenue will not change if price varies within a range where the elasticity coefficient is unity. D) Demand tends to be elastic at high prices and inelastic at low prices. Answer: B

Type: A Topic: 2 E: 360 MI: 116 46. In which of the following instances will total revenue decline? A) price rises and supply is elastic C) price rises and demand is inelastic B) price falls and demand is elastic D) price rises and demand is elastic Answer: D

Type: A Topic: 2 E: 360-361 MI: 116-117 47. If a firm's demand for labor is elastic, a union-negotiated wage increase will: A) necessarily be inflationary. C) cause the firm's total payroll to decline. B) cause the firm's total payroll to increase. D) cause a shortage of labor. Answer: C

Type: A Topic: 2 E: 360-361 MI: 116-117 48. The Illinois Central Railroad once asked the Illinois Commerce Commission for permission to increase its commuter rates by 20 percent. The railroad argued that declining revenues made this rate increase essential. Opponents of the rate increase contended that the railroad's revenues would fall because of the rate hike. It can be concluded that: A) both groups felt that the demand was elastic but for different reasons. B) both groups felt that the demand was inelastic but for different reasons. C) the railroad felt that the demand for passenger service was inelastic and opponents of the rate increase felt it was elastic. D) the railroad felt that the demand for passenger service was elastic and opponents of the rate increase felt it was inelastic. Answer: C

Type: A Topic: 2 E: 360 MI: 116

Page 680: Microeconomics Study Guide

49. If a firm finds that it can sell $13,000 of a product when its price is $5 per unit and $11,000 of it when its price is $6, then: A) the demand for the product is elastic in the $6-$5 price range. B) the demand for the product must have increased. C) elasticity of demand is 0.74. D) the demand for the product is inelastic in the $6-$5 price range. Answer: A

Type: C Topic: 2 E: 361 MI: 117 50. Suppose the price elasticity of demand for bread is 0.20. If the price of bread falls by 10 percent, the quantity demanded will increase by: A) 2 percent and total expenditures on bread will rise. B) 2 percent and total expenditures on bread will fall. C) 20 percent and total expenditures on bread will fall. D) 20 percent and total expenditures on bread will rise. Answer: B

Use the following to answer questions 51-52:

Type: G Topic: 2 E: 361 MI: 117 51. Refer to the above diagram which is a rectangular hyperbola, that is, a curve such that each rectangle drawn from any point on the curve will be of identical area. If this rectangular hyperbola was a demand curve, we could say that it would be: A) elastic at high prices and inelastic at low prices. C) impossible to generalize about its elasticity. B) elastic at low prices and inelastic at high prices. D) of unit elasticity throughout. Answer: D

Type: G Topic: 2 E: 361 MI: 117 52. Refer to the above diagram which is a rectangular hyperbola, that is, a curve such that each rectangle drawn from any point on the curve will be of identical area. In comparing the price elasticity and the slope of this demand curve we can conclude that the: A) slope of a demand curve measures its elasticity. B) elasticity of a demand curve measures its slope. C) slope and elasticity of the curve are both constant throughout. D) slope of the curve varies, but its elasticity is constant. Answer: D

Type: A Topic: 2 E: 361 MI: 117 53. Gigantic State University raises tuition for the purpose of increasing its revenue so that more faculty can be hired. GSU is assuming that the demand for education at GSU is: A) decreasing. B) relatively elastic. C) perfectly elastic. D) relatively inelastic. Answer: D

Page 681: Microeconomics Study Guide

Type: A Topic: 2 E: 361 MI: 117 54. If the demand for farm products is price inelastic, a good harvest will cause farm revenues to: A) increase. B) decrease. C) be unchanged. D) either increase or decrease, depending on what happens to supply. Answer: B

Type: A Topic: 2 E: 360 MI: 116 55. Other things the same, if a price change causes total revenue to change in the opposite direction, demand is: A) perfectly inelastic. B) relatively elastic. C) relatively inelastic. D) of unit elasticity. Answer: B

Type: A Topic: 2 E: 361 MI: 117 56. If the price elasticity of demand for a product is unity, a decrease in price will: A) have no effect upon the amount purchased. B) increase the quantity demanded and increase total revenue. C) increase the quantity demanded, but decrease total revenue. D) increase the quantity demanded, but total revenue will be unchanged. Answer: D

Type: A Topic: 2 E: 361 MI: 117 57. If a price reduction reduces a firm's total revenue:A) the demand for the product is inelastic in this price range. B) the product is an inferior good. C) in this price range the elasticity coefficient of demand is greater than 1. D) this price decline will increase the firm's profits. Answer: A

Type: A Topic: 2 E: 361 MI: 117 58. In which of the following cases will total revenue increase? A) price falls and demand is inelastic C) price rises and demand is inelastic B) price falls and supply is elastic D) price rises and demand is elastic Answer: C

Type: A Topic: 2 E: 360-361 MI: 116-117 59. A manufacturer of frozen pizzas found that total revenue decreased when price was lowered from $5 to $4. It was also found that total revenue decreased when price was raised from $5 to $6. Thus,A) the demand for pizza is elastic above $5 and inelastic below $5. B) the demand for pizza is elastic both above and below $5. C) the demand for pizza is inelastic above $5 and elastic below $5. D) $5 is not the equilibrium price of pizza. Answer: A

Use the following to answer questions 60-61:

Page 682: Microeconomics Study Guide

Type: G Topic: 2 E: 360 MI: 116 60. Refer to the above diagram. In the P1P2 price range demand is: A) of unit elasticity. B) relatively inelastic. C) relatively elastic. D) perfectly elastic. Answer: C

Type: G Topic: 2 E: 360 MI: 116 61. Refer to the above diagram. In the P3P4 price range demand is: A) of unit elasticity. B) relatively inelastic. C) relatively elastic. D) perfectly elastic. Answer: B

Type: A Topic: 2 E: 360 MI: 116 62. If the demand for a product is elastic, then total revenue will: A) increase whether price increases or decreases. C) fall as price falls. B) be constant in response to a price change. D) rise as price falls. Answer: D

Type: A Topic: 2 E: 361 MI: 117 63. The total-revenue test for elasticity: A) is equally applicable to both demand and supply. B) does not apply to demand because price and quantity are inversely related. C) does not apply to supply because price and quantity are directly related. D) applies to the short-run supply curve, but not to the long-run supply curve. Answer: C

Type: A Topic: 2 E: 361 MI: 117 64. If the University Chamber Music Society decides to raise ticket prices to provide more funds to finance concerts, the Society is assuming that the demand for tickets is: A) parallel to the horizontal axis. B) shifting to the left. C) inelastic. D) elastic. Answer: C

Type: A Topic: 2 E: 361 MI: 117 65. The state legislature has cut Gigantic State University's appropriations. GSU's Board of Regents decides to increase tuition and fees to compensate for the loss of revenue. The board is assuming that the: A) demand for education at GSU is elastic. B) demand for education at GSU is inelastic. C) coefficient of price elasticity of demand for education at GSU is unity. D) coefficient of price elasticity of demand for education at GSU is greater than unity. Answer: B

Type: A Topic: 2 E: 360 MI: 116 66. Which of the following is correct? A) If demand is elastic, an increase in price will increase total revenue. B) If demand is elastic, a decrease in price will decrease total revenue.

Page 683: Microeconomics Study Guide

C) If demand is elastic, a decrease in price will increase total revenue. D) If demand is inelastic, an increase in price will decrease total revenue. Answer: C

Type: A Topic: 2 E: 361 MI: 117 67. Suppose that the price of peanuts falls from $3 to $2 per bushel and that, as a result, the total revenue received by peanut farmers changes from $16 to $14 billion. Thus:A) the demand for peanuts is elastic. B) the demand for peanuts is inelastic. C) the demand curve for peanuts has shifted to the right. D) no inference can be made as to the elasticity of demand for peanuts. Answer: B

Type: A Topic: 2 E: 361 MI: 117 68. Which of the following is correct? A) If the demand for a product is inelastic, a change in price will cause total revenue to change in the opposite direction. B) If the demand for a product is inelastic, a change in price will cause total revenue to change in the same direction. C) If the demand for a product is inelastic, a change in price may cause total revenue to change in either the opposite or the same direction. D) The price elasticity coefficient applies to demand, but not to supply. Answer: B

Cash expenditures a firm makes to pay for resources are called:

Explicit costs

Which would be an implicit cost for a firm? The cost:

Of wages foregone by the owner of the firm

If a firm's revenues just cover all its opportunity costs, then:

Economic profit is zero

An industry is expected to expand if firms in the industry are earning positive:

Economic profits

In the short run:

Page 684: Microeconomics Study Guide

Output is raised or reduced by changing the levels of variable inputs

Which is most likely to be a long-run adjustment for a firm that manufactures cars on an assembly line basis?

A change in production to a redesigned and retooled facility

According to the law of diminishing marginal returns:

The additional output generated by additional units of an input will diminish

Diminishing marginal returns occurs as a firm adds more variable inputs to at least one fixed input because:

As more variable inputs are hired, the amount of the fixed input per variable input decreases

The law of diminishing returns in a manufacturing plant of a fixed capacity implies that, eventually, employing one:

More worker will decrease the average amount of output per worker

The marginal product of labor curve graphically shows the change in total product resulting from a:

One-unit increase in the quantity of a particular resource used, holding constant other resources

When a bakery manager reports that at her bakery, productivity of her 15 workers last month was 1,800 loaves per worker, she is referring to the:

Average product of labor

The range of diminishing marginal productivity begins when:

Marginal product reaches its maximum

At the Amarillo Piano Company, the average product of labor stays constant at 5, regardless of how much labor is employed. This implies that:

The marginal product of labor is constant

At what point does marginal product equal average product?

Where average product is equal to its maximum value

With fixed costs of $400, a firm has average total costs of $3 and average variable costs of $2.50. Its output quantity must be:

800 units

Page 685: Microeconomics Study Guide

At an output of 1,000 units per year, a firm's variable costs are $5,000 and its average fixed costs are $3. Its total costs per year are:

$8,000

The range over which average variable cost is increasing is the same as the range over which:

Average product is decreasing

If the long-run average total cost curve for a firm is horizontal in the relevant range of production, then it indicates that there:

Are constant returns to scale

Normal profit is an implicit cost.

True

In the long run, a firm can increase its output quantity, but it will be limited by the size of its existing production plant.

False

When diminishing marginal returns starts occurring, the addition of successive units of a variable resource to a fixed resource will cause the firm's production to diminish.

False

Marginal product is highest where marginal cost is lowest.

True

Mutual interdependence would tend to limit control over price in which market model?

Oligopoly

Under which market model are the conditions of entry into the market easiest?

Pure competition

Local electric or gas utility companies mostly operate in which market model?

Pure monopoly

The steel and automobile industries would be examples of which market model?

Oligopoly

Which idea is inconsistent with pure competition?

Page 686: Microeconomics Study Guide

Product differentiation

Which characteristic would best be associated with pure competition?

Price takers

In a purely competitive industry, each firm:

Can easily enter or exit the industry

Which is true under conditions of pure competition?

No single firm can influence the market price by changing its output

Price is constant or "given" to the individual firm selling in a purely competitive market because:

Each seller supplies a negligible fraction of total demand and supply

A purely competitive firm does not try to sell more of its product by lowering its price below the market price because:

It can sell all it wants to at the market price

The demand curve faced by a purely competitive firm:

Is the same as its marginal revenue curve

Sam owns a firm that produces tomatoes in a purely competitive market. The firm's demand curve is:

A horizontal line

In pure competition, each extra unit of output that a firm sells will yield a marginal revenue that is:

Equal to the price

In a graph for a firm in pure competition with the quantity of output measured on the horizontal axis, the total revenue curve is:

Upward-sloping

Which is necessarily true for a purely competitive firm in short-run equilibrium?

Marginal revenue less marginal cost equals zero

A firm sells a product in a purely competitive market. The marginal cost of the product at the current output of 1,000 units is $2.50. The minimum possible average variable cost is $2.00.

Page 687: Microeconomics Study Guide

The market price of the product is $2.50. To maximize profit or minimize losses, the firm should:

Continue producing 1,000 units

T-Shirt Enterprises is selling in a purely competitive market. It is producing 3000 units, selling them for $2.00 each. At this level of output, the average total cost is 2.50 and the average variable cost is $2.20. Based on these data, the firm should:

Shut down in the short run

In pure competition, price is determined where the industry:

Demand and supply curves intersect

If there are many firms in an industry, then it must be a purely competitive market.

False

In short-run equilibrium, a competitive firm cannot earn economic profits.

False

Which of the following distinguishes the short run from the long run in pure competition?

Firms can enter and exit the market in the long run, but not in the short run.

In a purely competitive industry:

there may be economic profits in the short run, but not in the long run.

Suppose a firm in a purely competitive market discovers that the price of its product is above its minimum AVC point but everywhere below ATC. Given this, the firm:

should continue producing in the short run, but leave the industry in the long run if the situation persists.

We would expect an industry to expand if firms in that industry are:

earning economic profits.

Which of the following will not hold true for a competitive firm in long-run equilibrium?

P equals AFC

Assume a purely competitive increasing-cost industry is initially in long-run equilibrium and that an increase in consumer demand occurs. After all economic adjustments have been completed product price will be:

higher and total output will be larger than originally.

Page 688: Microeconomics Study Guide

Assume a purely competitive, increasing-cost industry is in long-run equilibrium. If a decline in demand occurs, firms will:

leave the industry and price and output will both decline.

An increasing-cost industry is the result of:

higher resource prices which occur as the industry expands.

Under what conditions would an increase in demand lead to a lower long-run equilibrium price?

The firms in the market are part of a decreasing-cost industry.

In a decreasing-cost industry:

lower demand leads to higher long-run equilibrium prices.

When LCD televisions first came on the market, they sold for at least $1,000, and some for much more. Now many units can be purchased for under $400. These facts imply that:

the LCD television industry is a decreasing-cost industry.

Purely competitive industry X has constant costs and its product is an inferior good. The industry is currently in long-run equilibrium. The economy now goes into a recession and average incomes decline. The result will be:

an increase in output, but not in the price, of the product.

Suppose losses cause industry X to contract and, as a result, the prices of relevant inputs decline. Industry X is:

an increasing-cost industry.

Under pure competition in the long run:

both allocative efficiency and productive efficiency are achieved.

If for a firm P = minimum ATC = MC, then:

both allocative efficiency and productive efficiency are being achieved.

If a purely competitive firm is producing where price exceeds marginal cost, then:

the firm will fail to maximize profit and resources will be underallocated to the product.

Allocative efficiency occurs whenever:

Page 689: Microeconomics Study Guide

it is impossible to produce a net benefit for society by changing the combination of goods and services produced.

Entrepreneurs in purely competitive industries:

innovate to lower operating costs and generate short-run economic profits.

Innovations that lower production costs or create new products:

often generate short-run economic profits that do not last into the long run.

The theory of creative destruction was advanced many years ago by:

Joseph Schumpeter.

Creative destruction is least beneficial to:

workers in the "destroyed" industries.

Which of the following is an example of creative destruction?

Automobile production causes the wagon industry to shut down.

(Consider This) The average life expectancy of a U.S. business is approximately:

10.2 years.

(Consider This) Approximately what percentage of start-up firms in the U.S. go bankrupt within the first two years?

22

(Consider This) Which of the following statements is true about U.S. firms?

Over half are bankrupt within the first five years after starting up.

Barriers to entering an industry:

are the basis for monopoly.

What do economies of scale, the ownership of essential raw materials, and patents have in common?

They are all barriers to entry.

With respect to the pure monopolist's demand curve it can be said that:

price exceeds marginal revenue at all outputs greater than 1.

The demand curve faced by a pure monopolist:

Page 690: Microeconomics Study Guide

is less elastic than that faced by a single purely competitive firm.

The marginal revenue curve for a monopolist:

becomes negative when output increases beyond some particular level.

Which of the following is characteristic of a pure monopolist's demand curve?

It is the same as the market demand curve.

Because the monopolist's demand curve is downsloping:

price must be lowered to sell more output.

The pure monopolist's demand curve is relatively elastic:

in the price range where marginal revenue is positive.

For a pure monopolist marginal revenue is less than price because:

when a monopolist lowers price to sell more output, the lower price applies to all units sold.

If a pure monopolist is operating in a range of output where demand is elastic:

marginal revenue will be positive but declining.

Which of the following is incorrect? Imperfectly competitive producers:

do not compete with one another.

A pure monopolist:

will realize an economic profit if price exceeds ATC at the profit-maximizing/loss-minimizing level of output.

A pure monopolist's short-run profit-maximizing or loss-minimizing position is such that price:

may be greater or less than ATC.

The supply curve of a pure monopolist:

does not exist because prices are not "given" to a monopolist.

An important economic problem associated with pure monopoly is that, at the profit maximizing outputs, resources are:

underallocated because price exceeds marginal cost.

Page 691: Microeconomics Study Guide

Comparing a pure monopoly and a purely competitive firm with identical costs, we would find in long-run equilibrium that the pure monopolist's:

price and average total cost would be higher, but output would be lower.

There is some evidence to suggest that X-inefficiency is:

more likely to occur in monopolistic firms than in competitive firms.

Price discrimination refers to:

the selling of a given product at different prices that do not reflect cost differences.

Which of the following is not a precondition for price discrimination?

The commodity involved must be a durable good.

If a monopolist engages in price discrimination, it will:

charge a higher price where individual demand is inelastic and a lower price where individual demand is elastic.

Price discrimination is:

only illegal if used to lessen or eliminate competition.

A dilemma of regulation is that:

the regulated price that achieves allocative efficiency is also likely to result in losses.

(Consider This) Children are charged less than adults for admission to professional baseball games but are charged the same prices as adults at the concession stands. This pricing system occurs because:

(Consider This) Children are charged less than adults for admission to professional baseball games but are charged the same prices as adults at the concession stands. This pricing system occurs because:

(Last Word) DeBeers Consolidated Mines markets about:

55 percent of the world's rough-cut diamonds.

A pure monopolist will maximize profits by producing at that output where price and marginal cost are equal.

False

All of the following can file antitrust charges under the Sherman Act except:

the Federal Energy Regulatory Commission.

Page 692: Microeconomics Study Guide

Which of the following is least likely to violate the Sherman Act or the Clayton Act?

Competitive firms F and G independently charge lower prices to frequent customers than to occasional customers.

A function of the Federal Trade Commission is to:

investigate instances of faulty and misleading advertising.

Which of the following laws prohibited mergers by stock acquisition if the effect was to lessen competition?

Clayton Act of 1914

Tying agreements:

obligate a purchaser of product X to also buy product Y from the same seller.

The Celler-Kefauver Act of 1950:

amended the Clayton Act.

Price fixing:

is a per se violation of the antitrust laws.

The Sherman Act:

declared monopoly and restraints of trade to be illegal.

In which of the following cases did the final court decision result in a breakup of the firm into competing businesses?

Standard Oil case

In the Microsoft antitrust case, the Federal government said in essence that:

Microsoft was a "bad monopoly."

In the Alcoa case of 1945 the courts held that:

the mere possession of monopoly power is a violation of the antitrust laws.

In the U.S. Steel case of 1920 the courts held that:

although U.S. Steel possessed monopoly power, it had not violated the Sherman Act because it had not unreasonably used that power.

Page 693: Microeconomics Study Guide

Which one of the following is most likely to increase the Herfindahl index of a particular industry?

a horizontal merger

A merger between a maker of household detergents and a fast food chain would be an example of:

a conglomerate merger.

A conglomerate merger:

can extend the line of products sold, extend the territories in which products are sold, or combine totally unrelated products.

Conspiracies to fix prices are:

per se violations of the antitrust laws.

Critics of the regulation of natural monopolies contend that:

the industry may "capture" or control the regulatory commission.

The main purpose of industrial regulation is to:

lower price to average total cost such that the firm earns a fair return.

Critics of industrial regulation say that such regulation:

perpetuates monopoly long after new technology has eroded natural monopoly.

The largest efficiency gains from deregulation have occurred in the:

airlines, trucking, and railroad industries.

Social regulation differs from industrial regulation in that:

social regulation applies to virtually all industries, while industrial regulation applies to a restricted number.

Which one of the following is concerned with social regulation?

Equal Employment Opportunity Commission

(Consider This) The Consider This box "Of Catfish and Art (and Other Things in Common)" lists examples of recent antitrust cases involving:

price fixing.

Page 694: Microeconomics Study Guide

(Consider This) According to the Consider This box on catfish and art, which of the following firms were recently convicted of price fixing?

Sotheby's and Christy's (art auction houses)

(Last Word) In 2001, Microsoft was found guilty of violating:

Sections 1 and 2 of the Sherman Act.

The average household income in the United States in 2008 was:

$68,424

The percentage of total before-tax income received by the top 20 percent of households in 2008 was about:

50 percent

A Lorenz curve showing perfect equality in the distribution of income:

Is a straight line with a 45-degree angle

Which of the following is an example of a noncash transfer that is typically not included in the income-distribution data?

Medicare

Suppose that Jane earns $10,000 in year 1 and $15,000 in year 2, while Jim earns $15,000 in year 1 and $10,000 in year 2. Is there income equality for the two individuals?

The annual data indicate inequality, but the two-year data indicate equality

Two major criticisms of the Bureau of Census data as a portrayal of the degree of income inequality are that the income concept employed is too:

Narrow and the income accounting period is too short

The degree of inequality in income distribution based on single-year data is:

Overstated because it does not take into account income mobility

One of the major causes of income inequality is differences in:

Education

Which of the following would be considered part of wealth?

Corporate stock holdings

Page 695: Microeconomics Study Guide

Reasons for the growing income inequality in the United States since 1970 include the following, except:

Increase in the number of households headed by single or divorced women

"A more equal distribution of income in the United States would result in greater total consumer satisfaction." This statement is based on the concept that:

Incomes are subject to diminishing marginal utility

Of the following groups, which had the highest incidence of poverty in 2008?

Hispanics

Social insurance programs partially replace income that has been lost due to the following, except:

Resignation from a job

Government programs that pay benefits to those who are unable to earn income because of permanent disabilities or to those who have very low incomes and dependent children are called:

Public assistance programs

The Temporary Assistance for Needy Families (TANF) program:

Put a limit on receiving welfare payments and required able-bodied adults to work after receiving assistance for two years

One of the provisions of the Temporary Assistance for Needy Families (TANF) program was to:

Set a 5-year lifelong limit on welfare benefits

Suppose that a prejudiced white employer is willing to hire white workers at a rate of $16/hour, and this employer has a discrimination coefficient of $4. This implies that the employer would:

Hire a non-white worker only at a rate of $12/hour or less

Suppose that white workers are getting paid $21/hour, while similarly-productive African-American workers are getting paid $18/hour. A prejudiced white employer with a discrimination coefficient of $24/hour will:

Not hire African-Americans at all, even if they offer to work for free

Ending discrimination against minority groups in educational processes and in employment situations would cause total domestic output to:

Page 696: Microeconomics Study Guide

Rise, because of an increase in the productivity of the labor force

In statistical discrimination:

The average characteristics of the group are applied to individual members

The crowding model of occupational discrimination suggests that occupational segregation results in:

A lower domestic output than would otherwise be the case

The Lorenz curve is a graph that relates income to household spending.

False

The Supplemental Nutrition Assistance Program (formerly the food-stamp program) mostly pays out cash-vouchers to eligible households.

False

The Temporary Assistance for Needy Families (TANF) that replaced the old Aid for Families with Dependent Children welfare program succeeded in reducing the number of welfare recipients and increasing the employment rate among single mothers.

True

In 2008, about what percentage of U.S. households had personal incomes of less than $10,000?

About 7 percent

The Lorenz curve is helpful in visualizing the:

Degree of inequality in the distribution of income

As the area between the Lorenz curve and diagonal gets larger, the Gini ratio:

Rises to reflect greater inequality

When taxes and transfer payments are taken into account, the distribution of income in the United States:

Is more equally distributed

Since 1970, the distribution of personal income in the United States has:

Moved toward greater inequality

Entitlement programs include:

Page 697: Microeconomics Study Guide

Both public assistance programs and social insurance programs

Which of the following statements applies to the Social Security program?

It is financed by payroll taxes on employees and employers

The earned income tax credit (EITC) is, in essence:

A wage subsidy for low-income workers to offset Social Security taxes

Which of the following distinguishes the short run from the long run in pure competition: Firms can enter and exit the market in the long run, but not in the short run.           2primary force encouraging the entry of new firms into a PURELY COMPETITIVE INDUSTRY is:economic profits earned by firms already in the industry.          3In a purely competitive industry:here may be economic profits in the short run, but not in the long run.        

Page 698: Microeconomics Study Guide

  4Suppose a firm in a purely competitive market discovers that the price of its product is above its minimum AVC point but everywhere below ATC. Given this, the firmshould continue producing in the short run but leave the industry in the long run.          5We would expect an industry to expand if firms in that industry are:earning economic profits          6Assume a purely competitive, increasing-cost industry is in long-run equilibrium. If a decline in demand occurs, firms will:leave the industry and price and output will both decline.          7When a purely competitive firm is in long-run equilibrium:price equals marginal cost      

Page 699: Microeconomics Study Guide

    8A constant-cost industry is one in which:resource prices remain unchanged as output is increased.          910. An increasing-cost industry is associated with: B. an upsloping long-run supply curve.          10

Refer to the above diagrams which pertain to a purely competitive firm producing output q and the industry in which it operates. Which of the following iscorrect?The diagrams portray short-run equilibrium, but not long-run equilibrium.         

Page 700: Microeconomics Study Guide

 11

Refer to the above diagrams which pertain to a purely competitive firm producing output q and the industry in which it operates. In the long run we should expect:firms to leave the industry, market supply to fall, and product price to rise.          12A purely competitive firm is precluded from making economic profit in the long run because:of unimpeded entry to the industry.          13Under what conditions would an increase in demand lead to a lower long-run equilibrium price?the firms in the market are part of a decreasing-cost industry          14

Page 701: Microeconomics Study Guide

Refer to the above diagram showing the average total cost curve for a purely competitive firm. At the long-run equilibrium level of output, this firm's total revenue:is $400          15The MR = MC rule applies:in both the short run and long run          16

Page 702: Microeconomics Study Guide

Refer to the above diagram. Line (1) reflects the long-run supply curve for:an increasing-cost industry.          17

Refer to the above diagram. Line (2) reflects the long-run supply curve for:a constant-cost industry.          18

Page 703: Microeconomics Study Guide

Refer to the above diagram. Line (1) reflects a situation where resource prices:increase as industry output expands.          19

Refer to the above diagram. Line (2) reflects a situation where resource pricesremain constant as industry output expands.          20Allocative efficiency is achieved when the production of a good occurs where:

Page 704: Microeconomics Study Guide

P=MC          21

The term productive efficiency refers to:

the production of a good at the lowest average total cost          22Under pure competition in the long run:both allocative efficiency and productive efficiency are achieved.          2344. If for a firm P = minimum ATC = MC, then:

C. both allocative efficiency and productive efficiency are being achieved.        

Page 705: Microeconomics Study Guide

  24

The above diagram portrays:the equilibrium position of a competitive firm in the long run.          25

Refer to the above diagram. If this competitive firm produces output Q, it will:earn a normal profit.       

Page 706: Microeconomics Study Guide

   26

Refer to the above diagram. At output level Q1:neither productive nor allocative efficiency are achieved.          27

Refer to the above diagram. At output level Q1:resources are underallocated to this product and productive efficiency is not realized.      

Page 707: Microeconomics Study Guide

    28

Refer to the above diagram. At output level Q2:resources are overallocated to this product and productive efficiency is not realized.          29Allocative efficiency occurs whenever:it is impossible to produce a net benefit for society by changing the combination of goods and services produced.          30Which of the following outcomes is consistent with a purely competitive market in long-run equilibrium?consumer and producer surplus will be maximized.    

Page 708: Microeconomics Study Guide

      31Entrepreneurs in purely competitive industries:innovate to lower operating costs and generate short-run economic profits.          32Innovations that lower production costs or create new products:often generate short-run economic profits that do not last into the long run.          33The process by which new firms and new products replace existing dominant firms and products is called:creative destruction          34(Last Word) The entry of generic drugs into a previously monopolized pharmaceutical market will:increase efficiency by increasing consumer surplus    

Page 709: Microeconomics Study Guide

      35(Last Word) Patents give pharmaceutical companies exclusive rights to produce and sell the patented medications:for a maximum of 20 years

1. Marginal utility can be: A. positive, but not negative.B. positive or negative, but not zero.C. positive, negative, or zero.D. decreasing, but not negative. 

     2. Refer to the above data. The value for X is: A. 15.B. 5.C. 55.D. 10. 

3. Marginal utility is the: A. sensitivity of consumer purchases of a good to changes in the price of that good.B. change in total utility obtained by consuming one more unit of a good.C. change in total utility obtained by consuming another unit of a good divided by the change in the price of that good.D. total utility associated with the consumption of a certain number of units of a good divided by the number of units consumed. 

     

Page 710: Microeconomics Study Guide

4. Refer to the above diagram. The marginal utility of the third unit of X is: A. 5.B. 4.C. 2.D. 15. 

5. Where total utility is at a maximum, marginal utility is: A. negative.B. positive and increasing.C. zero.D. positive but decreasing. 

Page 711: Microeconomics Study Guide

6. To maximize utility a consumer should allocate money income so that the: A. elasticity of demand on all products purchased is the same.B. marginal utility obtained from the last dollar spent on each product is the same.C. total utility derived from each product consumed is the same.D. marginal utility of the last unit of each product consumed is the same. 

7. The marginal utility of the last unit of apples consumed is 12 and the marginal utility of the last unit of bananas consumed is 8. What set of prices for apples and bananas, respectively, would be consistent with consumer equilibrium? A. $4 and $6B. $6 and $4C. $8 and $12D. $16 and $9 

8. If a rational consumer is in equilibrium, which of the following conditions will hold true? A. MUa = MUb = MUc = ... = MUn.B. The marginal utility of each good purchased will be zero.C. The marginal utility of the last dollar spent on each good purchased will be the same.D. The total utility obtained from each good purchased will be the same. 

9. A consumer's demand curve for a product is downsloping because: A. total utility falls below marginal utility as more of a product is consumed.B. marginal utility diminishes as more of a product is consumed.C. time becomes less valuable as more of a product is consumed.D. the income and substitution effects precisely offset each other. 

 Answer the question on the basis of the following marginal utility data for products X and Y. Assume that the prices of X and Y are $4 and $2 respectively and that the consumer's income is $18.

    

10. Refer to the above data. What quantities of X and Y should be purchased to maximize utility? A. 2 of X and 1 of YB. 4 of X and 5 of YC. 2 of X and 5 of YD. 2 of X and 6 of Y 

11. An explicit cost is: A. omitted when accounting profits are calculated.B. a money payment made for resources not owned by the firm itself.C. an implicit cost to the resource owner who receives that payment.D. always in excess of a resource's opportunity cost. 

Page 712: Microeconomics Study Guide

 The following is cost information for the Creamy Crisp Donut Company:Entrepreneur's potential earnings as a salaried worker = $50,000Annual lease on building = $22,000Annual revenue from operations = $380,000Payments to workers = $120,000Utilities (electricity, water, disposal) costs = $8,000Value of entrepreneur's talent in the next best entrepreneurial activity = $80,000Entrepreneur's forgone interest on personal funds used to finance the business = $6,000 

12. Refer to the above data. Creamy Crisp's accounting profit is: A. $150,000.B. $380,000.C. $230,000.D. $294,000. 

13. Refer to the above data. Creamy Crisp's economic profit is: A. $150,000.B. $80,000.C. $230,000.D. $94,000. 

14. Refer to the above data. If, other things equal, Creamy Crisp's revenue fell to $286,000: A. its implicit costs, including a normal profit, would exceed its explicit costs.B. it would earn a normal profit but not an economic profit.C. it would suffer an economic loss.D. its accounting profit would fall to zero. 

 Answer the question on the basis of the following output data for a firm. Assume that the amounts of all non-labor resources are fixed.

   

15. Refer to the above data. The marginal product of the sixth worker is: A. 180 units of output.B. 30 units of output.C. 15 units of output.D. negative. 

16. Refer to the above data. Average product is at a maximum when: A. five workers are hired.B. four workers are hired.C. three workers are hired.D. two workers are hired.  

Page 713: Microeconomics Study Guide

17. To economists, the main difference between the short run and the long run is that: A. the law of diminishing returns applies in the long run, but not in the short run.B. in the long run all resources are variable, while in the short run at least one resource is fixed.C. fixed costs are more important to decision making in the long run than they are in the short run.D. in the short run all resources are fixed, while in the long run all resources are variable. 

     

18. In the above diagram the range of diminishing marginal returns is: A. 0Q3.B. 0Q2.C. Q1Q2.D. Q1Q3. 

 Use the following data to answer the question:

   19. Refer to the above data. The average product (AP) when two units of labor are hired is: A. 8.B. 9.C. 10.D. 18. 

20. Refer to the above data. Diminishing returns begin to occur with the hiring of the _________ unit of labor. A. firstB. secondC. thirdD. seventh 

21. Refer to the above data. Marginal product becomes negative with the hiring of the __________ unit of labor. A. thirdB. fourthC. sixthD. seventh 

Page 714: Microeconomics Study Guide

22. Fixed cost is: A. the cost of producing one more unit of capital, for example, machinery.B. any cost which does not change when the firm changes its output.C. average cost multiplied by the firm's output.D. usually zero in the short run. 

23. If a firm decides to produce no output in the short run, its costs will be: A. its marginal costs.B. its variable costs.C. its fixed costs.D. zero. 

24. A purely competitive seller is: A. both a "price maker" and a "price taker."B. neither a "price maker" nor a "price taker."C. a "price taker."D. a "price maker." 

25. The demand curve in a purely competitive industry is _____, while the demand curve to a single firm in that industry is _____. A. perfectly inelastic, perfectly elasticB. downsloping, perfectly elasticC. downsloping, perfectly inelasticD. perfectly elastic, downsloping 

26. A competitive firm in the short run can determine the profit-maximizing (or loss-minimizing) output by equating: A. price and average total cost.B. price and average fixed cost.C. marginal revenue and marginal cost.D. price and marginal revenue. 

27. When a firm is maximizing profit it will necessarily be: A. maximizing profit per unit of output.B. maximizing the difference between total revenue and total cost.C. minimizing total cost.D. maximizing total revenue. 

28. Suppose you find that the price of your product is less than minimum AVC. You should: A. minimize your losses by producing where P = MC.B. maximize your profits by producing where P = MC.C. close down because, by producing, your losses will exceed your total fixed costs.D. close down because total revenue exceeds total variable cost. 

Page 715: Microeconomics Study Guide

 Answer the question on the basis of the following data confronting a firm:

    29. Refer to the above data. If the firm's minimum average variable cost is $10, the firm's profit-maximizing level of output would be: A. 2.B. 3.C. 4.D. 5. 

30. If a firm is confronted with economic losses in the short run, it will decide whether or not to produce by comparing: A. marginal revenue and marginal cost.B. price and minimum average variable cost.C. total revenue and total cost.D. total revenue and total fixed cost. 

31. A firm finds that at its MR = MC output, its TC = $1,000, TVC = $800, TFC = $200, and total revenue is $900. This firm should: A. shut down in the short run.B. produce because the resulting loss is less than its TFC.C. produce because it will realize an economic profit.D. liquidate its assets and go out of business. 

     

 32. Refer to the above diagram for a purely competitive producer. The firm will produce at a loss at all prices: A. above P1.B. above P3.C. above P4.D. between P2 and P3. 

33. The primary force encouraging the entry of new firms into a purely competitive industry is: A. normal profits earned by firms already in the industry.B. economic profits earned by firms already in the industry.C. government subsidies for start-up firms.D. a desire to provide goods for the betterment of society.  

Page 716: Microeconomics Study Guide

34. If a purely competitive firm is producing at the MR = MC output level and earning an economic profit, then: A. the selling price for this firm is above the market equilibrium price.B. new firms will enter this market.C. some existing firms in this market will leave.D. there must be price fixing by the industry's firms. 

35. Long-run competitive equilibrium: A. is realized only in constant-cost industries.B. will never change once it is realized.C. is not economically efficient.D. results in zero economic profits. 

36. A purely competitive firm: A. must earn a normal profit in the short run.B. cannot earn economic profit in the long run.C. may realize either economic profit or losses in the long run.D. cannot earn economic profit in the short run. 

37. When LCD televisions first came on the market, they sold for at least $1,000, and some for much more. Now many units can be purchased for under $400. These facts imply that: A. the LCD television industry was once competitive, but is now monopolistic.B. fewer firms produce LCD televisions than was the case five or ten years ago.C. the demand curve for LCD televisions has shifted leftward.D. the LCD television industry is a decreasing-cost industry. 

38. The MR = MC rule applies: A. in the short run, but not in the long run.B. in the long run, but not in the short run.C. in both the short run and the long run.D. only to a purely competitive firm.

      39. Refer to the above diagrams which pertain to a purely competitive firm producing output q and the industry in which it operates. In the long run we should expect: A. firms to enter the industry, market supply to rise, and product price to fall.B. firms to leave the industry, market supply to rise, and product price to fall.C. firms to leave the industry, market supply to fall, and product price to rise.D. no change in the number of firms in this industry. 

 40. Creative destruction is: A. the process by which large firms buy up small firms.B. the process by which new firms and new products replace existing dominant firms and products.C. a term coined many years ago by Adam Smith.D. is applicable to planned economies, but not to market economies.

Page 717: Microeconomics Study Guide

41. Which of the following is correct? A. Both purely competitive and monopolistic firms are "price takers."B. Both purely competitive and monopolistic firms are "price makers."C. A purely competitive firm is a "price taker," while a monopolist is a "price maker."D. A purely competitive firm is a "price maker," while a monopolist is a "price taker."

42. Which of the following is a characteristic of pure monopoly? A. close substitute productsB. barriers to entryC. the absence of market powerD. "price taking"

43. The nondiscriminating pure monopolist's demand curve: A. is the industry demand curve.B. shows a direct or positive relationship between price and quantity demanded.C. tends to be inelastic at high prices and elastic at low prices.D. is identical to its marginal revenue curve.

44. When a firm is on the inelastic segment of its demand curve, it can: A. increase total revenue by reducing price.B. decrease total costs by decreasing price.C. increase profits by increasing price.D. increase total revenue by more than the increase in total cost by increasing price.Answer the question on the basis of the demand schedule shown below:

    45. Refer to the above data. The marginal revenue obtained from selling the third unit of output is: A. $6.B. $1.C. $3.D. $5.

46. For a pure monopolist marginal revenue is less than price because: A. the monopolist's demand curve is perfectly elastic.B. the monopolist's demand curve is perfectly inelastic.C. when a monopolist lowers price to sell more output, the lower price applies to all units sold.D. the monopolist's total revenue curve is linear and slopes upward to the right.

Page 718: Microeconomics Study Guide

47. A pure monopolist should never produce in the: A. elastic segment of its demand curve because it can increase total revenue and reduce total cost by lowering price.B. inelastic segment of its demand curve because it can increase total revenue and reduce total cost by increasing price.C. inelastic segment of its demand curve because it can always increase total revenue by more than it increases total cost by reducing price.D. segment of its demand curve where the price elasticity coefficient is greater than one.

Answer the question on the basis of the following table showing the demand schedulefacing a nondiscriminating monopolist:

    48. Refer to the above table. The monopolist will select its profit-maximizing level of output somewhere within the: A. 3-5 unit range of output.B. 1-3 unit range of output.C. 1-4 unit range of output.D. 2-4 unit range of output.

      49. Which of the above diagrams correctly portray a nondiscriminating pure monopolist's demand (D) and marginal revenue (MR) curves? A. AB. BC. CD. D

50. Which of the above diagrams correctly portray the demand (D) and marginal revenue (MR) curves of a purely competitive seller? A. AB. BC. CD. D

Page 719: Microeconomics Study Guide

51. Suppose that a pure monopolist can sell 5 units of output at $4 per unit and 6 units at $3.90 per unit. The monopolist will produce and sell the sixth unit if its marginal cost is: A. $4 or less.B. $3.90 or less.C. $3.50 or less.D. $3.40 or less.

 52. Refer to the above data for a nondiscriminating monopolist. This firm will maximize its profit by producing: A. 3 units.B. 4 units.C. 5 units.D. 6 units.

53. Refer to the above data for a nondiscriminating monopolist. At its profit-maximizing output, this firm's price will exceed its marginal cost by ____ and its average total cost by ___. A. $20; $27.33B. $10; $10.40C. $24; $27.33D. $30; $20.50

54. Refer to the above data for a nondiscriminating monopolist. At its profit-maximizing output, this firm's total costs will be: A. $300.B. $248.C. $198.D. $126.

55. Refer to the above data. At its profit-maximizing output, this firm's total revenue will be: A. $300.B. $198.C. $180.D. $280.

Page 720: Microeconomics Study Guide

56. Refer to the above data for a nondiscriminating monopolist. At its profit-maximizing output, this firm's total profit will be: A. $82.B. zero.C. $54.D. $27. 

1. An industry comprised of 40 firms, none of which has more than 3 percent of the total market for a differentiated product is an example of: A. monopolistic competition.B. oligopoly.C. pure monopoly.D. pure competition.

A. monopolistic competition.

2. An industry comprised of a small number of firms, each of which considers the potential reactions of its rivals in making price-output decisions is called: A. monopolistic competition.B. oligopoly.C. pure monopoly.D. pure competition.

B. oligopoly.

3. Which of the following is not a basic characteristic of pure competition? A. considerable nonprice competitionB. no barriers to the entry or exit of firmsC. a standardized or homogeneous productD. a large number of buyers and sellers

A. considerable nonprice competition

4. Which of the following statements is correct? A. The demand curve for a purely competitive firm is perfectly elastic, but the demand curve for a purely competitive industry is downsloping.B. The demand curve for a purely competitive firm is downsloping, but the demand curve for a purely competitive industry is perfectly elastic.C. The demand curves are downsloping for both a purely competitive firm and a purely competitive industry.D. The demand curves are perfectly elastic for both a purely competitive firm and a purely competitive industry.

Page 721: Microeconomics Study Guide

A. The demand curve for a purely competitive firm is perfectly elastic, but the demand curve for a purely competitive industry is downsloping.

5. Marginal revenue is the: A. change in product price associated with the sale of one more unit of output.B. change in average revenue associated with the sale of one more unit of output.C. difference between product price and average total cost.D. change in total revenue associated with the sale of one more unit of output.

D. change in total revenue associated with the sale of one more unit of output.

6. Suppose you find that the price of your product is less than minimum AVC. You should: A. minimize your losses by producing where P = MC. B. maximize your profits by producing where P = MC. C. close down because, by producing, your losses will exceed your total fixed costs.D. close down because total revenue exceeds total variable cost.

C. close down because, by producing, your losses will exceed your total fixed costs.

7. Refer to the above diagram. To maximize profit or minimize losses this firm will produce: A. K units at price C. B. D units at price J. C. E units at price A. D. E units at price B.

C. E units at price A.

8. Refer to the above diagram. At the profit-maximizing output, total revenue will be: A. 0AHE.B. 0BGE.C. 0CFE.D. ABGE.

A. 0AHE.

9. Refer to the above diagram. At the profit-maximizing output, the firm will realize: A. a loss equal to BCFG.B. a loss equal to ACFH.C. an economic profit of ACFH.D. an economic profit of ABGH.

D. an economic profit of ABGH.

10. Refer to the above diagram. At P2, this firm will: A. produce 44 units and realize an economic profit.B. produce 44 units and earn only a normal profit.C. produce 68 units and earn only a normal profit.D. shut down in the short run.

Page 722: Microeconomics Study Guide

B. produce 44 units and earn only a normal profit.

11. Refer to the above diagram. At P1, this firm will produce: A. 47 units and break even.B. 47 units and realize an economic profit.C. 66 units and earn only a normal profit.D. 24 units and earn only a normal profit.

B. 47 units and realize an economic profit.

12. Refer to the above diagram. At P4, this firm will: A. shut down in the short run.B. produce 30 units and incur a loss.C. produce 30 units and earn only a normal profit.D. produce 10 units and earn only a normal profit.

A. shut down in the short run.

13. Refer to the above diagram. At P3, this firm will: A. produce 14 units and realize an economic profit.B. produce 62 units and earn only a normal profit.C. produce 40 units and incur a loss.D. shut down in the short run.

C. produce 40 units and incur a loss.

14. If product price is $60, the firm will: A. shut down.B. produce 4 units and realize a $120 economic profit.C. produce 6 units and realize a $100 economic profit.D. produce 3 units and incur a $40 loss.

C. produce 6 units and realize a $100 economic profit.

15. If product price is $45, the firm will: A. shut down.B. produce 4 units and realize a $120 economic profit.C. produce 5 units and realize a $15 economic profit.D. produce 6 units and realize a $100 economic profit.

C. produce 5 units and realize a $15 economic profit.

16. If product price is $25, the firm will: A. shut down and incur a $90 loss.B. shut down and incur a $50 loss.C. produce 3 units and incur a $65 loss.D. produce 4 units and realize a $10 economic profit.

Page 723: Microeconomics Study Guide

B. shut down and incur a $50 loss.

17. The marginal cost column reflects: A. the law of diminishing returns.B. the law of diminishing marginal utility.C. diseconomies of scale.D. economies of scale.

A. the law of diminishing returns.

18. At 6 units of output, total fixed cost is ____ and total cost is ____ A. $25; $50.B. $50; $300.C. $100; $200.D. $150; $300.

D. $150; $300.

19. If the market price for this firm's product is $87, it will produce: A. 9 units at an economic profit of zero.B. 6 units at a loss of $90.C. 9 units at an economic profit of $281.97.D. 8 units at an economic profit of $130.72.

C. 9 units at an economic profit of $281.97.

20. If the market price for this firm's product is $68.10, it will produce: A. 8 units at an economic profit of zero.B. 6 units at a loss of $90.C. 9 units at an economic profit of $281.97.D. 8 units at an economic profit of $130.72.

D. 8 units at an economic profit of $130.72.

21. If the market price for this firm's product is $24, it will produce: A. 4 units at a loss of $150.B. 6 units at a loss of $90.C. 3 units at an economic profit of zero.D. 4 units at a loss of $138.

D. 4 units at a loss of $138.

22. If the market price for this firm's product is $15, it will produce: A. 0 units at a loss of $150.B. 3 units at a loss of $168.C. 3 units at an economic profit of zero.D. 4 units at a loss of $138.

Page 724: Microeconomics Study Guide

A. 0 units at a loss of $150.

1. Which of the following is true concerning purely competitive industries? A. There will be economic losses in the long run because of cut-throat competition.B. Economic profits will persist in the long run if consumer demand is strong and stable.C. In the short run, firms may incur economic losses or earn economic profits, but in the long run they earn normal profits.D. There are economic profits in the long run, but not in the short run.

C. In the short run, firms may incur economic losses or earn economic profits, but in the long run they earn normal profits.

2. A constant-cost industry is one in which: A. a higher price per unit will not result in an increased output.B. if 100 units can be produced for $100, then 150 can be produced for $150, 200 for $200, and so forth.C. the demand curve and therefore the unit price and quantity sold seldom change.D. the total cost of producing 200 or 300 units is no greater than the cost of producing 100 units.

B. if 100 units can be produced for $100, then 150 can be produced for $150, 200 for $200, and so forth.

3. The above diagrams show a purely competitive firm producing output q and the industry in which it operates. Which of the following is correct? A. The diagrams portray neither long-run nor short-run equilibrium.B. The diagrams portray both long-run and short-run equilibrium.C. The diagrams portray short-run equilibrium, but not long-run equilibrium.D. The diagrams portray long-run equilibrium, but not short-run equilibrium.

C. The diagrams portray short-run equilibrium, but not long-run equilibrium.

4. The above diagrams show a purely competitive firm producing output q and the industry in which it operates. In the long run we should expect: A. firms to enter the industry, market supply to rise, and product price to fall.B. firms to leave the industry, market supply to rise, and product price to fall.C. firms to leave the industry, market supply to fall, and product price to rise.D. no change in the number of firms in this industry.

C. firms to leave the industry, market supply to fall, and product price to rise.

5. The above diagrams show a purely competitive firm producing output q and the industry in which it operates. The predicted long-run adjustments in this industry might be offset by: A. a decline in product demand.B. an increase in resource prices.C. a technological improvement in production methods.D. entry of new firms into the industry.

C. a technological improvement in production methods.

Page 725: Microeconomics Study Guide

6. When LCD televisions first came on the market, they sold for at least $1,000 and some for much more. Now many units can be purchased for under $400. These facts imply that: A. the LCD television industry was once competitive, but is now monopolistic.B. fewer firms produce LCD televisions than was the case five or ten years ago.C. the demand curve for LCD televisions has shifted leftward.D. the LCD television industry is a decreasing-cost industry.

D. the LCD television industry is a decreasing-cost industry.

7. The above diagram shows the average total cost curve for a purely competitive firm. At the long-run equilibrium level of output, this firm's total revenue: A. is $10.B. is $40.C. is $400.D. cannot be determined from the information provided.

C. is $400.

8. The above diagram shows the average total cost curve for a purely competitive firm. At the long-run equilibrium level of output, this firm's economic profit: A. is zero.B. is $400.C. is $200.D. cannot be determined from the information provided.

A. is zero.

9. The term allocative efficiency refers to: A. the level of output that coincides with the intersection of the MC and AVC curves.B. minimization of the AFC in the production of any good.C. the production of the product-mix most desired by consumers.D. the production of a good at the lowest average total cost.

C. the production of the product-mix most desired by consumers.

10. Under pure competition in the long run: A. neither allocative efficiency nor productive efficiency are achieved.B. both allocative efficiency and productive efficiency are achieved.C. productive efficiency is achieved, but allocative efficiency is not.D. allocative efficiency is achieved, but productive efficiency is not.

B. both allocative efficiency and productive efficiency are achieved.

11. If for a firm P = minimum ATC = MC, then: A. neither allocative efficiency nor productive efficiency is being achieved.B. productive efficiency is being achieved, but allocative efficiency is not.

Page 726: Microeconomics Study Guide

C. both allocative efficiency and productive efficiency are being achieved.D. allocative efficiency is being achieved, but productive efficiency is not.

C. both allocative efficiency and productive efficiency are being achieved.

12. In the above diagram by producing output level Q: A. neither productive nor allocative efficiency are achieved.B. both productive and allocative efficiency are achieved.C. allocative efficiency is achieved, but productive efficiency is not.D. productive efficiency is achieved, but allocative efficiency is not.

B. both productive and allocative efficiency are achieved.

13. In the above diagram at output level Q1: A. resources are overallocated to this product and productive efficiency is not realized.B. resources are underallocated to this product and productive efficiency is not realized.C. productive efficiency is achieved, but resources are underallocated to this product.D. productive efficiency is achieved, but resources are overallocated to this product.

B. resources are underallocated to this product and productive efficiency is not realized.

14. In the above diagram at output level Q2: A. resources are overallocated to this product and productive efficiency is not realized.B. resources are underallocated to this product and productive efficiency is not realized.C. productive efficiency is achieved, but resources are underallocated to this product.D. productive efficiency is achieved, but resources are overallocated to this product.

A. resources are overallocated to this product and productive efficiency is not realized.

15. Allocative efficiency occurs whenever: A. consumer surplus is maximized.B. it is impossible to produce a net benefit for society by changing the combination of goods and services produced.C. firms have maximized their profits.D. it is impossible to make someone in society better off without making someone else worse off.

B. it is impossible to produce a net benefit for society by changing the combination of goods and services produced.

16. In long-run equilibrium, purely competitive markets: A. minimize total cost.B. maximize the sum of consumer surplus and producer surplus.C. yield economic profits to most sellers.D. inevitably degenerate into monopoly in increasing cost industries.

B. maximize the sum of consumer surplus and producer surplus.

Page 727: Microeconomics Study Guide

17. Entrepreneurs in purely competitive industries: A. have no incentive to innovate because in the long run they will earn no economic profits.B. innovate to lower operating costs and generate short-run economic profits.C. utilize pricing strategies to generate short-run economic profits.D. rarely try to innovate because of a lack of financial resources.

B. innovate to lower operating costs and generate short-run economic profits.

18. Creative destruction is: A. the process by which large firms buy up small firms.B. the process by which new firms and new products replace existing dominant firms and products.C. a term coined many years ago by Adam Smith.D. is applicable to planned economies, but not to market economies.

B. the process by which new firms and new products replace existing dominant firms and products.

19. (Consider This) Which of the following statements is true about U.S. firms? A. Over half are bankrupt within the first two years after starting up.B. Over half are bankrupt within the first five years after starting up.C. Nearly 65 percent last 10 years or more.D. The life expectancy of a U.S. firm is approximately 22 years.

B. Over half are bankrupt within the first five years after starting up.

20. (Last Word) The entry of generic drugs into a previously monopolized pharmaceutical market will: A. discourage the development of new drugs.B. increase efficiency by increasing consumer surplus.C. create inefficiency by introducing chemically-inferior medications.D. not affect the market price because pharmaceutical firms are "price takers."

B. increase efficiency by increasing consumer surplus.

1. Which of the following is correct? A. Both purely competitive and monopolistic firms are "price takers."B. Both purely competitive and monopolistic firms are "price makers."C. A purely competitive firm is a "price taker," while a monopolist is a "price maker."D. A purely competitive firm is a "price maker," while a monopolist is a "price taker."

C. A purely competitive firm is a "price taker," while a monopolist is a "price maker."

2. Pure monopolists may obtain economic profits in the long run because: A. of advertising.B. marginal revenue is constant as sales increase.C. of barriers to entry.D. of rising average fixed costs.

Page 728: Microeconomics Study Guide

C. of barriers to entry.

3. Which of the following best approximates a pure monopoly? A. the foreign exchange marketB. the Kansas City wheat marketC. the only bank in a small townD. the soft drink market

C. the only bank in a small town

4. Large minimum efficient scale of plant combined with limited market demand may lead to:

A. natural monopoly.B. patent monopoly.C. government franchise monopoly.D. shared monopoly.

A. natural monopoly.

5. For an imperfectly competitive firm: A. total revenue is a straight, upsloping line because a firm's sales are independent of product price.B. the marginal revenue curve lies above the demand curve because any reduction in price applies to all units sold.C. the marginal revenue curve lies below the demand curve because any reduction in price applies to all units sold.D. the marginal revenue curve lies below the demand curve because any reduction in price applies only to the extra unit sold.

C. the marginal revenue curve lies below the demand curve because any reduction in price applies to all units sold.

6. Refer to the above diagram for a pure monopolist. If the monopolist is unregulated, it will maximize profits by charging: A. a price above P3 and selling a quantity less than Q3.B. price P3 and producing output Q3.C. price P2 and producing output Q2.D. price P1 and producing output Q1.

B. price P3 and producing output Q3.

7. Refer to the above diagram for a pure monopolist. Suppose a regulatory commission is created to determine a legal price for the monopoly. If the commission seeks to provide the monopolist with a "fair return," it will set price at: A. P1.B. P3.

Page 729: Microeconomics Study Guide

C. P2.D. P4.

A. P1.

8. Refer to the above diagram for a pure monopolist. If a regulatory commission seeks to achieve the socially optimal allocation of resources to this line of production, it will set a price of: A. P1.B. P3.C. P2.D. P4.

C. P2.

9. Refer to the above diagram for a pure monopolist. If a regulatory commission sets the price to achieve the socially optimal allocation of resources, it will have to: A. tax the monopolist P3P1 per unit to prevent the monopolist from realizing an economic profit.B. subsidize the monopolist or the monopolist will go bankrupt in the long run.C. subsidize the monopolist P1P4 per unit to allow the monopolist to break even.D. tax the monopolist P1P2 per unit to prevent the monopolist from realizing an economic profit.

B. subsidize the monopolist or the monopolist will go bankrupt in the long run.

10. The profit-maximizing price for the monopolist will be: A. $5.00.B. $2.90.C. $3.35.D. $4.50.

D. $4.50.

11. The profit-maximizing monopolist will realize a: A. profit of $8.50.B. profit of $7.50.C. profit of $16.D. loss of $14.

C. profit of $16.

12. Refer to the above diagram. At the profit-maximizing level of output, total revenue will be: A. NM times 0M.B. 0AJE.

Page 730: Microeconomics Study Guide

C. 0EGC.D. 0EHB.

B. 0AJE.

13. Refer to the above diagram. At the profit-maximizing level of output, total cost will be: A. NM times 0M.B. 0AJE.C. 0CGC.D. 0BHE.

D. 0BHE.

14. Refer to the above diagram. At the profit-maximizing level of output, the firm will realize:

A. an economic profit of ABHJ.B. an economic profit of ACGJ.C. a loss of GH per unit.D. a loss of JH per unit.

A. an economic profit of ABHJ.

15. A pure monopolist is selling 6 units at a price of $12. If the marginal revenue of the seventh unit is $5, then: A. price of the seventh unit is $10.B. price of the seventh unit is $11.C. price of the seventh unit is greater than $12.D. firm's demand curve is perfectly elastic.

B. price of the seventh unit is $11.

16. In the short run, a monopolist's economic profits: A. are always positive because the monopolist is a price-maker.B. are usually negative because of government price regulation.C. are always zero because consumers prefer to buy from competitive sellers.D. may be positive or negative depending on market demand and cost conditions.

D. may be positive or negative depending on market demand and cost conditions.

17. Under which of the following situations would a monopolist increase profits by lowering price (and increasing output): A. if it discovered that it was producing where MC = MRB. if it discovered that it was producing where its MC curve intersects its demand curveC. if it discovered that it was producing where MC < MRD. under none of these circumstances because a monopolist would never lower price

C. if it discovered that it was producing where MC < MR

Page 731: Microeconomics Study Guide

18. Refer to the above diagrams. Firm A is a: A. pure competitor and Firm B is a pure monopoly.B. pure competitor, as is Firm B.C. pure monopoly and Firm B is a pure competitor.D. pure monopoly, as is Firm B.

A. pure competitor and Firm B is a pure monopoly.

19. Refer to the above diagrams. The demand for Firm B's product is: A. perfectly elastic over all ranges of output.B. perfectly inelastic over all ranges of output.C. elastic for prices above $4 and inelastic for prices below $4.D. inelastic for prices above $4 and inelastic for prices below $4.

C. elastic for prices above $4 and inelastic for prices below $4.

20. Economic profit in the long run is: A. possible for both a pure monopoly and a pure competitor.B. possible for a pure monopoly, but not for a pure competitor.C. impossible for both a pure monopolist and a pure competitor.D. only possible when barriers to entry are nonexistent.

B. possible for a pure monopoly, but not for a pure competitor.

21. Which of the following statements is correct? A. The pure monopolist will maximize profit by producing at that point on the demand curve where elasticity is zero.B. In seeking the profit-maximizing output the pure monopolist underallocates resources to its production.C. The pure monopolist maximizes profits by producing that output at which the differential between price and average cost is the greatest.D. Purely monopolistic sellers earn only normal profits in the long run.

B. In seeking the profit-maximizing output the pure monopolist underallocates resources to its production.

22. Confronted with the same unit cost data, a monopolistic producer will charge: A. the same price and produce the same output as a competitive firm.B. a higher price and produce a larger output than a competitive firm.C. a higher price and produce a smaller output than a competitive firm.D. a lower price and produce a smaller output than a competitive firm.

C. a higher price and produce a smaller output than a competitive firm.

23. Comparing a pure monopoly and a purely competitive firm with identical costs, we would find in long-run equilibrium that the pure monopolist's: A. price, output, and average total cost would all be higher.B. price and average total cost would be higher, but output would be lower.

Page 732: Microeconomics Study Guide

C. price, output, and average total cost would all be lower.D. price and output would be lower, but average total cost would be higher.

B. price and average total cost would be higher, but output would be lower.

24. Refer to the above diagrams. With the industry structure represented by diagram: A. (A) there will be only a normal profit in the long run, while in (B) an economic profit can persist.B. (A) price exceeds marginal cost, resulting in allocative inefficiency.C. (B) price equals marginal cost, resulting in allocative efficiency.D. (B) equilibrium price and quantity will be e and h, respectively.

A. (A) there will be only a normal profit in the long run, while in (B) an economic profit can persist.

25. Refer to the above diagrams. With the industry structure represented by diagram: A. (B) there will be allocative efficiency.B. (A) economic profit can persist in the long run.C. (B) output will be less than in diagram (A).D. (B) output will be the same as in diagram (A).

C. (B) output will be less than in diagram (A).

26. There is some evidence to suggest that X-inefficiency is: A. absent whenever two or more producers are competing with one another.B. not encountered in either competitive or monopolistic firms.C. more likely to occur in monopolistic firms than in competitive firms.D. more likely to occur in competitive firms than in monopolistic firms.

C. more likely to occur in monopolistic firms than in competitive firms.

27. Which of the following conditions is not required for price discrimination? A. Buyer with different elasticities must be physically separate from each other.B. The good or service cannot be profitably resold by original buyers.C. The seller must be able to segment the market, that is, to distinguish buyers with different elasticities of demand.D. The seller must possess some degree of monopoly power.

A. Buyer with different elasticities must be physically separate from each other.

28. (Consider This) Children are charged less than adults for admission to professional baseball games but are charged the same prices as adults at the concession stands. Which of the following conditions of price discrimination explain why this occurs? A. The seller must have some monopoly power; that is, it must be able to set the product price.B. The seller must be able to identify buyers by group characteristics such as age or income.C. Groups must have different elasticities of demand for the product.

Page 733: Microeconomics Study Guide

D. The items cannot be bought by people in the low-price group and transferred to members of the high-price group.

D. The items cannot be bought by people in the low-price group and transferred to members of the high-price group.

1. Which of the following is not a basic characteristic of monopolistic competition? A. the use of trademarks and brand namesB. recognized mutual interdependenceC. product differentiationD. a relatively large number of sellers

B. recognized mutual interdependence

2. A monopolistically competitive industry combines elements of both competition and monopoly. The monopoly element results from: A. the likelihood of collusion.B. high entry barriers.C. product differentiation.D. mutual interdependence in decision making.

C. product differentiation.

3. Use your basic knowledge and your understanding of market structures to answer this question. Which of the following companies most closely approximates a monopolistic competitor? A. Subway SandwichesB. Pittsburgh Plate GlassC. Ford Motor CompanyD. Microsoft

A. Subway Sandwiches

4. The price elasticity of a monopolistically competitive firm's demand curve varies: A. inversely with the number of competitors and the degree of product differentiation.B. directly with the number of competitors and the degree of product differentiation.C. directly with the number of competitors, but inversely with the degree of product differentiation.D. inversely with the number of competitors, but directly with the degree of product differentiation.

C. directly with the number of competitors, but inversely with the degree of product differentiation.

5. The monopolistically competitive seller maximizes profit by producing at the point where: A. total revenue is at a maximum.B. average costs are at a minimum.

Page 734: Microeconomics Study Guide

C. marginal revenue equals marginal cost.D. price equals marginal revenue.

C. marginal revenue equals marginal cost.

6. Excess capacity refers to the: A. amount by which actual production falls short of the minimum ATC output.B. fact that entry barriers artificially reduce the number of firms in an industry.C. differential between price and marginal costs which characterizes monopolistically competitive firms.D. fact that most monopolistically competitive firms encounter diseconomies of scale.

A. amount by which actual production falls short of the minimum ATC output.

7. Refer to the above diagram for a monopolistically competitive firm in short-run equilibrium. The profit-maximizing output for this firm will be: A. 100.B. 160.C. 180.D. 210.

B. 160.

8. Refer to the above diagram for a monopolistically competitive firm in short-run equilibrium. This firm will realize an economic: A. loss of $320.B. profit of $480.C. profit of $280.D. profit of $600.

B. profit of $480.

9. If all monopolistically competitive firms in the industry have profit circumstances similar to the firm shown above: A. new firms will enter the industry.B. some firms will exit the industry.C. all firms will exit the industry.D. no firms will exit the industry.

B. some firms will exit the industry.

10. Refer to the above diagram for a monopolistically competitive firm. Long-run equilibrium price will be: A. above A.B. EF.C. A.D. B.

Page 735: Microeconomics Study Guide

C. A.

11. In long-run equilibrium monopolistic competition entails: A. an efficient allocation of resources.B. an overallocation of resources due to inadequate capacity.C. an underallocation of resources due to excess capacity.D. production at the minimum attainable average total cost.

C. an underallocation of resources due to excess capacity.

12. Refer to the above data. If columns (1) and (3) of the demand data shown above are this firm's demand schedule, the profit-maximizing price will be: A. $9.B. $7.C. $11.D. $6.

A. $9.

13. Refer to the above data. If columns (1) and (3) of the demand data shown above are this firm's demand schedule, economic profit will be: A. $10.B. $19.C. $6.D. $8.

D. $8.

14. The economic inefficiencies of monopolistic competition may be offset by the fact that: A. advertising expenditures shift the average cost curve upward.B. available capacity is fully utilized.C. resources are optimally allocated to the production of the product.D. consumers have increased product variety.

D. consumers have increased product variety.

15. In which of these continuums of degrees of competition (highest to lowest) is oligopoly properly placed? A. pure competition, oligopoly, pure monopoly, monopolistic competitionB. oligopoly, pure competition, monopolistic competition, pure monopolyC. monopolistic competition, pure competition, pure monopoly, oligopolyD. pure competition, monopolistic competition, oligopoly, pure monopoly

D. pure competition, monopolistic competition, oligopoly, pure monopoly

16. Oligopolistic industries are characterized by: A. a few dominant firms and substantial entry barriers.B. a few dominant firms and no barriers to entry.

Page 736: Microeconomics Study Guide

C. a large number of firms and low entry barriers.D. a few dominant firms and low entry barriers.

A. a few dominant firms and substantial entry barriers.

17. The copper, aluminum, cement, and industrial alcohol industries are examples of: A. interproduct competition.B. homogeneous oligopoly.C. monopolistic competition.D. differentiated oligopoly.

B. homogeneous oligopoly.

18. Oligopoly is more difficult to analyze than other market models because: A. the number of firms is so large that market behavior cannot be accurately predicted.B. the marginal cost and marginal revenue curves of an oligopolist play no part in the determination of equilibrium price and quantity.C. of mutual interdependence and the fact that oligopoly outcomes are less certain than in other market models.D. unlike the firms of other market models, it cannot be assumed that oligopolists are profit maximizers.

C. of mutual interdependence and the fact that oligopoly outcomes are less certain than in other market models.

19. Refer to the above data. The four-firm concentration ratio for the above industry is: A. 100 percent.B. indeterminate, since we don't know which four firms are included.C. 80 percent.D. 20 percent.

C. 80 percent.

20. Refer to the above data. The Herfindahl Index for the above industry is: A. 1,600.B. 1,800.C. 18,000.D. 80.

B. 1,800.

21. Refer to the above data. Suppose that firms in this industry split up such that there were 100 firms, each with a one percent market share. The four-firm concentration ratio and the Herfindahl Index respectively would be: A. 100 percent and 10,000.B. 4 percent and 4.C. 100 percent and 16.D. 4 percent and 100.

Page 737: Microeconomics Study Guide

D. 4 percent and 100.

22. Game theory: A. is the analysis of how people (or firms) behave in strategic situations.B. is best suited for analyzing purely competitive markets.C. reveals that mergers between rival firms are self-defeating.D. reveals that price-fixing among firms reduces profits.

A. is the analysis of how people (or firms) behave in strategic situations.

23. If Beta commits to a high-price policy, Alpha will gain the largest profit by: A. also adopting a high-price policy.B. adopting a low-price policy.C. adopting a low-price policy, but only if Beta agrees to do the same.D. engaging in non-price competition only.

B. adopting a low-price policy.

24. With independent pricing the outcome of this duopoly game will gravitate to cell: A. A.B. B.C. C.D. D.

A. A.

25. If Alpha and Beta engage in collusion, the outcome of the game will be at cell: A. A.B. B.C. C.D. D.

D. D.

26. Refer to the above diagram. Equilibrium output is: A. j.B. h.C. g.D. f.

C. g.

27. Refer to the above diagram. Equilibrium price is: A. e.B. d.C. c.D. b.

Page 738: Microeconomics Study Guide

B. d.

28. Refer to the above diagram. This firm's demand and marginal revenue curves are based on the assumption that: A. the firm has no immediate rivals.B. rivals will match both a price increase and a price decrease.C. rivals will match a price increase, but ignore a price decrease.D. rivals will ignore a price increase, but match a price decrease.

D. rivals will ignore a price increase, but match a price decrease.

29. Refer to the above diagram. In equilibrium the firm: A. is realizing an economic profit of ad per unit.B. should close down in the short run.C. is incurring a loss.D. is realizing an economic profit of bd per unit.

A. is realizing an economic profit of ad per unit.

30. Cartels are difficult to maintain in the long run because: A. they are illegal in all industrialized countries.B. individual members may find it profitable to cheat on agreements.C. it is more profitable for the industry to charge a lower price and produce more output.D. entry barriers are insignificant in oligopolistic industries.

B. individual members may find it profitable to cheat on agreements.

31. If the several oligopolistic firms that comprise an industry behave collusively, the resulting price and output will most likely resemble those of: A. bilateral monopoly.B. pure monopoly.C. monopolistic competition.D. pure competition.

B. pure monopoly.

32. Other things equal, cartels and similar collusive arrangements are easier to establish and maintain: A. when there are ample opportunities for the firms to make secret price concessions to selected buyers.B. during periods of business-cycle stability and full employment.C. when the demand and cost conditions of the participating firms differ substantially.D. when the number of firms is relatively large.

B. during periods of business-cycle stability and full employment.

33. Advertising can enhance economic efficiency when it: A. increases brand loyalty.

Page 739: Microeconomics Study Guide

B. raises entry barriers.C. increases consumer awareness of substitute products.D. boosts average total cost.

C. increases consumer awareness of substitute products.

34. Suppose that a particular industry has a four-firm concentration ratio of 85 and a Herfindahl Index of 3,000. Most likely, this industry would achieve: A. both productive efficiency and allocative efficiency.B. allocative efficiency but not productive efficiency.C. neither productive efficiency nor allocative efficiency.D. productive efficiency but not allocative efficiency.

C. neither productive efficiency nor allocative efficiency.

1. Broadly defined, technological advance: A. can occur in the short run, long run, or very long run.B. comprises new and improved goods and services and new and improved ways of producing or distributing them.C. includes invention, but not innovation or diffusion.D. includes product innovation, but not process innovation.

B. comprises new and improved goods and services and new and improved ways of producing or distributing them.

2. Technological advance is shown as a(n): A. movement from a point inside a production possibilities curve to a point on the curve.B. movement along a production possibilities curve.C. outward shift of a production possibilities curve.D. inward shift of a production possibilities curve.

C. outward shift of a production possibilities curve.

3. The successful commercial introduction of a new product, the use of a new method, or the creation of a new form of business enterprise is called: A. innovation.B. invention.C. creative destruction.D. diffusion.

A. innovation.

4. Which of the following is a true statement? A. innovation normally follows invention and precedes diffusion.B. invention normally follows diffusion and precedes innovation.C. diffusion normally follows invention and precedes innovation.D. innovation normally follows diffusion and precedes invention.

Page 740: Microeconomics Study Guide

A. innovation normally follows invention and precedes diffusion.

5. Innovation: A. is the first discovery of a product or process, rather than its first successful commercial introduction.B. includes new products, but not new production methods.C. is also known as diffusion.D. can either increase or decrease the market share of a large firm, depending on whether it is introduced by the large firm or one of its competitors.

D. can either increase or decrease the market share of a large firm, depending on whether it is introduced by the large firm or one of its competitors.

6. In the United States, research and development spending as a percentage of GDP is: A. 1.5 to 2.0 percent, which is lower than that of most other industrial countries.B. 2.5 to 3.0 percent, which is higher than that of most other industrial countries.C. 4.5 to 5.0 percent, which is lower than that of most other industrial countries.D. 5.5 to 6.0 percent, which is higher than that of most other industrial countries.

B. 2.5 to 3.0 percent, which is higher than that of most other industrial countries.

7. When economists view technological change as internal to the economy, they mean that it: A. occurs randomly.B. occurs accidentally.C. arises deliberately from the profit motive and competition.D. arises mainly from government subsidies.

C. arises deliberately from the profit motive and competition.

8. Entrepreneurs: A. include everyone engaged in R&D work.B. are located in small enterprises only.C. differ from other innovators because of the risks entrepreneurs must bear.D. work exclusively in government and university R&D laboratories.

C. differ from other innovators because of the risks entrepreneurs must bear.

9. New scientific knowledge mainly comes from university and government laboratories, not private firms, because: A. large corporations do not have funds available to channel toward basic research.B. government pays scientists higher salaries than do private firms.C. entrepreneurs find it difficult to secure venture capital to finance innovation.D. basic scientific principles, as such, cannot be patented and do not always have commercial applicability.

D. basic scientific principles, as such, cannot be patented and do not always have commercial applicability.

Page 741: Microeconomics Study Guide

10. In exchange for a share of ZYX's profits if it succeeds, Firm ABC provides development funds to newly formed ZYX which is developing an innovative product. ABC funds are called ____________ while ZYX is known as a ____________. A. venture capital; startupB. retained earnings; entrepreneurial firmC. mutual funds; startupD. transfer payments; entrepreneurial firm

A. venture capital; startup

11. Suppose a firm anticipates that an R&D expenditure of $100 million will result in a new production process that will reduce costs and thus create a one-time added profit of $112 million a year later. The firm's expected rate of return is: A. 0.12 percent.B. 112 percent.C. 12 percent.D. 2 percent.

C. 12 percent.

12. A profit-maximizing firm should not undertake a R&D project for which the: A. expected rate of return exceeds its interest-rate cost of funds.B. interest-rate cost of funds exceeds the expected rate of return.C. expected returns are in the distant future.D. the expected returns, though potentially very large, are uncertain.

B. interest-rate cost of funds exceeds the expected rate of return.

13. Refer to the above data. The firm's optimal amount of R&D spending is: A. $20 million.B. $40 million.C. $60 million.D. $80 million.

D. $80 million.

14. Refer to the above data. At $100 million of R&D expenditures, the: A. marginal cost of R&D exceeds the marginal benefit.B. marginal benefit of R&D exceeds the marginal cost.C. expected rate of return from R&D is negative.D. firm has exceeded its affordable level of R&D.

A. marginal cost of R&D exceeds the marginal benefit.

15. A consumer will buy a new product rather than an existing product: A. when the MU/P of the new product is less than the MU/P of the existing product.B. when the substitution of the new product for the old product increases the consumer's total

Page 742: Microeconomics Study Guide

utility.C. only if the new product has a lower price than the existing product.D. only if the MU of the new product exceeds the MU of the existing product.

B. when the substitution of the new product for the old product increases the consumer's total utility.

16. Refer to the above diagram which relates to Firm A. Which of the following would shift A's average total cost curve from ATC1 to ATC2? A. replacement of old equipment with new, more productive equipment embodying technological advanceB. a decrease in the incomes of A's customersC. a move along A's total product curve (not shown)D. the increase in the price of one of the major inputs used to produce A's product

A. replacement of old equipment with new, more productive equipment embodying technological advance

17. All of the following increase the expected rate of return on R&D expenditures, except: A. patents.B. trademarks.C. imitation by others.D. trade secrets.

C. imitation by others.

18. Suppose that Marlen Fisher has legal protection against anyone producing and selling a fishing lure identical to his unique-action "MarFish" lure, whatever the competitor might name the lure. This legal protection is most likely to be a: A. trademark.B. restraining order.C. patent.D. copyright.

C. patent.

19. Suppose that Marlen Fisher has legal protection against anyone producing and selling a fishing lure specifically named "MarFish." This legal protection is most likely to be a: A. trademark.B. restraining order.C. patent.D. copyright.

A. trademark.

20. Suppose that Book-Cost Busters (BCB), without authorization, reproduced a best-selling novel and placed it for downloading on the BCB pay-for-use website. This action would violate the publisher's: A. profit rights.

Page 743: Microeconomics Study Guide

B. patent.C. copyright.D. trademark.

C. copyright.

21. Suppose that a firm's legal staff concludes that a new production process which a firm is developing is patentable. Graphically, this new information would shift the firm's expected rate of return curve on R&D to the: A. right and reduce its optimal amount of R&D.B. right and increase its optimal amount of R&D.C. left and increase its optimal amount of R&D.D. left and reduce its optimal amount of R&D.

B. right and increase its optimal amount of R&D.

22. Even where imitation is possible, a firm may gain advantage from being the first to introduce an innovative product because of: A. long-lasting brand-name recognition.B. a time lag between innovation and imitation by rivals.C. trade secrets that limit the ability of rivals to imitate the product.D. all of these.

D. all of these.

23. Which of the following supports the contention that pure competitors have a weak incentive to engage in R&D? A. Entry to purely competitive industries is easy and thus profit from innovation is quickly competed away.B. In pure competition, products are already highly differentiated.C. Most purely competitive industries are increasing-cost industries.D. Pure competitors are happy to earn only a normal profit.

A. Entry to purely competitive industries is easy and thus profit from innovation is quickly competed away.

24. Economists who contend that oligopolists have a strong incentive to engage in R&D say that: A. the undistributed profits of oligopolists give them a source of readily available, relatively low cost funds for financing R&D.B. entry barriers enable oligopolists to sustain the profit it gains from innovation.C. the large size of oligopolists' R&D departments allows them to use very specialized, expensive R&D equipment and employ teams of specialized researchers.D. all of these are true.

D. all of these are true.

Page 744: Microeconomics Study Guide

25. The process by which new firms and new products replace existing dominant firms and products is called: A. monopolistic competition.B. the inverted-U process.C. process innovation.D. creative destruction.

D. creative destruction.

26. Creative destruction is not automatic because: A. there are major obstacles to the entry of new innovative firms into concentrated industries.B. consumer tastes are highly unstable.C. corporate takeovers increase dynamic competition.D. large firms rarely are technologically progressive.

A. there are major obstacles to the entry of new innovative firms into concentrated industries.

Page 745: Microeconomics Study Guide

UNIT 4 PRACTCE TESTDirections: Do this test quickly and skip the questions you do not know the answers to. This will allow you to isolate

the topics that you need to devote the most time to studying and give us more time in class to review them.1. In which of the following industry structures is the entry of new firms the most difficult? A. pure monopoly B. oligopoly C. monopolistic competition D. pure competition2. An industry comprised of a very large number of sellers producing a standardized product is known as: A. monopolistic competition. B. oligopoly. C. pure monopoly. D. pure competition.3. Which of the following is not a characteristic of pure competition? A. price strategies by firms B. a standardized product C. no barriers to entry D. a larger number of sellers4. Which of the following is not a basic characteristic of pure competition? A. considerable nonprice competition B. no barriers to the entry or exodus of firmsC. a standardized or homogeneous product D. a large number of buyers and sellers In answering the next question(s), assume a graph in which dollars are measured on the vertical axis and output on the horizontal axis.5. Refer to the above information. For a purely competitive firm, total revenue: A. graphs as a straight, upsloping line. B. is a straight line, parallel to the vertical axis.C. is a straight line, parallel to the horizontal axis. D. graphs as a straight, downsloping line.6. Refer to the above information. For a purely competitive firm, marginal revenue: A. graphs as a straight, upsloping line. B. is a straight line, parallel to the vertical axis.C. is a straight line, parallel to the horizontal axis. D. graphs as a straight, downsloping line.7. Refer to the above information. For a purely competitive firm: A. marginal revenue will graph as an upsloping line.B. the demand curve will lie above the marginal revenue curve.C. the marginal revenue curve will lie above the demand curve.D. the demand and marginal revenue curves will coincide.8. Price is constant or given to the individual firm selling in a purely competitive market because: A. the firm's demand curve is downsloping. B. of product differentiation reinforced by extensive advertising.C. each seller supplies a negligible fraction of total supply. D. there are no good substitutes for its product.9. For a purely competitive seller, price equals: A. average revenue. B. marginal revenue. C. total revenue divided by output. D. all of these.10. A perfectly elastic demand curve implies that the firm: A. must lower price to sell more output.B. can sell as much output as it chooses at the existing price.C. realizes an increase in total revenue which is less than product price when it sells an extra unit.D. is selling a differentiated (heterogeneous) product.11. A purely competitive firm should produce in the short run if its total revenue is sufficient to cover its: A. total variable costs.B. total costs.C. total fixed costs.D. marginal costs.12. If a firm is confronted with economic losses in the short run, it will decide whether or not to produce by comparing: A. marginal revenue and marginal cost. B. price and minimum average variable cost.C. total revenue and total cost. D. total revenue and total fixed cost.

Page 746: Microeconomics Study Guide

13. Curve (1) in the above diagram is a purely competitive firm's: A. total cost curve. B. total revenue curve.C. marginal revenue curve. D. total economic profit curve.14. Curve (2) in the above diagram is a purely competitive firm's: A. total cost curve. B. total revenue curve.C. marginal revenue curve. D. total economic profit curve.15. Curve (3) in the above diagram is a purely competitive firm's: A. total cost curve. B. total revenue curve.C. marginal revenue curve. D. total economic profit curve.16. Curve (4) in the above diagram is a purely competitive firm's: A. total cost curve. B. total revenue curve.C. marginal revenue curve. D. total profit curve.17. Refer to the above diagram. Other things equal, an increase of product price would be shown as: A. an increase in the steepness of curve (3), an upward shift in curve (2), and upward shift in curve (1).B. a decrease in the steepness of curve (3), a downward shift in curve (2), and an upward shift in curve (1).C. an downward shift in curve (4) and an upward shift in curve (1), with no changes in lines (2) and (3).D. an upward shift in line (2) only.18. In the short run the individual competitive firm's supply curve is that segment of the: A. average variable cost curve lying below the marginal cost curve.B. marginal cost curve lying above the average variable cost curve.C. marginal revenue curve lying below the demand curve.D. marginal cost curve lying between the average total cost and average variable cost curves. 19. In the long run a pure monopolist will maximize profits by producing that output at which marginal cost is equal to: A. average total cost. B. marginal revenue. C. average variable cost. D. average cost. 20. Economic profit in the long run is: A. possible for both a pure monopoly and a pure competitor.B. possible for a pure monopoly, but not for a pure competitor.C. impossible for both a pure monopolist and a pure competitor.D. only possible when barriers to entry are nonexistent.21. If at the MC = MR output, AVC exceeds price: A. new firms will enter this industry.B. the firm should produce the MC = MR output and realize an economic profit.C. the firm should shut down in the short run.D. the firm should expand its plant.

Page 747: Microeconomics Study Guide

Answer the next questions on the basis of the following cost data for a firm selling in a purely competitive market:

22. Refer to the above data. If the market price for the firm's product is $12, the competitive firm will produce: A. 4 units at a loss of $109.B. 4 units at an economic profit of $31.75.C. 8 units at a loss of $48.80.D. zero units at a loss of $100.23. Refer to the above data. If the market price for the firm's product is $32, the competitive firm will produce: A. 8 units at an economic profit of $16.B. 5 units at a loss of $10.C. 8 units at a loss equal to the firm's total fixed cost.D. 7 units at an economic profit of $41.50.24. Refer to the above data. If the market price for the firm's product is $28, the competitive firm will: A. produce 4 units at a loss of $17.40.B. produce 7 units at a loss of $14.00.C. close down in the short run.D. produce 6 units at a loss of $23.80.

25. Refer to the above diagram. This firm will earn only a normal profit if product price is: A. P1. B. P2. C. P3. D. P4.26. Refer to the above diagram. The firm will realize an economic profit if price is: A. P1. B. P2. C. P3. D. P4.

Page 748: Microeconomics Study Guide

27. Refer to the above diagram. The firm will produce at a loss if price is: A. less than P1. B. P2. C. P3. D. P4.28. Refer to the above diagram. The firm will shut down at any price less than: A. P1. B. P2. C. P3. D. P4.29. Refer to the above diagram. The firm's supply curve is the segment of the: A. MC curve above its intersection with the AVC curve. B. MC curve above its intersection with the ATC curve.C. AVC curve above its intersection with the MC curve. D. ATC curve above its intersection with the MC curve.30. If the price of product Y is $25 and its marginal cost is $18: A. Y is being produced with the least-cost combination of resources.B. society will realize a net gain if less of Y is produced.C. resources are being underallocated to Y.D. resources are being overallocated to Y.31. Pure monopoly means: A. any market in which the demand curve to the firm is downsloping.B. a standardized product being produced by many firms.C. a single firm producing a product for which there are no close substitutes.D. a large number of firms producing a differentiated product.

32. Refer to the above diagram. This firm is selling in: A. a market in which there are an extremely large number of other firms producing the same product.B. an imperfectly competitive market.C. a market in which demand is elastic at all prices.D. a purely competitive market.33. Refer to the above diagram. Demand is relatively elastic: A. in the P2P1 price range. B. in the 0P1 price range. C. in the P2P4 price range. D. only at price P2.34. Refer to the above diagram. Demand is relatively inelastic: A. at price P3. B. at any price below P2. C. in the P2P4 price range. D. in the P2P3 range.35. Refer to the above diagram. If this somehow was a costless product (that is, the total cost of any level of output was zero), the firm would maximize profits by: A. selling the product at the highest possible price at which a positive quantity will be demanded.B. producing Q1 units and charging a price of P1.C. producing Q3 units and charging a price of P3.D. producing Q2 units and charging a price of P2.

Page 749: Microeconomics Study Guide

36. Refer to the above diagram. To maximize profits or minimize losses this firm should produce: A. E units and charge price C. B. E units and charge price A.C. M units and charge price N. D. L units and charge price LK.37. Refer to the above diagram. At the profit-maximizing level of output, total revenue will be: A. NM times 0M. B. 0AJE. C. 0EGC. D. 0EHB.38. Refer to the above diagram. At the profit-maximizing level of output, total cost will be: A. NM times 0M. B. 0AJE. C. 0CGC. D. 0BHE.39. Refer to the above diagram. At the profit-maximizing level of output, the firm will realize: A. an economic profit of ABHJ. B. an economic profit of ACGJ. C. a loss of GH per unit. D. a loss of JH per unit.40. If profits are maximized (or losses minimized), which of the following conditions are common to both unregulated monopoly and to pure competition? A. MC = P B. MC = ATC C. MR = MC D. P = MR41. In which one of the following market models is X-inefficiency most likely to be the greatest? A. pure competitionB. oligopolyC. monopolistic competitionD. pure monopoly42. The dilemma of regulation refers to the idea that: A. the regulated price which achieves allocative efficiency is also likely to result in persistent economic profits.B. the regulated price which results in a "fair return" restricts output by more than would unregulated monopoly.C. regulated pricing always conflicts with the "due process" provision of the Constitution.D. the regulated price which achieves allocative efficiency is also likely to result in losses.43. Monopolistic competition is characterized by a: A. few dominant firms and low entry barriers. B. large number of firms and substantial entry barriers.C. large number of firms and low entry barriers. D. few dominant firms and substantial entry barriers.44. A significant difference between a monopolistically competitive firm and a purely competitive firm is that the: A. former does not seek to maximize profits.B. latter recognizes that price must be reduced to sell more output.C. former sells similar, although not identical, products.D. former's demand curve is perfectly inelastic.45. The monopolistic competition model assumes that: A. allocative efficiency will be achieved. B. productive efficiency will be achieved.C. firms will engage in nonprice competition. D. firms will realize economic profits in the long run.

Page 750: Microeconomics Study Guide

46. In the long run, new firms will enter a monopolistically competitive industry: A. provided economies of scale are being realized.B. even though losses are incurred in the short run.C. until minimum average total cost is achieved.D. until economic profits are zero.47. Other things equal, if more firms enter a monopolistically competitive industry: A. the demand curves facing existing firms would shift to the right.B. the demand curves facing existing firms would shift to the left.C. the demand curves facing existing firms would become less elastic.D. losses would necessarily occur.48. In which of these continuums of degrees of competition (highest to lowest) is oligopoly properly placed? A. pure competition, oligopoly, pure monopoly, monopolistic competitionB. oligopoly, pure competition, monopolistic competition, pure monopolyC. monopolistic competition, pure competition, pure monopoly, oligopolyD. pure competition, monopolistic competition, oligopoly, pure monopoly49. Oligopolistic industries are characterized by: A. a few dominant firms and substantial entry barriers.B. a few dominant firms and no barriers to entry.C. a large number of firms and low entry barriers.D. a few dominant firms and low entry barriers.50. Concentration ratios measure the: A. geographic location of the largest corporations in each industry.B. degree to which product price exceeds marginal cost in various industries.C. percentage of total sales accounted for by the four largest firms in the industry.D. number of firms in an industry.51. Oligopolistic firms engage in collusion to: A. minimize unit costs of production.B. realize allocative efficiency, that is, the P = MC level of output.C. earn greater profits.D. increase production.

1. For economists, the word "utility" means: A. versatility and flexibility.B. rationality.C. pleasure or satisfaction.D. purposefulness.

Page 751: Microeconomics Study Guide

 

2. Joe sold gold coins for $1000 that he bought a year ago for $1000. He says, "At least I didn't lose any money on my financial investment." His economist friend points out that in effect he did lose money, because he could have received a 3 percent return on the $1000 if he had bought a bank certificate of deposit instead of the coins. The economist's analysis in this case incorporates the idea of: A. opportunity costs.B. marginal benefits that exceed marginal costs.C. imperfect information.D. normative economics.  

3. The assertion that "There is no free lunch" means that: A. there are always tradeoffs between economic goals.B. all production involves the use of scarce resources and thus the sacrifice of alternative goods.C. marginal analysis is not used in economic reasoning.D. choices need not be made if behavior is rational.  

4. The budget line shows: A. the amount of product A that a consumer is willing to give up to obtain one more unit of product B.B. all possible combinations of two goods that can be purchased, given money income and the prices of the goods.C. the minimum amount of two goods that a consumer can purchase with a given money income.D. all possible combinations of two goods that yield the same level of utility to the consumer.

      5. Refer to the budget line shown in the diagram above. If the consumer's money income is $20, the: A. prices of C and D cannot be determined.B. price of C is $2 and the price of D is $4.C. consumer can obtain a combination of 5 units of both C and D.D. price of C is $4 and the price of D is $2. 

6. Any point inside the production possibilities curve indicates: A. the presence of technological change.B. that resources are imperfectly substitutable among alternative uses.C. the presence of inflationary pressures.D. that more output could be produced with available resources.

Page 752: Microeconomics Study Guide

 7. Command systems are also known as: A. market systems.B. pure capitalism.C. laissez-faire capitalism.D. communism. 

8. The two basic markets shown by the simple circular flow model are: A. capital goods and consumer goods.B. free and controlled.C. product and resource.D. household and business. 

9. The law of demand states that, other things equal: A. price and quantity demanded are inversely related.B. the larger the number of buyers in a market, the lower will be product price.C. price and quantity demanded are directly related.D. consumers will buy more of a product at high prices than at low prices. 

10. The income and substitution effects account for: A. the upward sloping supply curve.B. the downward sloping demand curve.C. movements along a given supply curve.D. shifts in the demand curve. 

11. Which of the following would not shift the demand curve for beef? A. a widely publicized study that indicates beef increases one's cholesterolB. a reduction in the price of cattle feedC. an effective advertising campaign by pork producersD. a change in the incomes of beef consumers 

12. If the demand curve for product B shifts to the right as the price of product A declines, then: A. both A and B are inferior goods.B. A is a superior good and B is an inferior good.C. A is an inferior good and B is a superior good.D. A and B are complementary goods. 

13. If X is a normal good, a rise in money income will shift the: A. supply curve for X to the left.B. supply curve for X to the right.C. demand curve for X to the left.D. demand curve for X to the right. 

14. A decrease in the price of digital cameras will: A. cause the demand curve for memory cards to become vertical.B. shift the demand curve for memory cards to the right.C. shift the demand curve for memory cards to the left.D. not affect the demand for memory cards. 

15. Assume that the demand curve for product C is downsloping. If the price of C falls from $2.00 to $1.75: A. a smaller quantity of C will be demanded.B. a larger quantity of C will be demanded.C. the demand for C will increase.D. the demand for C will decrease. 

Page 753: Microeconomics Study Guide

16. A leftward shift of a product supply curve might be caused by: A. an improvement in the relevant technique of production.B. a decline in the prices of needed inputs.C. an increase in consumer incomes.D. some firms leaving an industry. 

17. A government subsidy to the producers of a product: A. reduces product supply.B. increases product supply.C. reduces product demand.D. increases product demand. 

18. Suppose that corn prices rise significantly. If farmers expect the price of corn to continue rising relative to other crops, then we would expect: A. the supply of ethanol, a corn-based product, to increase.B. consumer demand for wheat to fall.C. the supply to increase as farmers plant more corn.D. the supply to fall as farmers plant more of other crops. 

     

19. Refer to the above table. If demand is represented by columns (3) and (2) and supply is represented by columns (3) and (5), equilibrium price and quantity will be: A. $10 and 60 units.B. $9 and 50 units.C. $8 and 60 units.D. $7 and 50 units. 

20. Refer to the above table. Suppose that demand is represented by columns (3) and (2) and supply is represented by columns (3) and (5). If the price were artificially set at $9, A. the market would clear.B. a surplus of 20 units would occur.C. a shortage of 20 units would occur.D. demand would change from columns (3) and (2) to columns (3) and (1).

      21. Refer to the above diagram. The equilibrium price and quantity in this market will be: A. $1.00 and 200.B. $1.60 and 130.C. $0.50 and 130.D. $1.60 and 290. 

Page 754: Microeconomics Study Guide

22. Refer to the above diagram. A surplus of 160 units would be encountered if the price was: A. $1.10, that is, $1.60 minus $.50.B. $1.60.C. $1.00.D. $0.50. 

23. Refer to the above diagram. A shortage of 160 units would be encountered if price was: A. $1.10, that is, $1.60 minus $.50.B. $1.60.C. $1.00.D. $0.50. 

24. If there is a surplus of a product, its price: A. is below the equilibrium level.B. is above the equilibrium level.C. will rise in the near future.D. is in equilibrium. 

    25. Refer to the above diagram. A price of $60 in this market will result in: A. equilibrium.B. a shortage of 50 units.C. a surplus of 50 units.D. a surplus of 100 units. 

26. Refer to the above diagram. A price of $20 in this market will result in a: A. shortage of 50 units.B. surplus of 50 units.C. surplus of 100 units.D. shortage of 100 units. 

27. Assume in a competitive market that price is initially below the equilibrium level. We can predict that price will: A. decrease, quantity demanded will decrease, and quantity supplied will increase.B. decrease and quantity demanded and quantity supplied will both decrease.C. increase, quantity demanded will increase, and quantity supplied will decrease.D. increase, quantity demanded will decrease, and quantity supplied will increase. 

28. If the price elasticity of demand for a product is 2.5, then a price cut from $2.00 to $1.80 will: A. increase the quantity demanded by about 2.5 percent.B. decrease the quantity demanded by about 2.5 percent.C. increase the quantity demanded by about 25 percent.D. increase the quantity demanded by about 250 percent. 

Page 755: Microeconomics Study Guide

29. Suppose that as the price of Y falls from $2.00 to $1.90 the quantity of Y demanded increases from 110 to 118. Then the price elasticity of demand is: A. 4.00.B. 2.09.C. 1.37.D. 3.94. 

30. The price elasticity of demand of a straight-line demand curve is: A. elastic in high-price ranges and inelastic in low-price ranges.B. elastic, but does not change at various points on the curve.C. inelastic, but does not change at various points on the curve.D. 1 at all points on the curve. 

31. The price elasticity of demand is generally: A. negative, but the minus sign is ignored.B. positive, but the plus sign is ignored.C. positive for normal goods and negative for inferior goods.D. positive because price and quantity demanded are inversely related. 

32. If a demand for a product is elastic, the value of the price elasticity coefficient is: A. zero.B. greater than one.C. equal to one.D. less than one.

      33. Refer to the above diagram. Between prices of $5.70 and $6.30: A. D1 is more elastic than D2.B. D2 is an inferior good and D1 is a normal good.C. D1 and D2 have identical elasticities.D. D2 is more elastic than D1. 

34. Refer to the above diagram and assume a single good. If the price of the good decreases from $6.30 to $5.70, consumer expenditure would: A. decrease if demand were D1 only.B. decrease if demand were D2 only.C. decrease if demand were either D1 or D2.D. increase if demand were either D1 or D2. 

35. Refer to the above diagram and assume a single good. If the price of the good increased from $5.70 to $6.30 along D1, the price elasticity of demand along this portion of the demand curve would be: A. 0.8.B. 1.0.C. 1.2.D. 2.0.

Page 756: Microeconomics Study Guide

 36. Suppose the price of local cable TV service increased from $16.20 to $19.80 and as a result the number of cable subscribers decreased from 224,000 to 176,000. Along this portion of the demand curve, price elasticity of demand is: A. 0.8.B. 1.2.C. 1.6.D. 8.0 

37. A firm can sell as much as it wants at a constant price. Demand is thus: A. perfectly inelastic.B. perfectly elastic.C. relatively inelastic.D. relatively elastic. 

38. Suppose the price elasticity coefficients of demand are 1.43, 0.67, 1.11, and 0.29 for products W, X, Y, and Z respectively. A 1 percent decrease in price will increase total revenue in the case(s) of: A. W and Y.B. Y and Z.C. X and Z.D. Z and W. 

39. If a firm finds that it can sell $13,000 worth of a product when its price is $5 per unit and $11,000 worth of it when its price is $6, then: A. the demand for the product is elastic in the $6-$5 price range.B. the demand for the product must have increased.C. elasticity of demand is 0.74.D. the demand for the product is inelastic in the $6-$5 price range. 

40. Suppose the price elasticity of demand for bread is 0.20. If the price of bread falls by 10 percent, the quantity demanded will increase by: A. 2 percent and total expenditures on bread will rise.B. 2 percent and total expenditures on bread will fall.C. 20 percent and total expenditures on bread will fall.D. 20 percent and total expenditures on bread will rise. 

41. Gigantic State University raises tuition for the purpose of increasing its revenue so that more faculty can be hired. GSU is assuming that the demand for education at GSU is: A. decreasing.B. relatively elastic.C. perfectly elastic.D. relatively inelastic. 

42. Suppose the income elasticity of demand for toys is +2.00. This means that: A. a 10 percent increase in income will increase the purchase of toys by 20 percent.B. a 10 percent increase in income will increase the purchase of toys by 2 percent.C. a 10 percent increase in income will decrease the purchase of toys by 2 percent.D. toys are an inferior good. 

43. We would expect the cross elasticity of demand between dress shirts and ties to be: A. positive, indicating normal goods.B. positive, indicating complementary goods.C. negative, indicating substitute goods.D. negative, indicating complementary goods. 

Page 757: Microeconomics Study Guide

44. We would expect the cross elasticity of demand between Pepsi and Coke to be: A. positive, indicating normal goods.B. positive, indicating inferior goods.C. positive, indicating substitute goods.D. negative, indicating substitute goods. 

45. Assume that a 3 percent increase in income across the economy produces a 1 percent decline in the quantity demanded of good X. The coefficient of income elasticity of demand for good X is: A. negative and therefore X is an inferior good.B. negative and therefore X is a normal good.C. positive and therefore X is an inferior good.D. positive and therefore X is a normal good. 

46. The two main characteristics of a public good are: A. production at constant marginal cost and rising demand.B. nonexcludability and production at rising marginal cost.C. nonrivalry and nonexcludability.D. nonrivalry and large negative externalities. 

47. Unlike a private good, a public good: A. has no opportunity costs.B. has benefits available to all, including nonpayers.C. produces no positive or negative externalities.D. is characterized by rivalry and excludability. 

48. Nonexcludability describes a condition where: A. one person's consumption of a good does not prevent consumption of the good by others.B. there is no effective way to keep people from using a good once it comes into being.C. sellers can withhold the benefits of a good from those unwilling to pay for it.D. there is no potential for free-riding behavior. 

49. At the optimal quantity of a public good: A. marginal benefit exceeds marginal cost by the greatest amount.B. total benefit equals total cost.C. marginal benefit equals marginal cost.D. marginal benefit is zero. 

50. The ability of a good or service to satisfy wants is called: A. utility maximization.B. opportunity cost.C. revenue potential.D. utility. 

51. Marginal utility is the: A. sensitivity of consumer purchases of a good to changes in the price of that good.B. change in total utility obtained by consuming one more unit of a good.C. change in total utility obtained by consuming another unit of a good divided by the change in the price of that good.D. total utility associated with the consumption of a certain number of units of a good divided by the number of units consumed. 

Page 758: Microeconomics Study Guide

52. To maximize utility a consumer should allocate money income so that the: A. elasticity of demand on all products purchased is the same.B. marginal utility obtained from the last dollar spent on each product is the same.C. total utility derived from each product consumed is the same.D. marginal utility of the last unit of each product consumed is the same. 

 Answer the question on the basis of the following total utility data for products L and M. Assume that the prices of L and M are $3 and $4 respectively and that the consumer's income is $18.

    53. Refer to the above data. How many units of the two products will the rational consumer purchase? A. 3 of L and none of MB. 4 of L and 2 of MC. 3 of L and 5 of MD. 2 of L and 3 of M 

54. Refer to the above data. What level of total utility does the rational consumer realize in equilibrium? A. 87 utilsB. 104 utilsC. 51 utilsD. 58 utils 

55. Diminishing marginal utility explains why: A. the income effect exceeds the substitution effect.B. the substitution effect exceeds the income effect.C. supply curves are upsloping.D. demand curves are downsloping. 

56. Why do people tend to eat more at all-you-can-eat buffet restaurants than at restaurants where each item is purchased separately? A. Once the all-you-can-eat meal is purchased, consumers view additional trips back to the buffet as having a price of zero.B. MU/P is greater at all-you-can-eat restaurants.C. People who eat at all-you-can-eat restaurants do not experience diminishing marginal utility.D. Food at all-you-can-eat restaurants tends to have fewer calories, so consumers feel the need to consume a greater volume of food. 

57. To the economist, total cost includes: A. explicit and implicit costs, including a normal profit.B. neither implicit nor explicit costs.C. implicit, but not explicit, costs.D. explicit, but not implicit, costs. 

58. Suppose that a business incurred implicit costs of $500,000 and explicit costs of $5 million in a specific year. If the firm sold 100,000 units of its output at $50 per unit, its accounting: A. profits were $100,000 and its economic profits were zero.B. losses were $500,000 and its economic losses were zero.C. profits were $500,000 and its economic profits were $1 million.D. profits were zero and its economic losses were $500,000. 

Page 759: Microeconomics Study Guide

59. The basic difference between the short run and the long run is that: A. all costs are fixed in the short run, but all costs are variable in the long run.B. the law of diminishing returns applies in the long run, but not in the short run.C. at least one resource is fixed in the short run, while all resources are variable in the long run.D. economies of scale may be present in the short run, but not in the long run. 

 Answer the question on the basis of the following output data for a firm. Assume that the amounts of all non-labor resources are fixed.

    60. Refer to the above data. Diminishing marginal returns become evident with the addition of the: A. sixth worker.B. fourth worker.C. third worker.D. second worker. 

61. Refer to the above data. The marginal product of the sixth worker is: A. 180 units of output.B. 30 units of output.C. 15 units of output.D. negative. 

 Use the following data to answer the question:

    62. Refer to the above data. The average product (AP) when two units of labor are hired is: A. 8.B. 9.C. 10.D. 18. 

63. Refer to the above data. Diminishing returns begin to occur with the hiring of the _________ unit of labor. A. firstB. secondC. thirdD. seventh 

64. Refer to the above data. Marginal product becomes negative with the hiring of the __________ unit of labor. A. thirdB. fourthC. sixthD. seventh

Page 760: Microeconomics Study Guide

  Answer the question on the basis of the following cost data:

    65. Refer to the above data. Total fixed cost is: A. $6.25.B. $100.00.C. $150.00.D. $50.00. 

66. Refer to the above data. The average total cost of five units of output is: A. $69.B. $78.C. $3.D. $10. 

67. Refer to the above data. The total cost of four units of output is: A. $260.B. $77.50.C. $310.D. $215. 

68. Refer to the above data. If the firm closed down in the short run and produced zero units of output, its total cost would be: A. zero.B. $50.C. $150.D. $100. 

69. Refer to the above data. The marginal cost of the fifth unit of output is: A. $3.B. $62.C. $80.D. $78. 

70. In which of the following market structures is there clear-cut mutual interdependence with respect to price-output policies? A. pure monopolyB. oligopolyC. monopolistic competitionD. pure competition 

71. An industry comprised of 40 firms, none of which has more than 3 percent of the total market for a differentiated product is an example of: A. monopolistic competition.B. oligopoly.C. pure monopoly.D. pure competition.

Page 761: Microeconomics Study Guide

 72. An industry comprised of a small number of firms, each of which considers the potential reactions of its rivals in making price-output decisions is called: A. monopolistic competition.B. oligopoly.C. pure monopoly.D. pure competition. 

73. A purely competitive seller is: A. both a "price maker" and a "price taker."B. neither a "price maker" nor a "price taker."C. a "price taker."D. a "price maker."  

74. For a purely competitive seller, price equals: A. average revenue.B. marginal revenue.C. total revenue divided by output.D. all of these.

      75. Refer to the above short-run data. The profit-maximizing output for this firm is: A. above 440 units.B. 440 units.C. 320 units.D. 100 units. 

76. In the short run the individual competitive firm's supply curve is that segment of the: A. average variable cost curve lying below the marginal cost curve.B. marginal cost curve lying above the average variable cost curve.C. marginal revenue curve lying below the demand curve.D. marginal cost curve lying between the average total cost and average variable cost curves. 

77. Suppose you find that the price of your product is less than minimum AVC. You should: A. minimize your losses by producing where P = MC.B. maximize your profits by producing where P = MC.C. close down because, by producing, your losses will exceed your total fixed costs.D. close down because total revenue exceeds total variable cost. 

 Answer the question on the basis of the following data confronting a firm:

   

Page 762: Microeconomics Study Guide

 78. Refer to the above data. This firm is selling its output in a(n): A. monopolistically competitive market.B. monopolistic market.C. purely competitive market.D. oligopolistic market. 

79. Refer to the above data. If the firm's minimum average variable cost is $10, the firm's profit-maximizing level of output would be: A. 2.B. 3.C. 4.D. 5. 

80. Refer to the above data. At the profit-maximizing output the firm's total revenue is: A. $48.B. $32.C. $80.D. $64. 

81. Refer to the above data. Assuming total fixed costs equal to zero, the firm's: A. economic profit is $12.B. economic profit is $16.C. loss is $14.D. economic profit is $3. 

82. A firm finds that at its MR = MC output, its TC = $1,000, TVC = $800, TFC = $200, and total revenue is $900. This firm should: A. shut down in the short run.B. produce because the resulting loss is less than its TFC.C. produce because it will realize an economic profit.D. liquidate its assets and go out of business.

       83. Refer to the above diagram for a purely competitive producer. The lowest price at which the firm should produce (as opposed to shutting down) is: A. P1.B. P2.C. P3.D. P4. 

84. Refer to the above diagram for a purely competitive producer. The firm will produce at a loss at all prices: A. above P1.B. above P3.C. above P4.D. between P2 and P3.

Page 763: Microeconomics Study Guide

85. Refer to the above diagram for a purely competitive producer. If product price is P3: A. the firm will maximize profit at point d.B. the firm will earn an economic profit.C. economic profits will be zero.D. new firms will enter this industry. 

 Answer the question on the basis of the following cost data for a firm that is selling in a purely competitive market:

    86. Refer to the above data. If the market price for the firm's product is $12, the competitive firm will produce: A. 4 units at a loss of $109.B. 4 units at an economic profit of $31.75.C. 8 units at a loss of $48.80.D. zero units at a loss of $100. 

87. Refer to the above data. If the market price for the firm's product is $32, the competitive firm will produce: A. 8 units at an economic profit of $16.B. 6 units at an economic profit of $7.98.C. 10 units at an economic profit of $4.D. 7 units at an economic profit of $41.50. 

88. Refer to the above data. If the market price for the firm's product is $28, the competitive firm will: A. produce 4 units at a loss of $17.40.B. produce 7 units at a loss of $14.00.C. shut down in the short run.D. produce 6 units at a loss of $23.80. 

89. In a purely competitive industry: A. there will be no economic profits in either the short run or the long run.B. economic profits may persist in the long run if consumer demand is strong and stable.C. there may be economic profits in the short run, but not in the long run.D. there may be economic profits in the long run, but not in the short run. 

90. If a purely competitive constant-cost industry is realizing economic profits, we can expect industry supply to: A. increase, output to increase, price to decrease, and profits to decrease.B. increase, output to increase, price to increase, and profits to decrease.C. decrease, output to decrease, price to increase, and profits to increase.D. increase, output to decrease, price to decrease, and profits to decrease. 

Page 764: Microeconomics Study Guide

91. Which of the following conditions is true for a purely competitive firm in long-run equilibrium? A. P > MC = minimum ATC.B. P > MC > minimum ATC.C. P = MC = minimum ATC.D. P < MC < minimum ATC. 

92. Which of the following is correct? A. Both purely competitive and monopolistic firms are "price takers."B. Both purely competitive and monopolistic firms are "price makers."C. A purely competitive firm is a "price taker," while a monopolist is a "price maker."D. A purely competitive firm is a "price maker," while a monopolist is a "price taker." 

93. Which of the following is a characteristic of pure monopoly? A. close substitute productsB. barriers to entryC. the absence of market powerD. "price taking" 

 Answer the question on the basis of the demand schedule shown below:

   94. Refer to the above data. The marginal revenue obtained from selling the third unit of output is: A. $6.B. $1.C. $3.D. $5. 

95. Which of the following is characteristic of a pure monopolist's demand curve? A. Average revenue is less than price.B. Its elasticity coefficient is 1 at all levels of output.C. Price and marginal revenue are equal at all levels of output.D. It is the same as the market demand curve. 

96. Because the monopolist's demand curve is downsloping: A. MR will equal price.B. price must be lowered to sell more output.C. the elasticity coefficient will increase as price is lowered.D. its supply curve will also be downsloping. 

97. An unregulated pure monopolist will maximize profits by producing that output at which: A. P = MC.B. P = ATC.C. MR = MC.D. MC = AC. 

Page 765: Microeconomics Study Guide

98. Suppose that a pure monopolist can sell 5 units of output at $4 per unit and 6 units at $3.90 per unit. The monopolist will produce and sell the sixth unit if its marginal cost is: A. $4 or less.B. $3.90 or less.C. $3.50 or less.D. $3.40 or less. 

99. A pure monopolist is producing an output such that ATC = $4, P = $5, MC = $2, and MR = $3. This firm is realizing: A. a loss that could be reduced by producing more output.B. a loss that could be reduced by producing less output.C. an economic profit that could be increased by producing more output.D. an economic profit that could be increased by producing less output.

      100. Refer to the above diagram. To maximize profits or minimize losses this firm should produce: A. E units and charge price C.B. E units and charge price A.C. M units and charge price N.D. L units and charge price LK. 

101. Refer to the above diagram. At the profit-maximizing level of output, total revenue will be: A. NM times 0M.B. 0AJE.C. 0EGC.D. 0EHB.

102. Refer to the above diagram. At the profit-maximizing level of output, total cost will be: A. NM times 0M.B. 0AJE.C. 0CGC.D. 0BHE. 

103. Refer to the above diagram. At the profit-maximizing level of output, the firm will realize: A. an economic profit of ABHJ.B. an economic profit of ACGJ.C. a loss of GH per unit.D. a loss of JH per unit. 

104. If profits are maximized (or losses minimized), which of the following conditions is common to both unregulated monopoly and to pure competition? A. MC = PB. MC = ATCC. MR = MCD. P = MR 

Page 766: Microeconomics Study Guide

    105. Refer to the above diagram for a pure monopolist. Monopoly price will be: A. e.B. c.C. b.D. a. 

106. Refer to the above diagram for a pure monopolist. Monopoly output will be: A. between f and g.B. h.C. g.D. f. 

     107. Refer to the above diagram. If this industry is comprised of only one seller, the profit-maximizing price and quantity will be: A. P3 and Q3.B. P1 and Q3.C. P2 and Q2.D. indeterminate on the basis of the information given. 

108. Which of the following statements is correct? A. The pure monopolist will maximize profit by producing at that point on the demand curve where elasticity is zero.B. In seeking the profit-maximizing output the pure monopolist underallocates resources to its production.C. The pure monopolist maximizes profits by producing that output at which the differential between price and average cost is the greatest.D. Purely monopolistic sellers earn only normal profits in the long run. 

109. When economists say that the demand for labor is a derived demand, they mean that it is: A. dependent on government expenditures for public goods and services.B. related to the demand for the product or service labor is producing.C. based on the desire of businesses to exploit labor by paying below equilibrium wage rates.D. based on the assumption that workers are trying to maximize their money incomes. 

Page 767: Microeconomics Study Guide

110. Marginal product is: A. the output of the least skilled worker.B. a worker's output multiplied by the price at which each unit can be sold.C. the amount an additional worker adds to the firm's total output.D. the amount any given worker contributes to the firm's total revenue. 

111. Assume labor is the only variable input and that an additional input of labor increases total output from 72 to 78 units. If the product sells for $6 per unit in a purely competitive market, the MRP of this additional worker is: A. $6.B. $12.C. $36.D. $72. 

112. If one worker can pick $30 worth of grapes and two workers together can pick $50 worth of grapes, the: A. marginal revenue product of each worker is $25.B. marginal revenue product of the first worker is $20.C. marginal revenue product of the second worker is $20.D. data given do not permit the determination of the marginal revenue product of either worker.

Answer the question on the basis of the following information for Manfred's Shoe Shine Parlor. Assume Manfred hires labor, its only variable input, under purely competitive conditions. Shoe shines are also sold competitively.

    113. Refer to the above data. How many units of output are produced when 2 workers are employed? A. 4B. 16C. 24D. 10 

114. Refer to the above data. What is the marginal product of the sixth worker? A. 2 unitsB. 3 unitsC. 4 unitsD. 5 units 

115. Refer to the above data. At what price does each shoe shine sell? A. $1B. $2C. $3D. $2.50 

Page 768: Microeconomics Study Guide

116. Refer to the above data. If the wage rate is $11, how many workers will Manfred hire to maximize profits? A. 1B. 2C. 3D. 5 

117. Refer to the above data. If the wage rate is $11 and Manfred's only fixed input is capital, the total cost of which is $30, then what will be his economic profit? A. $62B. $42C. $28D. $32 

118. Assume that a restaurant is hiring labor in an amount such that the MRC of the last worker is $16 and her MRP is $12. On the basis of this information we can say that: A. profits will be increased by hiring additional workers.B. profits will be increased by hiring fewer workers.C. marginal revenue product must exceed average revenue product.D. the restaurant is maximizing profits. 

 Answer the question on the basis of the data contained in the following table. Assume that the firm is hiring labor in a purely competitive market.

    119. Refer to the above data. If the wage rate is $20, how many workers will the firm choose to employ? A. 5B. 4C. 3D. 2 

120. Refer to the above data. If the wage rate is $11, how many workers will the firm choose to employ? A. 5B. 4C. 3D. 2 

121. If the nominal wages of carpenters rose by 5 percent in 2010 and the price level increased by 3 percent, then the real wages of carpenters: A. decreased by 2 percent.B. increased by 2 percent.C. increased by 3 percent.D. increased by 8 percent. 

122. If the nominal wage rises by 4 percent, and the price level rises by 7 percent, the real wage will: A. be unaffected.B. rise by 3 percent.C. fall by 3 percent.D. rise by 11 percent.

Page 769: Microeconomics Study Guide

     123. Refer to the above diagram. Assuming no union or relevant minimum wage, the firm represented will hire: A. Q2 workers and pay a W4 wage rate.B. Q2 workers and pay a W1 wage rate.C. Q3 workers and pay a W2 wage rate.D. Q4 workers and pay a W1 wage rate. 

124. A progressive tax is such that: A. tax rates are higher the greater one's income.B. the same tax rate applies to all income receivers, so that the rich pay absolutely more taxes than the poor.C. entrepreneurial income is exempt from taxation.D. the revenues it yields are spent on transfer payments. 

 The following data represent a personal income tax schedule. Answer the question on the basis of this information.

    125. Refer to the above table. If your taxable income is $8,000, your average tax rate is: A. 25 percent and the marginal rate on additional income is also 25 percent.B. 25 percent and the marginal rate on additional income is 40 percent.C. 25 percent and the marginal rate on additional income cannot be determined from the information given.D. 20 percent and the marginal rate on additional income is 30 percent. 

126. Which of the following best reflects the ability-to-pay philosophy of taxation? A. a tax on residential propertyB. a progressive income taxC. an excise tax on gasolineD. an excise tax on coffee 

127. The incidence of a tax pertains to: A. the degree to which it alters the distribution of income.B. how easy it is to evade the tax.C. who actually bears the burden of a tax.D. the progressiveness or regressiveness of tax rates.

Page 770: Microeconomics Study Guide

1. Other things equal, in which of the following cases would economic profit be the greatest? A. an unregulated monopolist which is able to engage in price discrimination B. an unregulated monopolist C. a regulated monopolist charging a price equal to average total cost D. a regulated monopolist charging a price equal to marginal cost

2. Purely competitive firms and pure monopolists are similar in that: A. the demand curves of both are perfectly elastic. B. significant entry barriers are common to both. C. both are price makers. D. both maximize profit where MR = MC.

3. (Last Word) DeBeers Consolidated Mines markets about:A. 65 percent of the world's rough-cut diamonds.B. 80 percent of the world's rough-cut diamonds.C. 50 percent of the world's rough-cut diamonds.D. 33 percent of the world's rough-cut diamonds.

4. To maximize profit a pure monopolist must: A. maximize its total revenue. B. maximize the difference between marginal revenue and marginal cost. C. maximize the difference between total revenue and total cost. D. produce where average total cost is at a minimum.

5. X-inefficiency refers to a situation in which a firm: A. is not as technologically progressive as it might be. B. encounters diseconomies of scale. C. fails to realize all existing economies of scale. D. fails to achieve the minimum average total costs attainable at each level of output.

6. R-1 F24052

Refer to the above diagram for a nondiscriminating monopolist. Marginal revenue will be zero at output:

A. q4. B. q3. C. q2. D. q1.

Page 771: Microeconomics Study Guide

7. The vertical distance between the horizontal axis and any point on a perfectly discriminating monopolist's demand curve measures:

A. the quantity demanded. B. total revenue. C. product price and marginal revenue. D. average revenue and average total cost.

8. If a monopolist engages in perfect price discrimination, it will: A. realize a smaller profit. B. charge a higher price where individual demand is inelastic and a lower price where individual

demand is elastic. C. produce a smaller output than when it did not discriminate. D. charge a competitive price to all its customers.

9.  R-2 F24167

Refer to the above diagram for a natural monopolist. If a regulatory commission set a maximum price of P1, the monopolist would produce output:

A. Q2 and realize a normal profit. B. Q4 and realize a normal profit. C. Q3 and realize an economic profit. D. Q4 and realize a loss.

10. The nondiscriminating monopolist's demand curve: A. is less elastic than a purely competitive firm's demand curve. B. is perfectly elastic. C. coincides with its marginal revenue curve. D. is perfectly inelastic.

11. What do economies of scale, the ownership of essential raw materials, and patents have in common?

A. They must all be present before price discrimination can be practiced. B. They are all barriers to entry. C. They all help explain why a monopolist's demand and marginal revenue curves coincide. D. They all help explain why the long-run average cost curve is U-shaped.

12.  R-2 F24167

Page 772: Microeconomics Study Guide

Refer to the above diagram for a natural monopolist. If a regulatory commission set a maximum price of P2, the monopolist would:

A. produce output Q1 and realize an economic profit. B. produce output Q3 and realize an economic profit. C. close down in the short run. D. produce output Q3 and realize a normal profit.

13. If a regulatory commission wants to provide a natural monopoly with a fair return, it should establish a price that is equal to:

A. minimum average fixed cost. B. average total cost. C. marginal cost. D. marginal revenue.

14.  R-3 F24090

Refer to the above diagram for a pure monopolist. Monopoly price will be: A. e. B. c. C. b. D. a.

15. Answer the next question(s) on the basis of the following demand and cost data for a pure monopolist:

R-4 REF24078

Refer to the above data. The equilibrium level of output will be: A. 4 units. B. 7 units. C. 6 units. D. 5 units.

16. The pure monopolist's demand curve is: A. identical with the industry demand curve. B. of unit elasticity throughout. C. perfectly inelastic. D. perfectly elastic.

Page 773: Microeconomics Study Guide

17. If a pure monopolist is producing at that output where P = ATC, then: A. its economic profits will be zero. B. it will be realizing losses. C. it will be producing less than the profit-maximizing level of output. D. it will be realizing an economic profit.

18. Answer the next question on the basis of the following table showing the demand schedule facing a nondiscriminating monopolist:

R-5 REF24065

The monopolist will select its profit-maximizing level of output somewhere within the: A. 3-5 unit range of output. B. 1-3 unit range of output. C. 1-4 unit range of output. D. 2-4 unit range of output.

19. A pure monopolist: A. will realize an economic profit if price exceeds ATC at the equilibrium output. B. will realize an economic profit if ATC exceeds MR at the equilibrium output. C. will realize an economic loss if MC intersects the downsloping portion of MR. D. always realizes an economic profit.

20. When the pure monopolist's demand curve is elastic, marginal revenue: A. may be either positive or negative. B. is zero. C. is negative. D. is positive.

21. Pure monopoly means: A. any market in which the demand curve to the firm is downsloping. B. a standardized product being produced by many firms. C. a single firm producing a product for which there are no close substitutes. D. a large number of firms producing a differentiated product.

22. A nondiscriminating pure monopolist's demand curve: A. is perfectly inelastic. B. coincides with its marginal revenue curve.

Page 774: Microeconomics Study Guide

C. lies above its marginal revenue curve. D. lies below its marginal revenue curve.

23. The dilemma of regulation refers to the idea that: A. the regulated price which achieves allocative efficiency is also likely to result in persistent

economic profits. B. the regulated price which results in a "fair return" restricts output by more than would

unregulated monopoly. C. regulated pricing always conflicts with the "due process" provision of the Constitution. D. the regulated price which achieves allocative efficiency is also likely to result in losses.

24.  R-6 F24034

Refer to the above diagram. Demand is relatively inelastic: A. at price P3. B. at any price below P2. C. in the P2P4 price range. D. in the P2P3 price range.

25. Answer the next question(s) on the basis of the following demand and cost data for a pure monopolist:

R-4 REF24078

Refer to the above data. Equilibrium price for the monopolist will be: A. $5.00 B. $2.90. C. $3.35. D. $4.50.

26. The demand curve faced by a pure monopolist: A. may be either more or less elastic than that faced by a single purely competitive firm. B. is less elastic than that faced by a single purely competitive firm. C. has the same elasticity as that faced by a single purely competitive firm. D. is more elastic than that faced by a single purely competitive firm.

27. Suppose that a pure monopolist can sell 10 units of output at $5 per unit and 11 units at $4.90 per unit. The marginal revenue of the eleventh unit is:

Page 775: Microeconomics Study Guide

A. $3.90.B. $.10.C. $53.90.D. $4.90.

28. The marginal revenue curve for a monopolist:A. is a straight, upward sloping curve.B. rises at first, reaches a maximum, and then declines.C. becomes negative when output increases beyond some particular level.D. is a straight line, parallel to the horizontal axis.

29. The MR = MC rule: A. applies only to pure competition. B. applies only to pure monopoly. C. does not apply to pure monopoly because price exceeds marginal revenue. D. applies both to pure monopoly and pure competition.

30.  R-7 F24027

Refer to the above two diagrams for individual firms. In Figure 2 the firm's demand and marginal revenue curves are represented by:

A. lines B and C respectively. B. lines A and C respectively. C. lines A and B respectively.D. line B.

1. In an oligopolistic market:A. one firm is always dominant.B. products may be standardized or differentiated.C. the four largest firms account for 20 percent or less of total sales.D. the industry is monopolistically competitive.

2. Clear-cut mutual interdependence with respect to the price-output policies exists in:A. pure monopolyB. oligopoly

Page 776: Microeconomics Study Guide

C. monopolistic competitionD. pure competition

3. Assume six firms comprising an industry have market shares of 30, 30, 10, 10, 10, and 10 percent. The Herfindahl Index for this industry:

A. is 2,525.B. is 1,600.C. is 2,200.D. is 80.E. cannot be determined from the information given.

4.

R-1 F26063

The above diagram portrays:

A. pure competition.B. monopolistic competition.C. noncollusive oligopoly.D. pure monopoly.E. collusive oligopoly.

Page 777: Microeconomics Study Guide

5.

R-1 F26063

Refer to the above diagram. Equilibrium output is:

A. j.B. h.C. g.D. f .

Page 778: Microeconomics Study Guide

6.

R-1 F26063

Refer to the above diagram. Equilibrium price is:

A. e.B. d.C. c.D. b.E. a.

Page 779: Microeconomics Study Guide

7.

R-1 F26063

Refer to the above diagram. This firm's demand and marginal revenue curves are based on the assumption that:

A. the firm has no immediate rivals.B. rivals will match both a price increase and a price decrease.C. rivals will match a price increase, but ignore a price decrease.D. rivals will ignore a price increase, but match a price decrease.

Page 780: Microeconomics Study Guide

8.

R-1 F26063

Refer to the above diagram. In equilibrium the firm:

A. is realizing an economic profit of ad per unit.B. should close down in the short run.C. is realizing a loss.D. is realizing an economic profit of bd per unit.

9. In the United States cartels are:A. quite common in industries which produce nondurable goods.B. in violation of the antitrust laws.C. concentrated in monopolistically competitive industries.D. encouraged by government policy so that firms can realize economies of scale.

10. If the several oligopolistic firms which comprise an industry behave collusively, the resulting price and output will most likely resemble those of:

A. bilateral monopoly.B. pure monopoly.C. monopolistic competition.D. pure competition.

Page 781: Microeconomics Study Guide

11. Advertising can enhance economic efficiency when it:A. increases brand loyalty.B. expands sales such that firms achieve substantial economies of scale.C. keeps new firms from entering profitable industries.D. is undertaken by pure competitors.

12. The conclusion that oligopoly is inefficient relative to the competitive ideal must be qualified because:

A. industry price leaders often select a price equal to marginal cost.B. over time oligopolistic industries may promote more rapid product development and greater

improvement of production techniques than if they were purely competitive.C. increased output due to persuasive advertising may perfectly offset the restriction of output

caused by monopoly power.D. many oligopolists sell their products in monopolistically competitive or even purely

competitive industries.

CHAPTER 25

Monopolistic Competition and Oligopoly

Topic Question numbers

___________________________________________________________________________________________________

1. Monopolistic competition: definition; characteristics 1-17

2. Demand curve 18-24

3. Price-output behavior 25-78

4. Efficiency aspects 79-88

Page 782: Microeconomics Study Guide

5. Oligopoly: definition; characteristics 89-112

6. Concentration ratio; Herfindahl Index 113-140

7. Game theory 141-156

8. Kinked-demand curve model 157-176

9. Collusion; cartels; price leadership 177-194

10.Advertising 195-200

11.Efficiency aspects 201-204

12.Review of four structures 205-226

Consider This 227-228

Last Word 229-233

True-False 234-258

___________________________________________________________________________________________________

Multiple Choice Questions

Monopolistic competition: definition; characteristics

Type: D Topic: 1 E: 460 MI: 216

1. Monopolistic competition means:

A) a market situation where competition is based entirely on product differentiation and advertising.

B) a large number of firms producing a standardized or homogeneous product.

C) many firms producing differentiated products.

D) a few firms producing a standardized or homogeneous product.

Answer: C

Type: A Topic: 1 E: 461 MI: 217

2. Monopolistic competition is characterized by a:

A) few dominant firms and low entry barriers.

B) large number of firms and substantial entry barriers.

C) large number of firms and low entry barriers.

D) few dominant firms and substantial entry barriers.

Answer: C

Page 783: Microeconomics Study Guide
Page 784: Microeconomics Study Guide

Type: A Topic: 1 E: 461 MI: 217

3. Under monopolistic competition entry to the industry is:

A) completely free of barriers.

B) more difficult than under pure competition but not nearly as difficult as under pure monopoly.

C) more difficult than under pure monopoly.

D) blocked.

Answer: B

Type: A Topic: 1 E: 461 MI: 217

4. Monopolistic competition resembles pure competition because:

A) both industries emphasize nonprice competition.

B) in both instances firms will operate at the minimum point on their long-run average total cost curves.

C) both industries entail the production of differentiated products.

D) barriers to entry are either weak or nonexistent.

Answer: D

Type: A Topic: 1 E: 460-461 MI: 216-217

5. Which of the following is not a basic characteristic of monopolistic competition?

A) the use of trademarks and brand names C) product differentiation

B) recognized mutual interdependence D) a relatively large number of sellers

Answer: B

Type: D Topic: 1 E: 461-462 MI: 217-218

6. Nonprice competition refers to:

A) competition between products of different industries, for example, competition between aluminum and steel in the manufacture of automobile parts.

B) price increases by a firm that are ignored by its rivals.

C) advertising, product promotion, and changes in the real or perceived characteristics of a product.

D) reductions in production costs that are not reflected in price reductions.

Answer: C

Page 785: Microeconomics Study Guide

Type: A Topic: 1 E: 462-463 MI: 218-219

7. Which of the following is not characteristic of monopolistic competition?

A) relatively large numbers of sellers C) production at minimum ATC in the long-run

B) product differentiation D) relatively easy entry to the industry

Answer: C

Type: A Topic: 1 E: 462 MI: 218

8. The book publishing, furniture, and clothing industries are each illustrations of:

A) countervailing power. C) monopolistic competition.

B) homogeneous oligopoly. D) pure monopoly.

Answer: C

Page 786: Microeconomics Study Guide

Type: A Topic: 1 E: 461 MI: 217

9. If the number of firms in a monopolistically competitive industry increases and the degree of product differentiation diminishes:

A) the likelihood of realizing economic profits in the long run would be enhanced.

B) individual firms would now be operating at outputs where their average total costs would be higher.

C) the industry would more closely approximate pure competition.

D) the likelihood of collusive pricing would increase.

Answer: C

Type: A Topic: 1 E: 461 MI: 217

10. Economic analysis of a monopolistically competitive industry is more complicated than that of pure competition because:

A) the number of firms in the industry is larger.

B) monopolistically competitive firms cannot realize an economic profit in the long run.

C) of product differentiation and consequent product promotion activities.

D) monopolistically competitive producers use strategic pricing strategies to combat rivals.

Answer: C

Type: A Topic: 1 E: 461 MI: 217

11. A monopolistically competitive industry combines elements of both competition and monopoly. The monopoly element results from:

A) the likelihood of collusion. C) product differentiation.

B) high entry barriers. D) mutual interdependence in decision making.

Answer: C

Type: D Topic: 1 E: 462 MI: 218

12. Nonprice competition refers to:

A) low barriers to entry.

B) product development, advertising, and product packaging.

C) the differences in information which consumers have regarding various products.

D) an industry or firm in long-run equilibrium.

Answer: B

Page 787: Microeconomics Study Guide

Type: A Topic: 1 E: 461 MI: 217

13. A significant difference between a monopolistically competitive firm and a purely competitive firm is that the:

A) former does not seek to maximize profits.

B) latter recognizes that price must be reduced to sell more output.

C) former sells similar, although not identical, products.

D) former's demand curve is perfectly inelastic.

Answer: C

Type: A Topic: 1 E: 460 MI: 216

14. A monopolistically competitive industry combines elements of both competition and monopoly. It is correct to say that the competitive element results from:

A) a relatively large number of firms and the monopolistic element from product differentiation.

B) product differentiation and the monopolistic element from high entry barriers.

C) a perfectly elastic demand curve and the monopolistic element from low entry barriers.

D) a highly inelastic demand curve and the monopolistic element from advertising and product promotion.

Answer: A

Page 788: Microeconomics Study Guide

Type: A Topic: 1 E: 461 MI: 217

15. Monopolistically competitive and purely competitive industries are similar in that:

A) both are assured of short-run economic profits.

B) both produce differentiated products.

C) the demand curves facing individual firms are perfectly elastic in both industries.

D) there are few, if any, barriers to entry.

Answer: D

Type: A Topic: 1 E: 462 MI: 218

16. The monopolistic competition model predicts that:

A) allocative efficiency will be achieved.

B) productive efficiency will be achieved.

C) firms will engage in nonprice competition.

D) firms will realize economic profits in the long run.

Answer: C

Type: C Topic: 1 E: 460 MI: 216

17. Use your basic knowledge and your understanding of market structures to answer this question. Which of the following companies most closely approximates a monopolistic competitor?

A) Subway Sandwiches B) Pittsburgh Plate Glass C) Ford Motor Company D) Microsoft.

Answer: A

Demand curve

Type: D Topic: 2 E: 462 MI: 218

18. A monopolistically competitive firm has a:

A) highly elastic demand curve. C) perfectly inelastic demand curve.

B) highly inelastic demand curve. D) perfectly elastic demand curve.

Answer: A

Page 789: Microeconomics Study Guide

Type: A Topic: 2 E: 462 MI: 218

19. The monopolistically competitive seller's demand curve will become more elastic the:

A) more significant the barriers to entering the industry.

B) greater the degree of product differentiation.

C) larger the number of competitors.

D) smaller the number of competitors.

Answer: C

Type: A Topic: 2 E: 462 MI: 218

20. The larger the number of firms and the smaller the degree of product differentiation the:

A) greater the divergence between the demand and the marginal revenue curves of the monopolistically competitive firm.

B) larger will be the monopolistically competitive firm's fixed costs.

C) less elastic is the monopolistically competitive firm's demand curve.

D) more elastic is the monopolistically competitive firm's demand curve.

Answer: D

Page 790: Microeconomics Study Guide

Type: A Topic: 2 E: 462 MI: 218

21. The demand curve of a monopolistically competitive producer is:

A) less elastic than that of either a pure monopolist or a pure competitor.

B) less elastic than that of a pure monopolist, but more elastic than that of a pure competitor.

C) more elastic than that of a pure monopolist, but less elastic than that of a pure competitor.

D) more elastic than that of either a pure monopolist or a pure competitor.

Answer: C

Type: A Topic: 2 E: 462-463 MI: 218-219

22. A monopolistically competitive firm's marginal revenue curve:

A) is downsloping and coincides with the demand curve.

B) coincides with the demand curve and is parallel to the horizontal axis.

C) is downsloping and lies below the demand curve.

D) does not exist because the firm is a "price maker."

Answer: C

Type: A Topic: 2 E: 462-463 MI: 218-219

23. In comparing the demand curve of a pure monopolist with that of a monopolistically competitive firm, we would expect the monopolistic competitor to have a:

A) perfectly elastic demand curve and the monopolist to have a perfectly inelastic demand curve.

B) generally more elastic demand curve.

C) generally less elastic demand curve.

D) demand curve whose elasticity coefficient is 1 at all possible prices.

Answer: B

Type: A Topic: 2 E: 462 MI: 218

24. The price elasticity of a monopolistically competitive firm's demand curve varies:

A) inversely with the number of competitors and the degree of product differentiation.

B) directly with the number of competitors and the degree of product differentiation.

C) directly with the number of competitors, but inversely with the degree of product differentiation.

D) inversely with the number of competitors, but directly with the degree of product differentiation.

Page 791: Microeconomics Study Guide

Answer: C

Price-output behavior

Type: A Topic: 3 E: 464 MI: 220 Status: New

25. In short-run equilibrium, a monopolistically competitive firm sets it price:

A) equal to marginal revenue. C) above marginal cost.

B) equal to marginal cost. D) below marginal cost.

Answer: C

Type: A Topic: 3 E: 464 MI: 220 Status: New

26. In long-run equilibrium, a monopolistically competitive firm sets it price:

A) above marginal cost. C) equal to marginal revenue.

B) below marginal cost. D) equal to marginal cost.

Answer: A

Page 792: Microeconomics Study Guide

Type: A Topic: 3 E: 464 MI: 220 Status: New

27. In short-run equilibrium, the price charged by the monopolistically competitive firm:

A) must be less than ATC.

B) must be more than ATC.

C) may be either equal to ATC, less than ATC, or more than ATC.

D) must be equal to ATC.

Answer: C

Type: A Topic: 3 E: 464 MI: 220 Status: New

28. In long-run equilibrium, the price charged by the monopolistically competitive firm:

A) must be less than ATC.

B) must be more than ATC.

C) may be either equal to ATC, less than ATC, or more than ATC.

D) will be equal to ATC.

Answer: D

Type: A Topic: 3 E: 464 MI: 220

29. Monopolistically competitive firms:

A) realize normal profits in the short run but losses in the long run.

B) incur persistent losses in both the short run and long run.

C) may realize either profits or losses in the short run, but realize normal profits in the long run.

D) persistently realize economic profits in both the short run and long run.

Answer: C

Type: A Topic: 3 E: 464 MI: 220

30. The monopolistically competitive seller maximizes profit by producing at the point where:

A) total revenue is at a maximum. C) marginal revenue equals marginal cost.

B) average costs are at a minimum. D) price equals marginal revenue.

Answer: C

Page 793: Microeconomics Study Guide

Type: A Topic: 3 E: 464 MI: 220

31. In long-run equilibrium a monopolistically competitive firm's price will:

A) be less than both MC and ATC. C) exceed MC, but equal ATC.

B) exceed ATC, but equal MC. D) exceed both MC and ATC.

Answer: C

Type: A Topic: 3 E: 465 MI: 221

32. Which of the following is correct for a monopolistically competitive firm in long-run equilibrium?

A) MC = ATC B) MC exceeds MR C) P exceeds minimum ATC D) P = MC

Answer: C

Type: A Topic: 3 E: 465-466 MI: 221-222

33. In long-run equilibrium a monopolistically competitive firm will:

A) earn an economic profit. C) equate price and marginal cost.

B) realize all economies of scale. D) have excess production capacity.

Answer: D

Page 794: Microeconomics Study Guide

Type: D Topic: 3 E: 465-466 MI: 221-222

34. Excess capacity refers to the:

A) amount by which actual production falls short of the minimum ATC output.

B) fact that entry barriers artificially reduce the number of firms in an industry.

C) differential between price and marginal costs which characterizes monopolistically competitive firms.

D) fact that most monopolistically competitive firms encounter diseconomies of scale.

Answer: A

Use the following to answer questions 35-38:

Type: G Topic: 3 E: 463-464 MI: 219-220

35. Refer to the above diagram for a monopolistically competitive firm in short-run equilibrium. This firm's profit-maximizing price will be:

A) $10. B) $13. C) $16. D) $19.

Answer: C

Type: G Topic: 3 E: 463-464 MI: 219-220

36. Refer to the above diagram for a monopolistically competitive firm in short-run equilibrium. The profit-maximizing output for this firm will be:

A) 210. B) 180. C) 160. D) 100.

Answer: C

Type: G Topic: 3 E: 463-464 MI: 219-220

Page 795: Microeconomics Study Guide

37. Refer to the above diagram for a monopolistically competitive firm in short-run equilibrium. This firm will realize an economic:

A) loss of $320. B) loss of $280. C) profit of $480. D) profit of $600. E) profit of $360.

Answer: C

Type: G Topic: 3 E: 464 MI: 220

38. Refer to the above diagram for a monopolistically competitive firm in short-run equilibrium. Assume the firm is part of an increasing-cost industry. In the long run firms will:

A) leave this industry, causing both demand and the ATC curve to shift upward.

B) enter this industry, causing demand to rise and the ATC curve to shift downward.

C) enter this industry, causing demand to fall and the ATC curve to shift upward.

D) enter this industry, causing both demand and the ATC curve to shift upward.

Answer: C

Page 796: Microeconomics Study Guide

Type: A Topic: 3 E: 464 MI: 220

39. In the short run a monopolistically competitive firm's economic profit:

A) will be maximized where price equals average total cost.

B) may be positive, zero, or negative.

C) are always positive.

D) will always be zero.

Answer: B

Use the following to answer questions 40-42:

Type: G Topic: 3 E: 463 MI: 219 Status: New

40. In short-run equilibrium, the monopolistically competitive firm shown above will set its price:

A) below ATC. B) above ATC. C) below MC. D) below MR.

Answer: A

Type: G Topic: 3 E: 463-464 MI: 219-220

41. The monopolistically competitive firm shown in the above figure:

A) is in long-run equilibrium.

B) might realize an economic profit or a loss, depending on its choice of output level.

C) cannot operate profitably, at least in the short run.

D) can realize an economic profit.

Answer: C

Page 797: Microeconomics Study Guide

Type: G Topic: 3 E: 464 MI: 220 Status: New

42. If all monopolistically competitive firms in the industry have profit circumstances similar to the firm shown above:

A) new firms will enter the industry. C) all firms will exit the industry.

B) some firms will exit the industry. D) no firms will exit the industry.

Answer: B

Page 798: Microeconomics Study Guide

Use the following to answer questions 43-45:

Type: G Topic: 3 E: 463 MI: 219 Status: New

43. In short-run equilibrium, the monopolistically competitive firm shown in the above figure will set its price:

A) below ATC. B) above ATC. C) below MC. D) below MR.

Answer: B

Type: G Topic: 3 E: 463-464 MI: 219-220

44. The monopolistically competitive firm shown in the above figure:

A) will realize allocative efficiency at its profit-maximizing output.

B) cannot operate at a loss.

C) is in long-run equilibrium.

D) is realizing an economic profit.

Answer: D

Type: G Topic: 3 E: 464 MI: 220 Status: New

45. If all monopolistically competitive firms in the industry have profit circumstances similar to the firm shown above:

A) new firms will enter the industry. C) all firms will exit the industry.

B) some firms will exit the industry. D) no firms will enter the industry.

Answer: A

Page 799: Microeconomics Study Guide

Use the following to answer questions 46-48:

Type: G Topic: 3 E: 463 MI: 219

46. Refer to the above diagrams, which pertain to monopolistically competitive firms. Short-run equilibrium entailing economic loss is shown by:

A) diagram a only. B) diagram b only. C) diagram c only. D) both diagrams a and c.

Answer: C

Type: G Topic: 3 E: 463 MI: 219

47. Refer to the above diagrams, which pertain to monopolistically competitive firms. A short-run equilibrium entailing economic profits is shown by:

A) diagram a only. B) diagram b only. C) diagram c only. D) both diagrams b and c.

Answer: B

Type: G Topic: 3 E: 463 MI: 219

48. Refer to the above diagrams, which pertain to monopolistically competitive firms. Long-run equilibrium is shown by:

A) diagram a only. B) diagram b only. C) diagram c only. D) both diagrams b and c.

Answer: A

Page 800: Microeconomics Study Guide

Type: A Topic: 3 E: 464 MI: 220

49. Which of the following is not characteristic of long-run equilibrium under monopolistic competition?

A) price equals minimum average total cost C) price is equal to average total cost

B) marginal cost equals marginal revenue D) price exceeds marginal cost

Answer: A

Page 801: Microeconomics Study Guide

Use the following to answer questions 50-52:

Type: G Topic: 3 E: 463 MI: 219

50. Refer to the above diagram for a monopolistically competitive firm. Long-run equilibrium price will be:

A) above A. B) EF. C) A. D) B.

Answer: C

Type: G Topic: 3 E: 463 MI: 219

51. Refer to the above diagram for a monopolistically competitive firm. Long-run equilibrium output will be:

A) greater than E. B) E. C) D. D) C.

Answer: C

Type: G Topic: 3 E: 464 MI: 220

52. Refer to the above diagram for a monopolistically competitive firm. If more firms would enter the industry and product differentiation would weaken:

A) resource misallocation would become more severe.

B) the demand curve would become more elastic.

C) equilibrium output would decline and equilibrium price would rise.

D) equilibrium output would decline and equilibrium price would fall.

Answer: B

Type: A Topic: 3 E: 464 MI: 220

53. Long-run equilibrium for a monopolistically competitive firm where economic profits are zero results from:

Page 802: Microeconomics Study Guide

A) rising marginal costs. C) relatively easy entry.

B) a perfectly elastic product demand curve. D) product differentiation and development.

Answer: C

Page 803: Microeconomics Study Guide

Use the following to answer questions 54-55:

Type: G Topic: 3 E: 464 MI: 220

54. In long-run equilibrium, the firm shown in the diagram above will:

A) earn a normal profit. B) go bankrupt. C) incur a loss. D) realize an economic profit.

Answer: A

Type: G Topic: 3 E: 465 MI: 221

55. In long-run equilibrium, production for the firm shown in the diagram above is:

A) greater than would occur under pure competition.

B) less efficient than in a purely competitive market.

C) more efficient than in a purely competitive market.

D) optimally efficient.

Answer: B

Type: A Topic: 3 E: 463-464 MI: 219-220

56. When a monopolistically competitive firm is in long-run equilibrium:

A) production takes place where ATC is minimized.

B) marginal revenue equals marginal cost and price equals average total cost.

C) normal profit is zero and price equals marginal cost.

D) economic profit is zero and price equals marginal cost.

Answer: B

Page 804: Microeconomics Study Guide

Type: A Topic: 3 E: 464 MI: 220

57. In the long run, new firms will enter a monopolistically competitive industry:

A) provided economies of scale are being realized. C) until minimum average total cost is achieved.

B) even though losses are incurred in the short run. D) until economic profits are zero.

Answer: D

Type: A Topic: 3 E: 464 MI: 220

58. If some firms leave a monopolistically competitive industry, the demand curves of the remaining firms will:

A) be unaffected. B) shift to the left. C) become more elastic. D) shift to the right.

Answer: D

Page 805: Microeconomics Study Guide

Type: A Topic: 3 E: 463-464 MI: 219-220

59. When a monopolistically competitive firm is in long-run equilibrium:

A) P = MC = ATC. C) MR > MC and P = minimum ATC.

B) MR = MC and minimum ATC > P. D) MR = MC and P > minimum ATC.

Answer: D

Type: A Topic: 3 E: 464 MI: 220

60. Other things equal, if more firms enter a monopolistically competitive industry:

A) the demand curves facing existing firms would shift to the right.

B) the demand curves facing existing firms would shift to the left.

C) the demand curves facing existing firms would become less elastic.

D) losses would necessarily occur.

Answer: B

Type: A Topic: 3 E: 464 MI: 220

61. Which of the following statements is correct?

A) Purely competitive firms, monopolistically competitive firms, and pure monopolies all earn zero economic profits in the long run.

B) Purely competitive firms, monopolistically competitive firms, and pure monopolies all earn positive economic profits in the long run.

C) In the long run purely competitive firms and monopolistically competitive firms earn zero economic profits, while pure monopolies may or may not earn economic profits.

D) Monopolistically competitive firms earn zero economic profits in both the short run and the long run.

Answer: C

Type: A Topic: 3 E: 463 MI: 219

62. For a monopolistically competitive firm in long-run equilibrium:

A) price will equal marginal cost. C) marginal revenue will exceed marginal cost.

B) price will equal average total cost. D) economic profits will be some positive amount.

Answer: B

Page 806: Microeconomics Study Guide

Type: A Topic: 3 E: 463-464 MI: 219-220

63. In long-run equilibrium both purely competitive and monopolistically competitive firms will:

A) produce at minimum average total cost. C) achieve allocative efficiency.

B) earn economic profits. D) equate marginal cost and marginal revenue.

Answer: D

Type: A Topic: 3 E: 465-466 MI: 221-222

64. In long-run equilibrium monopolistic competition entails:

A) an efficient allocation of resources.

B) an overallocation of resources.

C) an underallocation of resources.

D) production at the minimum attainable average total cost.

Answer: C

Page 807: Microeconomics Study Guide

Type: A Topic: 3 E: 464 MI: 220

65. Which of the following statements concerning a monopolistically competitive industry is correct?

A) If there are short-run losses, firms will leave the industry and the demand curves of the remaining firms will shift to the right.

B) If there are short-run economic profits, firms will enter the industry and the demand curves of existing firms will shift to the right.

C) If there are short-run losses, firms will leave the industry and the demand curves of the remaining firms will shift to the left.

D) If there are short-run economic profits, firms will leave the industry and the demand curves of the remaining firms will shift to the right.

Answer: A

Use the following to answer questions 66-71:

Answer the next question(s) on the basis of the following demand and cost data for a specific firm:

D em a n d D a ta (1 ) (2 ) (3 ) To ta l

P r ice P r ice Q u a n tity O u tp u t co st $ 11 $ 1 0 6 6 $ 6 1

9 .9 9 8 .8 5 7 7 6 2 9 8 8 8 6 4 8 7 9 9 6 7

7 .1 0 6 .1 0 1 0 1 0 7 2 6 5 11 11 7 9

5 .1 5 4 .1 5 1 2 1 2 8 6

C o st D a ta

Type: T Topic: 3 E: 464 MI: 220

66. If columns (1) and (3) of the demand data shown above are this firm's demand schedule, the profit-maximizing level of output will be:

A) 12 units. B) 8 units. C) 10 units. D) 9 units.

Answer: B

Type: T Topic: 3 E: 464 MI: 220

67. If columns (1) and (3) of the demand data shown above are this firm's demand schedule, the profit-maximizing price will be:

A) $9. B) $7. C) $11. D) $6.

Answer: A

Page 808: Microeconomics Study Guide

Type: T Topic: 3 E: 464 MI: 220

68. If columns (1) and (3) of the demand data shown above are this firm's demand schedule, economic profit will be:

A) $10. B) $19. C) $6. D) $8.

Answer: D

Type: T Topic: 3 E: 464 MI: 220

69. Suppose that entry into the industry changes this firm's demand schedule from columns (1) and (3) shown above to columns (2) and (3). Economic profit will:

A) fall by $10. B) fall to $6. C) increase by $10. D) decline to zero.

Answer: D

Page 809: Microeconomics Study Guide

Type: T Topic: 3 E: 464 MI: 220

70. Suppose that entry into this industry changes this firm's demand schedule from columns (1) and (3) shown above to columns (2) and (3). We can conclude that this industry is:

A) a pure monopoly. C) a constant cost industry.

B) purely competitive. D) monopolistically competitive.

Answer: D

Type: T Topic: 3 E: 464 MI: 220

71. With the demand schedule shown above by columns (2) and (3), in long-run equilibrium:

A) price will equal average total cost. C) marginal cost will exceed price.

B) total cost will exceed total revenue. D) price will equal marginal revenue.

Answer: A

Type: A Topic: 3 E: 463-464 MI: 219-220

72. An important similarity between a monopolistically competitive firm and a purely competitive firm is that:

A) both face perfectly elastic demand schedules. C) both realize productive efficiency.

B) economic profit tends toward zero for both. D) both realize allocative efficiency.

Answer: B

Type: A Topic: 3 E: 462 MI: 218

73. An important similarity between a monopolistically competitive firm and a pure monopolist is that both:

A) realize an economic profit in the long run.

B) achieve allocative efficiency.

C) face demand curves which are less than perfectly elastic.

D) achieve productive efficiency.

Answer: C

Type: A Topic: 3 E: 462, 464 MI: 218, 220

74. The less elastic a monopolistic competitor's long-run demand curve, the:

A) less its excess capacity.

Page 810: Microeconomics Study Guide

B) higher its price relative to that of a pure competitor having the same cost curves.

C) higher its long-run profits.

D) lower its average total cost at its equilibrium level of output.

Answer: B

Page 811: Microeconomics Study Guide

Use the following to answer questions 75-77:

Type: G Topic: 3 E: 463-464 MI: 219-220

75. Refer to the above diagram for a monopolistically competitive producer. The firm is:

A) minimizing losses in the long run. C) realizing a normal profit in the long run.

B) minimizing losses in the short run. D) about to leave the industry.

Answer: C

Type: G Topic: 3 E: 465-466 MI: 221-222

76. Refer to the above diagram for a monopolistically competitive producer. This firm is experiencing:

A) a shortage of production capacity. C) excess capacity of DE.

B) excess capacity of CD. D) diseconomies of scale.

Answer: C

Type: G Topic: 3 E: 465 MI: 221

77. Refer to the above diagram for a monopolistically competitive producer. If this firm were to realize productive efficiency, it would:

A) also realize an economic profit. C) also achieve allocative efficiency.

B) incur a loss. D) have to produce a smaller output.

Answer: B

Type: A Topic: 3 E: 463-464 MI: 219-220

78. In the long run a monopolistically competitive firm:

Page 812: Microeconomics Study Guide

A) earns an economic profit. C) produces where MR exceeds MC.

B) produces where P = ATC. D) achieves allocative efficiency.

Answer: B

Efficiency aspects

Type: D Topic: 4 E: 465-466 MI: 221-222

79. Monopolistically competitive industries are inefficient because:

A) they realize diseconomies of scale.

B) advertising costs retard technological advance and product development.

C) monopolistically competitive industries are overpopulated with firms whose plants are underutilized.

D) monopolistically competitive sellers engage in misleading advertising.

Answer: C

Page 813: Microeconomics Study Guide

Type: A Topic: 4 E: 466 MI: 222

80. The economic inefficiencies of monopolistic competition may be offset by the fact that:

A) advertising expenditures shift the average cost curve upward.

B) available capacity is fully utilized.

C) resources are optimally allocated to the production of the product.

D) consumers have a number of variations of the product from which to choose.

Answer: D

Type: A Topic: 4 E: 465-466 MI: 221-222

81. Inefficiencies occur under monopolistic competition because:

A) each firm's demand curve becomes more elastic as we move down the curve.

B) each firm's marginal revenue curve coincides with its demand curve.

C) each firm's downsloping demand curve is tangent to the ATC curve in the long run.

D) entry barriers greatly restrict the entry of new firms.

Answer: C

Type: F Topic: 4 E: 466 MI: 222

82. A significant benefit of monopolistic competition compared with pure competition is:

A) less likelihood of X-inefficiency.

B) improved resource allocation.

C) greater product variety.

D) stronger incentives to achieve economies of scale.

Answer: C

Type: A Topic: 4 E: 466 MI: 222

83. Product variety is likely to be greater in:

A) monopolistic competition than in pure competition.

B) pure competition than in monopolistic competition.

C) homogenous oligopoly than in monopolistic competition.

D) homogenous oligopoly than in differentiated oligopoly.

Answer: A

Page 814: Microeconomics Study Guide

Type: A Topic: 4 E: 466 MI: 222

84. Which of the following is correct?

A) The excess capacity problem diminishes as the monopolistically competitive firm's demand curve becomes less elastic.

B) The excess capacity problem means that monopolistically competitive firms typically produce at some point on the rising segment of their average total cost curve.

C) The greater the degree of product variation, the lesser is the excess capacity problem.

D) The greater the degree of product variation, the greater is the excess capacity problem.

Answer: D

Type: A Topic: 4 E: 465 MI: 221

85. In monopolistically competitive markets resources are:

A) overallocated because long-run equilibrium occurs where price exceeds marginal cost.

B) underallocated because long-run equilibrium occurs where price exceeds marginal cost.

C) overallocated because long-run equilibrium occurs where marginal cost exceeds price.

D) underallocated because long-run equilibrium occurs where marginal cost exceeds price.

Answer: B

Page 815: Microeconomics Study Guide

Type: A Topic: 4 E: 465 MI: 221

86. In long-run equilibrium a monopolistically competitive producer achieves:

A) neither productive efficiency nor allocative efficiency.

B) both productive efficiency and allocative efficiency.

C) productive efficiency, but not allocative efficiency.

D) allocative efficiency, but not productive efficiency.

Answer: A

Type: A Topic: 4 E: 465-466 MI: 221-222

87. The less elastic a monopolistic competitor's long-run demand curve, the:

A) greater its excess capacity.

B) lower its price relative to that of a pure competitor having the same cost curves.

C) higher its long-run economic profit.

D) lower its average total cost at its equilibrium level of output.

Answer: A

Type: A Topic: 4 E: 465 MI: 221

88. The more elastic a monopolistic competitor's long-run demand curve, the:

A) greater its excess capacity.

B) the higher its price relative to that of a pure competitor having the same cost curves.

C) lower its long-run profit.

D) lower its average total cost at its equilibrium level of output.

Answer: D

Oligopoly: definition; characteristics

Type: A Topic: 5 E: 467 MI: 223 Status: New

89. In which of these continuums of degrees of competition (highest to lowest) is oligopoly properly placed?

A) pure competition, oligopoly, pure monopoly, monopolistic competition

B) oligopoly, pure competition, monopolistic competition, pure monopoly

Page 816: Microeconomics Study Guide

C) monopolistic competition, pure competition, pure monopoly, oligopoly

D) pure competition, monopolistic competition, oligopoly, pure monopoly

Answer: D

Type: D Topic: 5 E: 467 MI: 223

90. The term oligopoly indicates:

A) a one-firm industry.

B) many producers of a differentiated product.

C) a few firms producing either a differentiated or a homogeneous product.

D) an industry whose four-firm concentration ratio is low.

Answer: C

Type: A Topic: 5 E: 467 MI: 223

91. In an oligopolistic market:

A) one firm is always dominant.

B) products may be standardized or differentiated.

C) the four largest firms account for 20 percent or less of total sales.

D) the industry is monopolistically competitive.

Answer: B

Page 817: Microeconomics Study Guide

Type: D Topic: 5 E: 467-468 MI: 223-224

92. Oligopolistic industries are characterized by:

A) a few dominant firms and substantial entry barriers.

B) a few dominant firms and no barriers to entry.

C) a large number of firms and low entry barriers.

D) a few dominant firms and low entry barriers.

Answer: A

Type: A Topic: 5 E: 467 MI: 223

93. The automobile, household appliance, and automobile tire industries are all illustrations of:

A) homogeneous oligopoly. C) pure monopoly.

B) monopolistic competition. D) differentiated oligopoly.

Answer: D

Type: C Topic: 5 E: 467 MI: 223

94. Use your basic knowledge and your understanding of market structures to answer this question. Which of the following companies most closely approximates a differentiated oligopolist in a highly concentrated industry?

A) Subway Sandwiches B) Pittsburgh Plate Glass C) Ford Motor Company D) Kaiser Aluminum.

Answer: C

Type: C Topic: 5 E: 467 MI: 223

95. Use your basic knowledge and your understanding of market structures to answer this question. Which of the following companies most closely approximates a homogenous oligopolist in a highly concentrated industry?

A) Kellogg B) Pittsburgh Plate Glass C) Ford Motor Company D) Starbucks Coffee.

Answer: B

Type: A Topic: 5 E: 467 MI: 223

96. The mutual interdependence that characterizes oligopoly arises because:

A) the products of various firms are homogeneous.

B) the products of various firms are differentiated.

Page 818: Microeconomics Study Guide

C) a small number of firms produce a large proportion of industry output.

D) the demand curves of firms are kinked at the prevailing price.

Answer: C

Type: A Topic: 5 E: 467 MI: 223

97. Barriers to entry in oligopolistic industries may consist of:

A) diseconomies of scale. C) ownership of essential resources.

B) diminishing returns. D) patent expirations.

Answer: C

Type: A Topic: 5 E: 467 MI: 223

98. The copper, aluminum, cement, and industrial alcohol industries are examples of:

A) interproduct competition. C) monopolistic competition.

B) homogeneous oligopoly. D) differentiated oligopoly.

Answer: B

Page 819: Microeconomics Study Guide

Type: A Topic: 5 E: 467 MI: 223

99. Which of the following is the best example of oligopoly?

A) women's dress manufacturing B) automobile manufacturing C) restaurants D) cotton farming

Answer: B

Type: A Topic: 5 E: 467 MI: 223

100. If there are significant economies of scale in an industry, then:

A) a firm that is large may be able to produce at a lower unit cost than can a small firm.

B) a firm that is large will have to charge a higher price than will a small firm.

C) entry to that industry will be easy.

D) firms must differentiate their products to earn economic profits.

Answer: A

Type: A Topic: 5 E: 472 MI: 228

101. In which of the following market models do demand and marginal revenue diverge?

A) pure monopoly, oligopoly, and monopolistic competition

B) pure monopoly, oligopoly, and pure competition

C) pure monopoly only

D) oligopoly only

Answer: A

Type: A Topic: 5 E: 471 MI: 227

102. Oligopoly is difficult to analyze primarily because:

A) the number of firms is too large to make collusion understandable.

B) the price and output decisions of any one firm depend on the reactions of its rivals.

C) output may be either homogenous or differentiated.

D) neither allocative nor productive efficiency is achieved.

Answer: B

Type: A Topic: 5 E: 471 MI: 227

Page 820: Microeconomics Study Guide

103. Oligopoly is more difficult to analyze than other market models because:

A) the number of firms is so large that market behavior cannot be accurately predicted.

B) the marginal cost and marginal revenue curves of an oligopolist play no part in the determination of equilibrium price and quantity.

C) of mutual interdependence and the fact that oligopoly outcomes are less certain than in other market models.

D) unlike the firms of other market models, it cannot be assumed that oligopolists are profit maximizers.

Answer: C

Type: A Topic: 5 E: 467 MI: 223

104. Which of the following is an illustration of differentiated oligopoly?

A) the aluminum industry C) the soft drink industry

B) the steel industry D) retail stores in large cities

Answer: C

Type: A Topic: 5 E: 467 MI: 223

105. Which of the following industries is an illustration of homogeneous oligopoly?

A) household laundry products B) personal computers C) aluminum D) the auto industry

Answer: C

Page 821: Microeconomics Study Guide

Type: D Topic: 5 E: 467 MI: 223

106. Differentiated oligopoly exists where a small number of firms are:

A) producing goods that differ in terms of quality and design.

B) setting price and output collusively.

C) setting price and output independently.

D) producing virtually identical products.

Answer: A

Type: D Topic: 5 E: 467 MI: 223

107. Homogeneous oligopoly exists where a small number of firms are:

A) producing virtually identical products. C) setting price and output collusively.

B) setting price and output independently. D) producing differentiated products.

Answer: A

Type: D Topic: 5 E: 467 MI: 223

108. Oligopolistic industries:

A) are characterized by a relatively large number of small sellers.

B) may produce either standardized or differentiated products.

C) always produce differentiated products.

D) always produce standardized products.

Answer: B

Type: A Topic: 5 E: 467 MI: 223

109. Which of the following is a unique feature of oligopoly?

A) mutual interdependence C) product differentiation

B) advertising expenditures D) nonprice competition

Answer: A

Type: A Topic: 5 E: 467 MI: 223

110. Prices are likely to be least flexible:

Page 822: Microeconomics Study Guide

A) in oligopoly. C) where product demand is inelastic.

B) in monopolistic competition. D) in pure competition.

Answer: A

Type: D Topic: 5 E: 467 MI: 223

111. Mutual interdependence means that each oligopolistic firm:

A) faces a perfectly elastic demand for its product.

B) must consider the reactions of its rivals when it determines its price policy.

C) produces a product identical to those of its rivals.

D) produces a product similar but not identical to the products of its rivals.

Answer: B

Type: A Topic: 5 E: 467 MI: 223

112. Clear-cut mutual interdependence with respect to the price-output policies exists in:

A) pure monopoly B) oligopoly C) monopolistic competition D) pure competition

Answer: B

Page 823: Microeconomics Study Guide

Concentration ratio; Herfindahl Index

Type: D Topic: 6 E: 468 MI: 224

113. Concentration ratios measure the:

A) geographic location of the largest corporations in each industry.

B) degree to which product price exceeds marginal cost in various industries.

C) percentage of total sales accounted for by the four largest firms in the industry.

D) number of firms in an industry.

Answer: C

Type: A Topic: 6 E: 468 MI: 224

114. If the four-firm concentration ratio for industry X is 80:

A) the four largest firms account for 80 percent of total sales.

B) each of the four largest firms accounts for 20 percent of total sales.

C) the four largest firms account for 20 percent of total sales.

D) the industry is monopolistically competitive.

Answer: A

Type: A Topic: 6 E: 468 MI: 224

115. An industry having a four-firm concentration ratio of 85 percent:

A) approximates pure competition. C) is a pure monopoly.

B) is monopolistically competitive. D) is an oligopoly.

Answer: D

Type: A Topic: 6 E: 468 MI: 224

116. As a general rule, oligopoly exists when the four-firm concentration ratio:

A) exceeds the Herfindahl index. C) is 40 percent or more.

B) is less than the Herfindahl index. D) is 15 percent or more.

Answer: C

Page 824: Microeconomics Study Guide

Type: A Topic: 6 E: 469 MI: 225

117. Aluminum competes with copper in the market for power transmission lines. This illustrates:

A) mutual interdependence. C) interindustry competition.

B) differentiated oligopoly. D) homogeneous oligopoly.

Answer: C

Type: A Topic: 6 E: 469 MI: 225

118. The Herfindahl index for a pure monopolist is:

A) 100. B) 10,000. C) 100,000. D) 10.

Answer: B

Page 825: Microeconomics Study Guide

Type: A Topic: 6 E: 469 MI: 225

119. Industries X and Y both have four-firm concentration ratios of 65 percent, but the Herfindahl index for X is 1,500 while that for Y is 2,000. These data suggest:

A) greater market power in X than in Y.

B) greater market power in Y than in X.

C) that X is more technologically progressive than Y.

D) that price competition is stronger in Y than in X.

Answer: B

Type: A Topic: 6 E: 468 MI: 224

120. Suppose that total sales in an industry in a particular year are $600 million and sales by the top four sellers are $200 million, $150 million, $100 million, and $50 million, respectively. We can conclude that:

A) price leadership exists in this industry.

B) the concentration ratio is more than 80 percent.

C) this industry is a differentiated oligopoly.

D) the firms in this industry face a kinked demand curve.

Answer: B

Type: A Topic: 6 E: 468 MI: 224

121. The four-firm sales concentration ratio for an industry measures the:

A) geographic concentration of firms.

B) extent to which the four largest firms dominate the production of a good.

C) percentage of the industry's capital facilities owned by the four largest firms.

D) degree of X-inefficiency in the industry.

Answer: B

Type: A Topic: 6 E: 468 MI: 224

122. Concentration ratios:

A) may overstate the degree of competition because they ignore imported products.

B) may overstate the degree of competition because interindustry competition is ignored.

C) may understate the degree of competition because they ignore imported products.

D) provide detailed insights as to the price and output behavior of firms which comprise the various industries.

Page 826: Microeconomics Study Guide

Answer: C

Type: A Topic: 6 E: 468-469 MI: 224-225

123. If a product such as cement or bricks is costly to ship and, therefore, markets are very localized, the national concentration ratio for that industry:

A) will be greater than 50 percent.

B) may understate the degree of monopoly.

C) may overstate the degree of monopoly.

D) will yield an accurate impression of the degree of monopoly.

Answer: B

Page 827: Microeconomics Study Guide

Type: A Topic: 6 E: 469 MI: 225

124. Concentration ratios may be inaccurate indicators of the degree of monopoly power in an industry because:

A) they include interindustry competition.

B) foreign competition is not considered.

C) they are only calculated for local and regional markets.

D) they do not distinguish between normal and economic profit.

Answer: B

Type: A Topic: 6 E: 468 MI: 224

125. If an industry evolves from monopolistic competition to oligopoly, we would expect:

A) the four-firm concentration ratio to decrease.

B) the four-firm concentration ratio to increase.

C) the four-firm concentration ratio to remain the same.

D) barriers to entry to weaken.

Answer: B

Type: D Topic: 6 E: 469 MI: 225

126. Interindustry competition means that:

A) in oligopolistic industries a few large firms compete with one another in bidding down product price.

B) in some markets the producers of a particular product might face competition from products produced by other industries.

C) firms that sell a product at one stage of production are faced with firms that buy the product at the next stage of production.

D) in most industries there are usually a number of firms producing identical products.

Answer: B

Type: A Topic: 6 E: 469 MI: 225

127. If you sum the squares of the market shares of each firm in an industry (as measured by percent of industry sales), you are calculating:

A) the four-firm concentration ratio. C) the degree of collusion.

B) the Herfindahl index. D) the Lerner index.

Answer: B

Page 828: Microeconomics Study Guide

Type: A Topic: 6 E: 469 MI: 225

128. The Herfindahl Index:

A) measures the prices charged by oligopolistic manufacturers.

B) is another name for the four-firm concentration ratio.

C) tells us whether oligopolistic firms are engaging in collusion.

D) gives much greater weight to larger firms than to smaller firms in an industry.

Answer: D

Type: A Topic: 6 E: 469 MI: 225

129. If the four-firm concentration ratio in an oligopolistic industry is 100 percent and each firm has an equal percentage of sales, the Herfindahl Index is:

A) 10,000. B) 2,500. C) 3,750. D) 1,000.

Answer: B

Page 829: Microeconomics Study Guide

Type: A Topic: 6 E: 469 MI: 225

130. Assume six firms comprising an industry have market shares of 30, 30, 10, 10, 10, and 10 percent. The Herfindahl Index for this industry is:

A) 2,525. B) 1,600. C) 2,200. D) 80.

Answer: C

Type: A Topic: 6 E: 469 MI: 225

131. Suppose the Herfindahl Indexes for industries A, B, and C are 1,200, 5,000, and 7,500 respectively. These data imply that:

A) market power is greatest in industry A.

B) market power is greatest in industry B.

C) market power is greatest in industry C.

D) industry A is more monopolistic than industry C.

Answer: C

Use the following to answer questions 132-136:

F irm M a rk et sh a re (% ) A 2 0 B 2 0 C 2 0 D 2 0 E 1 0 F 1 0

Type: T Topic: 6 E: 468 MI: 224

132. The industry characterized by the above information is:

A) an oligopoly. C) a purely competitive industry.

B) a monopolistically competitive industry. D) a pure monopoly.

Answer: A

Type: T Topic: 6 E: 468 MI: 224

133. The four-firm concentration ratio for the above industry is:

A) 100 percent.

B) indeterminate, since we don't know which four firms are included.

Page 830: Microeconomics Study Guide

C) 80 percent.

D) 20 percent.

Answer: C

Type: T Topic: 6 E: 469 MI: 225

134. The Herfindahl Index for the above industry is:

A) 1,600. B) 1,800. C) 18,000. D) 80.

Answer: B

Type: T Topic: 6 E: 469 MI: 225

135. If all the firms in the above industry merged into a single firm, the Herfindahl Index would become:

A) 100. B) 1,000. C) 10,000. D) 100,000.

Answer: C

Page 831: Microeconomics Study Guide

Type: T Topic: 6 E: 468-469 MI: 224-225

136. Suppose that firms in this industry miraculously split up such that there were 100 firms, each with a one percent market share. The four-firm concentration ratio and the Herfindahl Index respectively would be:

A) 100 percent and 10,000. B) 4 percent and 4. C) 100 percent and 16. D) 4 percent and 16.

Answer: D

Use the following to answer questions 137-140:

F irm M ark et sh are (% ) A 4 0 B 3 0 C 2 0 D 5 E 5

Type: A Topic: 6 E: 468 MI: 224 Status: New

137. Refer to the above data. The four-firm concentration ratio for this industry is:

A) 90 percent.

B) 95 percent.

C) 100 percent.

D) indeterminate, because we don't know which four firms are included.

Answer: B

Type: A Topic: 6 E: 469 MI: 225 Status: New

138. Refer to the above data. The Herfindahl Index for this industry is:

A) 95. B) 1000. C) 2925. D) 2950.

Answer: D

Type: A Topic: 6 E: 467-468 MI: 223-224 Status: New

139. Refer to the above data. This industry illustrates:

A) pure competition. B) monopolistic competition. C) oligopoly. D) pure monopoly.

Answer: C

Page 832: Microeconomics Study Guide

Type: A Topic: 6 E: 468-469 MI: 224-225 Status: New

140. Refer to the data above. If Firm B merged with Firm C, the industry's four-firm concentration ratio would ____ and its Herfindahl Index would ____:

A) rise; rise. B) fall; rise C) remain the same; rise. D) remain the same; fall.

Answer: C

Game theory

Type: A Topic: 7 E: 469 MI: 225 Status: New

141. Game theory:

A) is the analysis of how people (or firms) behave in strategic situations.

B) is best suited for analyzing purely competitive markets.

C) reveals that mergers between rival firms are self-defeating.

D) reveals that price-fixing among firms reduces profits.

Answer: A

Page 833: Microeconomics Study Guide

Type: A Topic: 7 E: 469 MI: 225 Status: New

142. The study of how people (or firms) behave in strategic situations is called:

A) cost-benefit analysis. B) recursive analysis. C) normative economics. D) game theory.

Answer: D

Type: A Topic: 7 E: 469-470 MI: 225-226

143. The terms strategic behavior and payoff matrix both relate directly to:

A) the perfect competition model. C) game theory.

B) the monopolistic competition model. D) the price leadership model.

Answer: C

Type: A Topic: 7 E: 469 MI: 225

144. Game theory is best suited to analyze the pricing behavior of:

A) pure monopolists. B) pure competitors. C) monopolistic competitors. D) oligopolists.

Answer: D

Type: A Topic: 7 E: 470 MI: 226

145. Game theory can be used to demonstrate that oligopolists:

A) rarely consider the potential reactions of rivals.

B) experience economies of scale.

C) that oligopolists can increase their profits through collusion.

D) may be either homogeneous or differentiated.

Answer: C

Use the following to answer questions 146-151:

Page 834: Microeconomics Study Guide

Type: T Topic: 7 E: 470 MI: 226

146. The above matrix best illustrates:

A) game theory. B) the principal-agent problem. C) product differentiation. D) price discrimination.

Answer: A

Page 835: Microeconomics Study Guide

Type: T Topic: 7 E: 470 MI: 226

147. Refer to the above diagram where the numerical data show profits in millions of dollars. Beta's profits are shown in the northeast corner and Alpha's profits in the southwest corner of each cell. If both firms follow a high-price policy:

A) Alpha will realize a $10 million profit and Beta a $30 million profit.

B) each will realize a $20 million profit.

C) Beta will realize a $10 million profit and Alpha a $30 million profit.

D) each will realize a $15 million profit.

Answer: B

Type: T Topic: 7 E: 470 MI: 226

148. Refer to the above diagram wherein the numerical data show profits in millions of dollars. Beta's profits are shown in the northeast corner and Alpha's profits in the southwest corner of each cell. If Beta commits to a high-price policy, Alpha will gain the largest profit by:

A) also adopting a high-price policy.

B) adopting a low-price policy.

C) adopting a low-price policy, but only if Beta agrees to do the same.

Answer: B

Type: T Topic: 7 E: 470 MI: 226

149. Refer to the above diagram where the numerical data show profits in millions of dollars. Beta's profits are shown in the northeast corner and Alpha's profits in the southwest corner of each cell. With independent pricing the outcome of this duopoly game will gravitate to cell:

A) A. B) B. C) C. D) D.

Answer: D

Type: T Topic: 7 E: 470-471 MI: 226-227

150. Refer to the above diagram where the numerical data show profits in millions of dollars. Beta's profits are shown in the northeast corner and Alpha's profits in the southwest corner of each cell. If Alpha and Beta engage in collusion, the outcome of the game will be at cell:

A) A. B) B. C) C. D) D.

Answer: A

Page 836: Microeconomics Study Guide

Type: T Topic: 7 E: 470-471 MI: 226-227

151. Refer to the above diagram where the numerical data show profits in millions of dollars. Beta's profits are shown in the northeast corner and Alpha's profits in the southwest corner of each cell. If Alpha and Beta agree to a high-price policy through collusion, the temptation to cheat on that agreement is demonstrated by the fact that:

A) Beta can increase its profit by lowering its price.

B) Beta can increase its profit by increasing its price still further.

C) both Alpha and Beta can earn even more profits if both agree to a low-price policy.

D) Alpha can increase its profit by reducing its production costs.

Answer: A

Page 837: Microeconomics Study Guide

Use the following to answer questions 152-154:

Type: T Topic: 7 E: 470 MI: 226

152. Refer to the above profits-payoff table for a duopoly. If the firms are acting independently and firm X sets its price at $6, firm Y will achieve the largest profit by selecting:

A) a price higher than $6. B) a price between $5 and $6. C) $6. D) $4.

Answer: D

Type: T Topic: 7 E: 470 MI: 226

153. Refer to the above profits-payoff table for a duopoly. If initially firms X and Y are charging $5 and $4 respectively:

A) the two firms will be maximizing joint profits.

B) Y will find it advantageous to raise its price if it was certain X would not alter its price.

C) X will find it advantageous to raise its price if it was certain Y would not alter its price.

D) both firms would find it advantageous to collude to raise their prices by $1 each.

Answer: D

Type: T Topic: 7 E: 470 MI: 226

154. Refer to the above profits-payoff table for a duopoly. If initially firm X's price was $6 and Y's price was $5:

A) X would find it profitable to cut price, provided Y also cut price.

B) Y would find it profitable to cut price, provided X also cut price.

C) Y would find it profitable to raise price by $1, provided X would also raise price by $1.

D) both firms would profit by simultaneously lowering their prices by $1.

Page 838: Microeconomics Study Guide

Answer: C

Page 839: Microeconomics Study Guide

Use the following to answer questions 155-156:

BA

C D

$ 8 0 0

$ 8 0 0

$120 0

$600

$100 0

$100 0

$120 0

$600

L a rg eb u d g et

S m allb u d get

L a rg eb u d g et

S m allb u d get

A jax’s a d vertis in g stra tegy

Acm

e’s

adve

rtis

ing

s

trat

egy

Type: A Topic: 7 E: 470 MI: 226 Status: New

155. Refer to the above game theory matrix where the numerical data show the profits resulting from alternative combinations of advertising strategies for Ajax and Acme. Ajax's profits are shown in the upper right part of each cell; Acme's profits are shown in the lower left. Without collusion, the outcome of the game is cell:

A) A. B) B. C) C. D) D.

Answer: A

Type: A Topic: 7 E: 470-471 MI: 226-227 Status: New

156. Refer to the above game theory matrix where the numerical data show the profits resulting from alternative combinations of advertising strategies for Ajax and Acme. Ajax's profits are shown in the upper right part of each cell; Acme's profits are shown in the lower left. With collusion and no cheating, the outcome of the game is cell:

A) A. B) B. C) C. D) D.

Answer: D

Kinked-demand curve model

Type: A Topic: 8 E: 472 MI: 228

157. Suppose an oligopolistic producer assumes its rivals will ignore a price increase but match a price cut. In this case the firm perceives its:

A) demand curve as being of unit elasticity throughout.

Page 840: Microeconomics Study Guide

B) supply curve as kinked, being steeper below the going price than above.

C) demand curve as kinked, being steeper below the going price than above.

D) demand curve as kinked, being steeper above the going price than below.

Answer: C

Page 841: Microeconomics Study Guide

Type: A Topic: 8 E: 472 MI: 228

158. The kinked-demand curve of an oligopolist is based on the assumption that:

A) competitors will follow a price cut but ignore a price increase.

B) competitors will match both price cuts and price increases.

C) competitors will ignore a price cut but follow a price increase.

D) there is no product differentiation.

Answer: A

Type: A Topic: 8 E: 472 MI: 228

159. The kinked-demand curve describes a situation in which an oligopolist will be:

A) interested in maintaining the going price unless there is a rather large change in costs.

B) anxious to either increase or lower price.

C) anxious to increase price but not to lower price.

D) anxious to lower price but not to increase price.

Answer: A

Type: A Topic: 8 E: 472 MI: 228

160. If an oligopoly is faced with a kinked-demand curve that is relatively elastic above, and relatively inelastic below, the going price, then it will:

A) increase total revenue by increasing price, but lower total revenue by decreasing price.

B) decrease total revenue by either increasing or decreasing price.

C) increase total revenue by either increasing or decreasing price.

D) increase total revenue by decreasing price, but lower total revenue by increasing price.

Answer: B

Type: A Topic: 8 E: 472 MI: 228

161. The kinked-demand curve model of oligopoly is useful in explaining:

A) the way that collusion works.

B) why oligopolistic prices and outputs are extremely sensitive to changes in marginal cost.

C) why oligopolistic prices might change only infrequently.

D) the process by which oligopolists merge with one another.

Page 842: Microeconomics Study Guide

Answer: C

Type: A Topic: 8 E: 472 MI: 228

162. The kinked-demand curve model helps to explain price rigidity because:

A) there is a gap in the marginal revenue curve within which changes in marginal cost will not affect output or price.

B) demand is inelastic above and elastic below the going price.

C) the model assumes firms are engaging in some form of collusion.

D) the associated marginal revenue curve is perfectly elastic at the going price.

Answer: A

Type: A Topic: 8 E: 472 MI: 228

163. If competing oligopolists completely ignore oligopolist X's price changes, then X's:

A) demand curve will be less elastic than if the other oligopolists matched X's price changes.

B) demand curve will be more elastic than if the other oligopolists matched X's price changes.

C) marginal revenue curve will have a vertical gap.

D) demand and marginal revenue curves will coincide.

Answer: B

Page 843: Microeconomics Study Guide

Type: A Topic: 8 E: 472 MI: 228

164. If an oligopolist is faced with a marginal revenue curve that has a gap in it, we may assume that:

A) it is colluding with its rivals to maximize joint profits.

B) its demand curve is kinked.

C) it is selling a standardized product.

D) it is selling a differentiated product.

Answer: B

Type: A Topic: 8 E: 472 MI: 228

165. The kinked-demand curve model of oligopoly:

A) assumes a firm's rivals will ignore a price cut but match a price increase.

B) embodies the possibility that changes in unit costs will have no effect on equilibrium price and output.

C) assumes a firm's rivals will match any price change it may initiate.

D) assumes a firm's rivals will ignore any price change it may initiate.

Answer: B

Use the following to answer questions 166-169:

Type: G Topic: 8 E: 472-473 MI: 228-229

166. Refer to the above diagram for a noncollusive oligopolist. Suppose that the firm is initially in equilibrium at point E where the equilibrium price and quantity are P and Q. Which of the following statements is correct?

A) Demand curve D1 assumes that rivals will match any price change initiated by this oligopolist.

B) Demand curves D1 and D2 both assume that rivals will ignore any price change initiated by this oligopolist.

C) Demand curves D1 and D2 both assume that rivals will match any price change initiated by this oligopolist.

D) Demand curve D2 assumes that rivals will match any price change initiated by this oligopolist.

Page 844: Microeconomics Study Guide

Answer: A

Type: G Topic: 8 E: 472-473 MI: 228-229

167. Refer to the above diagram for a noncollusive oligopolist. Suppose that the firm is initially in equilibrium at point E where the equilibrium price and quantity are P and Q. If the firm's rivals will ignore any price increase but match any price reduction, then the firm's demand curve will be:

A) D1ED2. B) D2ED1. C) D1ED1. D) D2ED2.

Answer: B

Page 845: Microeconomics Study Guide

Type: G Topic: 8 E: 472-473 MI: 228-229

168. Refer to the above diagram for a noncollusive oligopolist. We assume that the firm is initially in equilibrium at point E where the equilibrium price and quantity are P and Q. If the firm's rivals will ignore any price increase but match any price reduction, the firm's marginal revenue curve will be:

A) D1ED2. B) MR2abMR1. C) MR2aMR2. D) MR1bMR1.

Answer: B

Type: G Topic: 8 E: 472-473 MI: 228-229

169. Refer to the above diagram for a noncollusive oligopolist. We assume that the firm is initially in equilibrium at point E where the equilibrium price and quantity are P and Q. If the firm's rivals will ignore any price increase but match any price reduction, over what range might marginal cost rise without disturbing equilibrium price and output?

A) bE B) ab C) Qa D) Qb

Answer: B

Use the following to answer questions 170-174:

Type: G Topic: 8 E: 473 MI: 229

170. The above diagram portrays:

A) pure competition. B) monopolistic competition. C) noncollusive oligopoly. D) pure monopoly.

Answer: C

Type: G Topic: 8 E: 473 MI: 229

171. Refer to the above diagram. Equilibrium output is:

A) j. B) h. C) g. D) f .

Page 846: Microeconomics Study Guide

Answer: C

Type: G Topic: 8 E: 473 MI: 229

172. Refer to the above diagram. Equilibrium price is:

A) e. B) d. C) c. D) b.

Answer: B

Page 847: Microeconomics Study Guide

Type: G Topic: 8 E: 472-473 MI: 228-229

173. Refer to the above diagram. This firm's demand and marginal revenue curves are based on the assumption that:

A) the firm has no immediate rivals.

B) rivals will match both a price increase and a price decrease.

C) rivals will match a price increase, but ignore a price decrease.

D) rivals will ignore a price increase, but match a price decrease.

Answer: D

Type: G Topic: 8 E: 472-473 MI: 228-229

174. Refer to the above diagram. In equilibrium the firm:

A) is realizing an economic profit of ad per unit. C) is incurring a loss.

B) should close down in the short run. D) is realizing an economic profit of bd per unit.

Answer: A

Type: A Topic: 8 E: 472-473 MI: 228-229

175. According to the kinked-demand curve model, an oligopolistic firm will produce where:

A) average total cost is minimized.

B) price equals marginal cost.

C) marginal revenue equals marginal cost.

D) the demand curve intersects the average total cost curve.

Answer: C

Type: A Topic: 8 E: 472-473 MI: 228

176. A kink may exist in an oligopolist's demand curve because:

A) products are differentiated.

B) an abrupt change in price elasticity occurs.

C) the firm will ignore price cuts by rivals, but will match their price increases.

D) there is a gap in marginal costs.

Answer: B

Page 848: Microeconomics Study Guide

Collusion; cartels; price leadership

Type: A Topic: 9 E: 475 MI: 231

177. OPEC provides an example of:

A) a tacit understanding. C) an international cartel.

B) noncollusive oligopoly. D) a monopolistically competitive industry.

Answer: C

Type: A Topic: 9 E: 474 MI: 230

178. Oligopolistic firms engage in collusion to:

A) minimize unit costs of production.

B) realize allocative efficiency, that is, the P = MC level of output.

C) earn greater profits.

D) increase production.

Answer: C

Page 849: Microeconomics Study Guide

Type: A Topic: 9 E: 476 MI: 232

179. The likelihood of a cartel being successful is greater when:

A) firms are producing a differentiated, rather than a homogeneous, product.

B) cost and demand curves of various participants are very similar.

C) the number of firms involved is relatively large.

D) the economy is in the recession phase of the business cycle.

Answer: B

Type: A Topic: 9 E: 476 MI: 232

180. Cartels are difficult to maintain in the long run because:

A) they are illegal in all industrialized countries.

B) individual members may find it profitable to cheat on agreements.

C) it is more profitable for the industry to charge a lower price and produce more output.

D) entry barriers are insignificant in oligopolistic industries.

Answer: B

Type: A Topic: 9 E: 474-476 MI: 230-232

181. Three major means of collusion by oligopolists are:

A) cartels, tacit understandings, and price leadership.

B) market sharing, mutual interdependence, and product differentiation.

C) cartels, kinked-demand pricing, and product differentiation.

D) tacit understandings, P = MC pricing, and mutual interdependence.

Answer: A

Type: A Topic: 9 E: 474 MI: 230

182. If the firms in an oligopolistic industry can establish an effective cartel, the resulting output and price will approximate those of:

A) a purely competitive producer.

B) a pure monopoly.

C) a monopolistically competitive producer.

D) an industry with a low four-firm concentration ratio.

Page 850: Microeconomics Study Guide

Answer: B

Type: F Topic: 9 E: 476 MI: 232

183. In the United States cartels are:

A) quite common in industries that produce nondurable goods.

B) in violation of the antitrust laws.

C) concentrated in monopolistically competitive industries.

D) encouraged by government policy so firms can achieve economies of scale.

Answer: B

Type: A Topic: 9 E: 475 MI: 231

184. Assume the several manufacturers of ceramic tile in a city reach a verbal agreement to establish the price of their product at 55 cents per tile. This best describes:

A) multiproduct pricing. B) a cartel. C) a tacit understanding. D) price leadership.

Answer: C

Page 851: Microeconomics Study Guide

Type: A Topic: 9 E: 476 MI: 232

185. One would expect that collusion among oligopolistic producers would be easiest to achieve in which of the following cases?

A) a rather large number of firms producing a differentiated product

B) a very few firms producing a differentiated product

C) a rather large number of firms producing a homogeneous product

D) a very few firms producing a homogeneous product

Answer: D

Type: D Topic: 9 E: 474 MI: 230

186. Suppose the only three existing manufacturers of video game players signed a written contract by which each agreed to charge the same price for products and to distribute their products only in the geographical area assigned them in the contract. This best describes:

A) cost-plus pricing. B) multiproduct pricing. C) a cartel. D) price leadership.

Answer: C

Type: D Topic: 9 E: 475 MI: 231

187. Which of the following statements is correct?

A) A cartel is usually a formal agreement among oligopolists that sets product price and determines each firm's market share.

B) The practice of price leadership is almost always based on a formal written agreement.

C) All oligopolists heavily advertise their products.

D) Active and frequent price competition between firms is a basic characteristic of oligopoly.

Answer: A

Type: D Topic: 9 E: 477 MI: 233

188. Suppose firms in a collusive oligopoly decide to establish their prices at a level that discourages new rivals from entering the industry. This is called:

A) mutual interdependence. B) pricing the demand curve. C) limit pricing. D) price leadership.

Answer: C

Type: A Topic: 9 E: 474 MI: 230

Page 852: Microeconomics Study Guide

189. If the several oligopolistic firms that comprise an industry behave collusively, the resulting price and output will most likely resemble those of:

A) bilateral monopoly. B) pure monopoly. C) monopolistic competition. D) pure competition.

Answer: B

Type: A Topic: 9 E: 476 MI: 232

190. Other things equal, cartels and similar collusive arrangements are easier to establish and maintain:

A) when there are ample opportunities for the firms to make secret price concessions to selected buyers.

B) during periods of cyclical stability and full employment.

C) when the demand and cost conditions of the participating firms differ substantially.

D) when the number of firms is relatively large.

Answer: B

Type: D Topic: 9 E: 477 MI: 233

191. A break-down in price leadership leading to successive rounds of price cuts is known as:

A) limit pricing. B) a price war. C) tacit pricing D) price discrimination

Answer: B

Page 853: Microeconomics Study Guide

Type: F Topic: 9 E: 475 MI: 231

192. Which of the following nations is not a member of the OPEC oil cartel?

A) Saudi Arabia. B) Iran. C) Venezuela. D) Norway.

Answer: D

Type: F Topic: 9 E: 475 MI: 231

193. Which of the following companies was convicted in the 1990s as part of a conspiracy to fix prices?

A) Archer Daniels Midland B) Boeing C) Wendy's D) IBM

Answer: A

Type: F Topic: 9 E: 475 MI: 231

194. Secret conspiracies to fix prices are examples of:

A) cartels. B) price leadership. C) overt collusion. D) covert collusion.

Answer: D

Advertising

Type: F Topic: 10 E: 477 MI: 233

195. In recent years advertising expenditures in the United States have been:

A) 10 to 12 percent of GDP per year. C) more than $215 billion per year.

B) $1 to $2 billion per year. D) about $20 billion per year.

Answer: C

Type: A Topic: 10 E: 478 MI: 234

196. Advertising can enhance economic efficiency when it:

A) increases brand loyalty.

B) raises entry barriers.

C) increases consumer awareness of substitute products.

D) boosts average total cost.

Page 854: Microeconomics Study Guide

Answer: C

Type: A Topic: 10 E: 478 MI: 234

197. Advertising can enhance economic efficiency when it:

A) increases brand loyalty.

B) expands sales such that firms achieve substantial economies of scale.

C) keeps new firms from entering profitable industries.

D) is undertaken by pure competitors.

Answer: B

Type: A Topic: 10 E: 478-479 MI: 234-235

198. Advertising can impede economic efficiency when it:

A) increases entry barriers.

B) reduces brand loyalty.

C) enables firms to achieve substantial economies of scale.

D) increases consumer awareness of substitute products.

Answer: A

Page 855: Microeconomics Study Guide

Type: A Topic: 10 E: 478-479 MI: 234-235

199. Advertising can impede economic efficiency when it:

A) reduces entry barriers.

B) reduces brand loyalty.

C) leads to greater monopoly power.

D) provides consumers with useful information about product quality.

Answer: C

Type: A Topic: 10 E: 479 MI: 235

200. The presence of advertising in a particular market:

A) tells us that the industry is an oligopoly.

B) tells us that the industry is monopolistically competitive.

C) means that barriers to entering the industry are high.

D) may or may not mean substantial monopoly power in the industry.

Answer: D

Efficiency aspects

Type: A Topic: 11 E: 479 MI: 235

201. We would expect a cartel to achieve:

A) both allocative efficiency and productive efficiency.

B) allocative efficiency, but not productive efficiency.

C) productive efficiency, but not allocative efficiency.

D) neither allocative efficiency nor productive efficiency.

Answer: D

Type: C Topic: 11 E: 479 MI: 235

202. Suppose that a particular industry has a a four-firm concentration ratio of 85 and a Herfindahl Index of 3000. Most likely, this industry would achieve:

A) both productive efficiency and allocative efficiency.

Page 856: Microeconomics Study Guide

B) allocative efficiency but not productive efficiency.

C) neither productive efficiency nor productive efficiency.

D) productive efficiency but not allocative efficiency.

Answer: C

Type: A Topic: 11 E: 479 MI: 235

203. Suppose that an industry is characterized by a few firms and price leadership. We would expect that:

A) price would equal marginal cost.

B) price would equal average total cost.

C) price would exceed both marginal cost and average total cost.

D) marginal revenue would exceed marginal cost.

Answer: C

Page 857: Microeconomics Study Guide

Type: A Topic: 11 E: 479 MI: 235

204. The conclusion that oligopoly is inefficient relative to the competitive ideal must be qualified because:

A) industry price leaders often select a price equal to marginal cost.

B) over time oligopolistic industries may promote more rapid product development and greater improvement of production techniques than if they were purely competitive.

C) increased output due to persuasive advertising may perfectly offset the restriction of output caused by monopoly power.

D) many oligopolists sell their products in monopolistically competitive or even purely competitive industries.

Answer: B

Review of four structures

Type: A Topic: 12 E: 461, 467 MI: 217, 223

205. Monopolistic competition and oligopoly are alike in that:

A) the number of firms is approximately the same in both cases.

B) the kinked-demand analysis is applicable in both instances.

C) strong mutual interdependence exists among firms in both market models.

D) nonprice competition is common to both.

Answer: D

Type: A Topic: 12 E: 461, 467 MI: 217, 223

206. Product differentiation is present in:

A) purely competitive markets only.

B) monopolistically competitive markets only.

C) oligopolistic markets only.

D) both monopolistically competitive and oligopolistic markets.

Answer: D

Type: A Topic: 12 E: 479 MI: 235

207. Under which of the following market structures will the long-run equilibrium price be equal to marginal cost?

A) oligopoly B) monopolistic competition C) pure monopoly D) pure competition

Page 858: Microeconomics Study Guide

Answer: D

Type: A Topic: 12 E: 439, 467 MI: 195, 223

208. In which of the following industry structures is the entry of new firms the most difficult?

A) pure monopoly B) oligopoly C) monopolistic competition D) pure competition

Answer: A

Type: A Topic: 12 E: 460 MI: 216

209. An industry comprised of 40 firms, none of which has more than 3 percent of the total market for a differentiated product is an example of:

A) monopolistic competition B) oligopoly C) pure monopoly D) pure competition

Answer: A

Page 859: Microeconomics Study Guide

Type: A Topic: 12 E: 413, 438 MI: 169, 194

210. A one-firm industry is known as:

A) monopolistic competition B) oligopoly C) pure monopoly D) pure competition

Answer: C

Type: A Topic: 12 E: 467 MI: 223

211. An industry comprised of four firms, each with approximately 25 percent of the total market for a product, is an example of:

A) monopolistic competition B) oligopoly C) pure monopoly D) pure competition

Answer: B

Type: A Topic: 12 E: 413 MI: 169

212. An industry comprised of a very large number of sellers that are producing a homogeneous or standardized product is called:

A) monopolistic competition B) oligopoly C) pure monopoly D) pure competition

Answer: D

Type: A Topic: 12 E: 414, 467 MI: 170, 223

213. An industry comprised of a small number of firms, each of which considers the potential reactions of its rivals in making price-output decisions is called:

A) monopolistic competition B) oligopoly C) pure monopoly D) pure competition

Answer: B

Type: A Topic: 12 E: 468 MI: 224

214. An industry producing a differentiated product whose four-firm concentration ratio is 18 percent is an example of:

A) monopolistic competition B) oligopoly C) pure monopoly D) pure competition

Answer: A

Type: A Topic: 12 E: 468 MI: 224

215. An industry producing a homogeneous product whose four-firm concentration ratio is 76 percent is an example of:

Page 860: Microeconomics Study Guide

A) monopolistic competition B) oligopoly C) pure monopoly D) pure competition

Answer: B

Type: A Topic: 12 E: 414 MI: 170

216. In which of the following market models do individual firms exert no control over product price?

A) oligopoly B) pure monopoly C) monopolistic competition D) pure competition

Answer: D

Page 861: Microeconomics Study Guide

Type: A Topic: 12 E: 414 MI: 170

217. Which of the following correctly arrays the various market structures in terms of their similarities to one another?

A) pure competition, oligopoly, monopolistic competition, pure monopoly

B) pure monopoly, oligopoly, monopolistic competition, pure competition

C) pure competition, pure monopoly, monopolistic competition, oligopoly

D) pure competition, oligopoly, pure monopoly, monopolistic competition

Answer: B

Use the following to answer questions 218-226:

Type: G Topic: 12 E: 426 MI: 182

218. The purely competitive market model is portrayed in the above figures by:

A) Figure A. B) Figure B. C) both Figures B and D. D) Figure C.

Answer: B

Type: G Topic: 12 E: 414, 461 MI: 170, 217

219. Refer to the above figures. We would expect industry entry and exit to be relatively easy in:

A) Figure A only. B) Figure C only. C) both Figures A and C. D) both Figures B and D.

Answer: D

Page 862: Microeconomics Study Guide

Type: G Topic: 12 E: 432 MI: 188

220. Refer to the above figures. Both allocative and productive efficiency are being realized in:

A) all four figures. B) Figures B and D. C) Figure D only. D) Figure B only.

Answer: D

Type: G Topic: 12 E: 474-475 MI: 230-231

221. Refer to the above figures. Collusion is most likely to occur in the industry(ies) represented by:

A) Figure A. B) Figure B. C) Figure C. D) both Figures B and D.

Answer: C

Page 863: Microeconomics Study Guide

Type: G Topic: 12 E: 461, 467 MI: 217, 223

222. Refer to the above figures. Product differentiation may be present in:

A) Figure A only. B) Figure B only. C) Figure C only. D) both Figures C and D.

Answer: D

Type: G Topic: 12 E: 453-454 MI: 209-210

223. Refer to the above figures. Government regulation of price and service is most likely to occur in:

A) Figure A only. B) Figure D only. C) both Figures A and C. D) both Figures A and D.

Answer: A

Type: G Topic: 12 E: 474 MI: 230

224. Refer to the above figures. Long-run economic profits are most likely to occur in:

A) Figures A and B. B) Figure B only. C) Figure D. D) Figures A and C.

Answer: D

Type: G Topic: 12 E: 414 MI: 170

225. Refer to the above figures. Industry entry is likely to be most difficult in:

A) Figure A. B) Figure B. C) Figure C. D) Figure D.

Answer: A

Type: G Topic: 12 E: 414 MI: 170

226. Refer to the above figures. A homogeneous or standardized product is most likely to be produced in:

A) Figure A. B) Figure B. C) Figure C. D) Figure D.

Answer: B

Consider This Questions

Type: A E: 470 MI: 226 Status: New

Page 864: Microeconomics Study Guide

227. (Consider This) The story about three sellers of Native American arts and crafts best illustrates the idea of;

A) strategic behavior. B) excess capacity. C) the role of advertising. D) product differentiation.

Answer: A

Type: A E: 470 MI: 226 Status: New

228. (Consider This) The Native American arts and crafts story illustrates the twin ideas of:

A) product differentiation and monopolistic competition.

B) excess capacity and monopolistic competition.

C) local oligopoly and strategic behavior.

D) pure monopoly and price discrimination.

Answer: C

Page 865: Microeconomics Study Guide

Last Word Questions

Type: F E: 480 MI: 236

229. (Last Word) The U.S. beer industry:

A) has become monopolistically competitive as the result of new production technologies.

B) has evolved from monopolistic competition to oligopoly in the past 50 years.

C) is populated by hundreds of relatively large, independent brewers.

D) approximates the purely competitive market model.

Answer: B

Type: F E: 480-481 MI: 236-237

230. (Last Word) Increased concentration in the beer industry has been caused by:

A) changes in consumer tastes from the strong beers of small breweries to the light beers of the large brewers.

B) a shift of beer consumption from bars to homes.

C) technological progress which has speeded up canning and bottling lines and lowered costs.

D) all of the above factors.

Answer: D

Type: F E: 480 MI: 236

231. (Last Word) The two largest U.S. brewers share about:

A) 10 percent of the beer market. C) 50 percent of the beer market.

B) 20 percent of the beer market. D) 70 percent of the beer market.

Answer: D

Type: F E: 480 MI: 236

232. (Last Word) The leading seller of beer in the United States is:

A) Coors. B) Anheuser-Busch. C) Miller Brewing Company. D) Corona.

Answer: B

Page 866: Microeconomics Study Guide

Type: F E: 469, 480 MI: 225, 236

233. (Last Word) The two leading producers of beer in the United States have market shares of 47 percent and 22 percent, respectively. On the basis of that information, we can conclude that:

A) the four-firm concentration ratio in the beer industry is nearly 100 percent.

B) the Herfindahl index in the beer industry exceeds 2500.

C) firms practice overt collusion to set prices in the industry.

D) the industry is monopolistically competitive.

Answer: B

True/False Questions

Type: A E: 464 MI: 220

234. In the long run monopolistically competitive firms make normal profits because they are forced to operate at the minimum point on their average total cost curve.

Answer: False

Page 867: Microeconomics Study Guide

Type: A E: 464-465 MI: 220-221

235. The monopolistically competitive seller maximizes profits by equating price and marginal cost.

Answer: False

Type: A E: 464-465 MI: 220-221

236. Monopolistically competitive firms are inefficient because they produce at a point on the rising segment of their average cost curves.

Answer: False

Type: A E: 462 MI: 218

237. The demand curve of a monopolistically competitive producer is less elastic than that of a purely competitive producer.

Answer: True

Type: D E: 464 MI: 220

238. The larger the number of firms and the less the degree of product differentiation, the greater will be the elasticity of a monopolistically competitive seller's demand curve.

Answer: True

Type: A E: 464 MI: 220

239. The economic profits earned by monopolistically competitive sellers are zero in the long run.

Answer: True

Type: A E: 465-466 MI: 221-222

240. The excess capacity problem associated with monopolistic competition implies that fewer firms could produce the same industry output at a lower total cost.

Answer: True

Type: A E: 462 MI: 218

241. The demand curve of a monopolistically competitive firm is more elastic than that of a pure monopolist.

Page 868: Microeconomics Study Guide

Answer: True

Type: A E: 464 MI: 220

242. The monopolistically competitive seller equates price and marginal cost in maximizing profits.

Answer: False

Type: A E: 464 MI: 220

243. Monopolistically competitive sellers realize economic profits in the long run because entry barriers are significant.

Answer: False

Type: A E: 464-465 MI: 220-221

244. Monopolistically competitive sellers produce efficiently because they obtain only normal profits in the long run.

Answer: False

Page 869: Microeconomics Study Guide

Type: A E: 472 MI: 228

245. The oligopolist's kinked-demand curve is highly elastic below and highly inelastic above the going product price.

Answer: False

Type: A E: 467 MI: 223

246. Mutual interdependence means that oligopolistic producers rely on price competition in determining their shares of the total market for their product.

Answer: False

Type: D E: 472 MI: 228

247. If an oligopolist's several rivals exactly match any price changes it initiates, the demand curve will be less elastic than if its price changes are ignored by its rivals.

Answer: True

Type: A E: 474 MI: 230

248. If three or four homogeneous oligopolists collude, the resulting price and production outcomes will be similar to those of pure monopoly.

Answer: True

Type: A E: 476 MI: 232

249. Generally speaking, the larger the number of firms in an oligopolistic industry, the more difficult it is for those firms to collude.

Answer: True

Type: A E: 467 MI: 223

250. Generally speaking, oligopolistic industries producing raw materials and semifinished goods usually offer differentiated products, while oligopolists producing consumer goods usually offer standardized products.

Answer: False

Page 870: Microeconomics Study Guide

Type: C E: 468-469 MI: 224-225

251. Two industries that have the same 4-firm concentration ratio can have significantly different Herfindahl indexes.

Answer: True

Type: F E: 469 MI: 225

252. As it relates to oligopoly, game theory focuses on the strategic behavior of rival firms.

Answer: True

Type: F E: 469 MI: 225

253. The highest possible value of the Herfindahl index is 1,000.

Answer: False

Page 871: Microeconomics Study Guide

Type: D E: 467 MI: 223 Status: New

254. The market structure called "oligopoly" includes industries with one or a small number of firms.

Answer: False

Type: F E: 467 MI: 223 Status: New

255. The U.S. breakfast cereal industry is an example of differentiated oligopoly.

Answer: True

Type: F E: 467 MI: 223 Status: New

256. The U.S. steel industry is an example of homogeneous oligopoly.

Answer: True

Type: F E: 472 MI: 228 Status: New

257. In the kinked demand curve model, the firm's marginal revenue curve and demand curve are identical.

Answer: False

Type: F E: 467 MI: 223 Status: New

258. Homogenous oligopolists tend to advertise more than do differentiated oligopolists.

Answer: False